Sei sulla pagina 1di 835

Table of Contents

Chapter 1 Basics of Control System ~(1-1 ) to (1- 24)


1.1 Background 1 - 1
1.2 Definitions 1 - 1
1. C!assification of Contro! Systems 1 -
1.4 "pen #oop System $ 1 - %
1.4.1 &d'antages 1 - (
1.4.2 Disad'antages 1- 8
1.4. )ea! *ime &pp!ications of an "pen !oop System 1 - (
1.4..1 Sprink!er used to +ater a lawn. . 1- 8
1.4..2 Stepper ,otor -ositioning System. 1-8
1.4.. &utomatic *oaster System. . . . 1-.
1.4..4 *raffic /ght Contro!!er . . . . . 1-.
1.4..0 &utomatic Door "pening and C!"SingSystem . 1- 9
1.0 C!osed #oop System 1 - .
1.5.1 &d'antages . . . . . . . . . . . . . . . . . . . . . . . . . . . . . . . . . . . . . . . . . . . . . . . . . . . . . . . . . . . 1 - 11
1.5.2 Disad'antages 1 - 11
1.0. )ea! *ime &pp!ications of C!osed !oop System 1 - 11
1.0..1 1uman Being . . . . . 1 - 11
1.0..2 1ome 1eating System. . . . 1 - 11
1.0.. Ship Sta2i!i3ation System. . . 1-12
1.0..4 ,anua! Speed Contro! System.
1-12
1.0..0 D.C. ,otor Speed Contro! .
1- 13
1.0..4 *emperature Contro! System
1 - 14
1.0..% ,issi!e !aunching System .
1 - 14
1.0..( 5o!tage Sta2i!i3er. . . . . 1 - 14
1.4 Comparison of "pen #oop and C!osed #oop Contro! System 1 - 14
1.% Ser'omechanisms 66 1 - 14
1.( )egu!ating Systems ()egu!ators) 1 - 1%
y...... :":v
~.~- 7 ~... 8~.~
99: n!
1.9 Feedback and Feed Forward System 1 - 19
1.9.1 Real Time Application of Feed Forward System 1 - 20
1.10 Multiariable !ontrol Systems 1 - 21
Reiew "uestions ........ . ......... ## ..# 1 - 2$
- %&9 ..... ~!'
!'apter( 2 )asics of *aplaceTransform+. h ,<
... .
, (21) to - 220)
2.1 )ack.round 2 - 1
2.2 /efinition of *aplace Transform 2 - 1
2.$ 0roperties of *aplace Transform 2 - 2
2.3.1 linearity . . . . . . . . . . . . . . . . . . . . . . . . . . . . . . . . . . . . . . . . . . . . . . . . . . . . . . . 1 . . . . .. 2 - 2
2.3.2 Scalin. T'eorem. . . . . . . . . . . . . . . . . . . . . . . . . . . . . . . . . . . . . . . . . . . . . . . . . . . . . .. 2 - 3
2.$.$ Real /ifferentiation -/ifferentiation in Time /omain2 2 - $
2.$.3 Real 4nte.ration + 2 - $
2.3.5 /ifferentiation by s. . . . . . . . . . . . . . . . . . . . . . . . . . . . . . . . . . . . . . . . . . . . . . . . . . . . .. 2 - 4
2.3.6 !omple5 Translation 2 - 4
2.$.6 Real Translation -S'iftin. T'eorem2 2 - 3
2 $ 7 4nitial8alueT'eorem 2-4
2 $ 9 Final 8allie T'eorem 2 - 9
2.3 4nerse *aplace Transform 2 - 7
2.4.1 Simple and Real Roots. . . . . . . . . . . . . . . . . . . . . . . . . . . . . . . . . . . . . . . . . . . . . . . . .. 2 - 9
2.4.2 Multiple Roots 2 -10
2.4.3 !omple5 !on:u.ate Roots 2 -12
2.9 ;se of *aplace Transform in !ontrol System 2 - 13
2.< Special !ase of 4nerse *aplace Transform 2 - 19
=5amples wit' Solutions ..> 2 - 16
.. & #&. %&1?@-& .0J>' .
!'apter..$& Transfer F;flction.and4mpulseResponse.&.%.&.% ',>.,~ . -$..12 to -$(2<2
$.1 )ack.round $ - 1
$.2 !oncept of Transfer Function $ - 1
$.$ Transfer =unctAon # $ - 2
$.$.1 /efinition + + . .. . ! ! $ - 2
3.3.2 Adanta.es and Features of Transfer Function 3 - 3
3.3.3 /isadanta.es 3- 9
3.3.4 0rocedure to /etermine t'e Transfer Func;cn of a !ontrol System 3 - 9
$.3 4mpulse Response and Transfer Function $ - 6
$.9 Some 4mportant Terminolo.ies Related to t'e T.F $ - 10
44 B nl
3.5.1 Pc|es c| a Trars|er Furc||cr 3-1J
3.5.2 C|arac|er|s||c Lua||cr c| a Trars|er Furc||cr 3 - 11
. I If., .. '.
3.5.4 Pc|e-Zerc P|c|. . . . . . . . . . . . . . . . . . . . .. . . .. . 3 - 12
. . . . . ... . ' .
3.5.6 0.C. 0a|r 3-12
3.6 Lap|ace Trars|crm c| L|ec|r|ca| Ne|wcrk 3 - 14
Lxamp|es w||| Sc|u||crs 3 - 15
Rev|ew Ques||crs... .. " ." " 3 - 24
C|ap|er 4. Ha||ema||ca| Hcde||rg c| Ccr|rc| Sys|ems ~,?,~,ry., ~,\: (4 .. 1) to (4 - 64)
4.1 W|a| |s Ha||ema||ca| Hcde|? .4 - 1
4.2 Ara|ys|S c| Hec|ar|ca| Sys|ems 4 - 2
4.2.1 Trars|a||cra| Hc||cr . 4 - 2
4.2.2 Hass (H) 4 - 2
4.2.3 urear Spr|rg 4 - 3
4 2 4 Fr|c||cr.. .. . "'''.'....... 4 - 4
4 3 Rc|a||cra| Hc||cr 4 - 6
4.4 Lu|va|er| Hec|ar|ca| Sys|em (Ncde Bas|s) .4 - 6
: - II -. ~ _ IJ - . -
4.6 0ear Tra|rs 4 - 8
.. .
'. '. r. ' ~~~ .... 1 '.
. - .
4.6.2 Be|| cr C|a|r 0r|ves ..........~........... ' ~. . . .. . 4 - 11
4 6 3 Levers .. .. . 4 - 11
4.7 L|ec|r|ca| Sys|ems 4 - 11
4.8 Ara|cgcus Sys|ems 4 - 13
4.8.1 Hec|ar|ca| Sys|em 4 -13
4.8.2 Fcrce vc||age Ara|cgy (loop Ara|ys|s) .................................~... 4- 13
4.8.3 Fcrce Currer| Ara|cgy (Ncde Ara|ys|s) .................................~~.. 4-16
4.9 S|eps |c Sc|ve Prcb|ems cr Ara|cgcus Sys|ems 4 -16
r
4.10 Servcmc|crs 4 - 20
4.1J.1 Reu|remer|s c| 0ccd Servcmc|cr 4 - 2J
4.11 Types c| Servcmc|crs 4 - 20
4.12 0.C. Servcmc|cr 4 - 21
4.12.1 F|e|d Ccr|rc||ed 0.C. Servcmc|cr 4 - 21
4. 12.1.1 Fea|ures c| F|e|d Ccr|rc||ed 0.C. Servcmc|cr . . . . . . . . . . . . . . . . 4 - 21
r
. 1t... 4.12.2 Araature Ccrtrc||ec 0.C. Servcactcr .......................... 4 - 21
: -u |:!' , ...!,: ,',.",,:.'. .:;| 0,11.' .'
. - ~
.-
4.12.4 App||cat|crs c| 0.C. Servcactcr .............................. 4 - 23
-. . '.' ., '. , -,
u' , .-
-.. '
I.'!. | .._
L~ .-

-
'|||||| '- ,
4.15.1 Ccrstruct|cr ' ' ,., 4- 2I
4.152 Rctcr ........................................................ 42I
4.15.3 Tcrue-speec C|aracter|st|cs .............................. 4 - 2I
4.15.4 | eatures c| A. C.Servcactcr ................................. 4 - 2
4.1 S .S App||cat|crs .................... 4 - 2
4.1 S . Trars|er | urct|cr c| A.C. Servcactcr ........................ 4 - 28
4.1 Ccapar|scr c| Servcactcrs .4 - 30
4.1 .1 Ccapar|scr c etw eer A.C. arc 0.C.Servcactcr ................ 4- 30
4.1 .2 Ccapar|scr c etw eer Araature Ccrtrc|tec arc | |e|c Ccrtrc||ec D .C. Servcactcrs 4 - 30
4.1 I Mcce|s c| Ccaacr|y usec E|ectrcaec|ar|ca| Systeas .4 - 30
4.17.1 0ereratcrs ........................................ 4 31
4.17.2 0ereratcr 0r|v|r ~tcr ............................ 4 - 32
4.1 I.3 Pcs|t|cr Ccrtrc| Systea .......................................... 4 - 34
4.17.4 Pcs|t|cr Ccrtrc| w |t| | |e|c Ccrtrc||ec Mctcr .............................. 4 - 35
4.1 I.S Speec Ccrtrc| Systea ......................................... 4 - 3I
4.1 I. Speec Ccrtrc| us|r 0ereratcr 0r|v|r ~tcr ............................. 4 - 39
4.17.7 A Typ|ca| Pcs|t|cr Ccrtrc| Systea usec |r rcustry ................... 4 - 41
4.1 0.C. Mctcr Pcs|t|cr Ccrtrc| Systea .4 - 42
4.18.1 Trars|er | urct|cr c| 0.C. Mctcr Pcs|t|cr Ccrtrc| Systea .................. 443
4.1 9 Mcce|s c| T|eraa| Systeas 4 - 49
4,19.1 neat Trars|er Systea ..................................... 4 - 49
4.1 9.2 T|eracaeter ........................................ 4 - S 1
4.20 Actuatcrs 4 - S 1
4.20.1 nycrau||c Actuatcr 4 - S 2
4.20.2 Preuaat|c Actuatcr ............................................... 4 - 53
4.20.3 Ccapar|scr c etw eer Preuaat|c arc nycrau||c Systeas .... . . .. . . .. .... ... 4 - S 4
Exaap|es w |t| Sc|ut|crs 4 - S 4
~:r.
Rev|ew Quest|crs :..: 4 - 1
| 'r|
.' . . . . . . . .
~ 0h.tor.;~J::'"~ 0|agram~ontat|on~':('; :~:.- "~.Y~. " ";. , ": :: ~ ~. , ,~. ~. : . ~. ' ~ (5 ..:l ", t~:(. 5-_|a,
5.l Background 5 - l
5.l .l l||ustrat|ng 0oncopt ol B|ock 0|agram Roprosontat|on .......................~... 5 - 2
5.l .2 Advantagos ol B|ock 0|agram 5 - 1
5. 1. 3 0|sadvantagos ....~...................................~................. 5 - 4
5.2 S|mp|o or 0anon|ca| Form ol 0|osod Loop Systom 5 - 1
5.2.l 0or|vat|on ol T.F. ol S|mp|o 0|osod Loop Systom .........................~~.... 5- 5
5.3 Ru|os lor B|ock 0|agram Roduct|on 5 - 5
5 3 l 0r|t|ca|R|||os 5 - l 3
5.3.2 0onvort|ng Nonun|ty Foodoack to un|ty Foodoack .................~.~....~.... 5 - l 5
5.3.3 Procoduro to Solve B|ock 0|agram Roduct|on P,roo|oms 5- l
5.1 Ana|ys|s ol Mu|t|p|o lnput Mu|t|p|o Output Systoms 5 - l 8
5.5 B|ock 0|agram lrom Systom Equat|ons 5 - 21
Examp|os w|th So|ut|ons 5 - 21
:.. ". .. . .~l
Chapter - .. S|gna('Flow 0raph Roprosontat|on :'~~~;:,l ,:~,,~:: ,': ,~~~ ~ :: ~~:? "~.( ..l , to ( ~,
.l Background - l
.2 Proport|os ol S|gna| F|ow 0raph - 2
..3 Torm|no|ogy usod |n S|gna| F|ow 0raph - 3
E>.1Mothods to Ot>ta|nS|gna| F|ow 0raph - 5
.1.l From the Systom Equat|ons .....................~......................... - 5
.1.2 From the 0|von B|ock 0|agram ......~..~...~.........~........... ' -
~
M '
.
~I III ~ .-
.E>0ompar|son ol B|ock 0|agram and S|gna| F|ow 0raph Mothods E>- 9
E>.7App||cat|on ol tho 0onora| 0a|n Formu|a t>otwoon Output
Nodos and Non lnput Nodos E>- l 0
.8 App||cat|on ol Mason's 0a|n Formu|a to E|octr|ca| Notwork - l 2
.9 Oota|n|ng B|ock 0|agram lrom S|gna| F|ow 0raph - l 1
Examp|os w|th So|ut|ons - 17
.- .
0haptor ..7 T|mo Rosponso Ana|ys|s ol 0ontro| Systoms Y ."! (7l , to (7.l 30,
7.l Background 7 - l
7.2 0ol|n|t|on and 0|ass|l|cat|on ol T|mo Rosponso 7 - l
.. ~.
.,
l l n|
Control System Engineering
I S B N 9788184314632
All rights reserved with Technical Publications. N o port af this book should be
reproduced in any form, ledronic. !echanical, Photocopy or any information slorage end
retrieval systemwilhout prior permission in writing, from Technical Publications, Pune.
Published by "
Technlcal Publications Pune#
$%, Amit & esidency, '%(, S haniwarPeth, Pune) '%% *+*, I ndia.
Printer:
A J ertD T Printa s
S r.no. %*,+,S inhagad& oed,
Pune) '%%*'%
:"..:, ..
, ".L -,', ",' :':" ,.,,:. ~.,...:. -.-,;{ .t.. .- ". ,",./ 0 0 / )/ / . ".,. 0 ,,. 1 ) ,," "(if)." .,-. :.:.,'-:: :,.. ..!. !:' h ./ / ".1 . , 22'." ": ::,' :.'.....#.. , : :., .: '. .. : :.::.. .:: '!,t/ 3/ . ../ . ./ / / . . A,,0 ,/ ./ . / .j ,.;',.
$",' ,'",',. .::"':.:..... :.' ::::.::%:,,':: ,," ......:: %.:' ', ' "" .':' ".:& -:,," -"t-: "" " .... .__ ~ .,., " " ... " .. '" / - :,'.
' (
7.3 Standard Test Inputs : 7 - 4
7.4 Steady State Analysis 7 - 6
7.5 Derivation of Steady State Error 7 - 6
7.6 Effect of Input (Type and a!nitude" on Steady State Error
(Stati#Error #oefficient et$od" 7 - %
7.7 Effect of #$an!e in &(s"'(s" on Steady State Error
(T()E of a Syste*" 7 - +,
7.% Analysis of T()E ,-+ and . Syste*s 7 -++
7./ Disadvanta!es of Static Error #oefficient et$od u _ , 7 - +6
7.+, &eneralised Error #oefficient et$od (or Dyna*ic Error
#oefficients" 7 - +7
7.++ Transient 0esponse Analysis 7 - .3
7.++.+ et$od to Deter*ine Total 1utput c(t" 7 - .3
7.+. Analysis of 2irst 1rder Syste* 7 - .4
7.+..+ 3nit Step 0esponse of 2irst 1rder Syste* 7 - .4
7.+... #losed loop )oles of 2irst 1rder Syste* 7 - .6
7.+3 Analysis of Second 1rder Syste* 7 - .%
7.+4 Effect of l4on Second 1rder Syste* )erfor*ance 7 - ./
7.+5 Derivation of 3nit Step 0esponse of a Second 1rder Syste* .
(3nderda*ped #ase" 7 - 34
7.+6 Tra5sient 0esponse Specifications 7 - 3%
7.+7 Derivations of Ti*e Do*ain Specifications 7 - 3/
7.+7.+ Derivation of )ea6 Ti*e TD 7 - 39
7.+7.. Derivation ofp 7 -4+
7.+7.3 Derivation ofT- ............................7.......................... 7 -42
7.17.4 Derivation ofTs ........................................ 7 -43
7.+% 2eed8ac6 #$aracteristics of #ontrol Syste*s 7 - 44
7.+%.+ Effect of )ara*eter 9ariations in an 1pen loop #ontrol Syste* 77.7.7.7.7.....7. 7 - 45
7.+%.. Effect of )ara*eter 9ariations in a #losed loop Syste* 7 - 45
7.18.3 Sensitivity of a #ontrol Syste* -........7...... 7 - 46
7.18.4 Effect of 2eed8ac6 on Ti*e #onstant of a #ontrol Syste* ...............7.....7 7 - 4%
7.18.5 Effect of 2eed8ac6 on 1verall &ain..7.7................................... 7 - 50
1.18.6 Effect of 2eed8ac6 on Sta8ility 7 - 50
7.18.1 Effect of 2eed8ac6 on Distur8ance .............77.777...7. : 7 - 5+
I I nl
7.19 Performance Criterion 7 - 57
7.19.1 Mean Square Error (EMS) 7 - 57
7.19.2 Integral Square Error Criterion (ISE) 7 - 58
7.19.3 lltegral of !ime Multi"lie# Square Error Criterion (I!SE) 7 - 58
7.19.$ Integral of Square# !ime Multi"lie# %& Square# Error (IS!SE) 7 - 58
7.19.5 Integral of '%(olute )alue of Error (I'E) 7 - 59
7.19.* Integral of !ime Multi"lie# %& '%(olute )alue of Error (I!'E) 7 - 59
E+am"le( ,it- Solution( 7 - *.
/e0ie, 1ue(tion( 7 - 12$
':.. .-. 'q~ :.):.1 2 l- --3 4 5 5 6 6 3 . ""1 '" :.r .-; : ~ ' , ... ... 3 6 7I8.C6 3 9 6 6 3 4 6 7. 6 3
C-a"ter5 8 Sta%ilit& 'nal&(i( . . . .. . .. 6 : 5 .5 . 2 . (8 -1) to (8 ;$*)
8.1 <ac=groun# 8 - 1
8.2 Conce"t of Sta%ilit& 8 - 1
8.3 Sta%ilit& of Control S&(tem( 8 - 3
8.$ >ero In"ut an# '(&m"totic Sta%ilit& 8 - 8
8.4.1 /emar=( a%out '(&m"totic Sta%ilit& 8 - 8
8.5 /elati0e Sta%ilit& 8 - 9
8 * /out--?ur,it@ Criterion . . 8 - 9
8.6.1 Aece((ar& Con#ition( 8 - 1.
8*2 ?ur,it@( Criteoon 6 4 8-1.
8.6.3 Bi(a#0antage( of ?ur,it@( Met-o# 8 - 11
8.7 /out-( Sta%ilit& Criterion 8 - 11
87.1 /out-( Criterion 5 5 8- 12
8.8 S"ecial Ca(e( of /out-( Criterion 8 - 13
8.8.1 S"ecial Ca(e 1 8 - 13
8.8.2 S"ecial Ca(e 2 8 - 1$
8.8.2.1 Proce#ure to Elirrinate t-i( Bifficult& . . . . . 8 -15
8.8.2.2Im"crtance of an 'u+iliar& Equation. . . . . . . . . . . . . . . . . . . . . 8 -15
8.8.2.3 C-ange in Criterion of Sta%ilit& in S"ecial Ca(e 2 . . . . . . . . C . . . . . . 8 - 1*
8.9 '""lication( of /out-( Criterion 8 - 17
8.9.1 /elati0e Sta%ilit& 'nal&(i( 8 - 17
8.9.2 Betermining /ange of )alue( of D 8 - 17
8.1. '#0antage( of /out-( Criterion 8 - 18
8 11 Eimitation( of /out-( Criterion 8 - 18
8.12 Marginal D an# Frequenc& of Su(taine# G(cillation( 8 - 2.
E+am"le( ,it- Solution( 8 - 21
/e0ie, 1ue(tion( C C5 8 - $3
.. .4
. c ~:::
_ " . -" .
........ 4
x .
l I
hi rIt-v:':1 'R ott .AA.f.' '>."." y . " , : ~ / _ c . . <. ~. " , 0 ? ' z: , 1' 1 ' 4\6. (a ' 1 I )
U|IHI.I'.|oor~:.'"~.. 1 ..~ ! ' ~ ~ '. . . ~ ~, ~ . , . : :" :, '. : '\ , a_ ".:~Q '.. J<I~
9. 1 Background 9 - 1
1 .I Basic Concopt of R oot Locus 1 - 1
1 .3 Ang|o and Magnitudo Condition 1 - 3
1 .3.' Ang|o Condition ........................................................ 1 - 3
1 .3.I Uso of Ang|o Condition ............................................... 1 - 1
1 .3.3 Magnitudo Condition 1 - 5
1 .3.1 Uso of Magnitudo Condition 1 - 5
1 .~ <:;raJ) t' i~'Mott|od of [) otorrT|ining't(1 1 - E5
.. . . . . : : .. .
- .
..: -

I
. .

.- .
1 .7 Grapt' i~' [) otormination of 'K' for Spocifiod [) amping R atio's' 1 - 20
1 .8 <:;onora| Stops to So|vo tho Prob|om on R oot |ocus 1 - 21
1 .1 Effoct of Addition of OJ) onLooJ) Po|os and Zoros 1 - I 8
1 1 ' Additionof Po|o 1 - I 8
!. . I' .t. t-
9. 10 Advantagos of R oot |ocus Mothod 1 - 31
9. 11 Obtaining G(s) H(s) from Ct|aractoristic Equation 1 - 31
9. 12 Canco||ation of Po|os of <:;(s)vith Zoros of H(s) 1 - 3I
1 .' I .' Gain Margin 1 - 33
9. 13 R oot Sonsitivity 1 - 33
9. 1~ R oot Contour 1 - ~
9. 15 Invorso R oot Locus "" " 1 -~
9. 1E5Systom vith Positivo Foodback (K is Positivo) 1 - 37
ExamJ) |osvith So|utions 1 - 40
R oviovQuostions 1 - 100
Chaptor ~:;' 1 : 1 a! ica: o" FtoguoriCY::i) omain Ana|ysis ' .'.-.' -;.' (' 1 ~' ) to ( ' 1 '_ I 8)
10. 1 Background 10 - 1
10. 2 Advantagos of Froquoncy DOrT|ainApproach 10 - 1
10. 3 Limitations of Froquoncy R osponso Mott|ods 10 - I
10. 4 ConcoJ) tua| Approact| to Froquoncy R osponso 10 - 3
' 1 .1.' Stoady Stato R osponso to Sinusoida||npu| : (Froquoncy R osponso) ' 1 - 3
10. 5 Apparatus R oquirod for Froquoncy R osponso 10 - 5
#~ ~ -. -_ . tt -: . 0- " . . . . - '- a, #, . ~ _ --# -: -, -_ , " ~ _ - . . ->
I n|
10.6 Relation between Transfer Function and Frequency Response 10 - 6
10.7 Transfer Function and Frequency Response of a
Series R-C Circuit.......................... .. .. 10- 6
10.7.1 Generating Frequency Response ata 10 - 7
10.! Frequency o"ain #et$ods 10 - 7
10.% Co-relation between Ti"e o"ain and Frequency o"ain for
Second &rder Syste" 10 - !
10.%.1 eri'ationsof#r andro( 10-8
10.9.2 Co""ents on Co-relation between Ti"e o"ain and Frequency o"ain 10- 9
10.10 ).*. +)andwidt$, 10 -11
-.a"ples wit$ Solutions 10 - 1/
Re'iew 0uestions 10 - 17
~~-.~er :.. ~1.1 -; BodlfPlots -. ,-:~'--: ----~ ~:~~':~'--.-: ~-,- _--. : -:-, :' -: ~-~- -..' --:'/_~"'~ - (11".; .1lto '(11:90)
2 3. 43.3 53.-.3
11.1.1#agnitude 6lot. 11-1
11.1.2 T$e 6$ase 7ngle 6lot. . . . . . . . . . . . . . . . . . . . . . . . . . . . . . . . . . . . . . . . . . . . . . . . .. 11-1
11.1 8ogarit$"ic Scales +Se"ilog 6apers, 11 - 1
11./ Standard For" of &pen 8oop T.F. G(jro)(jro) 11 - /
11.9 )ode 6lots of Standard Factors of G(jro)(jro) 11 - 9
11.9.1 Factor 1 : Syste" Gain 2:4. ;. . . . . . . . . . ;; . . ; . . . ; . . . . . . . . . . . . . . . . . . . . . . . . .. 11- !
11.9.1 Factor 1 56oles or < eros "t t$e &rigin &ro,=p 11- 6
11.#.$ Factor $ : Si"ple 6oles or < eros +First %rder Factors, . . . . . . . . . . . . . . . . . . . . . . .. 11- 1$
11.#.# &"'tor # : 0uadratic &"'tors. . . . . . . . . . . . . . . . . . . . . . . . . . . . . . . . . . . . . . . . . . .. 11-10
11.> Steps to S?etc$ t$e )ode 6lot 11 - 1>
11.6 Frequency Response Specifications 11 - 1>
11.7 Calculation of G.#. and 6.#. fro" )ode 6lo8........... 11 - 17
11.! *$at s$ould be @alues of G.#. and 6.#. of a Good Syste"A 11 - 1%
11.% Bow to 2"pro'e t$e G.#. and 6.#. ( 11 - /0
11.10 eter"ination of ro)' and 6.#. for Standard Second &rder
Syste" 3 3.... 11 - /0
11.11 Calculation of Transfer Function fro" #agnitude 6lot 11 - />
11.11 7d'antages of )ode 6lots 11 - />
11.1/ eter"ination of :p3 : ' and : a fro" )ode 6lot.......................... 11 - /7
J' _.o'" - .. ..*..,.~~:f+" ... 4CD4E. DF.4 GHI ~ : I J. ~ ~ . . . . . . . It): ..+,~ ~ o f ~ .~ .( - . ;N:~ );'- 1 .... ~ " '= .r ~ " (- - ..::. ) 40- o0CID4J4C4 .D ~/_~ ..'.:' . --E H
~'- ~ 0'."~ ~ J . "'. ,. : , : ' ~ ,: " ~ : "(: ~ '. : . . . 1+++ "'.~'.
2 nl
11.14 Bode Plot of Systems with Transportation Lag 11 - 39
Examples with Solutions 11 - 41
Review uestions .... ...! " ! 11 - ##
$hapter -1% -- "Polar"and &y'uist PlotS" "--
12.1Ba()ground 12- 1
12_2 Polar Plot 12- 1
12.3rogtand ropein Polar Plot 12- #
12.4*etermination of +.,. and P.,. from Polar Plot.. 12- -
1%.4.1 *eterminingo.p( ,athemati(ally 1% - 1/
12.5&y'uist Plot 0nalysis 11 12- 12
12.6Pole-2ero $onfiguration "3 12- 13
12.7En(ir(lement 12- 14
1%.4.1 $ounting &um5er of En(ir(lements ! ! . ! . . . . . . .. 1% - 14
12.80nalyti( 6un(tion and Singularities 1 12- 16
12.9,apping Theorem or Prin(iple of 0rgument.................................. 12- 16
12.10&y'uist Sta5ility $riterion ! 12- 19
12.11+enerali7ed &y'uist Path and its ,apping 12- 21
12.12Steps to Solve Pro5lems 5y &y'uist $riterion............................. 12- 23
12.13 Behaviour of Right 8alf Pole 12- 35
12.140dvantages of &y'uist Plot.......................................................... 12- 40
12.15Log ,agnitude - Phase Plots....................................................... 12- 41
1%.19.1 Sta5ility 0nalysis using ,agnitude-Phase Plot. : . . . . . . . . . . . . . . . . . .. 1% - 4%
Examples with Solutions.......... 12- 45
Review uestions 12- 85
$hapter ;-1< $losed Loop 6='uen(y Response3". >1< =1? t o ( 1< =%/?
13.1$losed Loop 6re'uen(y Response 13- 1
13.2, $ir(les @$onstant ,agnitude Lo(iA 1 13- 2
13.3 & $ir(les @$onstant Phase Lo(iA 13- 9
1<.4 Bse of , $ir(les 13- 4
1<.9 Bse of & $ir(les 1< - -
1<.C &i(holls $hart 1< - 1/
( xiv) ,.."t
DDE nl
13.7 Frequency Specifications from the Nichol's Chart 13 - 12
Review uestions................................................................................. 13 - 2!
Chapter" 1# . Compe-nsation $ of Control Systems% ~.! .. , .... ~ ~ ::' &1# -1' to & 1#7 .48).
1#.1 (ac)*roun+ 1# - 1
1#.2 ,ypes of Compensation 1# - 1
1#.2.1 Series Compensation 1# - 2
1#.2.2 -arallel Compensation 1# - 2
1#.2.3 Series--arallel Compensation 1# - 2
1#.3 Compensatin* Networ)s 1# - 3
1#.# .ea+ Compensator 1# - 3
1#.#.1 /a0imum .ea+ 1n*le m an+ !" 1# - #
1#.#.2 -olar -lot of .ea+ Compensator 1# - 2
1#.#.3 (o+e -lot of .ea+ Compensator 1# - 7
1#.#.# Steps to 3esi*n .ea+ Compensator 1# - 4
1#.#.5 6ffects of .ea+ Compensation 1# - 7
1#.#.2 .imitations of .ea+ Compensation 1# - 1!
1#.5 .a* Compensator 1# - 1#
1#.5.1 /a0imum .a* 1n*le an+ P 1# -15
1#.5.2 -olar -lot of .a* Compensator 1# -12
1#.5.3 (o+e -lot of .a* Compensator 1# - 17
1#.5.# Steps to 3esi*n la* Compensator 1# -14
1#.5.5 6ffects an+ .imitations of .a* Compensator 1# - 17
1#.2 .a*-.ea+ Compensator 1# - 2#
1#.2.1 -olar -lot of .a*-.ea+ Compensator 1# - 22
1#.2.2 (o+e -lot of .a*-.ea+ Compensator 1# - 27
1#.2.3 6ffects of .a*-.ea+ Compensator 1# - 27
1#.7 Compensation usin* Root .ocus 1# - 3#
1#.4 3esi*nin* .ea+ Compensator usin* Root .ocus 1# - 35
1#.7 3esi*nin* .a* Compensator usin* Root .ocus 14 -#!
1#.1! 3esi*nin* .a*-.ea+ Compensator usi%* Root .ocus 1# - ##
Review uestions " 1# - #2
Chapter..15 .."St4te$ 8ari%%le91nalysi:.;-9 .0 < ( . .. ; . :' h ! ; h ~ ': d".",-~., ( 15.1) to & 15."#)
15.1 (ac)*roun+ 15 - 1
15.1.1 1+vanta*es of State 8aria<le 1nalysis. . . . . . . . . . . . . . . . . . . . . . . . . . . . . . . . . . . .. 15 - 2
15.2 Concept of State 15 - 2
~-:.,!~.$~$ .. t% ... % $.$~.~::~~$~$ ..~~ ...=!>>%.".%" .. - " >>> % < .... -' ~% ~.% ~.% : .. ; . ( &' ..... :~~~-; .(~ .~~: .. .. .....%%%.%?" '9 ." t)..~ ....... :.:. : .. :..... . e
== nl
15.2.1 Important Definitions ............................................... " 15 - 4
15.3 State Model of Linear Systems 15 - 5
15.3.1 State Model of Single Input Single Output System ........................ , 15 -
15.4 State !aria"le #epresentation using $%ysi&al !aria"les 15 -
15.4.1 'd(antages ............................. " ..................... 15 -11
15.5 State Diagram #epresentation 15 - 11
15.5.1 State Diagram of Standard State Model ................................. 15 -12
15.) *on +ni,ueness of State Model 15 - 13
15.- State Spa&e #epresentation using $%ase !aria"les 15 - 14
15.-.1 State Model from Differential .,uation. . . . . . . . . . . . . . . . . . . . . . . . . . . . . . . . . .. 15 -14
15.-.2 State Model from /ransfer 0un&tion ..................................... 15 -11
15.-.3 'd(antages. . . . . . . . . . . . . . . . . . . . . . . . . . . . . . . . . . . . . . . . . . . . . . . . . . .. 15 - 21
15.-.4 limitations 15 - 21
15.1 State Spa&e #epresentation using 2anoni&al !aria"les 15 - 2
15.1.1 3ordan4s 2anoni&al 0onn . . . . . . . . . . . . . . . . . . . . . . . . . . . . . . . . . . . . . . . . .. 15 - 32
15.1.2 'd(antages of 2anoni&al !aria"les ..................................... 15 - 3)
15.1.3 Disad(antages of 2anoni&al !aria"les ................................... 15 - 3)
15. State Model "y 2as&ade $rogramming 15 - 31
15.15 Deri(ation of /ransfer 0un&tion from State Model 15 - 45
15.15.1 2%ara&teristi& .,uation ........................ . . . . . . . . . . . . . . . . . . .. 15 - 41
15.15.2 MIM6 System ..................................................... 15 -42
15.11 Solution of State .,uations 15-43
15.11.1 7omogeneous .,uation 15 - 44
15.11.2 *on%omogeneous .,uation .......................................... 15 - 44
15.12 #e(ie8 of 2lassi&al Met%od of Solution 15 - 44
15.12.1 9ero Input #esponse ........................................ 15 - 4)
15.13 Solution of *on%omogeneous .,uation 15 - 4)
15.14 $roperties of State /ransition Matri: 15-4-
15.15 Solution of State .,uation "y lapla&e /ransform Met%od 15 - 41
15.1) 2omputation of State /ransition Matri: 15 - 4
I .
15.1- lapla&e /ransform Met%od 15 - 55
.:amples 8it% Solutions 15 - 51
#e(ie8 ;uestions 15 - 5
l I nl
Chapter 16 Control Components and Controllers .~ (16 -1) to ( 16 64)
16.1 Introduction to Stepper Motors 16- 1
16.2 Permanent Magnet Stepper Motor 16- 1
16.3 Variable Reluctance Stepper Motor 16- 3
16.3.1 Reduction Gear Stepper Motor . . .. . . . .. . . . . . .. .. . . .. 16 - 4
16.3.2 Multistack Stepper Motor 16 - 4
16.4 Important e!initions Related to Stepper Motor 16- "
16.5 Stepper Motor #$aracteristics 16- 6
16.".1 Static #$aracteristics. .. .. . . . . .. .. 16 - 6
16.".2 %namic #$aracteristics 16 - 7
16.6 i!!erence bet&een Stepper Motor and .#. Ser'omotor 16- (
16.7 )pplications o! Stepper Motor 16 - *
16.8 S%nc$ros 16- *
16.*.1 S%nc$ro +ransmitter 16 - *
16.*.2 S%nc$ro #ontrol +rans!ormer 16 - ,
16.*.3 -rror etector using S%nc$ros ... / 16 - 9
16.9 Potentiometer 16 - 11
16.,.1 Potentiometer as an -rror etector / 16 -13
16.,.2 +%pes o! Potentiometers 16 - 13
16.,.3 #$aracteristics o! Precision Potentiometer 16 -14
16.,.4 0oading in Potentiometers 16 -1"
16.10 Rotating )mpli!iers 1 16 - 16
16.12.1 Single Stage )mpli!ier 16 -16
16.12.2 +&o State Rotating )mpli!ier 3)mplid%ne4 16 -16
16.12.3 +rans!er 5unction 16 -1(
16.11 Magnetic )mpli!ier 16- 18
16.12 Ser'oampli!iers 16- 21
- 16- 24
16.14 Properties o! #ontroller 16 - 25
16 141-rror . . . 16 - 2"
16.14.2 Variable Range 16 - 26
16.14.3 #ontroller 6utput Range 16 - 26
16.14.4 #ontrol 0ag 16 - 2(
16145 ead 7one 16 - 2(
16.1" #lassi!ication o! #ontrollers 16- 27
... }~,. ... 8.
I nl
16.16 Discontinuous Controller Modes 16 - 28
16.16.1 Two Position Mode (ON-OFF Controller) r 16 - 28
16.16.2 Multiposition Mode 16 - 29

.

- u- .-
16.18 Proportionl Control Mode 16 - !1
16.18.1 C"rcteristic o# Proportionl Mode 16 - !2
16.18.2 O##set. . . . . . . . . . . . . . . . . . . . . . . . . . . . . . . . . . . .. .. . 16- !!
16.18.! $pplictions 16 - !!
16.19 %nte&rl Control Mode 16 - !'
16.19.1 (tep )esponse o# %nte&rl Mode 16 - !*
16.19.2 C"rcteristics o# %nte&rl Mode 16 - !*
16.19.! $pplictions 16 - !6
16.2+ Deri,ti,e Control Mode - - ..16 - !6
% %../// I i .0. % I 11.1I
.-
16.2+.2 $pplictions 16 - !8
16.21 Co1posite Control Modes - 16 - !8
16.22 Proportionl 2 %nte&rl Mode (P% Control Mode) 16 - !8
16.22.1 C"rcteristics o# P% Mode - 16 - '+
16.22.2 $pplictions 16 - '+
16.2! Proportionl 2 Deri,ti,e Mode (PO Control Mode) 16 - '+
16.2!.1 C"rcteristics o# PO Mode.. - ..0- 16 - '1
16.2!.2 $pplictions 16 - '2
16.24 T"ree Mode Controller (P%D Control Mode) 16 - 42
16.2* )esponse o# Controllers to 3rious %nputs 16 - '4
16.26 5##ect o# Co1posite Controllers on 2nd Order (6ste1 16 - '4
16.24 PD T6pe o# Controller 16 - '8
16.28 P% T6pe o# Controller 16 - *+
16.29 P%O T6pe o# Controller 16 - *1
16.!+ )te Feed7c8 Controller (Output Deri,ti,e Controller) 16 - *1
591ples wit" (olutions 16 - *!
)e,iew :uestions 16 - 62
.- .;
($ -1) to ($./8)
l % nl
Basics of Control System
1.1 Background
Inrecent years, concept of automatic control has achieved a very important position in
advancement of modem science. Automatic control systems have played an important role
in the advancement and improvement of engineering skills.
Practically, every activity in our day to day life is influenced by some sort of control
system. Concept of control systems also plays an important role inthe working of space
vehicles, satellites, guided missiles etc. Such control systems are now integral part of the
modem industrialization, industrial processes and home appliances. Control systems are
found in number of practical applications like computerised control systems, transportation
systems, power systems, temperature limiting systems, robotics etc.
Hence for an engineer it is absolutely necessary to get familiar with the analysis and
designing methods of such control systems.
This chapter includes the concept of system and control system. Then it gives the
classification of control systems. It includes the discussion of various types of control
systems supported with number of real time applications.
1.2 Definitions
To understand the meaning of the word control system, first we will define the word
system and then we will try to define the word control system.
System A system is a combination or an arrangement of different physical components which
act together as an entire unit to achieve certa!n objective.
!very physical ob"ect is actually a system. A classroom is a good e#ample of physical
system. A room along with the combination of benches, blackboard, fans, lighting
arrangement etc. can be called a classroom which acts as an elementary system.
Another e#ample of a system is a lamp. A lamp made up of glass, filament is a
physical system. Similarly a kite made up of paper and sticks is an e#ample of a physical
system.
Similarly system can be of any type i.e, physical, ecological, biological etc.
(1 - 1)
I I nl
Control System Engineering 1 -10 Basics of Control System
Insuch system, output or part of the output is fedback to the input for comparison
with the reference input applied to it.
Closed loop system can be represented as shown in the Fig. 1.12.
Error detector
r(t)
~ Forward path
met) Process to be
controlled
Reference .: ! : .
inpt
bet)
c!t) Controlled
otpt
Command
inpt
Reference
transdcer """"""-t--.c
Feedbac#
$%%%%%%%%1 element t---...!
Feedbac# signal
bet) &'(''I Feedbac# path
Fig" 1"1) Representation of closed loop control system
"he #arious signal s are,
ret$ % &eference input e't$ % (rror signal
c)t$ % Controlled output m)t$: : * +anipulated signal bet$ % Feedback signal
It is not possible in all the systems that a#ailable signal can be applied as input to the
system. ,epending upon nature of controller and plant it is re-uired to reduce it or
amplify it or to change its nature i.e. making it discrete from continuous type of signal etc.
"his changed input as per re-uirement is called reference input which is to be generated
by using reference transducer. "he main e.citation to the system is called its command
input which is then applied to the reference transducer to generate reference input.
/ractically many electronic integrated circuits work on the d.c. #oltage range of
* to 1+ ," "he supply a#ailable is )-+, a.c, 0ence the reference input #oltage in the range
of * to 1+ , d.c. is obtained from the command input )-+, a.c. and proper rectifying nit"
"he part of output, which is to be decided by feedback element is fed back to the
reference input. "he signal which is output of feedback element is called 1feedback signal1
bet$.
2t is then compared with the reference input gi#ing error signal e't$ % ret$ .bet$
3hen feedback sign is positi#e, systems are caned positi#e feedback systems and if it
is negati#e systems are called negati#e feedback systems.
"his error signa4 is then modified by controller and decides the proportional
manipulated signal for the process to be controlled.
"his manipulation is such that error will approach 5ero. "his signal then actuates the
actual system and produces an output. 6s output is controlled one, hence called controlled
output c)t$,
I I nl
Control System Engineering 1 -11 Basics of Control System
1.5.1Advantages
The advantages of closed loop system are,
1) Accuracy of such system is always very high because controller modifies and
manipulates the actuating signal such that error in the system will be zero.
2) Such system senses environmental changes, as well as internal disturbances and
accordingly modifies the error.
3) In such system, there is reduced effect of nonlinearities and distortions.
) !andwidth of such system i.e. operating fre"uency zone for# such system is very
high.
1.5.2Disadvantages
The disadvantages of closed loop system are,
1) Such systems are complicated and time consuming from design point of view and
hence costlier.
2) $ue to feedbac%, system tries to correct the error from time to time. Tendency to
overcorrect the error may cause oscillations without bound in the system. &ence
system has to be designed ta%ing into consideration problems of instability due to
feedbac%. The stability problems are severe and must be ta%en care of while
designing the system.
1.5.3Real Time Alications of Closed !oo System
1.5.3.1 "#man Being
The best e'ample is human being. If a person wants to reach for a boo% on the table,
closed loop system can be represented as in the (ig. 1.13.
)osition of the boo% is given as the reference. (eedbac% signal from eyes, compares the
actual position of hands with reference position. *rror signal is given to brain. !rain
manipulates this error and gives signal to the hands. This process continues till the
position of the hands get achieved appropriately.
In#t
e
Brain
Desired
"ands t+++t, .. osition
of t$e $ands
Reference -%--I
osition
of &oo'
Eyes
(ig. 1.13 "#man &eing
1.5.3.2 "ome "eating System
In this system, the heating system is operated by a valve. The actual temperature is
sensed &y a thermal sensor and compared with the desired temperature. The difference
between the two, actuates the valve mechanism to change the temperature as per the
re"uirement.
l I nl
Control System Engineering 1 -12 Basics of Control System
Desired
temperature
Valve
Heaijng House
t------1 system t---...... - temperature
'--__----!emperaturet-------'
sensor
"ig# 1#1$ Domestic %eating system
1#&#'#'S%ip Sta(ili)ation System
In this system a roll sensor is used as a feedback element. The desired roll position is
S%ipsta(ilisationsystem
\
\
*
S%lp-
\
\
\
+oll position
"ig# 1#1& S%ip sta(ili)ation system
selected as 6r while actual roll position is Oc which is compared with Or to generate
controlling signal.This activates fin actuator in proper way to stabilize the ship.
Desired roll
9r
"in
actuator
,ctual controlled roll
t-----.--!9c S%ip
+oll
sensor
"ig# 1#1-
1#&#'#$.anual Speed Control System
" locomotive operator driving a train is a good e#ample of a manual speed control
system. !%e ob$ective is to maintain the speed e%ual to the speed limits set. The entire
system is shown in the block diagram in the &ig.'.'(.
nl
Control System Engineering 1 -13 Basics of Control System
Speed
limit
Brain Hands
Actual
._. . . . . -
Speed
Control 1----4Actuator t---1 Vehicle
action mechanism
Sensor
1..3. !.C. "otor Speed Control
#otentiometer
Eyes
$ig. 1.1%
Shaft
&achometer
'(cou#ling
&achometer
$or
speed
feed)ac*
$ield current
constant
$ig. 1.1+ Speed control system
The D. C. shunt motor is used where field current is kept constant and armature
voltage is changed to obtain the desired speed. The feedback is taken by speed tachometer.
This generates voltage proportional to speed which is compared with voltage required to
the desired speed. This difference is used to change the input to controller which
cumulatively changes the speed of the motor as required.
Error
,----i Amplifier t----i Armature t----t -oad
Actual
speed
.__---------. &ac iorneter 1---------1
$ig. 1.1/ Speed control system
nl
Control System Engineering 1 -14 Basics of Control System
1.5.3.6 Temperature Control System
The aim is to maintain hot water temperature constant. Water is coming with constant
flow rate. Steam is coming from a valve. Pressure thermometer 'P' is used as a feedback
element which sends a signal for comparison with the set point. This error actuates the
valve which controls the rate of flow of steam, eventuall controlling the temperature of
the water.
I. water intet
Set
point --_.. Comparator t-f---.
I
- - - - - - - - - - - - -
Fig. 1.! Temperature control system
"esire#
temperature
$al%e
Steam
flow
&ctual
1----1 'rocess t--!e!-
temperature
'ressure
t(ermometer
"ig. 1.#1
1.5.3.) *issile +aunc(ing System
This is sophisticated e$ample of militar applications of feedback control. The enem
plane is sighted b a radar which continuousl tracks the path of the aeroplane. The
launch computer calculates the firing angle intenns of launch command, which when
amplified drives the launcher. The launcher angular position is the feedback to the launch
computer and t(e missile is triggered when error between the command signal arid missile
firing angle becomes %ero. The sstem is shown in the "ig. 1.##.
1.5.3., $oltage Sta-ili.er
Suppl voltage re&uired for various single phase appliances must be constant and high
- fluctuations are generall not permitted. 'oltage stabili%er is a device which accepts
variable voltage and outputs a fi$ed voltage.
1I / nl
------- - - - - - - - - - - - - - - - - - - - - - - - - _ _ ---------
Control System Engineering 1-15 Basics of Control System
Flighlpalh
. - - - - 7 < - - - - - - - - - - - - - -
0'\
Q)L ..
't ...-: -,
~L \
~~\ Lhe ad angle ,,'
~,\ I
'iii \
-0 \ , Actual
01 \ .._ _ _
~ I poSIt ion
\ I
\ I
Trackmg
control
l a u n c h
computer
l a u n c h
command
Feedback
Poer
am plif ie r
Fig! "!## $issile launc%ing system
,
r in!iple of "#!h "t a$ili%e r i" $a"e d on !ont r olling n#m $e r of "e !ond ar & t #r n" a" pe r
r e '#ir e m e nt t o in!r e a"e or d e !r e a"e t he o#t p#t &olt age . (he a!t #al o#t p#t )olt age lS sensed
by a t r an"f or m e r and pot e nt ial d i)id e r ar r ange m e nt . (he r e f e r e n!e )olt age is "e le !t e d
pr opor t ional t o t he d e "ir e d o#t p#t le )e l. (he a!t #al o#t p#t i" !om par e d *it h t hi" t o
ge ne r at e e r r or *hi!h in tum is inp#t t e d t o t%e !ont r olle r . (he !ont r olle r t a+e " t he pr ope r
d e !i"ion t o in!r e a"e or d e !r e a"e t%e n#m $e r of t #r n" "o a" t o ad ,#"t t he o#t p#t )olt age .
T%e "!he m e i" "ho*n in t he Fig. 1.-..
/
Controlled signal
Controller
and
relays
&solationtransformer
Fig. 1.-. 0olt age "t a$ili%e r
& & nl
Control System Engineering 1-16 Basics of Control System
The other examples of closed loop system are machine tool position control,
positioning of radio and optical telescopes, auto pilots for aircrafts, inertial guidance
system, automatic electric iron, railway reservation status display, sunseeker solar system,
water level controllers, temperature control system. So in closed loop feedback control
systems cause and effect relationship between input and output exists.
1.6Comparison of Open Loop and Closed Loop Control System
Sr. Open loop Closed oop
!o.
1. "ny change in output has no effect on the #hanges in output, affects the input which is
input $.e. feedback does not exists. possible by use of feedback.
2 . Output measurement is not re%uired for Output measurement is necessary.
operation of system.
3 . &eedback element is absent. &eedback element is present.
4 . 'rror detector is absent. . 'rror detector is necessary .
(. It is inaccurate and unreliable. )ighlyaccurate and reliable.
6.
)ighlysensitive to the disturbances. ess sensitive to the disturbances.
7 . )ighlysensitive to the environmental changes. ess sensitive to the environmental changes.
*. +andwidth is small. +andwidth is large.
9. Simple to construct and cheap. #omplicated to design and hence costly.
1,. -enerally are stable in nature. Stability is the ma.or consideration while
designing
11.
)ighlyaffected by nonlinearities. /educed effect of nonlinearities.
1.7Servomechanisms
0efinition 1 It is a feedback control system in which the controlled variable or the output is a
mechanical position or its time derivatives such as velocity or acceleration.
A simple example of servomechanism is a position control system. #onsider a load
which re%uires a constant position in its application. The position is sensed and converted
to voltage using feedback potentiometer. It is compared with input potentiometer voltage
to generate error signal. This is amplified and given to the controller. The controller in
turn controls the voltage given to motor, due to which it changes its position.
l I
t
Control SystemEngineering 1-17 ~ Basics of Control System
The scheme is shown in the Fig. 1.24.
Amp.
Gears
Generator Motor
+
potentrorneter
Error
voltage
Fig. 1.24Position control system
Fewother examples of servomechanisms are,
1) Power steering apparatus for an automobile.
2) achine tool position control.
) issile launchers.
4) !oll stabili"ation of ships.
1.! "eg#lating Systems $"eg#lators)
#efinition $ 1t is a feedback control system iff which for a preset oalue of tire reference input,
the output is kept constant at its desiredvalue.
%nsuch s%stems reference input remains constant for long perio&s. ost of the times
the reference input or the &esire& output is either constant or slowl% var%ing with time. 'n
. a regulator, the &esire& value of the controlle& outputs is more or less fixe&. (imilarl% the
reference input is also fixe& an& calle& set point. Thus the regulator maintains a constant
output for a fixe& reference input. The problems &ue to &isturbances arc mainl% rectifie&
b% the regulator. ) simple example of such regulator s%stem is servostabili"er. *e have
+ seen earlier that in voltage stabili"er position of tap on secon&ar% is a&,uste& b% using
rela% controls. -ut instea& of fixe& tap, the entire secon&ar% can be smoothl% tappe& using
%% % nl
Control System Engineering 118 Basics of Conb'Ol System
a servomotor drive. The servomotor drives the shaft and controls the position of tap on
secondary as per the controller signal. Due to the fluctuations in the main input if the load
voltage changes, such effects are rectified by the regulator to keep load voltage constant.
The actual scheme is shon in the !ig. 1."#, hile its block diagram representation is
shon in the !ig. 1."$.
Feedback
Reference
voltage
Isolation
transformer
II
Controlled
Servo
sinnal
+-__",;;__:I_---.
motor
To load
%&&....... &&&&&O stabilised voltage
Reference
input
Fig. 1.! Regulating system
Error """"
1&&&&1Controller t"""I
Servo
motor
Secondary ..... #.. $ctual
tap load voltage
%..." "I %oad voltage t&&&&&'
sensing
Fig. 1."$
.nl
Control SystemEngineering
, .,
!' J Basics of Control System
Few other examples of regulating system arc,
1) Temperature regulators.
2) Frequency controllers.
3) Speed governors.
1.9 Feedback and Feed Forard System
In the control systems considered up till now, it is considered that the disturance has
affected the output adversely. Such! an output is measured and compared with the
reference input to generate an error. This error is fed to the controller which is
successively operating the system to correct the output.
Thus such systems in which the effect of the disturance must show up in the error
efore the controller can ta"e proper corrective action arc called feedac" systems.
!f t"e disturance is measurale, then the signal can e added to the controller input
to modify the actuating signal. Thus, a corrective action is initiated without waiting for the
effect of the disturance to show up in the output i.e, cumulatively in the error. Thus the
undesirale effects of measurale disturances y approximately compensating for them
efore they affect the output. This is much more advantageous as in normal feedac"
system the corrective action starts only after the output has een affected. .
#ey $oint% Such systems in whiclz such corrective action is taken before distutbances affect
the output are called feed forward system,
# loc" diagram with feed forward concept is shown in the Fig. 1.2$.
r&&&&&&&&&&&i Feed forard
block
'ist(rbance
Controller )ct(ator $rocess
*(t+(t
Fig. 1.2$
The two difficulties associated with feed forward system are,
i) In some systems, the distura%c% may not e measurale.
ii) ,"e feed forward compensation is an open loop technique and if actuator transfer
function is not "nown accurately, then such compensation cannot e achieved.
r
Control System Engineering 1-2 Basics of Control System
Control system: To control means to regulate, to direct or to command. Hence a control
system is an arrangement of different physical elements connected in such a manner so as to
regulate, direct or command itself or some other system.
For example, if in a classroom, professor is delivering his lecture, the combination
becomes a control system as; he tries to regulate, direct or command the students in order
to achieve the objective which is to impart good knowledge to the students. Similarly if
lamp is switched ! or FF using a switch, the entire system can be called a control
system. "he concept of physical system and a control system is shown in the Fig. #.# and
Fig. #.$.
Lamp %&'''''Supply '''''...%
Fig. 1.2 Control system Fig. 1.1 Physical system
(hen a child plays with the kite, he tries to control it with the help of string and
entire system can be considered as a control system.
)n short, a control system is in the broadest sense, an interconnectionof the physical
components to provide a desired function, involving some kind of controlling action in it.
*lant : Tire portion of a system whic11is to be controlled or regulated is called the plant or
the Process.
Controller: The element of the system itself or external to tire system uhid: controls the plant
or the process is called controller.
For each system, there must be an excitation and system accepts ~i: as an input. +nd
.for analy,ing the behaviour of system for such input, it is necessary to define the output
of a system.
)nput : It is an applied signal or an excitation signal applied to a control system from an
external energy source in order to produce a specified output.
utput : It is the particular signal of interest or the actual response obtainedfrom a control
system wizen input is applied to it.
-isturbances : -isturbance is a signal which tends to adversely affect the value of the
output of a system. If such a disturbance is generated within the system itself, it is called
an internal disturbance. "he disturbance generated outside the system acting as an extra
input to the system in addition to its normal input. affecting the output adversely is called
an external disturbance.
Control systems may have more than one input or output. From the information
regarding the system, it is possible to wen define all the inputs and outputs of the
svstems,
r
Control System Engineering 1-20 Basics of Control System
1.9.1 Real Time Application of Feed Forward System
Ina particular process control industry, it is necessary to maintain the temperature of
a molten metal constant before giving it to the next process. For this a general temperature
control feedback system is used as shown in the Fig. 1.28. -
Storage
I tank
- -
Feedack
!---"--"i-- Constant
/rrr,rrrrn#$..r--"""-%!--"- temperature
re!"ired
#eat
exchanger
Fig. 1.$% Feedack system for temperat"re control
&ractically if the rate of flow of metal from tank to heat exchanger gets disturbed, due
to change in level then the temperature gets affected. 'ut it takes a long time to see its
effect at the output to take the corrective action.
&ey &oint" In feedback system, the correctiveaction can not be taken unless and untill output
gets disturbed.
(ue to the time lag, it is not possible to keep the output temperature constant within
limits.
Insuch a case, feed forward system is used. )he inlet flow rate is measured with the
flowmeter. *mmediately when there is a change in rate of flow, it is indicated to the
controller through flowmeter before it is going to disturb the output. )he controller takes
the corrective action in advance by ad+usting the steam flow. )hus the output temperature
gets maintained constant within the limits. "
- .
&ey &oint" The feed forward compensation, compensates the effect of disturbance before it
actually disturbs the output.
)he system is shown in the Fig. 1.2, -a. while its block diagram representation is
shown in the Fig. 1.2,-b..
)he feed forward minimi/es the transient error due to measurable disturbances. 0hile
feedback compensates for unmeasurable disturbances and other effects. )hus it is advisable
'
to incorporate both feedback and feed forward schemes ina system.
- I
nl
Control System Engineering
I! Basics of Control System
Controller t-----,
Feedback
signal
...- ..... ., Temperature
'-...--' sensor
Feed forard
signal
Storage
tank I
! .. "I
Inlet
#'---r-#### Constant
temperature
$a% Feed forard system
Flo
&isturbance
meter
$c'ange In
flo rare%
( .
.
IFeed forard
)eference -
input
-* -- Controller f--
+alue
-
,eat
e-c'anger
Temperature
sensor
Constant
temperature
( %Feedback
$b% Block diagram
Fig. ../0 1ractical e-ample of feed forard system
...23ulti4ariable Control Systems
The control system in which there is only one output of the interest is called single
variable system. Butin many practical applications more than one variables are mvolved.
A control system with multiple inputs and multiple outputs in called a multivariable
system.
The block diagram representation of a multivariable control system is shown in the
Fig. l.30. The part of the system which is reuired to be controlled is called plant. The
controller provides proper controlling action depending on the reference inputs. There arc
n reference inputs rl, r!, rn-
There are n output variables "l#t$,"!#t$,,%%, ..... cn#t&. The values of these variables
represent the performance of the plant. The control signals produced by the controller are
applied to the plant. 'ith the help of feedback elements the closed loop control of the
plant is also possible. (ue to the feedback,the controller takes into account the actual
output values to decide the control signals.
5' nl
Control System Engineering Basics of Control System
Plant
Controller
or
(Analog or digital)
.
System to be
controlled
...
-
Feedback
:
elements
:
Control
signals
Fig. 1.30 Block diagram of mlti!ariable control system
"n case of multivariable systems, sometimes it is observed that a single input
considerably affects more than one outputs. The system is said to be having strong
interactions or coupling. nus coupling is nothing but the disturbances for the separate
systems. The interactions inherently present between inputs and outputs can be cancelled
by designing a decoupling controller. Thus the resulting multivariable system is
considered to have proper number of single input single output systems and the controller
is designed for each system. The another way is to design a controller which will take care
of all t#e inherent interactions present in the multivariable system.
In multivariable linear control system, each input is independently considered. Only
one input and one output is considered and t#e total effect on any output because of all
t#e inputs acting simultaneously is determined by addition of the outputs due to each
input acting alone. Thus law of superposition is used to analyse multivariable linear
control systems.
In many practical control systems, control is achieved by more than one input and the
system may have many outputs. "n chemical processes simultaneous control of pressure,
temperature and concentration is required by commanding various inputs. Air crafts and
space crafts are other examples where movement is controlled by various inputs. ower
generators, atomic reactors and !et engines are some of other examples of multivariable
systems.
"onsider the block diagram of multivariable autopilot system shown in the #ig. $.%$.
The system shown in the ig. $.%$ keeps a track of rocket vehicle in response to
reference inputs given to it.The pcsition, velocity and acceleration of the vehicle are fed to
the digital controller using motion st.&rs. The controller takes appropriate decision and
sends a controlling signal which will drive the actuator, which will move the engine. Thus
there are three output variables which art. to be observed and controlled and there are
corresponding reference inputs hence the system is multivariable system.
$ "
Control System Engineering Basics of Control System 1-23
Reference
Inputs
Control
signal
I
Angular
position
I
L
Digital
Digital
r---
to
-
Actuator
-~
Rocket f-----eoo Vehicle ~~
~
controller analog
converter
Position
feedback
Analog
Velocity
to
feedback
digital
converter
Acceleration
feedback
Fig. 1.31 Multivariable autopilot system
Review Questions
I, Dtftne thefolluwmg terms
(i) System (ii) Control system (iii) Input (iv) Output (v) Disturbance.
2. Eplain !lO"tile control systems are classifie#
$. Define linear an# nonlinear control systems.
%. "hat is lime variant system& 'ive suitable eample. (ow i f is #ifferent than time invariant
system&
). Dtftne open IDOp an# close#loopsystem by giving suitable eamples.
*. Differentiatebetween open loop an# close#loopsystems giving suitable eamples.
+. "ith referencetofee#bac,control system #efine tirefollowing terms
i) Comman# input (ii) -eferenceinput (iii) .orwar#pat!l (iv) .ee#bac,patll
/. Eplain thefollowi0lg terms giving sllitableeample
i) Seroomechanism (ii) -egulator
1. Distinguish betweenfee#bac,control system an#fee#forwar# control system.
23. Differentiate between4
2. 5inear an# 6onlinear systems 2. Continuous an# Discrete#ata systems
22. Eplain what is close#loopcontrol system.
22. "rite a note on multioariablecontrol systems.
:I
000
nl
(1 - 24)
nl
I~ Control System Engineering 1-3 Basics of Control System
The input variable is generally
referred as the Reference Input
and output is generally referred as
the Controlled Output.
Reference """'~ PLANT
Input " l
10-----''''-Controlled
v' output
Cause and effect relationship
beteen input and output for a plant can be shon as in the !ig. 1.3.
!ig. 1.3
!" Classification of Control Systems
"roadly control syste#s can be classified as$
# %atural Control &yste#s ' The biological syste#s$ syste#s inside hu#an being are
of natural type.
()a#ple $The perspiration syste# inside the hu#an being is a good e)a#ple of
natural control syste#. This syste# activates the secretion glands$ secreating seat and
regulates the te#perature of hu#an body.
*+ ,an#ade Control &yste#s ' The various syste#s$ e are using in our day to day
life are designed and #anufactured by hu#an beings. &uch syste#s li-e vehicles$
sitches$ various controllers etc. are called" #an#ade control syste#s.
()a#ple * ' .n auto#obile syste# ith gears$ accelerator$ bra-ing systc# is a good
e)a#ple of #an#ade control syste#.
3+ Co#binational Control &yste#s' Co#binational control syste# is one$ having
co#bination of natural and #an#ade together i.e. driver driving a vehicle. In such
syste#$ for successful operation of the syste#$ it is necessary that natural syste#s
of driver alongith syste#s in vehicles hich are #an#ade #ust be active.
"ut for the engineering analysis$ control syste#s can be classified in #any different
ays. &o#e of the classifications are given belo.
/+ Ti#e 0arying and Ti#e - Invariant &yste#s ' Ti#e varying control syste#s are
those in hich para#eters of the syste#s are varying ith ti#e. It is not
dependent on hether input and output are functions of ti#e or not. !or
e)a#ple$ space vehicle hose #ass decreases ith ti#e$ as it leaves earth. The
#ass is a para#eter of space vehicle syste#. &i#ilarly in case of a roc-et$
aerodyna#ic da#ping can change ith ti#e as the air density changes ith the
altitude. .s against this if even though the inputs and outputs are functions of
ti#e but the para#eters of syste# are independent of ti#e$ hich are not
varying ith ti#e and are constants$ then syste# is said to be ti#e invariant
syste#. 1ifferent electrical netor-s consisting of the ele#ents as resistances$
inductances and capacitances are ti#e invariant syste#s as the values of the
ele#ents of such syste# are constant and not the functions of ti#e. The co#ple)ity
of the control syste# design increases considerably if the control syste# is of the
ti#e varying type. This classification is shon in the !ig. 1./.
% I "nl
Control System Engineering 1-4 Basics of Control System.
I ,... ~~I ...... '
Parameters Output
of systemare
constantsand not f{t)
functionsof time
f(t
Parameters
of system
are functions
of lime
Input
Input Output
(a) !ime in"ariant system
Fig. 1.4
(#) !ime "ariant system
5) Linear and Nonlinear Systems: A control system is said to be linear if it satisfies
following properties.
a) Te principle of s!perposition is applicable to te system. Tis means te
response to se"eral inp!ts can be obtained by considering one inp!t at a time
and ten algebraically adding te indi"id!al res!lts.
#atematically principle of s!perposition is e$pressed by two properties%
i) Additi"e property wic says tat for $ and y belonging to te domain of te
f!nction f ten we a"e%
f&$ $ y) % f&$) $ fey)
ii) 'omogeneo!s property wic says tat for any $ belonging te domain of te
f!nction f and for any scalar constant e$we a"e%
f(a & % e&f(&
b) Te differential e(!ation describing te system is linear a"ing its coefficients
as constants.
c) )ractically te o!tp!t i.e. response "aries linearly wit te inp!t i.e. forcing
f!nction for linear systems. L
*eal time e$ample: A resisti"e networ+ sown in te Fig. 1.5 &a) is a linear system.
Te Fig. 1.5&b) sows te linear relationsip e$isting between inp!t and o!tp!t
'(
"ln
)
*,
~
"ln - Input,I%Output
(*) linear system (#) 'esponse of system
+ig. (., E&ample of linear system
- I nl
,'! ,~I I.If
Control System Engineering 15 Basics of Control System
A control system is said to be nonlinear, If.
a. It does not satisfy the principle of superposition.
b. The equations describing the system are nonlinear in nature.
The function f(x) =x2 is nonlinear because
f(XI +X2) = (Xl +x2f ;e(Xl)2 +(X2)2
and f(a x) = (a x);eo. x2 where a =constant
The equations of nonlinear system inoles such nonlinear functions.
c. The output does not ary linearly for nonlinear systems.
!The arious nonlinearities practically present in the system are shown in the
"ig. #.$(a)% (b) and (c).
!"t#"t
(a) Sat"rabon
!"t#"t
(b) $ead %one
!"t#"t
(c) Ex#onential or
s&"are law
'ig. (.) *onlinearities
The saturation means if input increases beyond certain limit% the output remains
constant i.e. it does not remain linear. The flux and current relation i.e. &'( cure shows
saturation in practice. In some big ales% though force increases upto certain alue% the
.ale does not operate. )o there is no response for certain time which is called dead *one.
The oltage'current equation of a diode is exponential and nonlinear thus diode circuit
is an example of nonlinear system. +,is is shown in the "ig. #.+.
,
(-) System
. $iode Ex#onential
rise
(b) .es#onse
'ig. (./ Exam#le of nonlinear system
II ~I.
Control System Engineering 16
. .
Basics of Control System ,~ I
It can be seen that as long as Vin increases upto certain value, current remains almost'
Zero. This is a dead zone and thereafter voltage-current are exponentially related to each
other which is a nonlinear function.
Key Point: In practice it is difficult to find perfectly linear system. Most of the
physical systems are nonlinear to certain extent.
But if the presence of certain nonlinearity is negligible and not affecting the system
response badly, keeping response within its linear limits then the nonlinearity can be
neglected and for practical purpose the system can be treated to be linear.
rocedures for finding the solutions of nonlinear system problems are complicated and
time consuming. Because of this difficulty most of the nonlinear systems are treated as
linear systems for the limited range of operation with some assumptions and
approximations. !he number of linear methods, then can be applied for analysis of such
linear systems.
"# $ontinuous !ime and %iscrete !ime $ontrol &ystems ' (n a continuous time
control system all system variables are the functions of a continuous time
variable 't'. !he speed control of a d.c. motor using a tachogenerator feedback is
an example of continuous data system. )t any time 't' they are dependent on time.
(n discrete time systems one or more system variables are known only at certain
discrete intervals of time. !hey are not continuously dependent on the time.
*icroprocessor or computer based systems use such discrete time signals. !he
reasons for using such signals in digital controllers arc,
1) &uch signals are less sensitive to noise.
+# !ime sharing of one e,uipment with other channels is possible.
-# )dvantageous from point of view of size, speed, memory, flexibilityetc.
!he systems using such digital controllers or sampled signals are called sampled data
systems.
$ontinuous time system uses the &ignals as shown in the .ig. 1.B /a# which are
continuous with time while discrete system uses the signals as shown in the .ig. 0.1/b#.
Signal
(a) Continuous signal
ig. 1.!
Signal
t
(") #iscrete signal
r
Control System Engineering 1-7 Basics of Control System
7) Deterministic and Stochastic Control Systems : A control system is said to be
deterministic when its response to input as well as behaviour to external
disturbances is predictable and repeatable. If such respon.c;eis unpredictable
system is said to be stochastic in nature.
!) "umped #arameter and Distributed #arameter Control Systems : Control system
that can be described by ordinary di$$erential e%uations is called lumped
parameter control system. &or example electrical networ's with di$$erent
parameters as resistance inductance etc. are lumped parameter systems. Control
systems that can be described by partial di$$erential e%uations are called
distributed parameter control systems. &or example transmission line havin( its
parameters resistance and inductance totally distributed alon( it. )ence description
o$ transmission line characteristics is always by use o$ partial di$$erential e%uations.
*he lumped parameters are physically separable and can be shown to be located
at a particular point while representin( the system. *he distributed parameters can
not be physically separated and hence can not be represented at a particular place.
+) Sin(le ,nput Sin(le -utput .S,S-) and /ultiple ,nput /ultiple -utput ./,/-)
Systems : A system havin( only one input and one output is called sin(le input
sin(le output system. &or example a position control system has only one input
.desired position) and one output .actual output position). Some systems may have
multiple type o$ inputs and multiple outputs these are called multiple input
multiple output systems.
10)-pen "oop and Closed "oop Systems : *his is another important classi$ication.
*he $eatures o$ both these types are discussed in detail in comin( sections.
1.4Open Loop System
De$inition: A system in which output is dependent on input but controllin( action or
input is totally independent o$ the output or chan(es in output o$ the system is called an
-pen "oop System.
In a broad manner it can be represented as in &i(. 1.+.
Reference I
inputr(t)--~ Controller
u
..
!rocess
Controlle"
1--- ....
output c(t#
u $ %ctuating signal
&ig. 1.' Open loop control system
1e$erence input 2ret)3 is applied to the controller which (enerates the actuatin( si(nal
.u) re%uired to control the process which is to be controlled. #rocess is (ivin( out the
necessary desired controlled output c.t4.
() nl
Control System Engineering 1-8 Basics of Control System
1.4.1 Advantages
The advantages of open loop control system are,
1) Such systems are simple in construction.
2) Very much convenient when output is difficult to measure.
3) Such systems are easy from maintenance point of view.
4) enerally these are not trou!led with the pro!lems of sta!ility.
") Such systems are simple to design and hence economical.
1.4.2 Disadvantages
The disadvantages of open loop control system are,
1) Such systems are inaccurate and unrelia!le !ecause accuracy of such systems are
totally dependent on the accurate precali!ration of the controller.
2) Such systems give inaccurate results if there are variations in the e#ternal
environment i.e. such systems cannot sense environmental changes.
3) Similarly they cannot sense internal distur!ances in the system, after the controller
stage.
4) To maintain the $uality and accuracy, recali!ration of the controller is necessary
from time to time.
To overcome all the a!ove disadvantages, generally in practice closed loop systems are
used.
The good e#ample of an open loop system is an electric switch. This is %open loop
!ecause output is light and switch is controller of lamp. &ny change in light has no effect
on the '(-')) position of the switch, i.e. its controlling action.
Similarly automatic washing machine. *ere output is degree of cleanliness of clothes.
+ut any change in this output will not affect the controlling action or will not decide the
operation time or will not decide the amount of detergent which is to !e used. Some other
e#amples are traffic signal, automatic toaster system etc.
1.4.3 Real TIme Alications of an !en "oo System
The various illustrations of an open loop system are discussed !elow,
1.4.3.1Srl#$ler %sed to &ater a "a'n
T(e system is ad,usted to water a given area !y opening the water valve and
o!serving the resulting pattern. -hen the pattern is considered satisfactory, the system is
.cali!rated. and no further valve ad,ustment is made.
1.4.3.2Steer )otor *ositioning System
The actual position in such system is usually not monitored. The motor controller
commands a certain num!er of steps !y the motor to drive the output to a previously
determined location. . I
+ I
Control System Engineering 1-9 Basics of Control System
1.4.3.3 Automatic Toaster System
p o
:l.
In this system, the quality o f to ast dep ends up o n the time fo r which the to ast is
heated. Dep ending o n the time setting, bread is simp ly heated in this system. The to ast
quality is to be judged by the user and has no effect o n the inp uts.
Power input
D
1
esired
elay or !eating process
time
Controller of "read
Actual
toast
Controller Process
#ig$ 1$1%
1.4.3.4 Traffic &ig't Controller
A traffic flo w co ntro l system used o n ro ads is time dep endent. The traffic o n the ro ad
beco mes mo bile o r statio nary dep ending o n the duratio n and sequence o f lamp glo w. The
sequence and duratio n arc co ntro lled by relays which arc p redetermined and no t
dep endent o n the rush o n the ro ad.
Power
Des
1
ired time
elays to
control se(uence
lig'ts
Actual
traffic
control
#ig$ 1$11
1.4.3. Automatic Door )pening and Closing System
In this system, p ho to sensiti!e de!ices are used. "hen a p erso n interrup ts a light,
p ho to de!ice generates actuating signal which o p ens the do o r fo r sp ecific time. "hen
p erso n p asses thro ugh the do o r, light beco mes co ntinuo us clo sing the do o r. The o p ening
and clo sing o f the do o r is the o utp ut which has no thing to do with the inp uts, hence an
o p en lo o p system.
$ $$
The ro o m heater, fan regulato r, auto matic co ffee ser!er, electric lift, theatre lamp
dimmer, auto matic dryer are e#amp les o f o p en lo o p system.
1$* Closed &oop System
Deflni$o n : A system in which the controlling action or input is somehow dependent on the
output or changes ip t output is called closed loop system.
To ha!e dep endence o f inp ut o n the o utp ut, such system uses the feedbac% p ro p erty.
&eedbac% : Feedback is a property of the system by which it permits the output to be
compared with the reference input to generate the error signal based o p t which tile appropriate
controlling action capt be decided.
I I nl
Basics of Laplace Transform
2.1 Background
The various methods used to solve the engineering problems are based on the
replacement of functions of time by certain frequency dependent variables. This makes the
computation job very easy. The known example of such method is the use of Fourier series
to solve certain electrical problems.
The transformation technique relating the time functions to frequency dependent
functions of a complex variable is called the Laplace transformation technique. Such
transformation is very useful in solving linear differential equations. The transfer function
of a system, which is heart of the control system analysis is based on the Laplace
transform. This chapter gives the definition of Laplace transform, some commonly used
functions and Laplace transform pairs and useful properties of Laplace and inverse Laplace
transform. Some examples are also included demonstrating the superiority of Laplace
approach over the conventional approach.
2.2 Definition of LaplaceTransform
The Laplace transform is defined as below
Let f!t" be a real function of a real variable t defined for t > #, then
F!s" = L$f!t"%= 1 f!t".e&st dt
o
'here F!s" is called Laplace transform of f!t". (nd the variable)s) which appears in
F!s" is frequency dependent complex variable. *t is given by,
I S + !,jm I
'here 0' + -eal part of complex variable ).).
eo = *maginary part of complex variable ).).
The time function f(t) is obtained back from the Laplace transform by a process called
*nverse Laplace transform and denoted as Ll. Thus,
. (2 1)
I I ! nl
Control System Engineering 2-10 Basics of Laplace Transform
".. Example 2.3 :
Find the inverse Laplace transform of given F(s).
F(s) =s(s+ 3)(s+ 4)
Solution: The degree of N(s) is less than 0(5). Hence F(s) can be exressed as!
"# "2 "$
F(s) = s% (s%&) % ('%()
(s+2)
)here
I ('%2) 2
1
"$ =
s. F(s) s= 0 =s . s(s% &)(s%() = &x( * + ,
5"!
I (s% 2"
(-&%2)
1
"2 -* (s%&) . F(s) -- * (s% &). ( &)( () =
(-&)( -&%() * &
5- & S s% s%
s= -&
I (s% 2)
(-(%2)
1
"& =
(s%() . F(s) * (s%(). ( &)( () *
(-()(-(%&) =2"
#=$% S s% s%
&&&&& %
1., 1.& 1.2
F(s) = $'( " ( "
5 s%& s%(
Ta/ing in0erse 1alace transfor2!
1 1 1
f(t) = , '&+ e$)t -32e$(t
2.%.2 *+ltiple ,oots
The gi0en function is of the for2!
-(s"
F(s) -
- (s-a) n 43(s)
Here there is 2ultile root of the order 3n3 existing at s =a. The 2ethod of 5riting the
artial fraction exansion for such 2ultile roots is!
"o "# "2 .n/1 N3(s)
F(s) = (s%a6+ % (s-a)n-$ % (s-a)n- 2 % ... % (s-a) % 43(s)
N'(s)
)here + 43(s)reresents re2aining ter2s of the exansion of F(s).
.ey 0oint: Thus a separate coeJfident is assumed for each power of repetaiioe Toot, starting
from its higllfSt power n to 1.
I I nl"
Control System Engineering -" 2 -11 Basics of Laplace Transfonn
Here find the L.C.M. of the entire right hand side and express numerator interms of
Ko ,K, ,K2, The numerator N{s) on left hand side is knon. Compare the !oeffi!ients of
all poers of s inthe numerator of "oth sides hi!h ill gi#e simultaneous e$uations
interms of Ko ,K. ,K2, %ol#ing these e$uations e !an o"tain the !oeffi!ients
Ko ,K1 ,K2,.
&or ease of sol#ing simultaneous e$uations, e !an find out the !oeffi!ient Ko "' the
same method as dis!ussed for simple roots.
Ko = ()* a) n &(s)t = a
%imilarl' !oeffi!ients for simple roots present if an', !an also "e !al!ulated "' the
method dis!ussed earlier, for ease of sol#ing simultaneous e$uations.
+hile finding Lapla!e in#erse transform. of expanded &(s) refer to standard transform
pairs,
n!
, --;;Jand
s
n! n!
and L-e./ 0 0 tn1 ,
(%*a)n.l
( )
n+1
s.a
,_ Example 2.4 :
Obtain the inverse Laplace transform of given F(s).
F(s) =
(1*2)
1(1. 0 )2
Soltion: The gi#en &(s) !an "e expressed as,
Ko K3 K4 K)
&(s) 55 (s. 0 )2 . (). If . (). I) . %
&inding L.C.M. of right hand side,
(s* 2) Ko (1). K. (). 1) s. K4 (s. 0 )2 s. K) (s. 0 )2
s(s. 0 )2 * s(s. 0 )2
:. (s - 2) , Kos +K. %2 +K6 s +K;!s3 +2K;!s2 +K2s +K2%2 +2K2%2 +2 K7s +K)
Comparing !oeffi!ients of #arious poers of s on "oth sides,
&or 12,
K2 + K7 , / .. (1)
&or %2,
K, + 2 K2 + 3 K7 =0 ... (2)
&or s8,
Ko + K, + K2 + 3 KJ
, 1
... (3)
&or s/ ,
K,7 = - 2 ... (9)
:s
K2
,
-2
from (3),
K2
,
2
.
from (2),
Kl
,
2 ..
I I nl
, Control System Engln.. rlng 2-12 Basics of Laplace Transfonn
. from (3),
Ko = 3
pes)
3 2 2 2
= + +----
(s+ 1)3 (s+ 1)2 (s+ 1) s
L[e-at t n]
n!
Now
=
(s+a)n+ I
[ 1 ]
e-a' t n
.
L-I
=
n!
(S+8)n+ I
1 1 1 !
.
F(s)
=
3 . " 1)3 + 2 . < 1)2 + 2 . "#$ - 2 . S
s+ s+ s+
3 2
.
f(t)
= L-I [F(s)]= 2!e-1 t2 + 1! e-! t + 2 e-t - 2
3
.
f(t)
=
-t2e-!+2te-t +2e-t-2
2
2.%.3 Comple& Con'(gate )oots
If t"ere e#$sts a %&a'rat$( term $n )(s) of F(s) w"ose roots are (omp*e# (on+&,ates t"en
t*e -(s.$s e#presse' w$t" a f$rst or'er po*/nom$a* in s $n t"e n&merator as,
+s+B ,-"s$
."s$ = +-
(s2+as+03) )'(s)
$
,-"s$
1"ere (s2 +(2 #s+$3 $s t"e %&a'rat$( w"ose roots are (omp*e# (on+&,ates w"$*e /-"s$
represents rema$n$n, terms of t"e e#pans$on. 3"e + an' B are part$a* fra(t$on (oeff$($ents.
3"e met"o' of fin0ing t"e (oeff$($ents in s&(" a (ase $s same as '$s(&sse' ear*$er for
t*e m&*t$p*e roots. )n(e 4 an' 5 are 6nown t"en &se t"e fo**ow$n, met"o' for (a*(&*at$n,
$n7erse 8ap*a(e transform.
9ons$'er 4 an' 5 are 6nown
Now (omp*ete t"e s%&are in t"e 'enom$nator 1y (a*(&*at$n, *ast term as,
"2.T, )2
8.3. = " $
: F.3.
1"ere L.T = 8ast term
;.3 = ;$''*e term
F.3 = F$rst term
l I -rrl
Control System Engineering r;.~ 2 -13 Basics of Laplace Transform
L.T.
Where
[(i2
(I) = VJj-T
Now adjust the numerator As + B in such a way that it is of the form,
or
L [e-a coscot! = ( "2 2
s#a #($
(s# a"
%&e-a' sin cot! ( ( "2 2
)# a #(1"
(l)
Key oint! Thus inverse Laplace transform of F(s) having complex conjugate roots of D(s),
always contains sine, cosine or damped sine or damped cosine functions.
"# E$ample %.&! Find the inverse Laplace transform oj
S !"
F(s) = (s +#s!$)(s! 2)
Sol'tion! The *i+en ((s)can ,e writte&1' -,
.. ~. . . .
I I
As#B /
((s) ) +)
- s2#20#) s#2
*s +% # 20# ) has com1e2 conju*ate ,roots. To find A, Band / find %. /. 3. of ri*ht
hand side,
((s) (
(s# 2"(As# B"#/(s
%
#2s# )"
(s
2
# 2s# )"(s# %)
=
()2 # 20#)"(s# %)
A42#2 As#Bs#2 B#/s2 # 2s/# )/
r (s
2
# 2s# )"(s#%)
. . . ,).
/om1arin* the coefficients of +arious 1owers of ), of the numerators of ,oth sides
-
&%#3
(
42 (A#/"#s(2A# B#2/"#(2B#4/"
-.
-
A#/ = . ... (.)
-.
I! /
-
%* # B + %C = $ ./. (%) ..
-.
%B + SC ( 3 . . . (3"
To so+e the e5uations 5uic6y, the coefficient / corres1ondin* to the sim1e, rea root
can ,e o,tained as,
-
-.
/ ( 7 s . (s#2"j = (s2#3"(s#2" = (8#3" ( -
(" )( -2 (s2#20#)"(0#2")( 92 (8-8#)" )
l I ,rrl
Control System Engineering 214 ~: ..~ Basics of Laplace Transfonn
Substituting in(1) and (2),
2
A = -5
and B = - 2
2 '7
- " ' 5 s- 2 " 5
= +--
s2 + 28+5 (8+2)
F(s)
Consider
Completing square inthe denominator,
2 2
- 5 s- 2 - 5 - 2
! l (s) = ., = "
s- +2s+1+5 - 1 (s+1)- +(2)2
= - i# (s+s$)' %
split & as 2 ' 2
.
.. F(s)
(
= _~{ (s+ 1 ) +2' 2 } +_l _
5 )s+ 1)2+(2)2 )s+ 1)2+(2)2 (s+ 2)
As
#
(s+a) % # %
L-I ., = e-al cosrot
(s+ a)$' +ro2
and
L-I ~ ! = # e- al sin rot%
(s+ a)- +(1)2
*en+e ta,ing in-erse .apla+e trans/orm o/ F(s),
2 " 7
#(t) = - 5 # e- t +os2t +2 e- t sm 2t%+S e- 2t
2.5 Use of Laplace Transform in Control System
$he +ontrol s0stems +an be +lassi/ied as ele+tri+al me+1ani+al, 10drauli+, t1ermal and
so on2 All s0stems +an be des+ribed b0 integrodi//erential equations o/ -arious orders2
31ile t1e output o/ su+1 s0stems /or an0 input +an be obtained b0 sol-ing su+1
integrodi//erential equations2 4at1emati+all0, it is -er0 di//i+ult to sol-e su+1 equations in
l I nl
Control System Engineering 2-15 Basics of Laplace Transfonn
time domain. The Laplace transform of such integrodifferential equations converts them
into simple algebraic equations. All the complicated computations then can be easily
performed ins domain as the equations to be handled are algebraic innature. Such
transformed equations are known as equations in frequency domain.
Then by eliminating unwanted variable, the required variable in s domain can be
obtained. Then by using technique of Laplace inverse, time domain function for the
required variable can be obtained. Hence making the computations easy by converting the
integrodifferential equations into algebraic is the main essence of the Laplace transform.
). Example 2.6: Obtain the expression for y(t) which is satisfying the differential equation
d2 y(t) dy(t)
dt2 + 6---cit + 8 y(t) =16e-'. eglect initial conditions.
Solution: Taking Laplace transform of both sides of the given differential equation and
. . .. ! di . . La ! " f d#y$t% d dy$t%
neg ectmg initia con lion terms n p ace transform & dt# an -at we get,
s#y$S% ' 6s($s% ' )($s% *
!6
s' 1
$S# ' 6s' 8) ($s%
!6
=
$+' !%
!6
yes%
=
$s' !% $s# ' 6s' ,%
(es%
!6
:=:
$s' !% $s' #% $s' -%
(es%
at a# a.
=
--'--'--
s' ! s'# s'-
+... , #.66
(es% /* /
---'--
s'l ,'# s'-
Taking inverse Laplace transform of ($s%,
y(t) = +...e-t - ,e-#t + #.66e--t
This is the required solution of differential equation.
2.6 Special Case of Inverse Laplace Transform
Let us see now if the order of pes% and 0$s% of the function 1$s% is same. Insuch case
pes% must be divided by 0$s%, to obtain the seperation of 1$s% as a constant term which is
result of the division and the remainder polynomial P' $s% having order less than 0$s%.
pes%
So 1$s% = 0$s% ... order of pes% and 0$s% same
2! nl
Control System Engineering 2 -16 Basics of Laplace Transfonn
P'(s)
=: K+ Q(s)
.... after dividing pes) by Q(s)
Now Laplace inverse of constant term is impulse function. Refer last pair in the
Tale !.!
L-l "K# = K$(t)
where %(t) =unit impulse. I
&hile '( (s) ) Q(s) can now e e*pressed to otain partial fraction e*pansion+ to get its
inverse ver, easil,.
Note: The same method is to e applied -(s) with order of numerator pol,nomial
pes) is greater than denominator pol,nomial Q(s) .
....... s3+18s2+3s+5
)..,... Example 2.7: Find the Laplace inverse of F(s) = :\ ..! /0 /%1
s + ~ + s+
Soltion: 2ivide pes) by Q(s#.
s3+.s2+/0s+/%) 43+/.42+ 34 + 4 (/5K
53 + .42+/04 + /%
.
..
/%6!7/84 7
4 --+ P'(s)]
F(s) /+
/%s!7/8s74
=
s3 +.s2 +/04+/%
/%s!7/8s74 9:;
=
/+(s+!) (4+/) (4+4) = /+s+!+s+/ +4+4
A
/%s!7/8674/
=
(s+/) (s+4) s =-2 = 7 !/
B
/%4!7/8674/
=
(4+!) (s+4) =8.04
. s --I
;
/%s!7/8674/
=
(s+ /) (s+!) = !<.!4
s ",-5
F(s)
!/ 8.04 !<.!4
=
/74+!+s+/ + 4+4
f(t)
=
L7/ t-(s=)=(!)-2"e7 !t + 8.04e7! +!<.!4e7 4t
L-l (#
= o;t) =>nit impulse function.
$he%e
nl
Control System Engineering 2 -17. Basics of Laplace Transfonn
Examples with Solutions
".. Example 2.8: In the circuit given, the values of R and Care !" and J.l1 respectively.
Obtaitl the expressionfor the current flowing in the circuit if it is supplied with an input oj
step voltage of 1 V at t #O.
R
f
"I'M
Ie
i$%&'
Step of
v a r = o
T
(ig. 2.
So lutio n: Le t us wr ite do wn the e qua tio n fo r the ir uit !iv e n" a sswnin! su##l$ v o lta !e
a s v e t%.
R
f
."&'v (
Ie
i$%&'
v e t)
)
T
(ig. 2.$a'
*##l$in! +ir hho ff's v o lta !e la w we !e t,
v e t) # i-t% R * + I i-t% dt
.he r e v o lta !e a r o ss a #a ito r is +I i-t% dt
.hile v e t) #ste # o f 1 ' a t t #/ a s sho wn in the 0i!. 2.1-1%.
Ta,ing La #la e tr a nsfo r 2 o f 1o th side s o f a 1o v e e qua tio n,
3 e s% = l-s% R * + 4 I5 %6
v 7 ll&)t- (
o
0i!. 2.1-1%
&e !le tin! initia l o nditio ns he ne ne !le tin! the te r 2 o f initia l o nditio n in La #la e
tr a nsfo r 2 o f I i-t% dt.
&o w 3 e s% # .). a s v e t% is ste # o f -
s
... Re fe r #a ir 1 in 8a 1le 2.2

s = 1-9% R * s: I-s%
l . nl
Control System Engineering 2 -18 Basics of Laplace Transfonn
1
I ( S > [ R + s ~ ] =
5
1
1( 5) [ 1+s RC]
-
=
s s C
I ( s )
C C
=
l+s RC
=
RC[ S + R~ ]
I ( s )
=
i [ s +l~ J
Taki ng Laplace i nves e! e"ei ng Ta#le 2.1
.() 1 -$ $ % $ $ &
I & = -c" ...
R
S '#s & i & '& i ng val'es (" R an) C!
i ( & * =
++ 1~ e-1)( 1(!)( 1)( 1"-!
, 1"+-
i ( & ) = 1# 1"-! e-& $
%ey &oint' It can b e observed in this example that solving the equation in time domain for
the current involves calculation of complementary function, then particular integral and
arbitrary constants, independently. In Laplaceapproachwe get the answers of all in a single
step. Hence Laplaceapproachproves & ( b e superior over a normal approach.
). E(ample 2.)' $ series circuit consisting of resistance R and an inductance of L henry is
connected to a supply of v(t) volts. ind the expression of the current in s domain. !lso
calculate the value o" current at t = #.$ msec with R = 1 # 1"* nL = 2+ mH and supply
is a step voltage o f +" %. &eglect initial condition.
Sol,tion' T-e ci c'i & i s s .(/n i n t-e .ig. 2.2.
0ppl1i ng 23L /e ge& !
ve& ) = i ( & ) R + L )i e& *
)&
R
i ( ~
L ve& )
Taki ng Laplace an) neglec& i ng i ni & i al
c(n)i & i (ns (" c'en& /e ge& !
4i g$ 2$ 2
3( s ) = l( s )R + Ls I ( s )
3( s ) = 1( 5) [ R + s L]
// nl
Control System Engineering 2-19 Basics of Laplace Transform
l{s) =
yes)
R+sL
Now R =lxl03 n L = 25xlO-JH
50
and yes) =
s
l{s) =
50
s[lx 103+sx 25x 10-3]
50
...Takn! 25xlO-3 o"#sde l{s) =
25x 10-3sx [s+ $x 10$]
%{s) = s [5+$x 10$]
.. A = 0.05 and & = - 0.05
2000 A B
= -+
s [s+$xl0$]
... 'a(#al )(a*#ons
0.05 0.05
1+5) = -s-- s+ $x 10$
Takn! n,e(se La-la*e #(ans)o(./
+#) = 0.05 - 0.05 e-olx 101 A
.
!.
+#) = 0.05 +10e-$xl0$#) 1
1# # =0.5 .se* =O.5x10-3 se*
. +#) = 0.05 +l0e-$2l0$xo.52lo-3 ) 1
{#) = 0.05 A.
".. Example 2.103 Calculate Laplaceinverse of F(s) = +2 24 5)
55 + s+
20
Sol#tion$
-es) = 5+52 + 2s+ 5)
Now 6"ad(a#* s2+2s+ 5 7as *o.-lex *on8"!a#e (oo#s.
T7e(e)o(e -a(#al )(a*#on ex-anson o) 9{s) s/
20 al a2 s+a 3
9+s) = s+s2+2s+5) =5+ 52+1:+5
!.
20 = al +s2+2s+5) s+ s+a2 s+a3)
!
20
= s2 +al +a2)+s+2al +a3)+5al
!.
! 5al = 20 e6"a#n! *ons#an#s
!.
2al + a %
=
0 e6"a#n! *oe))*en#s o) s
a1 +a2 =
0 e6"a#n! *oe))*en#s o) s
2
&&' nl
Controt System Engineering 22 Basics of Laplace Tran.fonn
L-l [F(s)] = L-l (L(f(t} = (t)
The time function f(t) and its Laplace transfotm F(s) is called transform pair.
'. Example 2.! Find the Laplace transform of rot and fo r t " #.
$olution! (i) f(t) =e%at
%
-
F(s) = L&f(t)'
=
I f(t) e%(lt dt = J e%at . e%st dt
) )
% [ ]%
=
I e%(s*a)tdt = + %%. e%(s*a)t
o (s*a) )
( -1)
=
o - s*a = s*a
L&e%at'
1
and L-l{ _ l }
= =
e%al
..
s*a s*a
(ti) f(t) = 1
% % [e%st ]-
1

F(s)
=
, f(t) e% st dt = , e- st dt = _ s
_
..
s
o 0 0
L&l'
1
and
L%l&"'=l
=
- ..
s
2. !ro"erties of L#"l#$e Tr#nsform
.um/er of important properties of t0e Laplace transform are discussed in t0is section.
T0e ta/le of Laplace transform pairs is de1eloped using t0ese properties.
2.2. Line#rity
T0e transform of a finite sum of time functions is t0e sum of t0e Laplace transforms of
t0e indi1idual functions.
$o if Ft(s)3 "(s)3 3 Fo(s) are t0e Laplace transforms of t0e time functions ft(t)3
f2(t)3 3 fn(t) respecti1el4 t0en3
%
L&fl(t) & 0(t) & & fn(t)' ='t(s) & '.2(s) & & Fn(s)
( T0e propert4 can /e furt0er extended if t0e time functions are multiplied )y t0e
constants i.e.
50ere a6' a 23.7777.777.77 3 a n are constants.
((* nl
Control System Engineering 2-20 Basics of Laplace Transform
equating coefficients of s2
al=4 a2=-4 a3=-8
F(s) =! + -45-8 =!_4 [ s+ 2 ]
S S2+ 2s+ 5 s S2+ 2s+ 5
Co!leteing squa"e in t#e $enoinato"
F%s) - 4 _ 4 [ s+ 2 ] _ 4 _ 4 [ s+ 2 ]
- S S2+ 2s+ & + 5 - & - s (s+ &)2 + (2)2
'o( t#e $enoinato" is in t#e fo" (s+ a)2 +)* +et us a$,ust -nue"ato" to get t#e
e.!"ession in stan$a"$ fo"*
F(s)=! 4 [(s/ 0 / / 0 )2]
.
4 4[ s+ 2 &]
= S - (5+ &)2 +(2)2 + (s+ &)2 +(2)2 (1
'o( fi"st te" is in t#e fo" (s0 a) ro2 (#ic# is +a!lace of e-at cos "ot*
(s+a)- +
)#ile t#e secon$ te" nee$s a$,ustent of constant to get in t#e fo" 0 *1
(s+ a)- + o"
(#ic# is +a!lace of e-al sin (l)t.
.
F( ) 4 (5+ !) ! 2 "
s = S - 4 (s+ &)2 +(2)2 +#2 $ (s+ &)2 +(2)2
1 2a3ing in4e"se +a!lace t"ansfo" (e get*
f%t) = 4- 4e-t cos 2t+ 0*5e-t sin 2t]
.
f(t) = 4- e- t (4cos 2t + 25in 2t)
000
%%& nl
Control System Engineering 2-3 Basics of Laplace Transfonn
2.3.2 Scaling Theorem
If K is a constant then the Laplace transform of K f(t) is given as K times the Laplace
transform of f(t).
I L {K f(t) = K F(s) I
... K is constant
2.3.3 eal !ifferentiation (!ifferentiation In Time !omain)
Let F(s) be the Laplace transform of f(t). Then,
L {""#t$$ = s F{s) - f(%-)
Where f (&-) inicates val!e of f(t) at t " &- i.e. #!st before the instant t =o.
The theorem can be e$tene for nth orer erivative as,
{
"nttt)%
L # =sn f(&)-&'-( fto-))sn-* f'(%-)- ...- f(II-() (&-)
Where f(ll- () (+-- ) is the val!e of (n - l)th erivative of f(t) at t =+-.
. {"*f{t)$
i.e for n =*, L E *
= s2 F(s)-s f(,-)- f-(,-)
{
".ftt/ %
for n =., L t.
= S
3
F(s)-s
2
f(+- )- sf- (+- )- f) (+- ) an so on.
This propert0 is most !sef!l as it transfonns ifferential time omain e1!ations to
simple algebraic e1!ations, along 2ith the initial conitions, if an0.
*3..4 5eal 6ntegration
If F(s) is the Laplace transform of f(t) then,
{
I$ F( )
L 7ftt)t = #
This propert0 can be e$tene for m!ltiple integrals as,
{6
ll 6n % F(s)
L 8 I ftt)t8 - t*, t n =-n
o + + s
l * -nl
Control System Engineering 2-4 Basics of Laplace Transform
2.3.5 Differentiation by s
If F(s) is the Laplace transform of f(t) then the differentiation by s in the complex
frequency domain corresponds to the multiplication by t in the time domain.
L{tf(t)} =
-dF(s)
ds
Thus
L{t}
d d ! " # " " $
% L{ tx! =""# rL{$!% =""# " ="&'"=""
& & & s- s'
d d ! " # 2 2$
L{t2! = L{txt!="#s (L{tl)="#s s2 % ('))% s2t"l
2.3.* ComplexTranslation
If F(s) is the Laplace transform of f(t) then by the complex translation property
F(s - a) % L{e*" ttt+!
and
F(s , a) = L {e-* + ttl+!
-.s % " % a+ = F(&(% $ +,a
-here F(s) is the Laplace transform of f(t).
2.3./ 0eal Translation .S1ifting T1eorem+
This theorem is useful to obtain the Laplace transform of the shifted or delayed
function of time.
If F(s) is the Laplace transform of f(t) then the Laplace transform of the function
delayed by time Tis
I ~ - L - { - f - ( t - - - T ) - ~ - = - - e - - T - S - F ( - ~ ~
2.3.2 Initial 3al4e T1eorem
The Laplace transform is .ery useful to find the initial .alue of the time function f(t).
Thus if F{s) is the Laplace transform of f(t) then
f(/,) % Lim f(t) % Lim s F(s)
" -.0, &-/}''
The only restriction is that f(t) must be continuous or at the most a step discontinuity
at t =/.
r
Control System Engineering 25
Basics of Laplace Transfonn
2.3.9 Final Value Theorem
Similar to the initial value, the Laplace transform is also useful to find the final value
of the time function f(t). Thus if F(s) is the Laplace transform of f(t) then the final value
theorem states that,
Lim f(t) = Lim s F(s)
t !! s" ....o
The only restriction is that the roots of the denominator polynomial of F(s) i.e. poles of
F#s$ have negative or %ero real parts.
&. E'ample 2.2( Find the Laplace transform of sin (J)t.
Solution( The sin (Iltcan be expressed using uler!s e"uation as,
#o$
e%(&lI( 'e'i(l&t
sin rot
(()
(%
L* sin cot)
* e+'e'+ )
=
L (%
* ei+ , - *e " .+ ,
= L !)! L (!
.... & &
,eal)
/
,e'0t, =
/
1
and
s-a s-a
.,. using Linearity property
.
..
L* sin rot)
l. / 0 / . / 0 / . / / 0
=
(% s'%ro ' 2. s-%ro 1 2. s'%ro ' s-%ro
/ . S-%ro'S-%ro0 / . (%re 0
=
2. (s'%ro)(s-%1o) = (% s(2(%ro)(
$
1
3( -14(
L* sin rot)
eo
1
s(-ro(
22 nl
Control System Engineering 2-6 Basics of Laplace Transfonn
Table of Laplace Transforms:
f(t) F(s) Wavefonn
1
1
1t
-
s
0
. .
Constant K
K
Kt
-
s
0

K f(t)! K is constant K F(s)
t
1
1/t S"
0
. .
t n
n#
Sn$1
a-al
1
%---
--
s$a
0
eat
1
&
-
s-a
0

e-al tn n#
(s $ a)n$ 1
sin rot
ro
s"$oo"
'/"'\. r:
0
(-)
(%
cos rot
S
r & )
s"$oo"
. .
0--0-0-
e-al sin rot
(*)
(s$a)"$r+"
" ' - - / Y - : n . - _ .
0 y,, y--(--%
. . . . . -
e-.1 cos rot
(s$a)
(s$a)"$r+"
Y~l\~-7'\-"_'
o . ')l. . . /--0 -
sin'rot
ro
sLr+"
cos'rot
s
s"-r+"
Table " 1Stan1ar1 Laplace transfonn pair
r
Control System Engineering 2-7 Basics of Laplace Transfonn
Function f(t) Laplace Wavefonns
Transform F(s)
Unit step = u(t)
1
1t
-
s
0

u(t)

t
-
5
0
!
"elaye# unit step $ u(t - T)
e-Ts
t
-
1
5
%
!
0
T
u(t-T)
e-Ts
+ - 1
- -
s
. .
0
T
Unit ramp = ret) &%'t u(t)
1
52
(lope$)
o . .
t u(t)

t L : : = A
52
0
"elaye# unit ramp '' r(t - T) $ (t - T) u(t - T)
e-Ts
52
t
(%*pe$)
0
T !
(t - T) u(t - T)
e-Ts
52
~ A
o T
Unit impulse = + (t) )
,
%
lL-
t$*
%% . nl
Control System Engineering 2-8 Basics of Laplace Transfonn
Delayed unit impulse = l)(t- T)
e-Ts
LL
_I
o t=T
Impulse of strengt ! i"e" ! # (t)$% L O ! : ' !
"
!L
t = 0
Ta&le 2"2 Laplace transfonns of standard time functions
2"' In(erse Laplace Transform
As mentio ned earlier, inverse L aplace transfo rm is calculated &y partial fractio n
metho d rather than co mplex integratio n evaluatio n L et !"s# is the L aplace transfo rm o f f(t)
then the inverse L aplace transfo rm is deno ted as,
!"t$ = L-I %!"s$&
'he !"s$, in partial fractio n metho d, is (ritten in the fo rm as,
pes$
) "*$
""
=
+"s$
I) s ,
,here )"s$
= )umerato r po l-no mial in s
and +"s$
=
+eno minato r po l-no mial in s
!ey *oint+ The given function F(s) can be expressed in partial fraction form only wizen
degree of N(s) is less than D(s).
.ence if degree o f )"s$ is e/ual o r higher than O "s$ then mathematicall- divide )0s$
1- +"s$ to express !"s$ in /uo tient and remainder fo rm as,
!"s$
= , - !I"s$
.%(s)
=
, - /%(s)
,here
,
=
2uo tient o 1tained &y dividing )"s$ 1- +"s$
and !I"s$
.%(s)
= l3"s$ = 4emainder
l I
Control System Engineering 2-9 Basics of Laplace Transfonn
Now in the remainder, degree of N'(s) is less than D'(5) and hence FI (s) can be
expressed in the partial fraction form. Once (s) is expanded intenns of partial fractions,
in!erse "aplace transform can be easil# obtained b# ad$%sting the terms and referring to
the table of standard "aplace transform pairs (&able2.').
&he roots of denominator pol#nomial D(s) pla# an important role in expanding the
gi!en (s) into partial fractions. &here are three t#pes of roots of D(s). &he method of
finding partial fractions for each t#pe is different. "et %s disc%ss these three cases of roots
of D(5).
2.4. Simple an! "eal "oots
&he roots of #($) are simple and real. (ence the f%nction (s) can be expressed as,
N(s) N(s)
(s) ) D(s) ) (5-a)(s-b)(s-c) ...
*here a, b, c ... are the simple and real roots of D(s). &he degree of N(s) sho%ld be
alwa#s less than O(s). &his can be f%rther expressed as,
(s)
N+) ,, ,2 ,-
% %&&'&&'&&'
(s-a)(s-b).s-c) '.. (s-a) (s-b) (s-c) ...
*here (if (2, ,-, /.. are called partial fraction coefficients. &he !al%es of (if ,2,
,- /// can be obtained as,
,, %
(0-d)
(0(1a
,2 % (s-b) . (0() b
,2 %
(0-3) . (0)t) c
In general,
,n % (s-sn) . (s+ and so on.
5='n
*here 5
%
n 4 4 4 root of #($)
L)e5al6

%
(s7 a)
is standard "aplace transform pair. (ence once (s) is expressed in terms of partial
fractions, with coefficients ,if ,2 ... ,n, the in!erse "aplace transform can be easil#
obtained.
II * nl
Transfer Function and
Impulse Response
3.1 Background
The mathematical indication of cause and effect relationship existing between input
and output means to decide the transfer function of the given system. Itis commonly used
to characterize the input-output relationship of the system.
Transfer function explains mathematical function of the parameters of system,
performing' on the applied input in order to produce the required output. Laplace
transform plays an important role in making mathematical analysis easy. Laplace transform
and its use in control system analysis is thoroughly discussed in hapter-!. In this chapter
concept of transfer function, transfer function models and impulse response models of the
systems are discussed.
3.2 Concept of Transfer Function
In any system, first the
system parameters are
designed and their values
are selected as per the
requirement. The input is
selected next, to see the
performance of the
designed system. This is
shown in the "ig. #.$.
%ow perfonnance of the system can be expressed in terms of its output as,
&utput ='ffect of system parameters on the selected input
Selection of
system parameters
Selected

System parameters $-----(.-- Output


input
Fig. 3.1
&utput =)nput x 'ffect of system parameters,
&utput
'ffect of system parameters * )nput
This effect of system parameters, role of system parameters in the performance of
system can be expressed as ratio of output to input. +athematically such a function
(3 - 1)
II \ nl
Control System Engineering 3-10 Transfer Function and Impulse Response
C(s) = 0.4 _ 0.4
s 5+5
Taking Laplace inverse of this equation
c(l) = 0.4 - 0.4 e-!t ... "utput
3.5 Some Important Terminologies Related to the T.F.
#s transfer function is a ratio of Laplace of output to Laplace of input an$ %ence can
&e e'presse$ as a ratio of pol(no)ials in's',
*(5)
T.+. = ,(s)
This can be further e'presse$ as
=
aosm + alSm! + a"Sm" + + am
&osn +&15n-1 +&-!n-- + +&n
T%e nu)erator an$ $eno)inator can &e factorise$ to get t%e factorise$ for) of t%e
transfer function as
~ - - - - - - - - - - - - - - - - - - - - - - - - - - ~
.
..
T.+. =
.(s - sa)(s - 5&) (5- !))
(!- !/) (5- 5-) (5- !n)
0%ere . is calle$ s(ste) gain fa$or. 1o0 if in the transfer function values of 252 are
su&stitute$ as 51 3- 53...44. !n inthe $eno)inator t%en value of T.+. 0ill &eco)e infinit(.
3.5.! #oles of a Transfer Function
5efinition 6 The valuesof '5',which make the T.F. infinite after 5ubstit"tion in the
denominator of a T.F. are called Ipoks' of that T.F.
!o values 51 5- 33 .44.445n are calle$ poles of t%e T.+.
T%ese poles are not%ing &ut t%e roots of t%e equation o&taine$ by equating
$eno)inator of a T.*. to 7ero.
+or e'a)ple let t%e transfer function of a s(ste) &e
T(s) 66 -(5+ -)
5(5+4)
T%e equation o&taine$ &( equating $eno)inator to 7ero is
s(s + 4) = 0
.
.. s=O an$ 58-4
If t%ese values are use$ in the $eno)inator t%e value of transfer function &eco)es
infinity. 9ence poles of this transfer function are 5 =0 an$ - 4.
l $ %nl
Control System Engineering 312 Transfer Function and Impulse Response
2(s+ 1}2 (5+ 2}(S2 + 2s+ 2) = 0
Simple zero at s = -2
Repeated zero at 5 = -1(twice)
Complex con!"ate zeros at s = -1l#
$%e zeros are indicated &' small circle or zero (0( in t%e s-plane#
i#e#
3#5#) *ole-+ero *lot
,e-inition . Plot obtained by locating all poles and zeros o f a T.F. in s-plane is
called pole-zero plot of a system.
3.. !rder of a Transfer Function
,e-inition . The highest po wer of 's' present in the characteristic equation i.e. in
the denominator polynomial of a closed loop transfer function o f a system is called
'Order' o f R system.
3#5#/ ,#C# 0ain
$%e 1al!e o- t%e trans-er -!nction o&tained -or s = 0 i#e# zero -re2!enc' is called t%e
d#c# gain o- t%e s'stem#
rl-,-#C-#-0-a-in-= -$-(-S}-3-s 4 (-0(-(5
"ey #oint$ It is not possible to indicate the value o f d.c. gain on pole-zero plot as it is a
constant value. It is required to be separately specified, alongith the pole-zero plot.
$%e d#c5 "ain decides t%e res!ltant constant o- t%e s'stem trans-er -!nction#
1
6or example5 consider example 3#1 disc!ssed earlier# $%e s'stem $#6# is 1+ sRC
So 1 + sRC = 0 is its c%aracteristic e2!ation and s'stem is -irst order s'stem#
$%en s = - lIRe is a pole o- t%at s'stem
and $#6# %as no zeros#
Imaginary
$%e correspondin" pole-zero plot can &e
s%own as in t%e 6i"# 3#7#
Similarl' -or example 3#25 t%e $#6#
calc!lated is5
1
-6re
3s - #lane 3
-&f-----+--'(Real
1 ll8e
$#6# = s28C+ sRC+ 1= R 1
) + s 8+ 8C
6i"# 3#7
*' nl
Control System Engineering 3-13 Transfer Function and Impulse Response
The characteristic equation is,
R 1
52 + s L + LC = 0
So system is 2nd order and the two poles are, -~ (2~ r -(kr
T.F. has no eros.
Pole2 Pole1
!ow i" #alues o" $, Land C selected
are such that %oth poles are real, unequal
and ne&ati#e, the correspondin& pole-ero
plot can %e shown as in the Fi&. 3.10.
Imaginary
's-(lanel
For a s)ste* ha#in& T.F. as,
Cs! s+ 2!
$(s+ , - . - 2 /2s/20 . 1 2/21 /120
T"e characteristic equation is,
Fi&. 3.10
- (32 / 2s / 24(s2 /21 /12+ = # $e% s(- 2 /2s/2+ (s/3+(s/5+ =#
i.e. - )ste* is 1 th order and there are 1 poles. (oles are 0, -1 &' % ( 3, - 5 while ero is
located at 6-26.
The correspondin& pole-ero plot can %e drawn as shown in the Fi&. 3.11.
7"ter &ettin& "a*iliar with introductor) re*arks a%out control s)ste*, now it is
necessar) to see how o#erall s)ste*s are represented and t"e *ethods to represent t"e
&i#en s)ste*, %ased on the trans"er "unction approach.
Imaginary
8---
I
I
()**+f(,-((I((((i ..(((+ Real
-1: 0
"k--- ('
's - (lane'
Fig/ 0/11
II 1 nl
Control System Engineering 314 Transfer Function and Impulse Response
3" Laplace Transfonn of Electrical Network
In the use of laplace inelectrical systems, it is always easy to redraw te system by
finding Laplace transform of te given network. Electrical network mostly consists of te
parameters , L and !. "he various e#pressions related to these parameters in time
domain and Laplace domain are given inte table below.
Element Time domain Laplace domain Laplace domain
e!pression for "oltage e!pression for "oltage #ea"iour
Resistance R I$l% ! R l$s%R R
Inductance L
di$t%
&
s'es% sL
L &&&at
Capacitance C
' ( i$t% dl
1 1
(I$s%
sC
"able 3.1
)rom te table it can be seen that after taking Laplace transform of time domain
e*uations, neglecting the initial conditions, the resistance behaves as , the inductance
1
behaves as sL, while the capacitance behaves as sc and all time domain functions get
converted to Laplace domain like i$t% to l$s%, vet+ to ,$s% and so on.
-y using these transformations, the parameters can be replaced by their Laplace
transform to get Laplace transform of the entire network. .nce this is obtained, simple
algebraic e*uations relating Laplace of various voltages and currents can be directly
obtained. Tis eliminates the step of writing the integrodifferential e*uations and taking
Laplace of them.
e.g. !onsider a network shown below,
r r
rto&% &/ & 0 0 !0 ' r
)ig. 3.11
"he Laplace of the above network can be obtained by following replacements.
l ' ( L ' sL
)
1 *+ 1 ! *+ s!
nl
Control System Engineering 3-15
The other variables then can be
introduced which will be directly
Laplace variables to obtain the
Laplace domain equations directly.
Such Laplace of network is shown in
the Fi. 3.13.
r
r
Examples with Solutions
Transfer Function and Impulse Response
!
1"5!
Fi. 3.13
1
sC
,,.. Example3.6 TI,e transfer function of a system is given by,
K(5 ! #!
T(s) " 5"5! $!"5 !5!"5$ ! 7s+12)
Determine i) Poles ii) Zeros iii) Characteristicequation and iv) Pole!ero"lot in s"lane
Solution
i! %oles are the roots of the equation obtained by equatin denominator to &ero i.e.
roots of'
5"s ! #$ "5!%$ "s$!(s !&#$ "'
ie. S"5 ! $!"s ! 5!"5 ! 3!"s ! )!"*
So there are 5poles located at 5 "*' - $' -5' -3 and -)
ii! (eros are the roots of the equation
obtained by equatin numerator to
&ero i.e. roots of + "5 ! #!"*
,mainary
i.e. s - - #
There is only one &ero.
ill! .haracteristic equation is one' whose
roots are the poles of the transfer )6 -5 )* )3 -$
function. So it is'
S "5 ! $!"5 ! 5!"5$ !(5 !1$! "*
Le' S"5$ !(5 !1*!"5$ ! (s! &#$ "*
i.e s5 !1)5)!(153 !15) 5$!1$*5 "*
iv! %ole-&ero plot
This is shown in the Fi. 3.1)
o
Fi. 3.1)
,s-%lanel
l + ,nl
Control System Engineering 3-16 Transfer Function and Impulse Response
'. Example 3.7: The unit impulse response of a system is given by T(t) = e: t (! cos "t#
Determine its transfer function.
Solution: Laplace transform of the impulse response is the transfer function.
T(s$ = % (T(t$&
' L {e-t (1 - cos 2t) 'L {c-t} - L {e-t cos 2tl
(s ($ (s ($
' s(1 - (5(1)2 ((2)2 'S(1 - (S2 ( "s ()$
T(s$ '** !!!'!+ *
(s+ l}(s2 +2s+5)
'. Example 3.,: Obtain the transfer function of the lead network shown in the Fig. 3.!.
-
-

!
Fig. 3"15
Solution: Ta#e Laplace transform of the net-or./ as sho$n in the Fig. 3.0.
" %

sC
Fig. 3"16
T1e parallel com&ination of l an' s( )i*es impe'ance of%

+l+2 'RI 2sc * l


+,++2 1 -1+s1!
(s!
- =
l I nl
Control System Engineering 3 -17 Transfer Function and Impulse Response
Once the parameters are replaced by corresponding Laplace domain conversions, all
the parameters R, 5L and 1/ sC behave as impedances. Hence their series and parallel
combinations can be easily obtained by the algebraic calculations.
~--~: z r------r----!
Fig. 3.17
"pplying #$L to the circuit,
%, &5' = ( 1&5' + )&s' *+ ... 1!
... &+' %o&s' = l&s' *+
)&s'
E.,s!
.
=
*+
.. ,rom &+'
-ubstituting in 1! .e get,
/ )&s' 0( + *+1 = %o&s' I(1 *+1
*+
E,s!
Eos!
E"s! /
*...
#.
-ubstituting (, 2. 3. =
*+ *+ 1+s *l C'
*l = *l + *+ &1 +s *l C'
1+s*) c+ *+
.here
s Rt *+ C+*+ s+a
/ *l + s *, *+ C+*+ =5 + 13
1 &*l +*+'
a = *l C4 $ = *l *+ C
2his circuit is also called lead compensator.
I I nl
Control System Engineering 3-18 Transfer Function and Impulse Response
II" Example 3.9: Find the transfer function of the given circuit. (M.U.: Dec.-2003)
t
Cl
R2
t
r
eo
C2
!
T
0 0
Fig 318
Solution! Draw the Laplace domain network of the gien !"!tem.
Rl #$%.. "arallel
Series
t
t
&
E#s$
sCl
t%&#s$
%
&
!
" r
sC2
I
0
'ig. 3.&9
Eo#s$
t
t
Et#s$
'
E(s$
I I
Fig 3.20
1
R( )-
*( + Rt II ,&, + !-(I
.-( R
1 )*
.-(
+
&/!R&-&
*2 + R0) /,&, +&/!R2-2
- .-2 .-2
1% #s$ - 1o(!)
((!) + #1$
1o(!) + &(2) *2 ... (2)
. 1 ( ) - 13(!)-1o(!) , *2
## 0 . - *t
r. 1o(!) 4&/ 5:6 + '! 13(!)
1o(!) *2
r. Ei#s$ + -i /*2
&/.R2-2

l I nl
Control Systems Englneerina. : I 3-19 Transfer Function and Impulse Response
n.... Example 3.10 : Find the transfer function ~~:~ o f a syst~ hIlving differential equation
given below.
2 d1 c(t) + 2 d c(tJ + c(tJ r(t) + 2r(t - 1)
d t2 dt
Solution: Taking Laplace transform of the given equation and assuming all initial
conditions zero we get,
! s2!s) +!s C"s# + !s) "!s) + ! e$ # "!s)
Laplace transform of dela$ed function is,
L %f"t $ T)% e-sT &!s)
.
.. L 'ret - ()% ) e-s*1"!s)
!"efer Ta+le 2*2)
om+ining terms of !s) and "!s) we get,
!2,2 - !s + 1) !s) "!s) !1+ 2 e-#)
!s) 1+ 2e-s

"!s) 2 S! + 2,+1
&.. Example 3.11 : Find V,,(s) / Vj(s). ssu!e gain of buffer a!"lifier as #.
1 '(
0.)*.lF
+uffer
amplifier
&ig* 3*21
. Solution: Taking ,aplace transform of the net-or./
1
-.
S#" 1#" 10
.
1#" 10 '2
1
$0
s#"0.)x10
.
1 x 10 n 1o"s#
+uffer
amplifier
&ig* 3*22
2& nl
Control System Engineering 3-2 Transfer Function and Impulse Response
explaining the effect of system parameters on input to produce output is called transfer
function. Due to the own characteristics of the system parameters, the input gets
transferred into output, once applied to the system. This is the concept of transfer function.
The exact definition of the transfer function is given in the next section.
3.3 Transfer Function
3.3.1 Definition
Mathematically it is defined as the ratio of Laplace transform of output (response) of the system
to the Laplace transform of input (excitation or driving function), under the assumption that all
initial conditions are zero.
Symbolically system can be represented as shown in the Fig. 3.2{a. !hile the transfer
function of system can be shown as in the Fig 3.2{b.
ret) --I System "##$c!t) %&s##'##( T!s) )*$#+&s
Fig. 3.2!") Fig. 3.2!#)
Transfer function of this system is ,,-,, where +&s is .aplace of c&t and %&s is
.aplace of r&t.
H T&s is the transfer fw.lction of the system then,
.aplace transform of output +&s
T&s/ 0 .aplace transform of input 0 R!s)
$.. E%ample3."- For a system shoum in the Fig. 3.3. calculate its transfer function where
V,l(t)is output and Vi(t) is input to the system.
R
f
&'(
(e
f
)
viet (o!t)
"
i&t
T
"
Fig. 3.3
Solution) !e can write for t*is system, e1uations by applying +&, as-
2i &t
0
R.i&t ,3 i&t dt
... !1)
and vo&t
/ ~I i{t dt
... &2
l 0 1nl
Control System Engineering 320 Transfer Function and Impulse Response
Let us divide the network into two parts,
Part 1) :
1
-6
Sx 1 x 10
6
1 x 10 n
Fig 323
!pp"#ing $%L,
%i &S) =
1
-s-x-"-x-"-0--'''6 (&s) +1 ) 106(&s)
&")

..
%( &S) =
(*s) =
1Xio-"&s)
%t&s)
1 X 106
&2)
++ ,10s6+106-(&S) ++,106 .ss 106- ,%110&6S)-
Su/stituting in &1) %i &s)

..
%( s! s
=
%ieS) 0. 1
Part 2) :
1
s
6
1x10 2
.
.
Fig 323
,2)(06 -
%2&S4
1
(&s) s +1 X 106
%o*s) =
(&s) 1 X 106
1&0)
%o*s)
1
106
... 1!
&2)
r
Control System Engineering - 3-21 Transfer Function and Impulse Response
Substituting in (1)
= Vo(SHL 2+SlJI06
106 s
VOs! s
V2"s! = s+ 2
Now gain of buffer amplifier is 1 unity!
VI (s) V2(s)
(
s ) (s #$!
! s #% Vi(s) = s Vo(S)
Vo"S) &$
Vi(s) =(s+ l)(s+ $!
!!! #!$!
'(( E)ample *(%$+ Determine the transfer junction if the d.c. gain is equal to %,for tile
system whose pole-zeroplot is shown below.
Imaginary
x - - - - - - - - -
I
I
I
I
o
-o---%--&'((-+(-(Real
-3 -2 -1
I
I
---------- -.
Fig( 3!2)
Solution+ From pole-*ero plot gi+en' t/e transfer fun,tion -as 3 poles at ) = -1' -2+.
an/ -2-0! 1n/ it /as one *ero at s -3!
2&+& 2&+&
#(s3 = (s+1)(s+2+.)(s+2-.) (s+1)4"s+2)2-5)2J
2(s+ 3)
=
s #%! 4)2 #0s + &!
Now /!,! gain is +alue of #()3 at s =0 w-i,- is gi+en as 10!
/!,! gain = #(s) lat &=,
I I nl
Control System Engineering 3 -22 Transfer Function and Impulse Response
10
Kx3
=
1xS
K = 5~ = 16.667
T(s)
16.667 (s + 3)
=
(5+ ) (52 + 45+ !)
This is the required transfer functin.
)"" E#ample 3"3$ If the system transfer function is
y(s) s%4
x(s) :: S 2 %25 %5
Obtain the differential equation representing the system.
S!utin"
#es)
$es)
s%&
=
!2 %2s+ 5
(S2 %2s %5) 'es) =(s % 4% )es)
s2 #es) %2s #es) %!'(!) = !)(s) % &)(s)
&e'!acin( )aria*!e 5 *# "t and #es) *# #et) and )es* *# x(t% +e (et,
d2 d d
-"+ y(t) %2-d yet) %!y(t) = -d x(t) %4x(t)
dt- t t
dx
- +4x
dt
... .ifferentia! equatin
/0 E#ample 3"&$ For the two port network shown in the Fig. 3"2,+ obtain the transfer
functions
.) V2(S) d ..) 12 (s)
1 12 ( ) an ! ! 1 1 ( )
I+
- -
v ,
1
/..
4
2F
Fig" 3"2,
. I nl
Control System Engln~nng 3-23 Transfer Function and Impulse Response
Solution: Draw the Laplace transformof the given network as shown by replacing L by
1
sL, C by sc and all time variablesby the Laplacevariables,
."..
"
11(5)
S s(s+2)
s+S+T =
s+~
J
_1_ + _1_
<
25 2(5+2)
!"# 1(s}
1$
5+1
$% oo
&'y
= s(5+2)
j
1
1
(
S) 1*
Parallel s+1
combination
++ t_ _,,,J
1
Fig. 3.26(a)
11(5}
+0...!
1
1
Fig. 3.26()
Parallel combination
___..__
+
v
(
,l-,1,*
" 2s
Parallel 1 1
combination ""+2s
1
+
s}
#
Parallel combination
Fig. 3.26(c)
r
Control System Engineering 324 Transfer Function and Impulse R.sponse

+
11(S)
1x s(s+2}
s(s+2)
(s+1)
~
1 5(5+2) S2+35+1
11(s)
+--
(5+1)
(s)
~
5+1
V2(
(s2+35+1)
+
s)
Fig. 3.26(d)
11(5) =
_::---:=:--V_I_(S_) = VI (S) = V1(5)
s(s+ 2) s+ 1 [s2 + 3s+ 1]
--:::---:---+-:---_
s2 + 3s+ 1 s2+ 3s+ 1 52 + 35+ 1
VIeS)
II(s) = 1
I ( )
(s+ 1) V ( ) (5+ 1)
= I5 X = I5X----='---
(s2 + 35+ 1) (s2 + 3s+ 1)
V2(s) s+ 1
=
VI(S) 82 + 35+ 1
.
.. V2(S)
'. Example 3.15: What will be the transferfunction oj a system whose unit step response
is a unit impulse function ? (M.U. : Dec.-2003)
Solution: The ou!u c() = "() = u#$ $%!u&se 'u#c$o#
(#) he $#!u *e) = 1 = u#$ se! 'u#c$o#
+(s) ,-"(). 1
T./. = 0(s) = ,-&. = I= S
s
1) Define the transfer function of a system.
2) ErplRin the signifiCll1lCe of II trllnsferfunction stating its advanltlges and features.
) What are the limitations of t!t"nsfer function approach#
4) $ow transfer function is related to unit impulse response of a system#
S) Define and erplRin the following terms related to the transfer function of a system.
(i) %oles (ii) 1e*os (iii) Characteristic e&uation 'iv) %ole()ero plot 'v) *rder.
+) !he unit impulse response of a system is e(,t- .ind its transfer function.
1
Ans.:--7
s+
Review Questions
\ I nl
l. II ;., r (' [ . I; . ;-1'" I
Control System Engineering 3 25 Transfer Function an Im!ulse "es!onse
7) A certain system is described by a differentilll equation
d~t~{t) #3d~~t)#l1yW = 5x(t)
where y(t) is the output and the x(t) is the input. btain the transfer function of tile system.
yes) 5
Ans.: $es% = S2 +3s+ &&
8) A certain system has its transfer function as
!is) 25#&
Rfs) = s2 #s #&
btain its differential equation.
A
. d2c(t) ddt) (l) = 2 'r(t( (t(
ns.. dt2 # dt #C dt #r
9) if a system equation is "i#en as
ddt)
3 lit#2c{t) ) r(t - T)
Where c(t) is output and rit) is input shifted by T seconds. btain its transfer function.
e sT
Ans.: 3s+ 2
10) A system when excited by unit step type of input "i#es followirl" response
c(t) = &* $e%1 #&e%31
btain its transfer function !(s)'((s) 3s2 #2s+ 3
An5.: (s+ 1) (s+ 3(
11) )eri#e the transfer function of the system shown in *i". 3.27. +he amplifier "ain is ,.
v
&
r o t ) C _ 1 _ = ~ = _ _ _ ;
+m!lifier
of
gain
K
Fi. 3.21
Eo(s( ,
Ans.: !i(5) = (& #s"tC1) (& #s"2C2)
12) +he transfer function o f a system is "i#en by
+(s) = &-(s #.(
#(# #&)(s$ #/s#25%
btain its (i) -oles (ii) .eros (iii) rder.
/0etch its pole%1ero plot. Ans. : $%les &t '( -41 -3 0 )* +er% &t -,( -rder .
r
Control System Engineering 3-26 Transfer Functloi1 ~d Impulse Response
13) Obtain the transfer /unction of the network shown in the Fig. 3.28.
O '
,
Fig. 3.28
I:o(s) RI R2CIC2s1+ R2(CI + C2) s+ t
Ans.: Ej(s) = Rl R2CICIsl + (R1cI + RZCI + R2CI)S+ 1
14) Obtain the tra~er function of the network shown in the Fig. 3.29.
--..-
R1
1
Lj,~
cJ
1
I
elt!
< -
eot!
~R"
-?
#
i!)
it!
Fig. $."%
Eos!
A".: Ej(s) =
#(l + $Is) %
1+ $IS) &'('
&' R)
RI + R2
Rl RlCI + RIR2Cl
Rt + R2
000
( I )nl
Control System Engineering 3 - 3 Transfer Function and Impulse Response
We are interested in ~~~:~where V,,(S) is Laplace of vo{t) and Vi (s) is Laplace of Vi (I)
and initial conditions are to be neglected.
So taking Laplace of aboe two e!"ations and ass"#ing initial conditions $ero we can
write,
Vi(s)
1
.., (%)
= &I(s) + sCI(s)
Vo(s)
1
.,. (')
=
sCI(s)
.
I(s)
(
)c, (s) ..
S"bstit"ting in e!"ation (%),
Vi(s) = sCVo(s)[ R+ s~*
Vi (+) = sC& Vo(s) ,Vo(s) =Vo(s)-l , sC&.
Vo(s) /
VieS) ) / +sC&
We can represent aboe s0ste# as in the 1ig. %.', which is called transfer f"nction
#odel of the s0ste#.
V,es ! Vo"s)
!
1ig. %.'
)I., #$ample %.&' Find out the T.F. of the given network.
())~2 2 2 ..3 2 C.4 ) !t) ) 1
F i g . 3 . 5
Sol"tion: 5ppl0ing we get the e!"ations as,
" )
*R (di 1I id
e, t = 1 + dt +cit
Inp"t = Ci (t) 6 7"tp"t =Co (t)
... "1)
+ I nl
Control System Engineering 3-4 Transfer Function and Impulse Response
I
F(s)
Laplace transform of F(t) dt = -5-'
df(t)
and Laplace transform of (ft=sF(s)
Take Laplace transform,
..... neglecting initial conditions
... neglecting initial conditions
Now
Ej(s)
r 11
=
I(s)l ! sL! sc"
I(s)
1
Ej(s)
=
r 1 1
L! sL! sc"
#o(t) =
$" idt
1
%o(s)
=
s# r e s)
1(5)
= s#%o(s)
... (&)
.
..
... (3)
... (')
()*stit)ting +al)e of 1(5) ine,)ation (&),
1
= r 11
L! sL! sc"
1 1
So we can represent t-e s.stem as int-e Fig. /.0.
Fig. /.0 Transfer function model
/./.& Advantages and Features of Transfer Function
T-e +ario)s feat)res of t-e transfer f)nction are,
i) It gi+es mat-ematical models of all s.stem components and -ence of t-e o+erall
s.stem. Indi+id)al anal.sis of +ario)s components is also possi*le *. t-e transfer
f)nction approac-.
il) 1s it )ses a Laplace approac-, it con+erts time domain e,)ations to simple
alge*raic e,)ations.
II nl
Control System Engineering 3-5 Transfer Function and Impulse Response
iii) It suggests operational method of expressing equations which relate output to
input.
iv) The transfer function is expressed only as a function of the complex variable
's'. It is not a function of the real variable time or any other variable that is
used as the independent variable.
v) It is the property and characteristics of the system itself. Its value is dependent
on the parameters of the system and independent of the values of inputs. In
the example 3.! if the output i.e focus of interest is selected as voltage across
resistance " rather than the voltage across capacitor # the transfer function
will be different. $o transfer function is to be obtained for a pair of input and
output and then it remains constant for any selection of input as long as
output variable is same. It helps in calculating the output for any type of input
applied to the system.
vi) %nce transfer function is &nown output response for any type of reference
input can be calculated.
vii) It helps in determining the important information about the system i.e. poles
'eros characteristic equation etc.
viii) It helps inthe stability analysis of the system.

ix) The system differential equation can be easily obtained by replacing variable IS'
hy d(dt.
x) )inding inverse the required variable can be easily expressed in the time
domain. This is much more easy than to analyse the entire system in the time
domain.
3.3.3 Disadantages
The few limitations of the transfer function approach are
i) !nly applicable to linear time invariant systems.
ii) It does not provide any information concerning the physical structure of the
system. )rom transfer function physical nature of the system whether it is
electrical mechanical thermal or hydraulic cannot be *udged.
ill) +ffects arising due to initial conditions are totally neglected. ,ence initial
conditions loose their importance.
3.3." #rocedure to Detennine the Transfer Function of a Control System
The procedure used in+x. 3.! and +x. 3.- can be generalised as below .
!) /rite -down the time domain equations for the system by introducing different
variables in the system.
II $ nl
Control System Engineering 3-6 Transfer Function and Impulse Response
2) Take the Laplace transform of the system equations assuming all initial conditions
to be zero.
3) Identify system input and output variables.
4) liminating introduced variables! get the resultant equation in terms of input and
output variables.
") Take the ratio of Laplace transform of output variable to Laplace transform of
input variable to get the transfer function model of the system.
). Example 3.3: Find out the T.F. of the given network.

#
r
r o o o o o o '
I
$
..
r
%
r!
..
&ig. 3.'
Solution: (pplying )*L +e can +rite!
e,-t)
diet) $ .
i-t) dt / i-t)% ... "#) ::
L-lt/ #
+hile eo-t)
0
i-t)% ... -2)
+here e, -t)
0
input and
eo-t)
0
output
Taking Laplace of equations -$ ) and -2)! neglecting the initial conditions.
#$ -")
, -s) 0 s1es) / # s/ %I-s) ... -3)
... -4)
2ubstituting
o-s)
$
I3s)%
,-s)
% # #
::
I3s) sL/ s#/ %& from -3)
l-s)
o-s)
$
4534 from -4) inthe above equation +e get!
,-s)
Eo"s) r # #
$ 66LsL/6/%.
% s#
l ' (nl
Control System Engineering 3-7 Transfer Function and Impulse Response
This is the required transfer function.
Key Point: The network in Ex. 3.2 and Ex. 3.3 is same but as focus of interest i.e. output is
changed, the transfer junction is changed. For a fixed output, transfer junction is constant and
independent of any type o f input applied to the system. If the output variable is changed, the
T.F. also changes accordingly.
3.4 Impulse Response and Transfer Function
The impulse function is defined as,
f(t) ! for t "
# 0 for t je 0
A unit impulse \ f~dion l)(t) can be
considered a narro pulse (of an! shape)
occurin" at #ero time such that area under the
pulse is unity and the time for hich the pulse
occurs tends to #ero. In the limit t $% 0, the
pulse reduces to a unit impulse l)(t). $onsider a
narro rectan"ular pulse of idth ! and hei"ht
Y A units, so that the area under the pulse %, as
shon in the &i". 3.'(a).
.
..
(j(s) (o(s) ) * + , -$% %% + $./
. s$
(o(s0 s.$
(j(s0 s, -$1s.$1l
2o if e "o on reducin" idth ! and
maintain the area as unity then the hei"ht Y A
ill "o on increasin". 3ltimatel! hen A -1 0,
Y A $% 00 and it results the pulse of infinite
ma"nitude. It ma! then be called an impulse of
ma"nitude unit! and it is denoted &y l)(t). It is
not possible to dra an impulse function on
paper, hence it is represented b! a 4ertical
arro at t 0 as shon in the &i". 3.'5b).
So mathematicall! unit impulse is defined
as,
l)(t) 6, t 0
0, t '0
'
....
1/A
...%---- ! ----.%%%7% t$%
&i". 3.' (a)
*
o
Sym&ol of ~t)
&i". 3.' (b)
t ++.,
nl
Control System Engineering 3-8 Transfer Function and Impulse Response
o
Symbol of K~t)
t-.
If in the above example the area under
the narrow pulse is maintained at K units
while the period of pulse is reduced, it is
called to be an impulse of magnitude 'K' and
is denoted by K B(t, as shown in the
!ig.3.8(c.
An important property of impulse
function is that. if it is multiplied by any
function and integrated then the result is the
value of the function att".
K
!ig. 3.8 !c)
"# t $"
#hus % f(t s!t) dt I f(t s(t dt % f(t s!t) dt f(t It&o$
%- %-
#his is called Isampling& property of impulse. 'ence if we define (aplace transform of
B(t as,
-
()8(t* I +(t e-,*dt ...by definition
o
e-,*It..o
e#' (
...by sampling property.
-+(t. (
#hus (aplace transform of impulse function ,(t *
/ow #(s
0(s

1(s
)) R!s)* T!s)
&
&& 0(s
So response 0(s can be determined for any input once #(s is determined.
Key +oint) The equation IC(s) I))) R(s) T(s)] is applicable only in Laplace domain and canrwt
b e used in time domain. The equation [c(t) ret) . t(t)] is not et all VQUd in time domain.
/ow consider that input be unit impulse i.e.
ret 22 l)!t) unit impulse input
R!s) , 38(t. (
&
&&
,ubstituting in above,
0(s *$ #(s 4 #(s
/ow c(t ,#l lees5 ,#l (#(s5 4 #(t
r
Control SystemEngineering 3-9 Transfer Function and Impulse Response
Thus we can say that for impulse input, impulse response C(s) equals the transfer
function T(s). So impulse response is c(t) = T(t) as C(s) =T(s) hence we can conclude that,
Key Point: Laplace transform of impulse response of a linear time invariant system is its
transferfunction with all the initial conditions assumed to be zero.
'. Example 3.4 The unit impulse response of a certain system is found
to be e!4t. Determine its transferfunction.
Solution Laplace of unit impulse response is the transfer function.
=T(s)
T(s) =
". Example 3.# The Laplace inverse of the transfer function in time domain of a certain
system is e-51 while its input is rit) =2. Determine its output cit).
Solution Let T(s) e the transfer function.
!ut
L-l "T(s)# = T(t$ = e!#t
ret$ = 2
c(t) $ ret) % T(t),
... &i'en
It is mentioned earlier that ()()=T(t) is not at all 'alid in time domain, so
c(t) ) 2e-*t
+ence the equation 'alid accordin& to the definition of transfer function must e used,
C(s)
R%s"
T(s) = L,T(t)$=&'e--t $ =s. *
T(s) =
so
/(s)
2
... as r(l) 0= 2
=
-
s
1
C(s)
.
s2*
=
..
(~)
C(s)
2 al a2
=
=!(!! ..
s(s2 *) s s2*
II ) nl
MathelTIatical Modeling of Systems
4.1 What is Mathematical Model ?
To study and examine a control system, it is necessary to have some type of
equivalent representationof the syste!!"Such a representationcan be obtained from the
mathematicalequations,governingthe behaviourof the system. Most of such mathematical
equations are differential equations whether the system may be electrical, mechanical,
thermal,hydraulicetc.
Key Point: The set of mathematical equations, describing the dynamic characteristics of a
system is called mathematical model of the system.
Obtainingthe mathematicalmodel is the first step in analysinga given system. n the
mathematical model, the various operations in the system are represented by the
mathematicalequations.
Most of the control systemscontainmechanicalor electricalor both types of elements
and components. To analyse such systems, it is necessaryto convert such systems into
mathematical models based on transfer function approach. !rom mathematical angle of
view, models of mechanicaland electricalcomponentsare exactlyanalogousto eachother.
"ot only this, but we#can show that for given mechanical system there is always an
analogouselectricalnetwor$ exists and vice versa. The mathematicalequations describing
both the systemsare exactlysamein nature.
%s we are well familiar with the behaviour of electrical networ$s and methods of
writingequations for it, it will be better if we can draw equivalent electricalnetwor$s for
given mechanical systems. This will help us inwriting system equations inSimplified
manner and with more detailedunderstanding.
This chapter explains the concept of analogous networ$s, method of writing
differential equations for various physical systems and derives the transfer functionsof
various commonlyused control systems.
(4 - 1)
II\ nl
-co Control System Engineering 4-10 Mathematical Modeling of Systems
Torque equation of side 1 is,
T-J d20t(t) B d01(t) 'E()
- t dt2 + 1 dt + t t
... (I)
Torque equation of side 2 is,
... (2)
Now
TI Nt _ 02
T2
=
N2 -~
T2
N2
TJ
=
Nt
u!stitutin" in equation (2)

..

..
11 = N# J., d202 + N# B$ d02 + Nt T%
N2 - dt2 N2 - dt N&
u!stitutin" 'a#ue of Tt inequation (1)
... (()
u!stitutin" )2 =
d201
*. T = It dt2 ++I
:. T
JIe
=
Equi'a#ent inertia referred to ,ri-ar. side
J#e
=
11 +(~~r 12
B#e
=
Equi'a#ent fri/tion referred to ,ri-ar. side
.
.
and
B#e
= BI + (~~r B2
T
d201 d)# (Nt)
=
#ie (It2 + Bit, 0it + N2 n
l I nl
Control System Engineering, 4-11 Mathematical Modeling of Systems
Similarly the equation can be written referred to load side also, where applied torque
gets transferred to load as (~: T )
(~: ) T
(
! " ) #
where 1 2 . ( . ' = 1# + N ; $ % and
4. . 2 . !elt or Chain "ri#es
&elt and chain dri'es perform same function as that of gear train (ssuming that there
is no slippage between belt and pulleys we can write,
T ) -o $%%%$&'
r,
*ig 4 1+
for such dri'e
4 + , (e#ers
)he le'er system is shown in the
*ig 4 1- T his transmits translational
motion and forces, similar to gear
trains
&y law of moment,
* fl 1 1 . f# h
&y wor/ done fl 0% = 1# 0#
*ig 4 1-
2ence
4. + Electrical Systems
Similar to the mechanical systems, 'ery commonly ,sed systems are of electrical type
T he beha'iour of such systems is go'erned by 3hm4s law T he dominant elements of an
electrical system are,
l * nl
Control System Engineering
~tt 4-12 Mathematical Modeling of Systems
i) Resistor ii) Inductor iii) Capacitor
i) Resistor : Consider a resistance carrying
current 'I' as shown, then the voltage drop
across it can be written as,
Suppose it carries a current 11 - 12) then
!or the polarity o! the voltage drop shown its
e"uation is,
Ir--V-=-( -11---1-2) -R --'1
ii# Inductor : Consider an inductor
carrying current $I$ as shown, then the voltage
drop across it can be written as,
% = & dl
dt
I = ~' % dt
or
I
o - - - - ~ . - - - - - ~ - - - - - - - - - - ~ o
+ R -
+------ V -----------+
(11
12-
o ~ - - - - - - - - - - ~ - - - - - - - - - ~ o
+ R -
------ V -----------+
)ig* 4*1+
L
$, - . ooo$

o o
!"------ V --#'#------- !!!
$ig! 4*1/
! If it carries a current 11- 12) then !or the polarity shown its voltage e"uation is %
% 0 & d11d~12)
or
11 -&') = (I % dt
iii) Capacitor : Consider a capacitor
carrying current 'I' as shown, then the voltage
drop across it can be written as,
or
V 0 ~ Iidt
I = C d%
dt
If it carries a current 11 -I') then !or the
polarity shown its voltage e"uation is,
or
V 0 (f 11 -1)) dt
II -12) = C dV
dt
o
(11
12(
+ -
' o O O O b O o '
o
l
-------- V -------+
)ig* 4*21
C 1
~ o - - - - - - - - - - ~ I I ~ - - - - - - ~ - - - - ~ o
+----------V -------+
)ig* 4*21
(11
12-- c
o - - - - - + ~ I ~ I - - - - - - - - - - ~ o
------- V ------+
)ig* 4*22
l * +nl
Control System Engineering 4-13 Mathematical Modeling of Systems
4.8 Analogous Systems
In between electrical and mechanical systems there exists a fixed analogy and their
exists a similarity between - their equilibrium equations. Due to this, it is possible to draw
an electrical system which will behave exactly similar to the given mechanical system, this
is called electrical analogous of given mechanical system and vice versa. It is always
advantageous to obtain electrical analogous of the given mechanical system as we are well
familiar with the methods of analysing electrical networ than mechanical systems.
!here are two methods of obtaining electrical analogous networs, namely
1) "orce - #oltage $nalogy i.e. Direct $nalogy.
%& "orce - 'urrent $nalogy i.e. Inverse $nalogy.
4.8.1 Mechanical System
~x(t)
'onsider simple mechanical system as
shown in the "ig. 4.%3.
Due to the applied force, mass ( will
displace by an amount x)t& in the direction
of the force f)t& as shown in the "ig. 4.%3.
$ccording to *ewton+s law of motion,
applied force will cause displacement x)t&
in spring, acceleration to mass ( against
frictional force having constant ,
"ig. 4.%3
.
f)t&
= (a ,v !x)t& ..
-here, a = $cceleration, v . #elocity
f)t&
( d% x)t& d x)t& /
.. = 0 "#d# x)t&
dt- t
!aing 1aplace, $ %(s) = (s% &es) ,s 2es& /2)s&
!his is equilibrium equation for the given system.
*ow we 'ill try to derive analogous electrical networ.
4.8.( %orce )oltage Analogy (*oo+ Analysis)
$ n this method, to the force in mechanical
system, voltage is assumed to be analogous one.
$ccordingly we 'ill try to derive other analogous
terms. 'onsider electric networ as shown in the
"ig. 4.%4.
"ig. 4.%4
r
Control System Engineering 414 Mathematical Modeling of Systems
The equation according to Kirchhoffs law can be written as,
diet) 1J .
vet) = let) R + L dt+C l(t)dt
Taing La!lace,
"es) # $(s) R+ Ls $(s) + $% &
'ut we cannot co(!are )(s) and "es) unless we *ring the( into sa(e for(.
)or this we will use current as rate of flow of charge.
.
i(t)
#
dq
..
(it
i. e. $(s)
=
s+(s) or
+(s) = $(s)
s
Re!lacing ina*ove equation,
I ,(s) # Ls-+(.) + Rs +(s) + ~ +(s) I
Co(!aring equations for )(s) and /es) it is clear that,
i) $nductance 0L0 is analogous to (ass 1
ill Resistance 0R0 is analogous to friction '.
iii) Reci!rocal of ca!acitor i. e, llC is analogous to s!ring of constant K.
Translational Rotational Electrical
orce Tor!"e T #oltage #
Mass M I nertia $ I nd"ctance L
riction constant % Tortional friction Resistance R
constant %
S&ring constant K N/m Tortional s&ring constant Reci&rocal of
K 'mlrad ca&acitor 1/C
(is&lacement )K 2 Charge !
*
d+
e# 3 # co
C"rrent I * d!
#elocity + # df
dt
Table ,.- Tab"lar form of force./oltage analogy
nl
Control System Engineering 4-15 Mathematical Modeling of Systems
4.8.3 Force Current Analogy (Node Analysis)
In this method, current is treated as
analogous quantity to force in the mechanical
system. So force shown is replaced by a current
source in the system shown in the Fig. 4.25.
The equation according to irchhoff!s
current law for abo"e system is,
I # $% $& Ie
%et node "oltage be ',
1 f ' d'
. I ! " #dt""C"
% & dt
I t
Fig. 4.25
Ta(ing %aplace,
yes) yes)
$*s) ! ""sC#(s)
sL R
+ut to get this equation in the similar form as that of F*s) we will use,
d41
$(t) # where 41 # flu,
#(s) # s %&(s)
-!*5) # #(s)
i.e. 'I(
s
Substituting in equation for $*s)
1 1
$*s) # .S2 /*s) & s /*s) % /*s)
.omparing equations for F*s) and l*s) it is clear that,
i) .apacitor (C( is analogous to mass 0.
ii) &eciprocal of resistance / is analogous to frictional constant +.
iii) &eciprocal of inductance / is analogous to spring constant ).
*ranslational +otationa. Electrical
F Force * Current I
00all 1 .
, friction , 1/R
) S-ring ) 1/L
. dis-lacement 2
/
0 . d. 0 d1
#oltage (e( # de)2
. #elocity # - 1!"!(3)
dt dt
dt
*a4le 4.3 *a4ular form of force"current analogy
Ie
c
5 I nl
Control System Engineering 4-16 Mathematical Modeling of Systems
Key Point: The elements which are in series in F - Vanalogy, get connected in parallel in
F - I analogous network and which are in parallel in F - Vanalogy, get connected in series in
F- 1analogous network.
4.9 Steps to Solve Prolems on !nalogo"s Systems
Step 1: Identify all the displacements due to the applied force. The elements spring and
friction between two moving surfaces cause change in displacement.
Step : !raw the e"uivalent mechanical system based on node basis. 11:.eelements under
same displacement will get connected in parallel under that node. #ach
displacement is represented by separate node. #lement causing change in
displacement $cither friction or spring% is always betwcen the two nodes.
Step &: 'rite the e"uilibrium e"uations. (t each node algebraic sum of all the forces
acting at the node is )ero.
Step 4: #n F-*analogy+ use following replacements and rewrite e"uations+
F,*+ -,.+
/
0, 1+ 2, l/C, 3 , "+ 3 , i $current%
Step 4: Simulate the e"uations using loop method. 5umber of displacements e"ual to
number of loop currents.
Step 6: #n F-6 analogy+ use foll7'ingreplacements and rewrite e"uations+
F ,I+
/
- , 8+ 0, I91+ 2, Il.+ 3 , 8%+ 3:e$e.m.f.%
Step ;: Simulate the e"uations using node basis. 5umber of displacements e"<811 to
number of node voltages. Infact the system will be e3actly same as e"uivalent
mechanical system obtained in step with appropriate replacements.
$$.. E%ample 4.1: Draw the equivalent mechanical system o f the given system. Hence write
the set o equili!rium equations or it and o!tain electrical analogous circuits using,
i) F-V "nalogy and ii# F-$ "nalogy
&1
f't(
# # nl
Control System Engineering 4-17 Mathematical Modeling of Systems
Solution: The displacement of Ml is Xl (t)
and as Bl is between M1 and fixed support
hence it is also under the influence of Xl (t).
While B chan!es the displacement from Xl(t)
to X(t) as it is between two mo"in! points.
#nd M and K are under the displacement
X(t) as K is between mass and fixed support.
:EF = 0
$ F
Equivalent system
F = M% s Xl + Bl s Xl + B &(Xl - X)
o ' M 5 X + $ X + B &(X- Xl)
#t node 1(
#t node (
... (1)
!!! "#
)ow (i) * - + #nalo!, M - . B - / $ - l/C $ % q
+(s)
'
.l s 0l + /l &0l + / &(0l - 0) ... {3j
1
'
. s0 2 "l&q0 2 / &(0 - 0t) ... (4)
l(s)
3(s) i.e( 1(4) ' 4 3(s) =
s
+(s)
= .l s5'"s# 2 /5ll (4) 2 / ())(4) - h(s)6 - .oop '
/eplacin!
)umber of loop currents e0ual to number
of displacements.
7ence(
C) /(
*oo+ ' *oo+
8
"'&K#
"))# F ,) -nalogy
* .+5 M - 8 B .+ l/R K.+ 1 1 L
5(s)
' '
' 8l & 91+ /l & : 1+ / & (-l - -)
1
;<;;
- &(.-9l)28&c=)2-. -
/eplacin! 4 4> (4) =
+(s)
,!! "'#
... ()
r
Control System Engineering 4-18 Mathematical Modeling of Systems
I(s)
Vl(S) 1
= CISV1(S)+~+R2 [Vl(S)-V2(S)]
1 1
o = R2 [V2(S)- Vl(s)]+ C2 s V2(S)+ sL V2(S)
...Node 1
...Node 2
Hence,
Nume! o" node #ol$%&es e'u%l $o nume!
L(1/K) o" d(s)l%cemen$s.
'.. Example 4.! Draw the equivalent mechanical system and analogous systems based on
F-V and F-J methods for the given system.
E"#i$alent system
Sol#tion! %&o d(s)l%cemen$s * No elemen$ unde! +l (t) %lone %s "o!ce (s d(!ec$l,
%))l(ed $o % s)!(n& -l. So ($ .(ll s$o!e ene!&, %nd /ence is $/e c%use $o c/%n&e $/e "o!ce
%))l(ed $o 02. Hence d(s)l%cemen$ o" 02 is +2 %nd %s 12 %nd -2 %!e connec$ed $o "(2ed
su))o!$s o$/ %!e unde! +2($) onl, %s s/o.n (n $/e e'u(#%len$ s,s$em.
3$ node 1,
3$ node 2,
... (1)
..'()
02 I 82 , -2 %!e unde! s%me d(s)l%cemen$.
( ) *nl
Control System Engineering 4-19 Mathematical Modeling of Systems
(i) F-Vanalogy: M ~ L B -4R
1
V = C1(ql - q2)
O
1 ) ., 1
= C1 (q2 - qI +L2 s- q2 + C2 q2 +R2 sQ2
I~
C2
(11K2)
~ Sam !is"la#mn$ sam #%&&n$.
'~l(C
... (3) Loo" (1)
... ()) Loo" (2)
*q%a$ions in $&ms o+ II an! 12 #an , -&i$$n ,y %sing i($) = ~~ L. l(s) :: s Q(s) as
."lain! a&li&.
(ii) F - I analogy : M ~
l(s)
C, B~ 1/R,
1
= 1/(~l - q,2)
K~ llL
... (1) 0o! (1)
... (2) 0o! (1)
*q%a$ions interms o+ VI(s) an! V2(s) #an be o,$ain! by %sing the &la$ion,
!)1
1$) = dt
i.. 2s) = s q,(s) as ."lain! a&li&.
v ,
Sam !is"la#mn$-sam 1ol$ag.
II nl
Control System Engineering 4-2 Mathematical Modeling of Systems
4.2 Analysis of Mechanical Systems
In mechanical systems, motion can be of different types i.e. Translational, Rotational or
combination of both. The equations governing such motion in mechanical systems are
often directly or indirectly governed by Newton's laws of motion
4.2.1 Translational Motion
Consider a mechanical system in which motion is taking place along a straight line.
Such systems are of translational type. These systems are characterised by displacement,
linear velocity and linear acceleration.
Key oint! According to Newton's law o f motion, sum o f forces applied on rigid bo dy or
system must be equal to sum of forces consumed to produce displacement, velocity and
accelerationin various elements o f the system.
The following elements are dominantly involved in the analysis of translational
motion systems.
i" #ass ii" Spring iii" $riction
4.2.2 Mass (M)
%&&& f(t) orce
This is the property of the system itself
!"t) #is$lacement which stores the kinetic energy of the
translational motion. #ass has no power to
store the potential energy. It is measured in
kilograms 'kg". The displacement of mass
always takes place in the direction of the
applied force results in inertial force. This force
is always proportional to the acceleration
produced in mass '#" by the applied force.
$ig. (.%
Consider a mass '#' as shown in the $ig. (.%having )ero friction with surface, shown
by rollers.
The applied force f't" produces displacement *'t" in the direction of the applied force
f't". $orce required for the same is proportional to acceleration.
.
..
. d+*'t"
f't" , M! acceleration , M +
dt
Taking -aplace and neglecting initial conditions we can write,
I f's" % #S+ .'s" I
/lso mass cannot store potential energy so there cannot be conswnption of force in the
mass e.g. if two masses are directly connected to each other as shown in the $ig. (.+ and if
force f't" is applied to mass #, then mass #+ &ill also displace by same amount as #l0
I
Control System Engineering 4-20 Mathematical Modeling of Systems
4.10 Servomotors
The servosystem is one inwhich the output is some mechanical variable like position,
velocity or acceleration. Such systems are generally automatic control systems which. work
on the error signals. The error signals are amplified to drive the motors used insuch.
systems. These motors used inservosystems are called servomotors. These motors arc
usually coupled to the output shaft i.e, load through gear train for power matching.
These motors are used to convert electrical signal applied, into the angular velocity or
movement of shaft.
4.10.1 Requirements of Good Servomotor
The servomotors which are designed for use infeedback control systems must have
following requirements :
i) Linear relationship between electrical control signal and the rotor speed over a
wide range.
ii) Inertia of rotor should be as low as possible. servomotor must stop runt).ing!
without any time delay, if control signal to it is removed. "or low inertia, it is
designed with large length to diameter ratio, for rotors. #ompared to its frame
si$e, the rotor of a servomotor has very small diameter.
iii) Its response should be as fast as possible. "or quickly changing error signals, it
must react with good response. This is achieved by keeping torque to weight ratio
high.
iv) Itshould be easily reversible.
v) Itshould have linear torque % speed characteristics.
vi) Its operation should be stable without any oscillations or overshoots.
4.11 y!es of Servomotors
The servomotors are basically classified depending upon the nature of the electric
supply to be used for its operation.
The types of servomotors are as shown inthe following chart :
"
A.C.Servomotors
Servomotors
t ~
Special Servomotors D.C.Servomotors
#
Armature controlled Field controlled
nl
Control System Engineering 4-21 Mathematical Modeling of Systems
'~
4.12 D.C. Servomotor
Basically d.c. servomotor is more or less same as normal d.c.motor. There are some
minor differences between the two. All d.c. servomotors are essentially separately excited
type. This ensures linear torque-speed characteristics.
The control of d.c, servomotor can be from field side or from armature side.
Depending upon this, these are classified as field controlled d.c, servomotor and armature
controlled d.c. servomotor.
4.12.1 Field Controlled D.C. Servomotor
In this motor, the controlled signal obtained from the servoamplifier is applied to the
field winding. ith the help of constant current source, the armature current is maintained
constant. The arrangement is shown in the !ig. ".#$.
la (Constant
!f(t from
servoam"lifier
#
Constant
c$rrent
....%%---------o so$rce
r-'''--%
Fig. 4.2& Field controlled d.c. servomotor
TIlls type of motor has large &e IRe ratio where &f is reactance and 'f is resistance
of field winding. Due to this the time constant of the motor is high. This means it can not
give rapid response to the quic( changing control signals hence this is uncommon in
practice.
4.12.1.1 Feat$res of Fie'd Controlled D.C. Servomotor
%t has following features )
i* +referred for small rated motors.
ii* It has large time constant
iii* %t is open loop system. This means any change in output has no effect on the
input.
iv* ,ontrol circuit is simple to design.
4.12.2 (nnat$re Controlled D.C. Servomotor
In this type of motor, the input voltage -.a- is applied to the armature with a
resistance of 'a and inductance &a. The field winding is supplied with constant current
Ie. Thus armature input voltage controls the motor shaft output. )he arrangement is shown
inthe !ig. ".#/.
r
Control System Engineering "'~I . 4-22 Mathe~atical Modeling of Systems
(Constant)
Constantr
current
source 0--------' __ ""'-t
Fig. 4.2 !rmature controlled d.c. ser"omotor
The constant field can #e supplied with the help of parmanent magnets. In such case
no field coils are necessary.
4.$2.2.$ Features of !rmature Controlled %.C. Ser"omotor
It has following features:
i) uita!le for large rated motors.
il) It has small time constant hence its response is fast to the control signal.
iii) It is closed loop system.
i") The !ac# e.m.f. pro"ides internal damping which ma#es motor operation more
&
sta!le.
") The efficiency and o"erall performance is !etter than field controlled motor.
$s the armature controlled d.c. ser"omotor is closed loop system% in comparison with
open loop field controlled system% generally armature controlled motors are used.
4.$2.'Characteristics of %.C. Ser"omotors
The characteristics of d.c. ser"omotors are mainly similar to the tor&ue-speed
characteristics of a.c% ser"omotor. The characteristics are shown in the 'ig. 4.2(.
(or)ue
(*m)
+a ) !mlature "oltage
*a4, *a+ , *a2 , *a,
*a2-........
+a /Ea, ---%"%
o S0eed (r.0.m.)
Fig. 4.21 (or)ue-s0eed characteristics for an annature controlled d.c. ser"omotor
2 I 'nl
Control System Engineering .. c ,
423 Mathematical Modeling of Systems
4.12.4 Applications of D.C. Servomotor
These are widely used in air craft control systems, electromechanical actuators, process
controllers, robotics, machine tools etc.
4.13 Transfer !nction of ield Controlled D.C. Motor
"
Ass!mptions #
+
Jm $ %nertia
Bm = riction
ig. 4.29
&1' onstant armature current is fed into the motor.
!2" (f cc##. $lu% produced is proportional to field current.
% (f $ &f #f %
&3' Tor'ue is proportional to product of flu% and armature current.
% Tm oc (#a %
Tm $ &) (#a = )* &f #f r+
Tm = &m Kill %
... &1'
*here &m $ &)#a $ onstant
+pply &irchhoff)s law to field circuit.
dif
,f dt + -f #f $ ef
... !2"
,
.ow shaft tor'ue Tm is used for dri/in0 load a0ainst the inertia and frictional tor'ue.
#nertia force
T = , d 21 m " B d2m
).3 m 4 tt2 m tit
... &3'
J
d25m imil t d2-
$ m662673 ar om d)8
dt t6
.+ nl
, Control System Engineering , 4-24 Mathematical Modeling of Systems
,.,
F
' I ' B dOm 'I B dx
nctiona rorce = m ""(it smu ar to dt
Finding Laplace Transforms of equations (1), (2) and (3) we get
Tm!s" = #m # If!s"
$f!s" = !sl% + &f) If!s"
Tm!s"
= ' ms2am!("+Bmsa m!s"
$liminate I!s" from equations !4" and !)"
Tm!s" =
#m #f $f!s*
!(Lf + Rf)
$liminate Tm!s" from equations !+" and !,"
!s2 ' m + sBm"a m!("
#m #f $f!s"
=
!sl% + &"
Input = $f!("
Output =
&otational displacement Om!s"
-% Transfer function
+m(s)
=
E(s)
ames"
#m #f
$f!s"
=
a m(2+ sBm* !&f + sLf)
=
#m #f
s&f Bm 1+ s'tml 1+ ('tf.
!here /tm
0 "# =1otor time constant
'tf = "# =Field time constant
T%F% = Om!(" =
x
#m
1
&rll +('tf"
%2
Ef(s)
Bm!l + s'tm" (
%%% !4"
%%% !)"
%%% !+"
%%% !,"
Bloc3 diagram for field controlled d%c% motor is as s4own in Fig% 4%56%
Et(s)
"
$ t
I7("
$ m
(%)m(s)
1
&
R,(1 + st,' (m(1 + (tm'
(
...))
Fig% 4%56 Bloc3 diagram
, %
- .1 Control System Engineering .~......,~.~. 4 - 25 Mathematical Modeling of Systems,. .~.);
4.14 Transfer !nction of "rmat!re Controlled #.C. Motor
"ss!m$tions %
(i) Flux is directly proportional to current through field winding,
& C j )m ' Kr I r 'C onstant &
(ii) or!ue produced is proportional to product of flux and arm ature current.
' K"m l#)Ia
$ K"m x, If r,
(iii) %ac& e.m .f. is directly proportional to shaft '(locity ()m , as flux ( is constant.
d()t)
as
rom
$
crt
E*
$
K* )l)m)s) $ K* s+ m,s)
+ t
+ a L a
- - la
Constant
-
Fig. ,.-1
.pply Kirchhoff"s law to annature circuit /
+ 0
dia
ea ' 1* -Ia ( a) - a dt
a&e 0aplace transform ,
2ow
1a(s) ' 1*(3) - Ia (s) ./, - 3 0ei4
l,(s)
1, (s) - 1p(s)
'
/01 -3 0a
1-(3)
1,(3) - K* s5m(s)
$
+ , -3 0a
m
$
K"m Kf If In
Tm
$
K" K I 61a - x, sam es)7
m f f / -s2
a a
& & nl
Control System Engineering 426 Mathematical Modeling of Systems
Also ... from equation (3)
Equating equations of T I t\ J
K' m Kf If Ea(s} K'm Kf If Kb sames) 0 2 )a ()
= + m! "s m m!
(#$ + ! %a) (#$ + ! %a )
K'm K( If
&K'ml'(l$Kt$s 2 )
..
(#a +! %a) E.$(s}
*
(#$ " + %a) " ,ms +!m ames)
Km
.
am (s) s #$ m -I +S'tm}(. +s 't'a) G(s)
=
*
..
Ell (s) KmsKb
. +/(s)0(s)
."
s #.. m(. " s 't'm)(l + s 't'a)
12ere r.$ =.m3Em an4
%a
't'a =-
#a
Km = K ' m Kf
G(s) =
Km
!#.$ m(. " !'t'm}-l + !'ta)
0(s) = !Kb
52erefore 6an be re7resente4 in its blo68 4iagram form as in 9ig. 4.32.
1
fig. 4.32 lo684iagram
Ke: ;oint< Field controlled d.c. motor is open loop while armature controlled is closed loop
system. Hence armature controlled d.c. motors are prejnTed over field controlled type.
4..+ A.C. Se!omoto
=ost of the ser>omotors use4 in ,o1 7o1er ser>ome62anisms are a.6$ ser>omotors.
T he a.6. ser>omotor is basi6all: t1o 72ase in4u6tion motor. 52e out7ut 7o1er of a$6$
ser>omotor >aries from fra6tion of 1att to fe1 2un4re4 1atts. 52e o7erating frequen6: is
+0 0? to 400 0?.
" I
Control System Engineering 4 -27 Mathematical Modeling of Systems
The stator carries two windings,
uniformly distributed and displaced by 90, in
space. One winding is called main winding
or fixed winding or reference winding. This
is excited by a constant voltage a.c. supply.
The other winding is called control winding.
It is excited by variable control voltage, which
is obtained from a scrvoamplifier. This
voltage is 900 out of phase with respect to the voltage applied to the reference winding.
This is necessary to obtain rotating magnetic field. The schematic stator is shown in the
Fig4..
4.15.1 Construction
Control
winding
Fig. 4. Stator of !.C. ser"omotor
4.15.2 #otor
It is mainly divided into two parts
namely stator and rotor.
The rotor is generally of two types. The one is usual s!uirrel cage rotor. This has small
diameter and large length. "luminium conductors are used to #eep weight small. $ts
resistance is very high to #eep tor!ue%speed characteristics as linear as possible. "ir gap is
#ept very small which reduces magnetising current. This cage type of rotor is shown with
s#ewed bars in the Fig. 4.4 &a'. The other type of rotor is drag cup type. There are two
air gaps in such construction. (uch a construction reduces inertia considerably and hence
!luminium$ars
Shaft
&a' (!uirrel cage rotor
)*tator
%$& 'ragcu(ty(e rotor
Fig. 4.4
such type of rotor is used in very low power applications. The aluminium is used for the
cup construction. The construction is shown inthe Fig. 4.4 +b,.
4.-.. Tor!ue%speed Characteristics
The tor!ue%speed characteristics of a two phase induction motor, mainly depends on
the ratio of reactance to resistance. For small / to 0 ratio i.e. high resistance -0110 reactance
r
Control System Engineering 4-28 Mathematical Modeling of Systems
large ~
Torque
(Nrn)
Control voltage
E24>E23>E22>E2l
t
X
Small R
- Speed
Speed (rpm)
Fig. 4.35 Fig. 4.36
motor, the characteristics is much more linear while it is nonlinear for large X to Rratio as
shown in the Fig. 4.35.
In ractice, !esign of the motor is so as to get almost linear tor"ue-see!
characteristics. #he Fig. 4.36 shows the tor"ue-see! characteristics for $arious control
$oltages. #he tor"ue $aries almost linearl% with see!. &ll the characteristics are e"uall%
sace! for e"ual increments of control $oltage. !t is generall% oerate! with low see!s.
"#$" %eatures of & C Servomotor
#he a.c, ser$omotor has following features '
i( )ight in weight. ii( Ro*ust construction. iii( Relia*le an! sta*le oeration.
i$( +mooth an! noise free oeration. $( )arge tor"ue to weight ratio. $i( )arge R to X
ratio i.e, small X to R ratio. $ii( ,o *rushes or sli rings hence maintenance free. $iii(
+imle !ri$ing circuits.
"#$$&pplications
-ue to the a*o$e features it is wi!el% use! in instrument ser$omechanisms, remote
ositioning !e$ices, rocess control s%stems, self *alancing recor!ers, comuters, trac.ing
an! gui!ance s%stems, ro*otics, machine tools etc.
"#$' Transfer %unction of &C Servomotor
#he $arious aro/imations to !eri$e transfer function are,
0i( & ser$omotor rarel% oerates at high see!s. 1ence for a gi$en $alue of control
$oltage, # oc' , characteristics are erfectl% linear.
0ii( !n or!er that # ee , characteristics are !irectl% roortional to $oltage alie! to
its control hase, we assume # oc , characteristics are straight lines an! e"uall%
sace!.
#or"ue at an% see! 2,3 is,
nl
Control System Engineering 4-29 Mathematical Modeling of Systems
T. K E
dem
m = tm 2t + mdt"
where, d!; is speed of motor.
If load consists inertia 1 m and friction Bm we can write,
Tm(s) = 1m s26m + Bm s6m
Now Laplace transform of equation (1) is
Tm{s) Ktm!2(s) + m s6m(s)
!quatin" equations #2$ and #%$
(1)
(!)
(")
# Ktm!2 (s) + m s6m (s) =$m s2ames)+ %ms&mes)
6m(s) Ktm
=
!2#&$ s#s $m' m+ %m)
(
s$m)
s(%m' m) 1 + #Bm ' m)
am (s)
Km
=
!2 (s) s (1+ *tms)
Km
Ktm
=
%m'm
*tm =
1m
%m'm
where
and
2
& (m &9m
E2~O------ ---~~~---'_---~~~.--~09m
Kl$m 1/s 1+s
tm
(m'%m)
$m
)i". 4.%* &i"nal flow"raph of a.c.
ser,omotor
Key -oint # As slope is negative, in the
above equation +Bm - m) shows that total
friction increases due to m .s it adds more
friction, the damping improoes, improoing
stability of'the motor. This is called Internal
Electric Damping of 2 ph A.C. servomotor.
!2(s)
, tm
1 1
9m#-$
'
$m s s
%m'm
----
$m
/ig 0"1 %loc2 diagram of ac ser,omotor
34 nl
Control System Engineering 4-3 Mathematical Modeling of Systems
Due to mass, there cannot be any change in force from one mass to other hence no
change in displacement.
Rigid
connection
(No friction and no elastic action)
Fig. 4.2
Key Point: The displacement of rigidly connected masses is always same.
4..3 !inear S"ring
In actual mechanical system there may be an actual spring or indication of spring
action because of elastic cable or a belt. Now spring has a property to store the potential
energy. The force required to cause the displacement is proportional to the net
displacement in the spring. All springs are basically nonlinear in nature but for small
deformations their behaiours can #e
appro!imated as linear one. "ence
assuming linear spring constant $K$ for the
spring, we can write equation for the
%-&ft'mrcrnif$-----f(t) spring in the system.
#onsider a spring haing negligible
mass and connected to a rigid support. Its
spring constant be $K$ as shown in the
Fig. 4.$.
Fig. 4.$
i
%& !,(t)
. Force required
displacement !'t( in the
proportional to displacement.
( f't( ). K !'t(
(
to cause
spring is
Fig. 4.4
Now consider the spring connected
between the two moing elements haing
masses )l and )2 as shown in the Fig. 4.4
where force is applied to mass )I.
nl
Control System Engineering 4-30 Mathematical Modeling of Systems
Signal flow graph for A.C. servomotor is as shown in the Fig. 4.37. Hence block
diagram of A.C. servomotor is as shown in Fig. 4.3
4.16 Comparison of Servomotors
4.16.1Comparison between A.C. and D.C. Servomotor
Sr. A.C. Servomotor D.C. Servomotor
No.
1
!ow power o"tp"t of abo"t Deliver high power o"tp"t
1
! " w to #00 $.
#
Efficiency is less abo"t $ to #% %. &'gh efficiency.
(
D"e to absence of comm"tator )re*"ent maintenance re*"ired d"e to
maintenance is less. comm"tator.
4
Stability problems are less+ More problems of stability.
$
No radio fre*"ency noise ,r"shes prod"ce radio fre*"ency noise.
6
-elatively stable and smooth operation. Noisy operation.
7)
A.C. amplifiers "sed have no drift. Amplifiers "sed have a drift.
4.16.# Comparison between Armat"re Controlled and
)ield Controlled D.C. Servomotors
Sr. )ield Controlled Armat"re Controlled
No.
1 D"e to low power re*"irement amplifiers &igh power amplifiers are re*"ired to
are simple to design. design.
2)
Control voltage is applied to the field. Control voltage is applied to the armat"re
3&
.ime constant is large. .ime constant is small.
4
.his is open loop system. .his is closed loop system.
5)
Armat"re c"rrent is /ept constant. )ield c"rrent is /ept constant.
'&
0oor efficiency. ,etter efficiency.
7)
S"itable for small rated motors S"itable for large rated motors.
1
Costly as field coils are m"st 0ermanent magnet can be "sed instead
of field coils which ma/es the motor less
e2pensive.
4.13Models of Commonly "sed Electromechanical Systems
'n this article the transfer f(nctions of ver) commonl) (sed s)stems are derived. *his
will help the reader to find o(t the transfer f(nctions of the different practical s)stems.
+ ' 4nl
Control System Engineering 4 -31 Mathematical Modeling of Systems
4.11.1 Generators
Consider a separately excited generator
which is many times used in various practical
mechanical systems. Generators are required to
drive the motors because vacuum diodes,
transistor amplifiers are not suitable because of
their low ratings. Consider a generator as shown
in the figure.
f
I '
....--o
!ig. 4.3" Generator
#f = !ield resistance
Lf = !ield inductance
ef = $pplied voltage %input&
eg = Generated voltage %output&
'ow for a generator,
where, =flux
Flux is directly proportional to current passing through the field winding say if.
(et ), be the generator constant in *+ $
... (1)
$pplying )irchhoff,s La to field circuit.
. # ( dif
ef = -f f ! f dt
... %.&
/a0ing (aplace of both the equations %l& and %.&
Eg(s) 1 x, -f%s&
2r%s& 1 #, -f3s& ! (r3s& --%s& neglecting -r%4&
-,%s& 1
)a Er"s)
#f !5Lr
Eg(s) =
.
..
2g%s&
1
Er"s) #, !s (,
/his is the /.!. of separately excited generator.
r
Control System Engineering 4-32 Mathematical Modeling of Systems
4.17.2 Generator Driving Motor
. It is very common to find generator driving motor in practical mechanical systems. So .
let us discuss the T.F. of a system with generator driving motor.
+
Fig. 4.40 Generator driving motor
R, and f Resistance and inductance of generator field
eg !enerated voltage
" g - !enerator constant in !" #
R, and a - Resistance and inductance of motor armature
# - M.$. of oad and f is frictional force
$ow. T. F. of generator is,
Eg%s& ' " g
Ef%s& (f +S )f
$ow consider armature controlled motor,
Tor%ue produced &y motor is dependent on ia and let " T &e tor%ue constant.
T * " T ia
This tor%ue is utilised to drive a load.
+ * ,d2e + f de
dt2 dt
$ow 'g is voltage applied to armature.
.
..
p( * i R + dia + e-
o..g a a a)it
where e& is &ac* e.m# .
nl
Control System Engineering 4-33 Mathematical Modeling of Systems
Let Kb be the back e.m.f, constant
,
K
J d20 + f dO
T ta = dt2 dt '
Taking Laplace transform
KT I, (5) = J 52 e (s) + f s (5)
J !"#. $ R L di , K dO
~ = I... a + ... dt+ '" dt
... (3)
Taking Laplace transform
%g(5) = la(s) n, &L.l sla(s)&KI!' s((s) ... ())
finding * + *(5) from e,-ation (.)
(J s2 +fs) ((s/
I., (s) =
Kr
0-bstit-ting in e,-ation ())
sO (5) U 5 +f)
1g(5) = K (2, +0 La) + Kb s( (5)
T $
%g(0) = + (s) s [ (f +3#0) c : : +5 La) + Kh 3
%g(s) 4 0 Kb e (s) = 5(2a + 5 La) (s J +f) + (5)
KT
+ (5)
E g (s) - 0 Kh ((0)
=
0(2, +5 L# ) (5 J +f)
1
sJ+f f--04'__- 5(5)
6ig. ).)*
...
!r
Control System Engineering 4-34 Mathematical Modeling of Systems
Therefore using both the transfer functions of generator and armature controlled motor
we can develop the combined block diagram as below :
1
sJ +f ~,__ ....8(s}
Fig. 4.42
Reducing the block diagram, solving feedback loop of motor which is
T
=
.
..
!"#$
=
Ef(s
"Rf + s %t&'s ("Ra + # %a $ (J s+ f + b T ) *
!.1".# $osition Control System
+nother ver, common s,stem used in practice is position control s,stem. This is used
to control position of shaft, b, use of potentiometer as error detector. The error is to be
amplified b, amplifier and then must be given to armature controlled motor whose shaft
position will get controlled as per the controlled signal. The motor shaft is coupled to the
load through gearing arrangement with ratio -t.-2.
%oad has /.l. 0 and friction as f while ! r is the reference position while 1n is the
actual position of shaft.
The circuit diagram can be drawn as below :
L a
e =error
~
2 % ,const. )
---3ear
~
-2
&ig. !.!# $osition control system
_, nl
Control System Engineering 4-35 Mathematical Modeling of Systems
Let Kp be the potentiometer sensitivity, in V /rad
The corresponding block diagram can be drawn as shown in the Fig 444
Ores)
1
sJ+f
Fig. 4.44
!ed"cing the block diagram as shown in thc Fig 445
fl,(s)
#$s% &
Fig 445
K Kp KT $'l/'(%
K Kr KT $')/'(% = Ks = *ystem gain constant
+$s% = 1
r. ,ver all T F can be calc"lated as below,
O,(s)
&
,r-s%
s .-!a +* La% s +f) + Kb Kr %
)/ Ks
s .$!a +* La +0s + 1% +Kb Kr 2
,a $s% 3 Ks
OrCs) ! s .$!a +* La% s +f) +Kb KT 2 +K,
4.1".4 #osition Control $ith Field Controlled Motor
%n the above case if instead o1 armat"re controlled motor, 1ield controlled motor is
"sed then derive the T4 o1 overall dosed loop system
-onsider 1ield controlled motor,
L
5 di1
c( = i1 &f + f dt
'ow T e6 7 ia and ia is constant
T oc 8 , oc i1
r
Control System Engineering 4 .. 36 Mathematical Mod.Ung of Systems
ia = constant
Fig. 4.46
Let Kf be constant in N-m/ A
T = K, if
This torque is utilised to drie a load of moment of inertia ! and friction f.

.
d"# de
T = ! dt" $ f %&t
Kf if = ! d"e + f de
dt" dt

.
Finding La'lace of all the equations,
(f)s*
=
&f+s* ,f + # Lf &f)s*
and -, &f)s*
=
! s" .)s*+s f.)s*

If(s)
# .)s* /s + f*
. =
Kf
#u0stituting in (f)s*
(f)s*
s .)s* /s ! + f* /,, +s L,1
=
Kf

6)s*
Kf
.-- =
.
Ef(s) s /s ! + f2,i +s If I
No3 using the same 0loc4 diagram as deried in case )ill* re'lacing armature
controlled motor T. F. 05 the field controlled 3e can get the ne3 0loc4 diagram.
s (s + f) ),, +s!f)
"ig. 4.46
nl
Control System Engineering 4-37 Mathematical Modeling of Systems
Let
G(s)
Ks
- s (s J +f) CRt +s Lf) H(s) = 1
Ks
G(s) S(5 J + f) (R + sLd
--=--,--=--------
!+G(s)"(s) !+ K,
S(5J +f)(Rf +5 L()
Oa (5)
=
OreS) S(5 J +f) (R, +s Lf ) +K,
4#!7#$ S%eed Control System
&ns~me of the practical applicatios it is ecessar! to "ri#e a loa" at a "esire" spee"
(1) ra"$sec% &or this applicatio a electromechaical s!stem ca 'e (se" )hich is* sho) as
'elo)% %
' Constant
Ra~
\1\I\.r\,--. .... -_,
(achometer
%&i+% ,%,- Spee" control system
S!stem (ses armat(re cotrolle" motor a" tachometer fee"'ac.% Let K/ 'e the
amplifier +ai% Let )s "eri#e the trasfer f(ctio of this s!stem% 0he o'1ecti#e of s!stem is
to mo#e the loa" at a "esire" spee"%
(he "%c, tachometer o(tp(t #olta+e is proportioal to o(tp(t spee" (1)%
e, is referece #olta+e a" et is tachometer #olta+e%
error
%~%(5)
%%% (2)
et = K, (1) )here K, is costat%
3e+lecti+ i"(ctace of armat(re
l * +nl
Control System Engineering ;. 4-38 Mathematical Modeling of Systems
eb = Kb (1)
ca
=
ia Ra + Kb ro
.
Ka (e, - et)
=
ia Ra + Kb (1)
-. (ol
K, e, - K,
K,(1)
=
ia R, + Kb (1)
substituting from equation (5),
substituting from equation (6),
Taking Laplace
K.. Er(s) :: (K 1 K, + Kb ) roes)+ i.l (s) R,
!o" torque pro#uce# b$ motor
%& (')
T oc la
... (()
!o" t)is #ri*es a loa#
.
..
dO
")ere ro=
#t
'" (9)
+. Taking Laplace
KT ia (s) = Js roes)+ f (1)(5)
.) U s, f] f ()
1 1(5 = KT + (-) .
... (10)
.ubstituting inequation (7)
/s,f0(1)(s)
= (K, K, + Kb ) (1)(5) + KT R,
(K 1 K. + Kb ) (5) / s + f] roe.) n,
Er{s) = + -~------
K 1 Ka KT
m(s)
=
Er(s)
%T)is is t)erequire# transfer function of t)espee# control s$stem.
n
Control System Engineering I 4-39 Mathematical Modeling of Systems
4.17.6Speed Control using Generator Driving Motor
A position control system described in Fig. 4.49 in which the armature of motor is
applied with a control voltage through a generator. The field current of generator controls
the voltage generated by the generator.
e1(l
!
Fig. 4.49 "peed control system
#et us determine the transfer function of the system
$nput is e% (t.
&ow
d ( O )
cc't ( ( )l t!" *+ dt
... (11
Ta,ing #aplace transform
Ecs! # El s! " $% -. s!
&ow / ( * $
die !0
1c t2 3 # &f dt ' $ $f
... 1(!
applying *irchhoff0s law to the field circuit of generator.
&ow C.1 # *+ if where *4 is constant.
Ta,ing #aplace transform
/a s! ( 54 $f s!
Ta,ing #aplace transform of 1(!
/c(s *3 # $f (s 6!f 's)I *
... 1+!
/liminating $f (7
/c(" *3
14(7 #
(!e 's #r
/. s!
ic4 /c(s *3
(
(!4 's #f!
E..s! *a *3
#
Ec,s! -r 's)f!
r
Control System Engineering
" .
4-4 Mathematical Modelingof Systems
Now mass Ml will get displaced by
XI(t) but mass M2 will get displaced by
X2(t)as spring of constant K will store
some potential energy and will be the
cause for change in displacement.
onsider free body diagram of spring as
shown in the !ig. 4.".
Net displacement in the spring is Xt(t)- X2(t) and opposing force by the spring is
proportional to the net displacement i.e. XI(t) - #2(l).
Fig. 4. 5 Spring between two moving points
cases change in displacement
$a%ing &aplace'
!sprin() ! K* XI(l) - X2(t)+
!o)prinr. , K*XI (s) " X2(-)+
#ey $oint% The spring between the moving points causes a change ill displacement from one
point to another.
-pring beha.es e#actly same in rotational systems' only the linear spring constant
becomes torsional spring constant but denoted as /K/ only.
4. &. 4 Friction
0hene.er there is a motion' there e#ists a friction. !riction may be between mo.ing
element and fi#ed support or between two mo.ing surfaces. !riction is also nonlinear in
nature. 't can be di.ided into three types'
i) 1iscous friction ii) -tatic friction
(ash"potl
. . """)''*""n)2 p2r
+,+
!ig. 4.3
!ig. 4.4
iii) oulomb friction
1iscous friction as dominant out of the
three is generally considered' neglecting other
two types. 1iscous friction is assumed to be
linear' with frictional constant /5/. $his has
linear relationship with relati.e .elocity
between two mo.ing surfaces.
$he friction is generally shown by a
dash-pot or a damper as shown in the !ig. 4.3.
$his is the symbolic representation of a
friction.
onsider a mass M as shown in the !ig. 4.4
ha.ing friction with a support with a constant /6/
represented by a dash-pot.
!riction will oppose the motion of mass M
and opposing force is proportional to .elocity of
mass M.
'' - nl
Control System Engineering ~~! 1_- !i-. 4 - 40 Mathemabcal Modeling of Systems
Now consider field constant armature controlled motor. Armature is energized by
output of the generator.
Applying Kirchhoffs law to armature circuit!
.
"# (s) $% "&'() +*a (s) +,a +s -# .
/ac0 e.m.f is proportional to e m
"&'() $ K d1m
bdt
2a0ing -aplace 2ransform
"t3's4 = K& s5m 's)
Inarmature controlled d.c! motor as field current is constant so tor6ue produced is
directly proportional to the armature current.
2 = K *# 's) where K is motor constant.
.2his tor6ue is utilized to dri7e the load with moment of inertia 8 and friction 9m:
T = 8 d;0m +9. d5m 2a0ing -aplace transform from
dt; m dt
K*!!'s) = 8 s;5m's)<s9m5m=s)
fa '>) = ~O s +9m)5m. 's)
(u&stituting ine6uation f?r "..'s)
"a's)
1mes) K
$
"a=s) s KK& < se,a < s -a)'8 s< 9m)
Km
=
s+l<2ms)
... Neglecting L. of armature
K
Km $ +KK& +,a 9m1 $ @otor constant
I
'T' a! 8 T' f
1m = 'KK , F . ) $% ime constant 0 motor
s< a m A
Neglecting -a of armature motor!
1m's) _ Km
"a 's) -- s '1 +2m s)
.
..
l I
f ~ Control System Engineering' nl,Jl!ttC(I
4 -41 Mathematical Modeling of Systems
Ea{s)
K" K2
=
Ec(s) [ R e + s Ld
0
E... (s)
x, K2 Ec(s) ames) s(1 +Tms)
.0
=
:::
(R, + SLf )
Km
and Ec(s) = El (s) Kb sam
[
K x, K2 x! s"
= am (#) s (1+ Tms) + (Rr + SLf )
= $ (#) [R% S(1+ Tm s) (1+Tr s) + Kb K" K2 K& s"
m !1+ Tf "s)#
ames)
=
E'(S)
K
()$%$ *! = R! +$n$%at% ,nstant.
$.1%.% & Ty'ical (osition Control System )sed in *nd)stry
- .s/t/n ,nt%l 0$$dba,1 s2st$& )as a .t$nt/&$t$% b%/d+$ (/t) a s$ns/t/3/t2 0
K. 45%ad/an as $%%% d$t$,t%. It 0$$ds a d,.a&.l/0/$% +ith an .$n ,/%,6/t +a/n K.7)/s
S6..l/$s t a 0/$ld 0 +$n$%at% ()/,) )as %$s/stan,$ R0 and /nd6,tan,$ L0 8$n$%at%
,nstant /s K+45 - .7)/s /s ,nn$,t$d t an a%&at6%$ ,nt%ll$d d,.&t% (/t) &t%
t%96$ ,nstant K7 and ba,1 $.&.0 ,nstant Kb 45%ad5s$,. *t d%/3$s a lad 0 /n$%t/a J and
0%/,t/n 0.
:%a( the S/&.l/0/$d bl,1 d/a+%a& and )$n,$ ,al,6lat$ /ts t%ans0$% 06n,t/n.
;r
*~ Consti
R. ~ m
$
</+.4.#0Ty'ical 'osition control system
r
Control System Engineering 4-42 Mathematical Modeling of Systems
e r =Reference position , e a =Actual position
T. F of generator
Eg (s) _ Kg
Ec(s) - Rf +SLf
Block diagram can be drawn from te anal!sis of te different cases discussed.
Fig. 4."#
$eglecting te inductance of armature winding of motor.
Fig. 4."2
G(s) =
%, K ~ &T
'(s) = #
s (R, +s*f) +Ra s + f) + &b &T !
*et
T.F =
"#! $ %#C# Motor &osition Control System
,n industr! to conrol te position of te saft, a d.c. motor position control s!stem is
commonl! used.
'' nl
Control System Engineering 443 Mathematical Modeling of Systems
4.18.1Transfer Function of D.C. Motor Position Control System
Consider the D.C. position control system which is controlling position of the shaft.
Assume that the input and output of the system are the input shaft position and output
shaft position respectively.
Assume following system constants,
r = Angular displacement of the reference input shaft
c = Angular displacement of the output shaft
e = Angular displacement of the d.c. motor shaft used
K} = Gain of potentiometric error detector
Kp == Amplifier gain
eol
=
Applied armature voltage
e
= =
!ac" e.m.f.
#ol =
Armature winding resistance
$a =
Armature winding inductance
ia = Armature winding current
K
=
!ac" e.m.f, constant
K
=
%otor tor&ue constant
m
= %oment of inertia of motor.
m
=
'iscous friction coefficient of motor
I l
=
%oment of inertia of load
b L = 'iscous friction coefficient of load
n
=
Gear ratio (t)(*
!constant
+ig. 4.,3 D.C. %otor position control system
r
Control System Engineering 4-44 Mathematical Modeling of Systems
Equations describing above system can be written as follows :
Output shaft position is to be controlled so as to keep that at required position. Output
is sensed by angular displacement 'c' and compared with input which is r. Error is
amplified by amplifier with gain 'Kp' and given as input to the d.c, motor which in tum
controls the angular position of the shaft of the motor 'e' which in tum controls output
position of shaft 'c' so as to modify the error.
For potentiometric error detector we can write.
E(s) = l [R(s) - C(s)]
For amplifier
E, (s) = Kp E(s)
For armature controlled d.c. motor
1 1 = constant so flu! q , is constant
T
=
!t, whcre K = motor torque constant
"b
ee
e
.
"b =
K de
..
b i l i
For armature circuit
#
di, . $
"b + ,% dt+la a
&aking #aplace transforms
Eb(') = b () (s)
s,(() = s, (s) +*a (s)+$a +'#a 1
& = *a (s)
,ow torque is utilised to drive load + shaft of motor.
-eq
= - rn +n . -# = Equivalent moment of inertia
beq = bm +n . b/, = Equivalent frictional coefficient
&
d.' d)
= *a (s) = -eq 0. +beq d
dt t
= +-c.tq '. +beq'%)(s) #aplace transform
r,(s)
=
1 2 eq '. +bt.3qs%)(s)
Ea4s)
)(()
= b s) (() + 2 eq'. +bcqs5+$a +' #a )
nl
Control System Engineering 4-45 Mathematical Modeling of Systems
6(s)
K
=
Ea (s) s [J eqS + bcq ](Ra + sola 1 + KKb S
K
::;:
'La' is generally small hence neglected.
K
=
=
s (1 + Tm s)
K
Km = (bl.'qRa + KKb) = otor constant
R! J. . - q .
Tm ::;: 0 R KK) ::;:otor time constant
cq a + b
"o# C(s)
::;:
n 6 (s)
C(s) 6(s)
Ea(s)
=
n$$
Ea(s)
C(s) nKm
=
a!(s) s(l + Tm s)
C(s} nKm
#here Ea (s) = Kp E(s)
=
KpE(s} s(l + Tm s)
C{s)
=
n Km %! E(s)
s(1 + Tm &)
C{s) =
n Kill K' K( (R(s) - C(s)]
s()+Tnt s)
n Km Kl' K! n Km K' K! R(s)
:. C(s) + (1 T. .)$ C(s) = $$. (1 T )
s + m* * + m*
+
*()+ Tm &) + n Km K' Kl] n Km K' Kl R(s)
:.,(*) &()+ Tm *) = s()+Tm *)
C(s)
=
R(s)
nKmK'Kl
s(l + Tms)
nKmK'Kt
1 +~~=- - - :-
&()+ Tms-
.(s)
= $$/$$
)0,(s)1(s)
_ nl
Control System Engineering 4-46 Mathematical Modeling of Systems
Block diagram :
R(s)
Fig. 4.54
'-. Example 4.3: A d.c. motor drives a pointer which is spring loaded, to return to the
referenceposition. If Kb =Back e.m.f. constant, KT = Torque constant J < . ~ =Spring constant
and J =Moment of inertia. Find the transferJunction.
1
f o- - - - _ _ _ . I
(M. U. : Nov. - 93, Nov. - 94)
Soltion: ri!i"g !#$ %&%!$m $'(a!io"%.
)a(!)
a!,
= I a *, + +a crt+%,
,aki"g +a-lac$,
)a(%) = ./(0) +I ii (%) 1*a +0 +a2
... (1 )
.
r,(3)
)a(0) - ./(0)
.... (2)
=
..
(*, +%+a)
No4 Tm = 5, r. .
Tm ::: Mo!or !or'($
... (3)
a"d ./(0) 5/ %$$%)
as %,
d6
. . . (4)
=
oc -
d!
r,(%)
)a(%)- 6(%)
..
=
(*, + 0 +a)
a"d Tm{s)
=
5, 1),(0) - 5/06(0)2
. . (3)
. (*a + 0 +a)
, I
Control System Engineering 4-47 Mathematical Modeling of Systems
This torque is used to drive a pointer with inertia J and spring load of constant Ks.
d20{t)
Tm :: dt2 + K:! "#t)
Ta$ing %aplace
Tm{s) = 52 O(s) + &' 6(s)
(quating #)) and #*)
(6)
:. s2 "{s)+ &' 6(5)
= !T , - a#./-K0.1{.)2
3' + sl.'
4 !T !"s#(s) !T
)- ##))+ !$#(s)+ (% & ) 5 #3 & ) v,(s)
6'+. a .'+s .7
8 * ) , s2 #3a +. %a) + K9#3a + s%a) + KT K0.2 6 Kr .
'' (s #3 % ) - #3 % ) - : (s)
a+) a .'+s a
So transfer function is'
0#)) Kl

1#s)
)6 (&ample4.4: A d.c. generator supplies its output to a separately excited d.c. motor. The
field current of the motor is constant. The voltage applied to the field circuit of d.c. generator
is e/ (f). Write the differential equations of this Ward-Leonardsystem relating the input ef (I)
to the output *((( (f). (Refer to figure shoum elo!" #ence ootain expressionfor *((( ($".
#;.<.-;a=-1*, >T?@2 and April-1*#(lectrical2)
!
A )enerator
%a
*
Motor if Constant
C
+
r
e m
&, ,( -ard,&eonard system
Sol*tion. Torque produced "y motor'
T = !TBa#t)
(+ )
- oltage applied to the motor
ar'
(g#t) = r/ 3a +% dt+ E"(t)
... #2)
r
Control System Engineering 448 Mathematical Modeling of Systems
Back emf is proportional to the angular velocity
:. Eb{t) = x,dad~(t)
... (3)
Torque produced T is used to drive a load of inertia J and friction B
T = J d2a m +B dem
.. dt2 dt
... (4)
quating (1) and (4)
... (5)
!u"stituting in equation (2) #e can get resultant differential equation $relating g(t) and
am(t).
%o# f(t) '" (6)
and g(t) ... (&)
. !u"stituting If(t) = ~t) in equation (6) #e can get' equation relating Ef(t) and
g(t).
!u"stituting this in equation (2) #e can o"tain the final differential equation relating
t(t) and emet). It is very difficult to o"tain in time domain so let us o"tain it in (aplace
domain.
Taking (aplace transform of all the equations
T(s)
=
)T I,(*)
g(!)
= + , + (s)-.a / 5 (a0/ "(!)
"(!)
=
)" !em(s)
T (s)
= emes)-J s2 + B!0
.
)T r,(*)
=
ame!)-J s/ B0 *
.
1a (s)
2333
ames4s-J s/ B0
.
)T
g(s)
em(s) s-J s + B0-.a / ! (a0
.
= )T / )" sames)
and f(s)
=
1f(s) -.' + s (t0
... (1)
... (2)
... (3)
... (4)
... (5)
... (6)
... (&)
Control System Englneenng 4 ..49 Mathematical Modeling of Systems
So
Eg(s)
= K g If(s)
IE(S)
=
Eg(s)
~
Ef(s) =
EgCs)[Rf +sLt]
.(8)
K g
Substituting Eg(s) in equation (8) we get
Ef(s) K g = ames) [sas +B)~~a +S La +K bS]
[Rf +s Ltl
.
..
(R, +S Lf) {s (Ra +S La) Us +B) +SK bK !
K g K Ef(s)
4.19 Models of Thermal Systems
4.19.1 Heat Transfer System
, - - - - - - - I I I
Outlet w""# : r - - - " " "
o
$% C
I& I
Sur r ounding tem. 8 .
! "#er mal insulation
I
'()= ))* # I nlet : , ater
$% C
+ate'
&eater
Fig. 4.55
, t-e'.al s/ste. use0 fo' -eating flow of wate' is s-own below(
Ele1t'i1 -eating ele.ent is 2'o3i0e0 in t-e tan4 to -eat t-e wate'( -e tan4 is
.sulate0 to 'e0u1e -eat to t-e su''oun0ings(
-e ne1essa'/ si.2lif/ing assu.2tions a'e )
5) -e'e is no -eat sto'age int-e insulation(
6) ,ll t-e wate' in t-e tan4 is 2e'fe1tl/ .i7e0 an0 -en1e at a unifo'. te.2e'atu'e(
r
Control System Engineering 4-5 Mathematical Modeling of Systems
.
..
d x(t)
FfrictJonal = B~
Taking Laplace and neglecting initial conditions,
I Ffrictiollill (s) = Bs (s) I
Similarly if friction is between two moving surfaces, it is shown in the Fig. 4.8.
!ig. 4." !riction #et$een t$o mo%ing &oints ca'ses change in dis&lacement
I n such a case, opposing force is given by,
~ . . = ! d "t#t$ % & "'#t$(
)rlctional dt dt
Taking Laplace,
I F*rictional (s) = Bs ()(s) - "'#S$(
Thus if the force applied to mass +' is f#t$ then due to friction between the masses +l
and +', the force getting transmitted to +l is always less than f#t$. ,ence the
displacement of mass +l is different than the displacement of mass M'-
*ey +oint, The friction between two moving points, causes a change in displacement from
one point to other.
Frictional force also behaves e.actly in same manner, in rotational systems, only linear
frictional constant becomes torsional frictional constant but denoted by same symbol -5- only.
4.. /otational Motion
0his is the motion about a fi.ed a.is. I n such systems, the force gets replaced by a
moment about the fi.ed a.is i.e. #force . distance from fixed a.is$ which is called Tor/ue.
So e.tension of 0ewton1s law states that the sum of the tor/ues applied to a rigid
body or a system must be e/ual to sum of the tor/ues consumed by the different elements
of the system in order to produce angular displacement #e$, angular velocity #ro$ and
angular acceleration #n$ in them. As previously stated, spring and friction behaves in same
manner in rotational systems. 0he property of system which stores kinetic energy in
12 nl
Control System Engineering 4-50 Mathematical Modeling of Systems
6 i = Inlet water temperature in "C,
00 = Outlet water temperature in"C.
o = Surrounding temperature.
q = Rate of heat flow from heating clement inJ Isec.
qi = Rate of heat flow to the water.
qt = Rate of heat flow through tank insulation.
C = hermal capacit! inJ le.
R = Resistance of thermal insulation.
So rate of heat flow for the water in tank is,
d60
ql = Cdt
... (1)
he rate of heat flow from the water to the surrounding atmosphere through
insulation is,
... "#$
%s per the heat transfer principles,
&= &'(&,
Su)stituting e&uation (1) and (2)
C
d6" 00*6
q = ***crt + R
... (3)
... "+$
,eglecting the term aIR from the e&uation "+$ this is )ecause the -ariation of water
temperature a 0 is o-er and a)o-e am)ient temperature 0(I).
. & = C dOo + 00
dt R
aking .aplace transform,
60 (s)
/"s$ = Cs60"s$(*,r*
0. ransfer function is,
60(s) R
=
/"s$ 1+sCR
he time constant of the s!stem is Re.
nl
Control System Engineering 451 Mathematical Modeling of Systems
4.19.2 Thermometer
Consider a thermometer placed in a water
bath having temperature eJ I as shown.
eo is the temperature indicated by the
thermometer. The rate of heat flow into the
thermometer through its wall is,
dq
=
cit
here ! = Thermal
resistance of the thermometer wall.
The indicated temperature, rises at a rate of
d"o # 1 d$
%it &Cdt
where C is thermal capacity of the thermometer.
deo = ~. '"i &(()
dt C R
Ta*ing +aplace of the e$uation,
1
5,0s! = RC ',.%5-&.%1%5-)
1
=
1" s RC
The time constant is RC.
4.20#ct$ators
%ath
/ig. 4.5,
The actuator is a device which receives input signal from the controller and it
produces the input signal to the plant according to control signal so that the output will
approach the reference input signal to reduce the error to 0ero. Thus an actuator is
generally after the controller and before the plant in the control system. #n actuator can be
of t&o types 1
i! 2ydraulic actuator 3- 4neumatic actuator
'( nl
Control System Engin nng 452 Mathematical ModeUngof Systems
4.20.1 Hydraulic Actuator
The structure of an hydraulic actuator is shown in the Fig. 4.57.
Inut!!"!!!!!"!"!!!!!!"!"!!!!!!"!"!#f!$
dislacement
Main iston
%o sum &rom high
ressure source
%osum
'al(e iston
Fig. 4.57
They are piston devices in which motion of the spool regulates the oil flow to either
side of the power cylinder. When the spool moves to the right, the high pressure oil enters
the power cylinder to the left of the piston.
The differential pressure produced causes the power piston to move to the right,
pushing the oil infront of it into the sump.
The load coupled rigidly to the piston moves a distance y from its reference position
corresponding to the displacement x of the valve piston from its neutral position. The oil is
pressurised y a pump and is recalculated in the system.
E)uation of motion and transfer function
The rate of flow of oil into the piston is proportiona! to the rate of the movement of
the piston.
*
" A dy A " area of piston
dt#
... $l%
If & is the differential pressure across the piston then the force on the piston is '&.
%his moves the load of mass ( against friction ).
d2 y dy
' + & " ( dt2 ") dt
... $2%
For small values of the displacement *, if & is the differential pressure on the piston
and * is the oil flow then,
..,-.
dy
+l +!A$
dt
Control System Engineering 4-53 Mathematical Modeling of Systems
Taking Laplace transform,
K.,M K2 B
A52 Y (s) +--p;- SY(s) = K !eS)- A"es) s
#es) $s2 K2 M% K2 B5 +& 2 5' = K & !es)
"es) ( K)&
!es) - s2 K2 M +s K2 B +& 2 5
"es)
Xes)
(~)
!"
=
T*is transfer f+nction is similar to t*e electric motors.
Taking Laplace transform,
Apes) = Ms2 yes) +as Yes)+KY(s)
"es) A
=
pes) Ms2 +#s+K
$!2%!2 &ne'matic Act'ator
,ne+matic acting -al-e is +se. to
o/tain linear .isplacement of a pl+nger
0it* press+rise. air as inp+t.
T*e air at press+re , is in1ecte.
t*ro+g* inlet. ,ress+rise. air p+s*es t*e
.iap*ragm an. pl+nger. T*e pl+nger *as
a mass M an. friction on B 0it* spring
constant K.
Let A/e t*e area of .iap*ragm t*en
transfer f+nction can /e o/taine. as
/elo0.
2orce e3erte. on t*e pl+nger is & 3 ,.
(his force is oppose. )y mass, friction
an. spring.
d2y dy
&3, = M *+#*+Ky
.t2 .t
press+re
2ig. 4.54
(he a.-antages of pne+matic s"stems are fire proof, e3plosion proof, simplicit" an.
eas" to maintain.
+ , nl
Control System Engineering .. 4-54 Mathematical Modeling of Systems
4.20.3 Comparison between Pnematic and !ydralic Systems
Sr.No. Pnematic systems !ydralic systems
1. "he flid sed is air. "he flid sed is oil.
2. #ir is compressible. $il is incompressible.
3. #ir does not ha%e lbricating property. $il acts as a lbricator.
4 . "he otpt power is mch &esscompared to "he otpt power is mch higher than
hydralic. pnematic.
S . #t low %elocities' the accracy is poor. #t all %elocities' the accracy is satisfactory.
6. (o retrn pipes are re)ired when air is sed. "he retrn pipes are mst.
7 . Can be operated for the temperatre range of *t is sensiti%e to the temperatre changes and
00Cto 200$C.*t is insensiti%e to temperatre the range is 20$Cto +0$C.
changes.
8. "hese are fire proof and e,plosion proof. "hese are not fire and e,plosion proof.
9 . Easy from maintenance point of %iew. -ifficlt from maintenance point of %iew.
Note: The state space method of modeling the systems is separately co%ered in the
chapter 15.
E,amples with Soltions
E,ample 4.5. The voltage generated b y a d.c. generator is filtered b y a low pass filter
consisting R : z and L2 as shown below. This filtered voltage is controlled b y the voltage
applied to the field of generator.
If Rf=40!l, Lf=60H, Ro=O.!, Lo=lH, [.2=lH,R2=2Q and
generator constant Kg = 120 V / field "#p. $eter#ine the transfer f%nction
&'()
*+(s) of the s,ste#.
(M.U.: De.!"#$
r - - - - - - - - -
/
e,(t$ $ 0'
1o!!!%&&&&&,
_________ J
lo'pass filter
Constant speed
Control System Engineering 4-55 Mathematical Modeling of Systems
Solution: For field circuit,
dit
Cf(t) = ij(t) Rf + Lf --
dt
... (1)
For armature circuit, ( )
. () R . ( ) R ( d i2 L d i., (2)
',1 I= 12 t c l ' 12 t '2 ~ ',. -cit' + : It ...
For gencrator, Ca (I) = K g if (1)
' " (3)
... ()
!a"ing La#lace tran$form of all t%e e&uation$,
'f($) = It($) l R, +$ Lt)
'* ($) = 12(s) +(R.. + R2) +$ (La + L2))
a, (,) = ~ If{s)
... (5)
... (6)
... (-)
... (8) '2(.) = l//0($) R2
'&uation (1) and (-),
K g It($) = %($) l(Rd +R2) + $(L* + L/d)
.u2$tituting I a ($) from e&uation (3) and If (s) from e&uation (,),
.u2$tituting 12(s) from e&uation (3) and Id$) from (,),
.u2$tituting t%e 4alue$,
~ - - - - - - - - - - - - - - - - - - - - - - - - - - - - - - - - - - - - - - ~
120x2
2.
(0 +60s) !(2.5+2s))
=
(1 +1.,$) (1 +0.8s)
,,... Example 4.6: A higJrgain speed control system uses a tachogeneratorfor speed. sensing.
!%e iachogenerator produces , V per 155 r.p.m. This voltage is compared with reference
voltage to produce error signal. If the reference voltage is set to 10.8 volt. What is the talue
of expected speed? (6.7. / 8o4.-9)
Solution: !%e tac%ogenerator #roduce$ , : #er 155 r.#.m.,
It$ con$tant i$
5
155 = 5.5, :Ir.#.rn.
r
Control System Engineering "1. 4-56 Mathematical Modeling of Systems
.1
Now for the expected speed, error should be zero,
i.e VI - tachogenerator output = error
Tachogenerator output = Vr
Let N be expected speed in r.p..
.
.. N x !.!5 = 1!."
N = #16 r.p.. ... $xpected speed
". Example 4.7 %n armature controlled d.c. motor has an armature resistance of 0.37 n.
The moment of inertia is !." # 10-6 kg - 1112. $ back emf of 2.09 V is generated per 1%%
r.p.m. of the motor speed. The torque constant of the motor is 0.2 -m &$. !etermine tire
transferfunction o f the motor relating the motor shaft shift and the input "oltage.
&'.(. ) *ec.-+6,
Sol'tion The otor can (e shown as,
) n* nl
1
Vaet, ,
1 i.+I,
Now bac- ef is #.!+ V per 1!! r.p.. , N =1!! r.p.. is,
#1t N
+I, . ---- . 1!.4/1+ rad0sec
). 1ac- ef constant 2b . # $ % $ 9. !.1++5 V 0rad0sec
2T
.
!.# N-0%
3or arature circuit Va&t, =4a 5, . $to
Ta-ing Laplace Va &s, . 16 &7, 5, . $l8&s,
$b&t, =
2 d+
b d t
Ta-ing Laplace $b&7,
.
2b7+ +s,
Tm
.
2r4a9t,
... +1,
... &#,
... +/,
nl
Control System Engineering 4-57 Mathematical Modeling of Systems
Taking Laplace l".n(s) = Kr 1 .1 (s)
This torque drives a load of inertia J .
dze
= J dt2
Taking Laplace Tm(s)
=
J 52 (s)

Kl !a (s)
=
J s2 B (s)
.
fa (s)
2.5"1 #$
52 e (s) = 1 .25"1 #$552 e (5)

=
..
#.2
%.!(s) =
1 .25" &2 1 #$5e (&) "#.'( + K) & (5)
.

%a(s)
=
*.25"1 #$tl &2 (s) + #.1 ++5&(5)
.
.
(5)
1 21 21 .22
..
%ol(s)
=
=
*.25,l#$fi 52 + #.1 ++5s &(5+ *'1 '5.1 '5)
.......E I 48 I I bId . E~(s)
n...,... xamp e . : For t te system 5toum e ow etermine E~ ( ).
~fs
Separately excited d.c.generator
Solution:
-or field circuit.
... (*)
(/.0. 1 2ec.$+()
-or annature circuit.
-or generator. ea (l) = Kg if3t)
e2(t) = i2(t) 42 -or output.
, I nl
Control System Engineering 4-58 Mathematical Modeling of Systems
Taking Laplace of all the equations we get,
Ef (s) = If (S)[Rf + s Lrl
Ea(s) = 1 (!) [(Ra + R) + s(La + L")# = Kg I$s)
E(S) = h(s) R
%ence equating Ea (s) equations,
& g I$s) = 1(s) [I" (!) (Ra + R) +!(La + L)#
an$ hence using 'alues of If(s) an$ 1(s) fro( re(aining equations we get,
".. Eam!le 4."# $ 5% &'( ) phase a.c. servomotor has the following parameters :
Starting torque = 0.186 Nm
Rotor inertia =1)l*+S kg=m?
Supply voltage = 120 V
No load angular velocity = 304 rad/s
Assuming straight line torque-speed characteristics of the motor and ero friction! o"tain its
transfer function. (*ate)
Solution, The starting torque is nothing -ut locke$rotor torque.
Locke$rotor torque / *.101
Kt( = ---
Rate$ 'oltage 1*
.
..
= 1.!!)l2+3
Let ( -e the slope of linearise$ torque+spee$ characteristics.
Locke$ rotor torque *.101
4o loa$angular spee$ = 3*5
( =
= - +.,,8l--5
The torque at an6 angular spee$ rois gi'en -6,
. de
T( = & t( El +m $t
... (,)
r
Control System Engineering 4-59 Mathematical Modeling of Systems
where [2t = rotor voltage
Taking Laplace, Tm(s) = Ktm E2t (s) + m sO(s)
This torque is used to drive load of inertia J m The friction is given !ero"
""" (2)
t,
d2e ,
Tm
.,
I =
# m dt2

Tm(s)
=
s2Jm$(s) ..
.,.(3)
""" (%)
Equating (2) and (%),
&tmE2t (s) + m s$(') = s2J m( (')

.
&im E2t(s) = sO(s)(sJm-m]
)ence the transfer function of the motor is,
8(s)
E2t(s)
=
# "''X # *3
=
'[# +l$' s(,"# # (+l*%)-
E2t(s) s(s.,# "# ()
$(s)
# ''
=
This is the required transfer function"
1 _ Example 4.1 0: two phase a.c. servomotor haoing a torque constant of D.lUS Nm/V
controls a position load tI,rough a gear ratio of 10:1. The effective moment of inertia and
coefficient of viscous friction referredto load side are 0.! "g#m and 1.0 N#m/$rad/sec%. &he
s'nchro error detector produces an error signal of 0.1V per degree error in misalignment.
Develop the (loc" diagram representation of the control s'stem and there from o(tain the
transferfunction. (/"0" 1 /a233)
Sol!tion: 4or the error detector, &J = *"# 56degree error

.
&l =
*"# + # (*
7t
=
'"723' 56 radian
Ktm
=
Torque constant
=
*"*%' 8m65
8J 1
82
=
# *
J l
= *"2' kg m2 on load side
J c q = /otor side = (~ ~ r + J t = # 9*+$"2:

.
r
Control System Engineering
46 Mathematical Modeling of Systems
rotational system is called Inertia and denoted by T i.e, moment of inertia. Opposing
torque due to inertia T is proportional to the angular acceleration (a) of that inertia.
= J d28(t}
dt2
Tdue to inertia
Taking aplace,
~ - - - - - - - - - - - - - - - - - - - - ~
Tduc to inertia(!" = J #2 O(!"
Sr. No. Translational Motion Rotational Motion
$ Mass (M) Inertia (J)
2 riction (!) riction (")
% S#ring ($) S#ring ($)
% orce () Tor&'e m
# (is#lacement (& " )ng'lar dis#lacement (*)
'
+elocity , = (-f)
)ng'lar ,elocity ((J " = -n
(
)cceleration () t2& "
)ng'lar acceleration (e& = d2O"
dl2
Table %.. )nalogo's Elements
%.% E&'i,alent Mechanical System (Node "asis)
*hile dra+ing analogous net+orks, it is al+ays ,etter to dra+ the equi-alent
mechanical system from the gi-en mechanical system. To dra+ such system use follo+ing
steps.
Step $ . /ue to applied force, identify the displacements in the mechanical system.
Step 2 . Identify the elements +hich are under the influence of different
displacements.
Step % . 0epresent each displacement ,y a separate node, using 1odal 2nalysis,
Step 3. !ho+ all the elements in parallel under the respecti-e nodes +hich are
under the influence of respecti-e displacements.
Step #. 4lements causing same change in displacement +ill get connected in parallel
in ,et+een the respecti-e nodes.
l / 0nl
Control System Engineering 4-60 Mathematical Modeling of Systems
=
2.5 X 10-3 kg-m2
BL = 1 N-m/(rad/sec)
(N r
1
.
Beq = Motor side = N~ x B.
= 100 xl
= 0.01 N-m/ (rad/sec)
Te !lock diagram o" te s#stem is$
Nt
G(s) = K. x motorT.%.x N2
&.'.
Motor
%or motor$
Tm = (im )2
.
Tm(s) = Ktm )2 (5)
.
a*d Tm+t)
d2, de
=
- eq dt2 + B(. / q dt
.
Tm(s) = (02 - cq + 0 Bcq),(s)
.
... (1)
... (2)
)q1ati*g eq1atio*s (1) a*d (2)$
Ktm )2 (s) = (02 - eq + 5 Bl/ q) 0(5)
0(5) Ktm
= = =
)2(0) 0(0- eq 2 Beq) 5(03 2.5 x 10-4 +0.01) 0(52 5)
0.055 1,
G(s)
= 5.6275x 1, x. 8. = 10.4141
s(s25) 10 5(s25)
ac(s)
=
0i'0)
10.4141
5(02 5)
12 10.4141
s(s2 5)
10.4141
=
02 +5s+ 10.4141
nl
Control System Engineering 4-61 Mathematical Modeling of Systems
Review Questions
1. Explain the derivation of analogous networks using
i) Force-voltage ii) Force-current analogy
2. Write a shorf note on direct and inverse analogous networks.
3. Draw the analogous electrical networks based on
a) F - V analogy b F - I tl11alogy of the following !echanical s"ste!s
B
\
I-_f(t)
K
81
\ I nl
Control System Engineering 462 Mathematical Modeling of Systems
4. Distinguish between AC. servomotor and D.C. servomotor.
5. Derive transfer function o f a.c. servomotor stating the assumption made.
6. State the applications of a.c. servomotor.
7. Derive the transfer function of field controll~d d.c. seroomotor.
8. Answer the following giving reasons :
aJ A.c. servomotor has a smaller diameter and more length.
bJ Field controlled D.C. servomotor is premed than armature controlled.
!. Develop bloc" diagram for armature controlled D.#. seroomoior and find its transfer function.
$%. An armature controlled d.c. motor is supplied in series with a resistance from a
&' ( d.c. suppl). *he motor ta"es current of SA on smiling and the stalling tor+ue being %.!$5
,sm. *he motor runs at $%%% r.p.m. ta"ing a current of 1 A. The ualue of J = 4 X10-3 -g . ",2
and friction constant as 1.5 x 10- 3Nm/ /radsec0
Determine the transfer function of tile motor.
o (s) 2.9
(Ans. : v(s) = s(1 + 0.3s
$$. f o r the closed loop s)stt.1$72 shoton heloto3 draw the 4oc" diagram and determine
transfer function O["(s)/Or(S). *ile gioen ualues are 5rror detector gain K" = 8 (rad3 Amplifier
gain KA = 10 (26. Rf = = 5 , I--f == %.&5 H, Kf == %.%5 ,mA3 !",o "o r = %.%& -g . m2,
It== 3-g . m2# $%!&%''%$ == %.%7 ,m/rad sec0.
ft, = 5.5 ,m /radsec0
%( Constant
+
o CIH
Or
Am.) O*(s) = 32.12 )
( +,(s) (sl + 21.5s2 + 30.35 +32.12)
12. *he moment of inertia !m and the coeffi#ient of viscous friction $m f o r a field controll~d d.c. motor
are motor respectivel) 5 x1O-" ,- - m2 (./ 12.5 x%O-4Nm /mdsec0. *he motor tor+ru constant
"C, being 2.5 ,mA. Determine the transfer function relating the angular sp eed of the shllft and
th f i Id ' (l)(s) 2000
e e curren. (A.s.) ",(5) = 1+0.4s)
nl
Control System Engineering 463 Mathematical Modeling of Systems
13. Figure given below shows a block diagram of speed regulator system. The accelerating torque is the
difference between the torque developed by the controller and load TL. The controller gaitl is
0.00102 Nm/rad and the taClogenerator constant is 0.191 !/rad/."#ec.Tile load speed is ad$ltstl%d to
1000 r.p.m. The moment of inertia is J and friction is negligible.
Calculate a& The reference voltage. b& TIll' speed if a constant load torque of '.''1 Nm is
suddenly applied. (Ans.: a) 20V b) 951r.p.m.)
V(s) +
1--+--w{s)
14. (n instrument servo consisting of motor) spring loaded shaft) etc is shown below
R=1rad +
*here ! = !oltage in volts
+ = ,otor resistance 1Q
L = ,otor inductance 0.1 H
K/\ :;; (mpl-fier gain = 1' !/!
K = Spring C'1l.t/01t = 0.001 Nmfrcd
KI.I = 2ack. e.m3. constant = = 0.01 V (rdt!ec"
KT = Torque content = 0.01 N4m/(
] = ,oment of inertia = '.''5 Nms2
if tire input is 1radians) what is the steady state error3
l I
Control System Engineering 464 Mathematical Modeling of Systems
15. The positions seroomedumism is shown in figure below. 8r is rtforence position and 9 c actual
angular displaament oj s J u i f t i n radiam. Obtain i t s closed loop tramJer junction ~~:~
+
v-=-
~ constant rl
Given e
=
angular displacement of the motor shllft
Kf = gain f error detector = 7.64 V/ rad
n,
=
212, L " =negligible
Kb
=
5.5 X 1-2 V/ rad / s !c
K" =
6#l$-% Nm!
1 m =
1 #l&-5 K'- (2
f ( =
negligible
1L =
4.4 X 1-3 K' - (2
fr . =
4 #l)-2
*(/ +rad/s !c ,
*t
=
1
*2
=
1
(AN. ' %c +s , ::I 42.- ,
. 6r (s) %2 /-7.7 % / 42.-
1". Derive the transfer junction of a t#pical d.c. position control s#stem.
17. $erive the transfer fundion oj a t#pical d.c. speed control s#stem.
1%. Obtain the mathematical model of heat transfer s 0s t !(.
1&. Obtain the mllfhematical model of thermometer.
'(. ) 1u*t is *l+tuator 1 E:r pwn h#dmulic and pneumatic actuator s#stems.
'1. +omptlre h#draulic and pneu,*1-*tic actuators.
DOD
2l nl
Control System Engineering 4-7 Mathematical Modeling of Systems
4.5 Remarks on Nodal Method
a} The terms for an element
connected to a node 'X' and
stationary surface (reference) is,
d2x
For mass ~ M dt2
For friction ! B "~tl - d!t2 #
r--X(l)
s t
!"r(l# $$$$%
77777777$77
!"f(t)
f
&nction 'S'
....
....
/777//7l777
(
i
t---)(t)
~~%l
& & & & &
'
(a)
. B
F
fricti ' dx
or tenon -4 dt
For s(rin) ~ *x
+) The term for an element
connected +et,een the t,o nodes
'Xl - and ' X2' ie +et,een t,o
mo.in) surfaces is,
..... X* r-)+
( (
,f(t)$$$$%
/7~
t,o nodes as due to mass there &nction -S-
r
X* t-', X+
cannot +e chan)e in force as mass
cannot store (otential ener)y ~ f(t) $$$%
.*/ S01,,, -2- ,
c) 0ll elements ,hich are /77777777777777777777777
under the influence of same
dis(lacement )et connected in
(arallel under that node indicatin)
the corres(ondin) dis(lacement
e) consider the (art of the system, sho,n in the Fi) 4$1(a)
2o mass can +e +et,een the
For s(rin) -42 3 Xl - X2#
(4)
Fi) 43
5ll elements
6nder same
dis0lacement
-
-
B
M
)(t)
(a) (4)
Fi) 4$1
nl
Control System Engineering 4-8 Mathematical Modeling of Systems
Here M, Band K all are under the
influence of x(t). Hence in equivalent system
all of them will et connected in !arallel
under the node 'X'. "onsider another
exam!le of the system shown inthe #i. 4.$$.
In this system M, , Bl and K% all are under
the influence of dis!lacement &l' (his is
)ecause all are connected to riid su!!ort.
*hile there is chane from &l to &+ due
to simultaneous effect of B+ and K+. ,o B+
and K+ are under the influence of (&l - &+)'
But mass M+ is under the influence of &+
alone. Mass cannot )e under the influence
of difference )etween dis!lacements. ,o in equivalent system the elements B$ , K- and
MI,all in !arallel under &l while B+ and K+ in !arallel )etween &l and &+ and clement
M+ is under node &+ as shown in the #i. 4.$+ .
Fig. 4.11
4.6 Gear Trains
Fig. 4.1 Gear system
Fig. 4.1!
. ear train is a mechanical device that
transmits enery from one !art of a system to
another in such a way that force, torque, s!eed
and dis!lacement may )e altered. (he inertia and
friction of the ears are nelected in the ideal case.
"onsider a ear system as shown inthe #i. 4.$/.
(he num)er of teeth on the surface of the
ears is !ro!ortional to the radii (% and f+ of the
ears.
nl
Control System Engineering ~\' 4 - 9 II" Mathematical Modeling of Systems 1~;
The distance travelled along the surface of each gear is same.
i.e. I 01 rl = 02r2 I
The work done by one gear is same as the other.
i.e,
: !e can say
=
N2
"emar#s:
1$ The numbers of teeth % are
proportional to the radius r
of a gear.
2) The distance travelled on
each gear is same.
) !ork done & 'ill by each
gear is same.
\
'istance = re
1
r"""""#
(ig $.1$
4)1 *ear +rain !ith Inertia and (riction
In practice, gears do have inertia and friction which can not be neglected. %onsider
such practical gear arrangement connected to the load, shown below.
,rimary side
load tor&ue T-
load side
(ig 41.
T &'pplied tor&ue 01 ,02 &'ngular displacements
T. I T2 = Tor&ue transmitted to gears ( 1 , ( 2 = )nertia of gears.
N1 , N2 = Number of teeth *l , *2 = +riction coefficients
I I nl
Control System Engineering I , 4 - 9 .~ Mathematical Modeling of Systems i~:
The distance travelled along the surface of each gear is same.
i.e, I 81rl = 82r2 I
The work done by one gear is same as the other.
:. we can say
i.e,
Remarks
!" The numbers of teeth # are
proportional to the radius r
of a gear.
2 The distance travelled on
each gear is same.
$" !ork done = 1"8 by each
gear is same.
#
%istance & r'
1
r-----~
$ig. 4.!4
4.(.!. )ear *rain +ith I nertia and ,riction
%n practice, gears do have inertia and friction which can not be neglected. &onsider
such practical gear arrangement connected to the load, shown below.
8!
#!
.
I "
~
I
'!
~
I n-.ttor/.e* *!.0!
1rimaryside
T292~
~
(2
82
load side
,ig. 4.!2
T & )pplied tor*ue + ,+2 & )ngular displacements
!i , T2 = Tor*ue transmitted to gears 'l , J z = %nertia of gears.
( ! , ( 2 = (umber of teeth , , ,2 = $riction coefficients
-#1 , , : r %,
cControl System Engineering 4-10 Mathematical Modeling of Systems
Torque equation of side 1 is,
T
- J d201(t) B dOI(t) " Ii ( )
- 1 dt2 + 1 -----e r r - + t t
... (1)
Torque equation of side 2 is,
To J d202(1} B d02(t) r ()
2 = 2 dt2 + 2 ---cit + L t
... (2)
Now
'Ii
Nl _ O2
T2

N2 ! O t"
T2

N2
T) 00
NI
"u#stitutin$ inequation (2)
... (%)
"u#stitutin$ &alue of Tl in equation (1)
T J d2'I B dOl NI J d2(2 Nt B d02 N1 )Ii
I dt2 + l(it *N2 2" " " " dt2" *N2 2dt" + N2 I.
00 T

0
T

..
0
and
.
J Ie +qui&alent inertia referred to ,ri-ar. side
B1e +qui&alent fri/tion referred to,ri-ar. side
_01 , , : r I,
Control System Engineering 4-11 Mathematical Modeling of Systems
Similarly the equation can be written referred to load side also, where applied torque
gets transferred to load as (~: T)
where ! " l# $ h+(~: r! , and % " e $ % " +(~: r% &
4.6.2. Belt or Chain Drives
% elt and chain dri'es perform same function as that of gear train (ssuming that there
is no slippage between belt and pulleys we can write,
T() -* --=------r:C +))
r,
,ig 41-
for such dri'e
4.6. !evers
The le'er system is shown in the
,ig 41. This transmits translational
motion and forces, similar to gear
trains
% y law of moment,
" f. &/ $ f2 1"
% y wor0 done fl 1& $ f" 1"
,ig 41.
2ence
4.# Elec$ical Systems
Similar to the mechanical systems, 'ery commonly used systems are of electrical type
The beha'iour of such systems is go'erned by * hm3s law The dominant elements of an
electrical system are,
4 3& , , :1 " ,
Control System Engineering
~tt4-12 .. Mathematical Modeling of Syetems
i) Resistor ii) Inductor iii) Capacitor
i) Resistor: Consider a resistance carrying
current 'I' as shown, then the voltage drop
across it can e written as,
!uppose it carries a current "I, -12) then
for the polarity of the voltage drop shown its
e#uation is,
,,..--v-$ -" -1,---1-2)-R---'1
ii) Inductor : Consider an inductor
carrying current 'I' as shown, then the voltage
drop across it can e written as,
% $ & dI
dt
I $ ~' %dt
or
,
~O--~t-----~ __------~o
+ R -
-------v------
-'I
(2-
~o ---------~~------~o
+ R -
------V------
)ig. 4.1*
l 1
o ~----~rOmDmOO~DO~O~'--~.----o
------v---'-----
Fig. 4.1+
. Ifit carries a current "I, -12) then for the polarity shown its voltage e#uation is ,
% $ & d"lld~12)
or
iii) Capacitor : Consider a capacitor
carrying current 'I' as shown, then the voltage
drop across it can e written as,
or
% $ ~ ' I dt
I $ C d%
dt
If it carries a current "It - h) then for the
polarity shown its voltage e#uation is,
or
% ~' "I, -12) dt
"I,-12) $ Cd%
dt
o
!Ii
(2-
+ "oo, - - o( - o
l
------v------
)ig. 4.2.
c
~o --------~II~--~-~o
------v--------
)ig. 4.21
c
o ~--------+ ~I~I----------~o
------v------
Fig. ".22
/01 , , : r I,
Control System Engineering 4-13 Mathematical Modeling of Systems
4.8 Analogous Systems
In between electrical and mechanical systems there exists a fixed analogy and their
exists a similarity between their equilibrium equations. Due to this, it is possible to draw
an electrical system which will behave exactly similar to the given mechanical system, this
is called electrical analogous of given mechanical system and vice versa. It is always
advantageous to obtain electrical analogous of the given mechanical system as we are well
familiar with the methods of analysing electrical network than mechanical systems.
here are two methods of obtaining electrical analogous networks, namely
1) !orce - "oltage #nalogy i.e, Direct #nalogy.
$% !orce - &urrent #nalogy i.e. 'nverse #nalogy.
4.8.1 Mechanical System
!ig. 4.$3
&onsider simple mechanical system as
shown in the !ig. 4.$3.
Due to the applied force, mass M will
displace by an amount x(t% in the direction
of the force f() as shown in the !ig. 4.$3.
#ccording to *ewton+s law of motion,
applied force will cause displacement x,t%
in spring, acceleration to mass M against
frictional force having constant -
.. f,t%
.
/a 0 -v 0 K xt)
1here, a
.
#cceleration, v . "elocity
f(t%
d$ x,t% d x,t%
..
. M dt$ ! -,2't0 3x,t%
aking 4aplace, I !,s% . /s$ 5es%0 -s5es% 0 35,s%
"his is equilibrium equation for the given system.
*ow we will try to derive analogous electrical network.
4.8.# $orce %oltage Analogy &oo' Analysis)
&
i(
!ig. 4.$4
'n this method, to the force in mechanical
system, voltage is assumed to be analogous one.
#ccordingly we will try to derive other analogous
terms. &onsider electric network as shown in the
!ig. 4.$4.
671 , , : r ',
Control System Engineering Mathematical Modeling of Systems
The equation according to Kirchhoffs law can be written as,
vet)
'( ) R L diet) 1 I '()d
= It + (it + cit t
Taking Laplace,
es) = 1(!) R + Ls 1(!) + "#$
%ut we cannot co&pare '(s) and (es) unless we bring them into sa&e for&,
'or this we will use current as rate of flow of charge,
i(t)
=
dq
..
dt
i)e) "(s)
=
s*(s) or
*(s+ = "(s)
s
Replacing in above equation,
I ,"s) = L s'*(s) + Rs *(s) + ~ *(s) I
-o&paring equations for '(s) and es) it is dear that,
i) "nductance 'L' is analogous to &ass .
ii) Resistance 'R' is analogous to friction %)
iii) Reciprocal of capacitor i)e) lie is analogous to spring of constant K)
Translational
Force
Rotat"onal /lectrical
Torque T oltage
Mass M Inertia 0 Inductance L
Friction constant ! Tomonal friction "eSistanceR
constant !
S#ring constant I$ %lm Tortional s#nng constant "eci#rocal of
& %rnfrad ca#acitor 1/C
'is#facement '~ o Charge q
elocity( ) ~
1 IIIde *I*) C+rrent I, dq
df cit
Ta-le ../ Ta-ular form of force01oltage analogy
231 , , : r ",
Control System Engineering 415 Mathematical Modeling of Systems
4.8.3 Force Current Analogy (Node Analysis)
In thls method, current is treated as
analogous quantity to force in the mechanical
system. So force shown is replaced by a current
source in the system shown in the Fig. 4.25.
The equation according to irchhoff!s
current law for abo"e system is,
I # I$ +I% +Ie
et node "oltage !e &,
.. I # iI & dt +"+' ((
Ta)ing $aplace,
#t
Fig. 4.$%
&(s) &(s)
I*s+ # '+'+sC&(s)
,ut to get this equation in the similar form as that of F*s+ we will use.
d41
"et+ # dt where - ( flu.
yes+ # s "(s)
. )*(s) ( /es+
i.e, ...
s
0ubstituting in equation for I*s+
1 1
I*s+ # 's2 (*s+ +% s (*s+ +#,*s+
'omparing equations for F*s+ and 1*5+ it is clear that.
i+ 'apacitor!'! is analogous to mass 1.
ii+ %eciprocal of resistance ( is analogous to frictional constant ,.
iii+ %eciprocal of inductance iis analogous to spring constant +.
'ranslational ,otational Electr#cal
F Forca ' Current #
MM... 2 '
- friction - 1/R
+ S.ring + 1/L
/ dis.lacement 0
1
2 &elocity ( d/
- d3
&oltage *e* # d"
3))4((5)
dt dt
dt
'a!le 4.3 'a!ular form of force4current analogy
#e
c
451 , , : r I,
Control System Engineering 4-16 Mathematical Modelingof Systems
Key Point: The elements which are in series in F - V tmIllogy, get connected in parallel in
F - ItmIllogous network and which are in parallel in F - Vanalogy, get connected in series in
F- J analogous network.
4.9 Steps to Sole Pro!lems on "nalogo#s Systems
Step 1: Identify all the displacements due to the applied force. The elements spring and
friction between two moving surfaces cause change in displacement.
Step 2: raw the e!uivalent mechanical system based on node basis. The elements under
same displacement will get connected in parallel under that node. "ach
displacement is represented by separate node. "lement causing change in
displacement #either friction or spring$ is always between the two nodes.
Step %: &rite the e!uilibrium e!uations. 't each node algebraic sum of all the forces
acting at the node is (ero.
Step 4: In)-*analogy+ use foll,wingreplacements and rewrite e!uations+
) --*+ . --/+
$
0 --1+ 2 --lIe, 3 --!+ 3 -- % #current$
Step 4: Simulate the e!uations using loop method. 5umber of displacements e!ual to
number of loop currents.
Step 6: In )-6 analogy+ use following replacements and rewrite e!uations+
I
) --I+
$
.--7+ 0-- l81+ 2-- l8/+ & --41+ &'e(e.m.f.)
- - - - - - - - - - - - - - - - - - - - - - - - - - - - - - - - ~ - - - - - - ~ - - - ~ ~
Step 9: Simulate the e!uations using node basis. 5umber of displacements e!ual to
number of node voltages. Infact the system will be e3actly same as e!uivalent
mechanical system obtained in step 2 with appropriate replacements.
*...*. E&ample 4.+: Draw the equivalent mechanical system of the given system. Hence write
the set of equilirium equationsfor it and otain electrical analogous circuits using,
i) F-V !nalogy and ii) F-J !nalogy
-r
3+#t:
f#t$
;<1 + ," r I+
Control System Engineering 4-17 Mathematical Modeling of Systems
Solution: The displacement of Ml is Xl(t)
and as 81 is between MI and fixed support
hence it is also under the influence of Xl[t).
While 82 chanes the displacement from Xt(t)
to X2(t) as it is between two mo!in points.
"nd M2 and K are under the displacement
X2( t) as # is between mass and fixed support.
K F
Equivalent system
:EF = 0
"t node 1$ % & M' s2 Xt +81 s Xl +(2 )(XI *X2)
o & M2 52 X2 + #X2 + (2 )(X2* XI)
. (1)
... (2) "t node 2$
+ow (i) % * , "nalo- M -+ . (-+ / # -+ 11C x -+ q
/eplacin
0es) = .I s2 1l +/) 2 1t +/2 )(1l * 12) ..(3j
3 = .2 s212 +(1fQ 14 +/2 )(12 * 1l) 5.5(6)
I(s)
7(2) i.e. 1(2) & s Q(s) -
&
s
-es)
= .I sl)(s) + /) 11()) + /2 [ll()) * h(s)8 -+ .oop 1
0
1
= .2)I2()) + sC h(s) + /2 [h(s)*Il())8 -+ .oop2
9ence$
+umber of loop currents e1ual to number
of displacements.
Loop 1 Loop 2
:
(1/K)
(II) ! I "nalogy
(-+ l/R I< -+ Il. x -+
1 1
& CJ )29) + /l ) 91 + /2 ) (.1 * .2)
.(1) I(s)
..5(2)
/eplacin s 5 (s) = 0es)
;<1 $ , : r I$
Control System Engineering 4-18 Mathematical Modeling of Systems
I(s) ...Node 1
...Node 2
Hence,
Number of node voltages equal to number
L(1/K) of displacements.
,..,. Example 4.2: Draw the equiVlllent mechanical system and analogous systems based on
F-V and F-I methods for the given system.
1
X1t!
X1
" t!
2
82 2
E#$i%alent system
Sol$tion: &'o displacements ! No element under "l t! alone as force is directl#
applied to a spring l. So it $ill store energ# and %ence is t%e cause to c%ange t%e force
applied to &2. Hence displacement of &2 is "2 and as '2 and 2 are connected to fi(ed
supports bot% are under "2(t) onl# as s%o$n in t%e equivalent s#stem.
)t node 1,
)t node 2,
(((1!
... (2)
&2 , '2 , 2 are under same displacement.
*+1 , , : r I,
Control System Engineering 419 Mathematical Modeling of Systems
(i) FVanalogy: M_. L B_. R
1
V= C1(QI-Q2)
o = ~1 (q2-ql)+L22q2+~2 Q2+R2!Q2
K41/C
... (3) Loo" (1)
... (4) Loo" (2)
a#$ %i!"la&$#$n' !a#$ &())$n'.
*q(a'ion! in '$nn! o+ 11 an%12 &an ,$ -)i''$n ,y (!ing i(') = ~~ i.$. I(!) = ! Q(!) a!
$."lain$% $a)li$).
(ii) F - I analogy: M-' C/ B-' l/R, 0_. IlL
1
1(1) = Ii2 (~I - ~2)
... (1) 3o%$ (1)
... (2) 3o%$ (1)
*q(a'ion!in '$nn! o+ VI (!) an% Vz(s) &an ,$ o,'ain$% ,y (!ing '4$ )$la'ion/
%41
5(') = %'
i.e. V(s) = s cj(s) as $."lain$% $a)li$).
a#$%i!"la&$#$n'-!a#$5ol'ag$.
_61 / ,: r I/
Control System Engineering 4-20 Mathematigl Modeling of SY8tems
4.10 Servomotors
The servosystem is one inwhich the output is some mechanical variable like position.
velocity or acceleration. Such systems are generally automatic control systems which work
on the error signals. The error signals are amplified to drive the motors used insuch
systems. These motors used inservosystems arc called servomotors. These motors arc
usually coupled to the output shaft i.e. load through gear train for power matching.
These motors are used to convert electrical signal applied, into the angular velocity or
movement of shaft.
4.10.1 Requirements of ood Servomotor
The servomotors which are designed for use in feedback control systems must have
following requirements :
i) Linear relationship between electrical control signal and the rotor speed over a
wide range.
ii) Inertia of rotor should be as low as possible. A servomotor must stop running
without any time delay, if control signal to it is removed. or low inertia, it is
designed with large length to diameter ratio, for rotors. !ompared to its frame
si"e, the rotor of a servomotor has very small diameter.
iii) Its response should be as fast as possible. or quickly changing error signals, it
must react with good response. This is achieved by keeping torque toweight ratio
high.
iv) It should be easily reversible.
v) It should have linear torque # speed characteristics.
vi) Its operation should be stable without any oscillations or overshoot s.
4.11 !y"es of Servomotors
The servomotors are basically classified depending upon the nature of the electric
supply to be used for its operation.
The types of servomotors are as shown inthe following chart :
. .
AC.Servomotors
Servomotors
i
+
Speclal Servomotors D.C.Servomotors
~
Armature controlled Field controlled
$%& , , : r I,
Control System Engineering 421 Mathematical Modeling of Systems
'. .
4. 12 D. C. Servomotor
Basicallyd.c, servomotor is more or less same as normal d.c.motor. There are some
minor differences between the two. All d.c. servomotors are essentially separately excited
type. This ensures linear torque-speedcharacteristics.
The control of d,c, servomotor can be from field side or from armature side.
Depending upon this, these are classifiedas field controlled d.c. servomotor and armature
controlledd.c. servomotor.
4. 12. 1 Field Controlled D. C. Servomotor
In this motor, the controlled signal obtained from the scrvoamplifier is applied to the
field winding. ith the help of constant current source, the armature current is maintained
constant. The arrangement is shown in the !ig. ".#$.
Vft! from
servoam"lifier
la Constant!
%
Constant
c#rrent
&..----------o so#rce
r$%%%%'%
Fig. 4. 2& Field controlled d. c. servomotor
This type of motor has large 'f ()f ratio where 'f is reactance and )f is resistance
of field winding. Due to this the time constant of the motor is high. This means it can not
give rapid response to the quic' changing control signals hence this is uncommon in
practice.
4. 12. 1. 1Feat#res of Field Controlled D. C. Servomotor
*t hass followingfeatures (
i) *referred for small rated motors.
ii) It has large time constant
iii! *t is open loop system. This means any change in output has no effect on the
input.
iv) +ontrol circuit is simple to design.
4. 12. 2 +nnat#re Controlled D. C. Servomotor
In this type of motor, the input voltage 'Va I is applied to the armature with a
resistance of ,a and inductance &a. The field winding is supplied with constant current
If. Thus armature input voltage controls the motor shaft output. The arrangement is shown
inthe !ig. ".#-.
.%/ , , : r I,
Control System Enginearing tll~1 .4-22 Mathematical Modeling of Systems
I,
(Constant)
Constantr
current
source0-------'
........ . . . _ .
Fig. 4. 27 Armature controlled d. c. seromotor
The constant field can !e supplied with the help of parmanent magnets. In such case
no field coils are necessary.
4. "2. 2. " Features of Annature Controlled #. C. Seromotor
It has following features
i! "uita#le for large rated motors.
ii! It has small time constant hence its response is fast to the control signal.
iii) It is closed loop system.
i$! $he #ac% e.m.f. pro$ides internal damping which ma%es motor operation more
sta#le.
$! The efficiency and o$erall performance is #etter than field controlled motor.
As the armature controlled d.c. ser$omotor is closed loop system& in comparison %ith
open loop field controlled system. generally armature controlled motors are used.
4. "2. & Characteristics of #. C. Seromotors
The characteristics of d.c. ser$omotors arc mainly similar to the tor'ue-speed
characteristics of a.c& ser$omotor. The characteristics are shown in the (ig. 4.2).
Tor'ue
('m)
*a (Armature oltage
Ea4 + Ea, + Ea2 + Ea&
S)eed(r. ). m. )
Fig. 4. 2* $or+ue,s)eed characteristics for an amature controlled d. c. seromotor
-.1 & , : r I&
Control System Engineering .,. 423 Mathematical Modeling of Systems
4.12.4 Applications of D.C. Servomotor
These are widely used in air craft control systems, electromechanical actuators, process
controllers, robotics, machine tools etc.
4.13 ransfer !"nction of !ield Controlled D.C. Motor
#
Ass"mptions $
#
Jm% &nertia
'm% !riction
!ig. 4.2(
)1* Constant armature current is fed into the motor.
(2) cI I f acI f. Flux produced is proportional to field current.
& .f = f I f &
(!) Tor"ue is proportional to product of flux and armature current.
& Tm oc cilI a &
Tm % # #I a = # f I f r,
Tm = m +t I f &
,.. )1*
$here m % #la % Constant
%pply irchhoff#s la- to field circuit.
dif
&f dt #'f I f = ef
... (2)
(
)ow shaft tor"ue Tm is "sed for dri*in+ load a+ainst the inertia and frictional tor"ue.
I nertia force
% . d2,m - d.m
& r., m dt2 # m dt
..,)3*
/(0 , , : r I ,
Control System Engineering , 4-24 Mathematical Modeling of Systems
F
1 " B dOm imil B dx
nctiona rorce m'"(it" sarto dt
Finding !a"lace #ransforms of e$%ations &1', &2' and &(' )e get,
Tm(s) = KmKf If(S)
Ef(s)

(SLf + Rf) If(s)
Tm(s)

Jms2* m(s)+ BmsOm(s)
Eliminate If(S) from e$%ations &4' and &+'
Tm{s)
KmKf Ef(s)
(sLf +Rf'
Eliminate Tm{s) from e$%ations &,' and &-',
&S2 . m+ sBm)0m(s)
Kmx, Ef(s)

(sLr +Rf)
/n"%t

Ef(s)
O%t"%t

0otational dis"lacement Om(s)
12 #ransfer f%nction
ames)

Ef&S'
Om(s)
KmKf
Ef(S)

UmS2 + sBm) (Rt + sLr)
=
KmKf
sRr Bm!+ s'tm" !+ s'tr"
#$ere 'tm

Jm Motor time constant
Bm
'tf

i1 Field time constant
#2F2 3m&S' Kr
Km 1
2-
Ef(s) Rrl + S!'r) Bm(!+ s'tm) S
222 &4'
%%&(')
224&,'
222 &-'
Bloc5 (ia)ram for field controlled d2c2 motor is as sho)n in Fig2 42(* 2
Et(s)
*
K+
It{s)
6 2n
,m(s)
!
-
--
R+{! 7 s!!. Bm{! +Slm)
S
+///
Fig2 42(* Bloc5 diagram
891 , , : r /,
, :, Control SystemEngineering ~",,r"mr. 4 -25 Mathematical Modeling of Systems,. ,~,~,
4.14 Transfer !nction of "rmat!re Controlled #.C. Motor
"ss!m$tions :
(i) Flux is directly proportional to current through field winding,
% cI I m = Kr I r = onstant %
(li) !or"ue produced is proportional to product of flux and armature current.
! = K#m K, I f r,
! = K#m I lI a
(iii) $ac% e.m.f. is directly proportional to shaft &elocity rom, as flux '(is constant.
rom
d)(t)
as
=
crt
*+
=
K+ (,-m(s)= K+ s.mes)
/, /a L a
0
Constant
ig. 4.&1
"$$ly 'irchhoff(sla) to armat!re circ!it :
1
di2
ea = *+ 0I a (/a)0 a dt
!a%e 1aplace transform,
3ow
. *a(s)
=
*+(4) 0 la (s) l/.a 0 4 1a5
i,*s+
*..(s) 6 *+(4)
=
/ .. 0s 1a
la *s+
*a (s) 6 K+ s.mes)
=
/a 0 4 1a
!m
=
K#m Kr I r r,
!m
=
K# K % 7*a 6 K+ 4.mes)-
m f( /a 0 4 1a
89( , , : r I ,
Control System Engineering 426 Mathematical Modeling of Systems
A l s o ... from equation (3)
Equating equations of TIt\I
K'm Kr If Ea(s) K'm x, r, Kb sOm(S) 0 2 B)O )
(Ra + S a) ! (Ra + S a) + mS + S m m(S
K'm Kr If
." (R,#Sa)Ea(s)
em(s)
!
Ell (s)
G(s) s Ra Bm($ + stm%& +S 'fa)
1+ Km'sKa,
S Ra Bm(1+S 'tm)(& +S 'fa)
!
$ + ((s)%(s)
)*ere r., ! Im+Bm and
a
'ta =
Ra
((s)
S R, Bm($ + S 'tm) ($ +s'tot)
%(s) !
T*erefore ,an be re-resente. in its b&o,/ .iagram form as in 0ig. 1.22.
!ig" 4"32 #loc$ diagram
Ke3 %oint& Field controlled d.c. motor is open loop while armature controlled is closed loop
system. Hence armature controlled d.c. motors are preferred over {reId controlled type.
4"1' ("C" Ser)omotor
4ost of t*e ser5omotors use. in &o) -o)er ser5ome,*anisms are a,,, ser5omotors..
TI&e a.,. ser5omotor is basi,a&&3 t)o -*ase in.u,tion motor. T*e out-ut -o)er of a.,.
ser5omotor 5aries from fra,tion of )att to fe) *un.re. )atts. T*e o-erating frequen,3 is
'* +, to 4** %6.
7 'I , , : r' .
Control System Engineering 4-27 Mathematical Modeling of Systems
4.15.1 Construction
A.C.Supply
C9n~ol T~ Reference
W31ndlng ~ winding
Control 9(?-: Rotor
V O ! " ti" # #"
servo V-_
!plifier :
"t is mainly di$ided into t%o &arts
namely stator and rotor.
'he stator carries t%o %indings(
uniformly distri)uted and dis&laced )y *+,( in
s&ace. O ne %inding is called main %inding
or fi-ed %inding or reference %inding. 'his
is e-cited )y a constant $oltage a.c( su&&ly.
'he other %inding is called control %inding.
"t is e-cited )y $aria)le control $oltage( %hich
is o)tained from a scr$oam&lifier. 'his
$oltage is 9#$ out of &hase %ith res&ect to the $oltage a&&lied to the reference %inding.
'his is necessary to o)tain rotating magnetic field. 'he schematic stator is sho%n in the
.ig 4.//.
.ig. 4.// Stator of 0.C. ser$omotor
4.15.2 1otor
'he rotor is generally of t%o ty&es. 'he one is usual s2uirrel cage rotor. 'his has small
diameter and large length. 0luminium conductors are used to 3ee& %eight small. ts
resistance is $ery high to 3ee& tor2ue-s&eed characteristics as linear as &ossi)le. 0ir ga& is
3e&t $ery small %hich reduces magnetising current. 'his cage ty&e of rotor is sho%n %ith
s3e%ed )ars in the .ig. 4./4 4a5. 'he other type of rotor is drag cu& ty&e. 'here are t%o
air ga&s in such construction. Such a construction reduces inertia considera)ly and hence
Alu!iniu!%rs
S&ft
4a5 S2uirrel cage rotor
'rg
(%( 'rg cup ty&e rotor
.ig. 4./4
such ty&e of rotor is used in $ery lo% &o%er a&&lications. 'he aluminium is used for the
cu& construction 'he construction is sho%n in t&e .ig. 4./4 4)5.
4.15./ 'or2ue-s&eed Characteristics
'he tor2ue-s&eed characteristics of a t%o &hase induction motor( mainly de&ends on
the ratio of reactance to resistance. .or small 6 to 1 ratio i.e. high resistance 1#)) reactance
# 71 ( , : r (
Control System Engineering 428 Mathematical Modeling of Systems
Large ~
Torque
eNm)
Control voltage
E24>E23 >E22>E21
t
X
Small"R
Speed r!p!m!"
# Speed
Fig. 4.35 Fig. 4.36
motor, the characteristicsis much more linear while it is nonlinear for large Xto ratio as
shown in the Fig. 4.35.
!n "ractice, #esign of the motor is 5$ as to get almost linear tor%ue&s"ee#
characteristics. 'he Fig. 4.36 shows the tor%ue&s"ee# characteristics for (arious control
(oltages. 'he tor%ue (aries almost linearl) with s"ee#. *ll the characteristics are e%uall)
s"ace# for e%ual increments of control (oltage. !t is generall)o"erate# with low s"ee#s.
$!%&!$ 'eatures of (! C! Servomotor
'he a.c, ser(omotor has followingfeatures +
i) ,ight in weight. ii) o-ust construction. iii) elia-le an# sta-le o"eration.
i() .mooth an# noise free o"eration. () ,arge tor%ue to weight ratio. (i) ,arge to X
ratio i.e, small X to ratio. (ii) No -rushes or sli" rings hence maintenance free. (iii)
.im"le #ri(ing circuits.
$!%&!& (pplications
/ue to the a-o(e features it is wi#el) used in instrument ser(omechanisms, remote
"ositioning #e(ices, "rocess control s)stems, self -alancing recor#ers, com"uters, trac0ing
an# gui#ance s)stems, ro-otics, machine tools etc.
$!%&!) Transfer 'unction of (!C! Servomotor
'he (arious a""ro1imations to #eri(e transfer functionare,
2i) * ser(omotor rarel) o"erates at high s"ee#s. 3ence for a gi(en (alue of control
(oltage, ' oc N characteristicsare "erfectl)linear.
2ii) !n or#er that ' oc N characteristicsare #irectl) "ro"ortional to (oltage a""lie# to
its control "hase, we assume ' c4c N characteristicsare straight lines an# e%uall)
s"ace#.
'or%ue at an) s"ee# 5N6 is,
781 , , : r !,
Control System Engineering 4-29 Mathematical Modeling of Systems
T. K E
dem
m = tm 2t + m"""'dt
where, d:; is speed of motor.
Ifload consists inertia 1m and friction Bm we can write,
Tm(s) = 1m52 m+ Bm sm
!ow "aplace transform of e#$ation (1) is
Tm(s) % Ktm&2 (s) + m sem(s)
&#$atin' e#$ations (2) and (3)
... (1)
.... (2)
... (3)
:. (im &2 (s) + m sm (s) % 1m )2 m())+ Bmsmes)
m()) Ktm
% %
&2()) s(slm - m + Bm)
m(*)
(m
&2 ())
% ..
s(l + 'tms)
where (m
%
Ktm
m!m
and 'tm
%
1m
Bm-m
2
) em sem
+,,-- .... ---1i).o%-/...--o-:;.;./---t .. ----ooem
Kit " 1/s
(m-Bm)
.m
#ig. $.3% Signal flow gra&h of a.c.
ser'omotor
0
s.m1
s (Bm- m) 1+ (Bm/ m)
Kay (oint) As slope is negative, in the
above equation 0Bm - m . shows that total
friction increases due to m. As it adds more
friction, the damping improves, improving
stability of tire motor. This is allied InterJUll
Electric Damping of 2 ph A.. servomotor.
#ig. $.3* loc+ diagram of a.c. ser'omotor
/-1 , ,! r 2,
Control System Engineering 4-30 Mathematical Modeling of Systems
Signal flow graph for A.c. servomotor is as shown in the Fig. 4.37. Hence block
diagram of A.C. servomotor is as shown in Fig. 4.3
4.16 Comparison of Servomotors
4.16.1 Comparison between A.C. and D.C. Servomotor
Sr. A.C. Servomotor D.C. Servomotor
No.
1
!ow power o"tp"t of abo"t Deliver high power o"tp"t
1
# $ to 1%% $.
#
Efficiency is less abo"t & to #% !. 'igh efficiency.
()
D"e to absence of comm"tator *re+"ent maintenancere+"ired d"e to
maintenanceis less. comm"tator.
4)
Stability problems are less. More problems of stability.
&)
No radio fre+"ency noise. ,r"shes prod"ce radio fre+"ency noise.
"#
-elativelystable and smooth operation. Noisy operation.
7)
A.C. amplifiers "sed have no drift. Amplifiers "sed have a drift.
4.16.# Comparison between Armat"re Controlled and
*ield Controlled D.C. Servomotors
Sr. Field Controlled Annat$re Controlled
%o.
1) D"e to low power re+"irement amplifiers 'igh power amplifiers are re+"ired to
are simple to design. design.
2)
Control voltage is applied to the field. Control voltage is applied to the armat"re
3&
.ime constant is large. .ime constant is small.
4& .his is open loop system. .his is closed loop system.
5)
Armat"re c"rrent is /ept constant. *ield c"rrent is /ept constant.
"& 0oor effiCiency. ,etter efficiency.
7)
S"itable for small rated motors S"itable for large rated motors.
1)
Cos2yas field coils are m"st 0ermanent magnet can be "sed instead
of field coils which ma/es the motor less
e3pensive.
4.14 Models of Commonly "sed Electromechanical Systems
'n this article the transfer f$nctions of ver( commonl( $sed s(stems arc derived. )his
will help the reader to find o$t the transfer f$nctions of the different practical s(stems.
*+, - , : r '-
Control System Engineering 4-31
4.11.1 Generators
Consider a separately excited generator
which is many times used in various practical
mechanical systems. Generators are required to
drive the motors because vacuum diodes,
transistor amplifiers are not suitable because of
their low ratings. Consider a generator as shown
inthe figure.
f ! "ield resistance
Lf ! "ield inductance
ef ! #pplied voltage $input%
eg ! Generated voltage $output%
&ow for a generator,
where. ! flux
"lux is directly proportional to current passing through the field winding say if.
'et (, be the generator constant in )* #
#pplying (irchhoff+s 'aw to field circuit.
. ' dir
ef ! ,f r + r dt
-a.ing 'aplace of both the equations $1% and $/%
Mathematical Modeling of Systems
I
rr
"ig. 4.30 Generator
Eg(s)
!
(a ,1$s%
2,$s%
!
f ,f$s% + 'r$s% ,t$s% neglecting ,r$3%
,f$4%
2f$s%
..
:::
r +s ',
.
Eg(s) =
x, Efs)
..
r + s 'f
..
2g$s%
=
(a
2,$s% f + 5 'f
!his is the -.". of separately excited generator.
... (1)
... (")
671 , , : r ,,
- Control System Engineering 4-32 Mathematical Modeling of Systems
4.17.2 Generator Driving Motor
. It is very common to find generator driving motor inpractical mechanical systems. So .
let us discuss the T.F. of a system with generator driving motor.
Fig. 4.4 !enerator driving motor
"f and #$ - "esistance and inductance of generator field
eg - !enerated voltage
% g - !enerator constant in &' (
"$ and #a - "esistance and inductance of motor armature
) - *.I. of #oad and f is frictional force
+ow. T. F. of generator is$
Eg(s) % g
Ef(s) !f " ,#f
+ow consider armature controlled motor$
Tor-ue produced $y motor is dependent on ia and let I. T $e tor-ue constant.
% / % T ia
This tor-ue is utilised to drive a load.
T ) d2 " f d0
.. dt2 dt
+ow Eg is voltage applied to armature.
p1 -1 &' " " # dia " e$
---g a a a crt
where e$ is2ac3 e.m) .
d0
es ee c.o ee Cit
145 $ , : r I$
Control System Engineering 4-33 Mathematical Modeling of Systems
Let Kb be the back e.m.f. constant
,
eb = Kb d6
dt
d20 dO
KT ia = J dt2 + f (ft.
Taking Laplace transform
KT la(s) = Js26(s)+fs6(s)
. . ! L diil v. dO
""g = #ii a + a $it + %%#& dt
... (3)
Taking Laplace transform
'g(s) = la(s) !a + Lol s la(s) + Kto s O(s) ... (()
finding ).*(s) from e+,ation (3)
(J s2 +f s) O(s)
).*(-) =
.,bstit,ting in e+,ation (()
s6(s)(Js+f) *
g(s) = K (!* +. La) + K" sO (s)
T .
Eg(s) = O(.)s/(f+f.)$01+.La)+Kh2
Eg(s) _ . Kb 6 (s) = s(!a + . Lo#) (sJ +f)6 (s)
Kr
6(s)
=
s(!a +. La )(- J +f)
)
sJ + f I---'IP-" 9(s)
3ig. (.()
. . .
_\1' ir I
Control System Engineering
434 Mathematical Modeling of Systems
Tnerefore using both the transfer functions of generator and armature controlled motor
we can develop the combined block diagram as below :
~
R,+sL,
1
t--4,__... 9(s
s!+ f
E,(s
Fig. 4.4"
Reducing the block diagram, solving feedback loop of motor which is
KT

s !Ra + "#a$ # s + f$
1+ s$t, KT
s!R, + "#a$ # "+ f$
KT
% s& ' ( ( !R' a% ) % s & ( & & & & #' a $& * & ( & & & & s% % % ) % f$ ) s + b + T

%(s + T + g
, f !-$ % :::!R % f% ) % s% # % ,% :' & $% ( :.s/ !:% ! % R% a % ) % s% % #% a:( 0 $% ::* :% "% ) / f$ % % ) % % + % b% % + % % T% % :::( & l1
4.&'.( )osition Control System
2nother ver3 common s3stem used in practice is position control s3stem. This is used
to control position of shaft, b3 use of potentiometer as error detector. The error is to be
amplified b3 amplifier and then must *e given to armature controlled motor whose shaft
position +ill get controlled as per the controlled signal The motor shaft is coupled to the
load through gearing arrangement with ratio 4t546.
#oad has 7.8. 9 and friction as f while :r is the reference position while all is the
actual position of shaft.
,he circuit diagram can be drawn as below :
La
% % ;ear
e - error
~
l& const.l
.ig. 4.4( )osition control system
0 <1 , , : r 8,
Control System Engineering 4-35 Mathematical Modeling of Systems
Let Kp be the potentiometer sensitivity, in V /rad
The corresponding block diagram can be drawn as shown in the Fig 444
1
sJ+f
Fig 444
!ed"cing the block diagram as shown in the Fig 445
#r$%&
'$s& (
Fig 445
KKp KT$)l/)*&
K Kr KT $)l/)*& = K, =%ystem gain constant
H(s) ( 1
r. +ver all T F can be calc"lated as below,
Ks
s,$!a -sLa&.s-/&-KbKT0
1+ 1 2 s
s,$!a +% La& .s +/& + Kb KT 3
4a$s&
(
+res& s,$!a - s La5.s- /&- Kb KT J+ K,
43 64 7osition 8ontrol with Field 8ontrolled 9otor
In the above case if instead o/ armat"re controlled motor, /ield controlled motor is
"sed then derive the TF o/ overall closed loop system
8onsider /ield controlled motor,
)ow T :e 43 ia and i; is constant
<=3 , , : r 1 ,
Control System Engineering 4-36 Mathematical ModeUng of Systems
ia = constant
Fig. 4.46
Let Kf be constant in N-m/ A
This torque is utilised to drive a load of moment of inertia 1and friction f.
d20 de
= 1 dt ! f dt
.
K! if
=
" d6 ! f d6
dt2 dt
Finding La#lace of all the equations!
$f%&' = (!%s' "f ! s Lf (f%s'
and #$ (f%s) = * +26%+' ! + f6%s'
(f%s'
s6%s' ,+" ! f%
.. =
Kf
&ubstituting in $f%s'
$!%s'
s 6%s' ,+& ! f' ("f ! s L!-
=
#f
6%.'
Kf
..
/ $!%s'
=
s ,s * 0 /0 f)"f ! s *f +
No1 using the same bloc2 diagram as derived in case %iii' re#lacing armature
controlled motor T. F. b3 the field controlled 1e can get the ne1 bloc2 diagram.
,ig. 4.4-
45" ! , : r (!
. Control System Engineering 4-37 Mathematical Modeling of Systems
Let K!' = K Kp
Kf (~;)
.
G(S)
K,.
H(s) =1
-
sCsI+ f)(Rf + sLf)
(s) s(s! + f) ("f + s#f~
- = - - - : : : : : : : - : ~ = - : - = - . . . . ; _ ; _ _ . . . . : . . . . . : ~ - _ _ _ _ : _ ; _ _ _
1+ (s)$(s) %+ K~
s (s & + f) (R, + s #r)
0'1(5)
=
OrCS) s (s ' + f) (R, + S Lr ) + K,
4.17.5 Speed Control Sste!
"n s#!e of t$e pr%&t'&%l %ppl'&%t'ons 't is ne&ess%r to dr'(e % lo%d %t % des'red speed
(l)r%d)se&. *or t$'s %ppl'&%t'on %n ele&tro!e&$%n'&%l sste! &%n +e ,sed -$'&$ 's' s$o-n %s
+elo-. .
IConstant
R%.
(achometer
-)ig. *.*+ S,eed control system
Sste! ,ses %r!%t,re &ontrolled !otor and t%&$o!eter feed+%&/. Let I- . +e t$e
%!pl'f'er 0%'n. Let ,s der'(e t$e tr%nsfer f,n&t'on of this/ sste!. (he o+1e&t'(e of system is
to !o(e t$e lo%d %t %. des'red speed.
(he d.&, t%&$o!eter o,tp,t (olt%0e is proport'on%l to o,tp,t speed (l).
er 's referen&e (olt%0e %nd et 's t%&$o!eter (olt%0e.
error
('% = K2 0 e =K1I (e, - et )
e, 3 Kt (") -$ere K, 's &onst%nt.
.#. (5)
... (4)
5e0le&t'n0 'nd,&t%n&e of %r!%t,re
671 , , : r ",
Control System Engineering :. 4-38 Mathematical Modeling of Systems
substituting from equation (5),
substituting from equation (6), ia Ra + Kb m
~ f;J
K, e, - K, K, m = ia Ril + Kb1 0
Taking Lala!e
K, "r(s) # (K" K, $Kb) m(s) $iii (s) R%%
&o' torque ro(u!e( by motor
--- (7)
T DC ia
)%*(+)
&o' t,is (ri-es a loa(
dB
',ere m# .ft
___(9)
r,Taking Lala!e
KT iii (s) # Js m(s) + f m(s)
U
s$ f)
""---=;----" + f m( s)
KT
/ubstituting inequation (0)
ia (s) #
___(10)
U s$ f1m(s)
# (K, K, + Kb) m(s) + KT Ra
(K...Kt + Kt% ) m(s) Us + f] m(s) Rd
"r2s) = + --=--=c:;---
3, 3, KT
m(s)
=
"r(s)
nus is th !"#i!d transfer fun!tion of t,e s$d !ontrol s4stem%
*51 , , : r I,
Control System Engineering! 4-39 Mathematical Modeling of Systems
4.17.6 Speed Control using Generator Driving Motor
A position control system described in Fig. 4.49 in which the armature of motor is
applied with a control voltage through a generator. The field current of generator controls
the voltage generated by the generator.

L !oad
. -\--o::!"m
"m
"ig. 4.4# Speed control system
#et us determine the transfer function of the system
$nput is e% &t'.
(ow
d ( O )
ce&t' ) l$l%t& ' () dt
... %11&
Ta*ing #aplace transform
Ee%s& * E+ &+' - () +, , &+'
(ow - &' . #
di% /0
ct0 1 ) &&ff2 $ $f
... %1-&
applying .irchhoff3s law to the field circuit of generator.
(ow e.! ) .% il where .4 is constant.
Ta*ing #aplace transform
-4 %s& ) .4 $f %s&
Ta*ing #aplace transform of %1-&
Ee%s& (- ) $f %s& .f /s!f 0
-liminating $f %s&
... %11&
-c&s' (-
&/f /s#%'
-.!&+'
.4 -c&+' (-
)
&/% / +#&'
E22%s& .a (-
-c&s'
)
&/t /s#&'
5\! % , : r $%
Control System Engineering ..\: '.. !i-.. 4 - 40 Mathematical Modeling of Systems
.j
Now consider fieJd constant armature controlled motor. Armature is energized by
output of the generator.
Applying Kirchhoff's Jaw to armature circuit
,
!il"s# $ !b"s#%&a"s#'(a%s)*+
,ac- e.m.f. is proportional to em
!b./# $ K d0m
b"ft
1a-ing )aplace 1ransform
!b"/2 = Kb s0m"s#
Inarmature controlled d.c. motor as field current is constant so tor3ue produced is
directly proportional to t4iearmature current.
1 $ K& a"s# where K is motor constant .
.1his tor3ue is utilized to dri5e the load with moment of inertia J and friction 6ro.
1 $ J d70m % 8 o d0m 1a-ing )aplace transform from
dt7 m dt
K4a"s# $ Js70m"s#%s6m0m"s#
t "s# $ 9 "ls +6m#0m "s#
. .
/ubstituting ine3uation f9 r ! "s#
EOl(s)
:m(s) K
..
Ea(s)
$
sKKb%s"(a %s)a#;s% 6m2
$
Km
... Neglecting L. of armature
<'4 % 1ros+
Km
K
$ 'KKb% ( 6m4$ =otor constant
Tm
(a> T f t
= &KK 8 o 4 = une constant 0 mo or
b%(* m ?
Neglecting ')a' of armature motor
ames2 @ Km
!a"s#A- s"l%1ms#
@B4 , : r I,
r~"- Control System Engineering ~n"iJ~ttw 4 - 41 Mathematical Modeling of Systems
Ea(s) x, K2
Ec(s)
=
JRr + s Lr
E"!(s)
K" K2 Ec(s) Om(s} s (1 + Tm s)
""
= =
(R, + s Lf)
Km
and Ec(s) = El (s) - Kb sOm
K" K2 lEI (s) - Kb sam (5)] Om (5) 5(1 +r#5)
. . . ~~~~ ~ =
$R% + ! L%)
x, K2 Km
" x, x, K. , Km !]
"R, + s Ltl EI (5)
= Om (!) s(l +Tm !) + (Rf +5Lf)
x, K2 &%
a (s) $Rr s (1+r%s) (1 +T# s) + Kb Kit K2 Km s]
""
$1 +sTd EI (5)
=
m $1+'f(s)
Om(!) Ka K2 Km
E((s)
)
!"K, Ka K2 Km + Rf (1 +Tm s) (1 + Tf s)
Om(!)
=
E((s)
*
w$%#% K& = R' & %n%#at(# )(nstant.
4"1+"+, 'y-ical .osition Control System /sed in 0nd/stry
, *(siti(n )(nt#(l +%%dba), s-st%m $as a *(t%nti(m%t%# b#id& % wit$ a s%nsiti.it- (+
*-/0#adian as %##(# d%t%)t(#. It +%%ds a d. ). am*li+i%# wit$ an (*%n )i#)1it & ain K. 'his
!1**li%s t( a +i%ld (+ & %n%#at(# w$i)$ $as #%sistan)% Rf and ind1)tan)% Lf" 2%n%#at(#
)(nstant is K& /0 3. . T$is is )(nn%)t%d t( an a#mat1#% )(nt#(ll%d d. ), m(t(# wit$ m(t(#
t(#41% )(nstant KT and ba), %. m. + )(nstant Kb /0#ad0s%). It d#i.%s a l(ad (+ in%#tia J and
+#i)ti(n f"
5#aw t$% sim*li+i%d bl(), dia& #am and $%n)% )al)1lat% its t#ans+%# +1n)ti(n.
1ig" 4"23 'y-ical -osition control system
671 , , : r I,
Control System Englnearing 4-42 Mathematical Modeling of Systems
e r = Reference position , e a = Actual position
T. F of generator
Eg (5) K g
Ef(s) ::::: Rf + S Lf
Bloc !iagra" can #e !ra$n fro" t%e anal&sis of t%e !ifferent cases !iscusse!.
'
sJ + f ....... ...,_.. ()(s)
Fig. 4.5'
*eglecting t%e in!uctance of ar"ature $in!ing of "otor.
Fig. 4.52
G(s) =
+, K K T
,-s) = '
SeRf . S/f) 0Ra (1s. f) + K # K T1
/et
T.F =
(a(s)
=
!r(s)
".#$ %.C. Motor &osition Control System
2n in!ustr& to conrol t%e position of t%e s%aft, a !.c, "otor position control s&ste" is
co""onl& use!.
34' 4 , : r '
Control SystemEngineering 443 Mathematical Modeling of Systems
4.18.1 Transfer Function of D.C. Motor Position Control System
Consider the D.C. position control system which is controlling position of the shaft.
Assume that the input and output of the system are the input shaft position and output
shaft position respectively.
Assume following system constants,
r Angular displacement of the reference input shaft
c
=
Angular displacement of the output shaft
e
=
Angular displacement of the d.c. motor shaft used
K)
=
Gain of potentiometric error detector
Kp
=
Amplifier gain
e!
=
Applied armature voltage
e"
=
ac! e.m.f.
"a
=
Armature winding resistance
#a #
Armature winding inductance
ia # Armature winding current
$ =
ac! e.m.f! constant
K
#
$otor tor%ue constant
% m # $oment of inertia of motor.
b m
=
&iscous friction coefficient of motor
&'
=
$oment of inertia of load
b L
=
&iscous friction coefficient of load
n # Gear ratio 't(')
$constant
*ig. 4.+3 D.C. $otor position control system
,-. , , : r /,
Control System Engineering 4-44 Mathematical Modeling of Systems
Equations describing above system can be written as follows :
Output shaft position is to be controlled so as to keep that at required position. Output
is sensed by angular displacement 'c' and compared with input which is r. Error is
amplified by amplifier with gain 'Kp' and given as input to the d.c. motor which in tum
controls thc angular position of the shaft of the motor '0' which in turn controls output
position of shaft' c' so as to modify the error.
For potentiometric error detector we can write.
E(s) = K !"(s) # C(s)]
For amplifier
E.$ (s) % Kr E(s)
For armature controlled d$c$ motor
if = constant &O flu' ( ) is constant
* = Kia where K = motor torquc constant
eb
ee
a
dO
..
+b
% Kb,,dt
For armature circuit
ea % -
di$ . "
+b + ) Tt+ la a
*aking -aplace transforms
Eb(&) % l<j, s.(s)
Ea (s) = Eb(s) +/a (s)!"a +s -a0
* % K/a (s)
1ow torque is utilised to drive load +shaft of motor.
J e q
%
J m+n 2 J - % Equivalent moment of inertia
beq
%
bm+n 2 b- % Equivalent frictional coefficient
*
d23 dO
% Kia (s) % eq dt2 +beqdt
%
lJ .. 'q s2 +beqs4O(s) -aplace transform
la (s)
)
%
K5eq &2 +beqs0O(s)
Ea(s)
.(s)
%
Kbs. (6)7 ,,,'8(5eqs2+bcqsl/"a +&-a0
9:) $ , : r /$
Control System Engineering 4-45 Mathematical Modeling of Systems
o(S) K
=
Ea(s) s Ueqs + bcq](Ra + sola J+ KKb s
K
=
'La' is generally small hence neglece!"
K
=
S#KKb + s n$ J l' < j + bc"-q n$ J
Km
=
s(l + %ms)
K
Km = (b"" -qRa + KKb) = &oor consan
Ra J , " ' I .
%m = O l' < j a + KK!" = &oor ime consan
'o( )(s)
=
n* (5)
)(s) #$s"
. .
Ea(s)
=
n--
Ea$s"
)(s) nKm
=
. .
Ea(S) s(l + %ms)
)(s) nKm
(here Ea(s) = K+ E(s)
=
K,E(s)
s(l + %ms)
C$s%
=
n Km K+ E(s)
s $&+ &" " m s)
n Km K+ Kl I $s" ' C$s""
)(s) =
s(l + %mS)
#
5(-+ %m5)+ n Km Kr Kl] n Km K+ Kl $s"
(. C$s" s $&+ %ms) = s $&+%ms)
C$s"
=
$s"
nKmK+K
5(-+ %ms)
nKmK+K
&+--.-= /000-,1
s(- +%ms)
2(s)
)'. . . . , . . . '*. " . . *'+
l+2(s)3(s)
45- $ , : r 6$
Control System Engineering 4-46 Mathematical Modeling of Systems
Block diagram :
R(s)
Fig. 4.54
'. Example 4.3: A d.c. motor drives Q pointer which is spring loaded, to retum to the
reference position. If Kb = Back e.m.f. constant, Kr = Torque constant K~ = Spring constant
and J =Moment of inertia. Find the transferfunction.
1
fo-----.
(M.U. : Nov.-93, Nov.-94)
Solution: Writig t!" #$#t"m "%&atio#.
'a(t)
ar,
=
( a )(( + *a dt+ s,
+akig *a,lac",
v,(5) = -.(/)0.12(5)3)45+/*a6
r,(5)
v,(5) - -.(/)
..
=
()a +#*.,) 7
No8 +m
=
9+(a, +m = Motor tor%&"
ad -.(/) 9. #:(#) a# -.
d"
=
oc -
dt
(a (#)
'a(#)- :(5)
..
=
(), +/ *a)
ad +m(#)
=
9+ 3'a (#) - 9./"(/)6
.()a +5*a)
..(1 )
7... (;)
.(3)
7.. (4)
.. (5)
<=1 , , : r I,
Control System Engineering 4-47 Mathematical Modeling of Systems
This torque is used to drive a pointer with inertia J and spring load of constant K",.
d20(t)
Tm = J dt2+ K0(l)
Taking aplace
Tm(s) = J !2 "#!$ + K, O(s)
%quating (5) and #&$
...()
KT
'
(a#s$ ) K* sees$+
!. "s2O(s) + K, . O(s) =
#$$ + sl.,
2 KTK*-.#-$) KT .
" s #!$ + /, 0#!$ + (# % ) & (# % ) '.,#-$
),0! a ),0- .1
So transfer function is,
"#!$
52J #2il +sa $ +K, #2, +sa$ + Kr K...s
KT
=
(a#-$
.#s$
(
(a#!$
,& E)am*le 4A: A d.c. generator supplies its output to a separately excited d.c. motor. The
field current of the motor is constant. The voltage applied to the field circuit of d.c. gene!ator
is eI (t). Write the differential equations of this Ward-eot!lrdsystem relating the inpuief (I)
to the output .m (t). ("efer to figure sho#n $elo#) %ence o$tain expression for "III (s).
#3.4.-3a5-.&#6T78$ and 9pril-.&#%lectricalJ $
Motor if Constant
8
e m
- --l +ard,%eonard system
Sol-tion! Torque produced .y :otor,
T = KT ;a#t$
(oltage applied to the :otor
d;,
%g#t$ ( r, 2a + Tt+ %*#t$
... (/)
... #2$
)<1 , & :r ;,
Control System Engineering Mathematical Modeling of Systems
Back emf is proportional to the angular velocity
E (t) = K dam(t)
b b dt
Torque produced T is used to drive a load of inertia J and friction B
T = J dZSm +B dOm
.. dt2 dt
.
.. ... ()
.!. (")
#quatin$ (1) and (4)
%Tla(t) = J d&0m +B dOm
dt& dt
J d1am B dam
la(t) = %T'(ft2+ %T"dt
... (5)
Substitutin$ in equation (2) 'e can $et resultant differential equation (relatin$ #$(t) and
Om(t).
)o'
ar*
#f+t) = ,f -* +.* dt
... (/)
... (0) and #$(t) = .Kg ,*(t)
.. Substitutin$ ,f+l) = 1t) in equation (/) 'e can $el* equation relatin$ #r(l) and
#$(t).
Substitutin$ this in equation (&) 'e can obtain the final differential equation relatin$
#*(t) and 2met).,t is very difficult to obtain in time domain so let us obtain it in .aplace
domain.
Takin$ .aplace transform of all the equations
T+s)
=
%Tr,(s)
#$(s)
=
la (3) 4-a 5 S .a65 #b(S)
#b(s)
=
%bSOm(s)
T (s)
=
em(s)4J s& +BS6
.. %T,a(s)
=
Om(s) 4J s +B6 S
,a (3)
=
Om(s)s4J s5 B6
..
%T
#$(s)
ames) s4J s 5 B64-a +S .a6
..
= %T + %bsOm(s)
and #(s)
=
7*(3) 4-r +s .t6
... (1)
... (&)
..()
... (")
... (3)
... (/)
... (0)
897 * , : r ,*
Control System Engineering Mathematical Modeling of Systems
Eg(s)
= K g If(s)
So If(s)
=
Eg (s)
~
Ef(s) =
Eg(s) [Rf +sLr]
() !!
K g
Substituting Eg (s) in equation (8) we g et
Ef(s) K g = Om(s) [sOS + B)~~a + S La + K bS]
[Rf + S L,l
(Rf + s Lf) seRa + s La)Os+ ")+SK bK !"
K g K ! Ef(s)
Om(S) =
#!$% Models of &hermal Systems
#!$%!$ 'eat &ransfer System
II
f
s(rro(nding tem)! % *
&hermal ins(lation
+ater
,(tlet -a~C. ==~II i / * In0et-ater
%#$% ~& ' = = = = ( ( t) l= = = = = = ( ( t( ( = = ( ( ( ( * ' + , - - .%
'eater
) ig - /- 00
1 t2e3mal s4stem use5 fo3 2eating flow of wate3 is s2own below-
Ele%t3i% 2eating element is 63o7i5e5 in t2e tan1 to 2eat t2e wate3- !2e tan1 is
insulate5 to 3e5u%e 2eat to t2e su33oun5ing s-
!2e ne%essa34 sim6lif4ing assum6tions a3e (
+ ) !2e3e is no 2eat sto3ag e in t2e insulation-
8) 1ll t2e wate3 in t2e tan9 is 6e3fe%tl4 mi2ed an5 2en%e at a unifo3m tem6e3atu3e-
&:+ , , : r ;,
Control System Engineering 4-50 Mathematical Modeling of Systems
01 = Inlet water temperature in"C.
00 = Outlet water temperature in"C,
o Surro unding temperature.
q
=
Rate o f heat flo w fro m heating clement inJ I sec.
qi =
Rate o f heat flo w to the water.
ql =
Rate o f heat flo w thro ugh tank insulatio n.
C
=
Thermal capacit in JI"C .
R
=
Resistance o f thermal insulatio n.
So rate o f heat flo w fo r the water in tan! is,
ae,
qi = C"""(ii'""
.. (1)
The rate o f heat flo w fro m the water to the surro unding atmo sphere thro ugh
insulatio n is,
... "#$
%s per the heat transfer principles,
q = qi +ql
Su&stituting equatio n "'$ and "#$
C
dOo 00(0
q = "ft)(R(
... (3)
... "*$
+eglecting the term O,R fro m the equatio n "*$ this is &ecause the -ariatio n o f water
temperature 00 is o -er and a&o -e am&ient temperature 0"'$.
q
= CdOo )Oo
.. dt R
Ta!ing /aplace transfo rm,
0"s1= CsOo "s$) Oo 2s$
3. Transfer functio n is,
~ - - - - - - - - - - - - - - ~
Oo "s$ R
=
0"s1 l)sCR
The time co nstant o f the sstem is RC.
45' , , : r I,
Control System Engineering 451 Mathematical Modeling of Systems
4.19.2 Thermometer
Consider a thermometer placed in a water
bath having temperature eJ' as shown.
00 is the temperature indicated by the
thermometer. The rate of heat flow into the
thermometer through its wall is
d!
di=
"here # $ Thermal
resistanceof the thermometer wall.
The indicated temperature rises at a rate of
au 1 dq
crt $ Cdt
where C is thermal capacity of the thermometer.
d: to $ b . [ O i % 00&
Ta'ing (aplaceof the e!uation
1
s)* +5, $ #C -)i+.,/0+5,&
0o+s, 1
$
01+s, l2s#C
The time constant is C.
4.2! "ct#ators
$ath
3ig. 4.54
The actuator is a device which receives input signal hom the controller and it
produces the input signal to the plant according to control signal so that the output will
approach the reference input signal to reduce the error to 5ero. Thus an actuator is
generallyafter the controller and before the plant in the control system. 6n actuator can be
of t%o types 7
i, 8ydraulic actuator 9, :neumaticactuator
;<1 , : r =
Control SystemEngineering 452 Mathematical Modeling of Sy.....
4.20.1 Hydraulic Actuator
The structure of an hydraulic actuator is shown inthe Fig. 4.57.
Input--il----+-il------t-t------i--t--t-
displacement
Mainpiston
Tosump From i!
pressure source
To sump
"al#episton
Fig. 4.57
They are piston devices in which motion of the spool regulates the oil flow to either
side of the power cylinder. hen the spool moves to the right! the high pressure oil enters
the power cylinder to the left of the piston.
Te differential pressure produced causes the power piston to move to the right!
pushing te oil infront of it into the sump.
The load coupled rigidly to the piston moves a distance y from its reference position
corresponding to the displacement " of the valve piston from its neutral position. The oil is
pressurised #y a pump and is recalculated in the system.
$%uationof motion and transfer function
The rate of flow of oil into the piston is proportional to the rate of the movement of
the piston.
dy
& $ A dt% A $ area of piston
.'. (1)
If & is the differential pressure across the piston then the force on the piston is '&.
This moves the load of mass M against friction (.
d2 * dy
' + & $ M dt2 +( Cit ... )2*
For small values of the displacement X, if & is the differential pressure on the piston
and +is the oil flow then!
..,)-*
dy
,l+-A
dt
./0 ! , : r 1!
Control System Engineering 4-53 Mathematical Modeling of Systems
Taking Laplace transform,
K M K.,B
+S2 Y(S)+ --j:-S YeS) K) !(s) -"# Y(s) s
$es) %s2 K2 M+ K2 Bs-i- &2 s' K) A !es)
$es) K(&
!es) S2 K2 M+sK2 B+&2 S
$es)

!es)
(* 2&)
T+is transfer f,nction is similar to t+e electricmotors.
4.20.2 Pneumatic Actuator
-ne,matic acting .al.e is ,se/ to
o0tain linear /isplacement of a pl,nger
1it+ press,rise/ air as inp,t.
T+e air at press,re P is injecte/
t+ro,g+ inlet. -ress,rise/ air p,s+es t+e
/iap+ragm an/ pl,nger. T+e pl,nger has
a mass M an/ friction on B 1it+ spring
constant K.
Let A be t+e area of /iap+ragm t+en
transfer f,nction can be o0taine/ as
0elo1.
2orcee3erte/ on t+e pl,nger is & X -.
This force is oppose/ by mass, friction
an/ spring.
Fig. 4.5
Taking Laplace transform,
A -(s) MS2 Yes) +Bs Y(s) +K$es)
Y(s) A

pes) Ms2 + Bs+ K


T+e a/.antages of pne,matic s$stems are fire proof, e3plosion proof, simplicit$ an/
eas$ to maintain.
4#' , , : r I,
Control System Engineering 4-54 Mathematical Modeling of Systems
4.20.3 Comparison between Pneumatic and Hydraulic Systems
Sr.o. Pneumatic systems Hydraulic systems
1. !he fluid used "s air. !he fluid used is oil.
2. #ir is compressible. $il is incompressible.
3. #ir does not ha%e lubricatingproperty. $il acts as a lubricator.
4. !he output power is much less comparedto !he output power is much higher than
hydraulic. pneumatic.
5. #t low %elocities& the accuracy is poor. #t all %elocities& the accuracy is satisfactory.
6. o return pipes are re'uired when air is used. !he return pipes are must.
7. Can be operated for the temperaturerange of "t is sensiti%e to the temperaturechanges and
00 Cto 200 OC. It is insensiti%eto temperature the range is 20 OC to 70 OC.
changes.
8 . !hese are fire proof and e(pl$Sionproof. !hese are not fire and e(plosion proof.
9 . Easy from maintenancepoint of %iew. )ifficult from maintenancepoint of *iew.
ote + The state space method of modeling the systems is separately co%ered in the
chapter 15.
E(amples with Solutions
E(ample 4.,+ The voltage generated by a d.c. generator is filtered by a luw p ass filter
consisting Rz and Lz as shan," below. This filtered voltage is controlled by the voltage
ap p lied to the field of genenuor.
If Rf = 40 n, Lf -60 H, Ro = 0.5 n, Lo = lH, la = IH, Rz = 2n and
gerator constant !g - "#0 $ I field %p . &eterine the transfer function
E2(s)
'I(s) of the s*ste.
(M..! "e#.-$7%
r - - - - - - - - -
.&
Ra
&a
t
'% ~)
!
e,(t%
/ r
e20t1
2
1
.._ - - - - - - - -
/ ow)ass filter
Constant s)ee*
+,1 , ,+ r I,
Control System Engineering 4-55 Mathematical Modeling of Systems
Solution: For field circuit, ... (1)
For armature circuit,
For generator, Ca (l) = K g ir(l) ... (3)
... (4) For output, ez(t) = iz{t) R2
Taking Laplace transform of all the euations,
!r{s) = ",(s) lR, + s "., #
!a ($) = %2(s) &(Ra ' R2) ' s (La ' %(2)#
s,($) = ) K g %,($)
... (5)
... (6)
... (*)
... (8) !2(+, = %2(+, R2
!uation (-) and (*),
K g lr(s} = h(s, l(RII'R2)'s(L,,'L2)1
+u.stituting la ($) from euation (/) and %,($) from euation ($),
, & !f{s) # !2(+)
0 R L = (R( l(R., ' R2)'s(La + L2)#
1" + + f 2
+u.stituting %2(+, from euation (/) and "f(s) from ($),
!2(+, 2 3 g R2
!c(s) ( (R, + + Lr) &(3 , + R2)' s(La + L2)#
+u.stituting the 4alues,
r - - - - - - - - - - - - - - - - - - - - - - - - - - - - - - - - - - - - - ~
E2(s)
=
Ef(s)
%2562
=
(45 ' -5s) &(2.$' 2s)J
2.4
(% + %.$$) (% + U .8; ; )
,_ Example 4.6: A high gain speed control system uses a tadtogeneraior for speed. sensing.
The tachogenerator produces $ V per %55 r.p.m. This voltage is compared witll reference
voltJlge to produce error signal. If the reference voltage is set to 10.8 volt. What is the value
of expected speed? (7.8. ) 9o4.(:4)
Solution: The tachogenerator produces $ ; per %55 r.p.m.,
"ts constant is
$
%55 = 5.5$ ;"r.p.m.
2<% , , : r I,
Control System Engineering' I. 4-56 .J Mathematical Modeling of Systems
.1
Now for the exected seed! error sho"ld be #ero!
i.e $r - tachogenerator o"t"t % error
&achogenerator o"t"t % $r
'et N (e exected seed inr..m.
N x ).)5 = 1).*
N % +16r..m. ... Exected seed
I, ... Examle 4.,- An armature controlled d.c. motor has .tf armature resistance of 0.37 n.
The moment of inertia is 2, 5 x 10-6 kg - m2 A back emf of 2.09 V is generated p er 100
rp m. o f the motor sp eed. !ire tor"ue constant of the motor is 0.2 #-m $A. %etermine the
transfer function o f the motor relating the motor shaft shift and the inp ut &oltage.
/M.0. - 1ec.-263
Sol"tion - &he motor can (e shown as!
.a
G : -
1
4 ./1'3
$a/t3
l o a d
1
J
Now (ac5 emf is +.)2 $ er 1)) r..m. ! N % 1)) r..m. is!
+1tN
(1 ) % 666) % 1).4,12rad7sec
:. Back emf constant x! % l894)92% ).1225$7rad7sec
:& % ).+ N-ml ;
<or armat"re circ"it $a/l3 = =a .a + E(
$ats3 % =a/s3 .a +E(/S3
... (1 )
&a5ing 'alace
... (2)
&a5ing 'alace E(/S3 = :(S6 /s3
&m = :& loa/t3
... (3)
>?1 ! ' r =!
Control System Engineering 4-57 Mathematical Modeling of Systems
Taking Laplace 1".n(5) = Kr I., (s)
This torque ri!es a loa o" inertia # .
Taking Laplace Trn(s) # s2 $ (5)
.. Klla(s)
=
# %&$ (s)
"a (s)
&.5X 1'-6
52 $ (s) = 1.&5 X 1'-5 S2 $ (s)
=
..
'.&
.. (.,(s)
=
1.&5 X S2 1'-5$ (s) X '.)7 +K* 5$ (s)
.. (a(%)
=
4.+&5 X 1'"" %&$(5) +'.1$$5s$(s)
..
$(s)
1 &1+&1+.&&
= =
(a(s) 4.+&5X 1'-+ S2 +'.1$$5s s(s +4)1)5.1)5)
..... , I h h bid . -,(5)
II...,... xamp e 4../ For t e system s own e ow eiermine E- ( ).
_f 5
ett)
Separately excited d.c. generator
Solution
0or "iel circuit,
i(
ef (t) = if (t) 1f +L, dt
... (4)
(2.3. / 4ec.-$7)
0or ar5ature circuit,
0or generator,
0or output,
6a (t) = !g i"(t)
6&(l) = i&(t71&
891 , , : r I,
Control System Engineering 4-58 Mathematical Modeling of Systems
Taking Laplace of all the equations we get,
Et(s) = If(S) [Rf + s Lr]
E,,(s) = I;(5) !("a + "#+ s(La + L)$ = Kg I%(s)
E(S) = # (5) "
&ence equating Ea (s) equations,
' g If(s) = #(s) !# (s)("a + "( + s(La + L)$
an* hence using +alues of If(s) an* #(s,f%o-%e-aining equations we get,
'.. Eam!le ".# $ A %& '() * phase a.c. servomotor has tile following parameters :
Starting torque = 0.186 .-
Rotor inertia = # /l0-5 kg-m2
Supply voltage = 120 V
o load angular velocity = 124 ra!"s
Assuming straight line torque#spee! characteristics of the motor an! $ero friction% o&tain its
transfer 'unction. (3ate)
Sol+tion$ ,he sta%ting to%que is nothing 4ut loc-ed %oto% to%que(
Locke*%oto% to%que 2(#85
Ktrn = "ate*+oltage = #2
( 1.))*l0#3
Let m4e the slope of linca%ise* to%que-spee*cha%acte%istics(
Locke*%oto% to%que 2(#85
.o loa*angula% spee* = 124
- =
= . 5(##8/l0-4
The to%que at an6 angula% spee* (7) is gi+en 46,
. dB
,m = Kim E# +md't
... (/)
89# , %:r I,
Control System Engineering 4-59 Mathematical Modeling of Systems
where 21 = rotor voltage
Taking Laplace, Tm(s) =Kim E21(s) + m sO(s)
This torque is used to drive load of inertia 1m- The friction is given zero.
... (!
d"
!
Tm
"" #
# #
1m dt
"" Tm(s)
#
s2$mO(s%
$quating (! and (4!,
KlmE21(S)+ msO(s)
#
s2$mO(s)
"" Kim E2&(s)
#
sO(s) &s$m'm(
... (%!
... (4!
&ence the transfer function of the motor is,
1.55'l(-%
#
s)l'l(-5 s-(-*.11+'l"-,-!
O(s%
#
$t(s)
155
s(s +*1.1+!
This is the required transfer function.
. . E)am*le +"1,- . two phase a.c. servomotor having a torque constant of 0.045 Nm/V
controls a position load through a gear ratio of 10:1. The effective moment of inertia and
coefficient of viscous friction referred to load side are 0.25 kgm2 and 1.0 Nm/!rad/sec". The
s#nchro error detector produces an error signal of $.lV per degree error in misalignment.
%evelop the &lock diagram representation of the control s#stem and there from o&tain the
transfer function. (/.0. 1 /a2-99!
Sol/tion- 3or the error detector, 4. # ".1 01 degree error
.. 4l # ".1' 1+"
1t
#
5.5956 7radian
Kim
#
Torque constant
#
"."458m76
8. 1
8
#
1"
. L #
".59g .. m

on load side
(8 r 1 ..
J e q
# /otor side # 81 ' :t, # 1""' ".5
;<1 , ' :r :,
Control System Engineering 460 Mathematical Modeling of Systems
=
2.5x 10-3 kg-m2
BL
=
1 N-m/(rad/sec)
Beq Motor side =( ~ ~ r x B,
1
..
=
= 100x
= 0!01 N-m/ (rad/sec)
"#e $ock diagram o% t#e s&stem is,
Nt
'(() = Kt x motor "!)!x N2
)or motor,
"m = *+m ,2
.. "m(s) = *tm ,2 (()
a-d "m(t) =
d2e de
.eq dt2 + Beq dt
.. "m(s) = (s2 .cq + s Bcq)e(s)
... ( 1
!!! (2)
,q/ati-g eq/atio-s (1) a-d (2),
*im ,2 (s) = (s2 .cq + s Bcq)e(s)
0(s) = *im 1 0!04( 1 12
,2(() s(s.eq 3 Beq) - s(sx 2!(x 10-3 3 0!01) - (((3 4)
!( s
= 4!526(x 12 x!!7!!!= 10!3131
s(s3 4) 10 4((3 4)
10!3131
((43 4)
13 10!3131
4((3 4)
10!3131
= !!!!!!8!!!!----
423 4s3 10!3131
191 , , : r +,
. "
Control System Engineering 4-61 Mathematical Modeling of Systems
Review Questions
1. Explain the derivation of tm410gous networks using
i) F orc e- voltage ii) F ora- c urrent analogy
2. Write a short note 071 direc t and inverse analogous networks.
. !raw the tm"llogous elec tric al nehvorks #ased on
a$ F- % analogy #$ F- " analogy o f the following mec hanic lll systems
B
\
I-_f(t)
K
81
_\1 , , : r I,
Control System Engineering 4-62 Mathematical Modeling of Systems
4. Distinguish between A.C. servomotor and D.C. servomotor.
5. Derive transfer /unction o f a.c. servomotor stating the tlSSumption made.
6. State the applications of a.c. SmJOmotor.
7. Derive the transfer /unction of field controlled d.c. seroomotor.
8. Ansrver the following gimng reasons :
a AC. seroomotor has a smaller diameter and more length.
b) !ield controlled D.C. str"Omotor is pref emd than armature controlled.
9. Develop bloc# dillgram for armature controlled D.C. servomotor and find its transfer function.
10. An armature controlled d.c. motor is supplied in series with a resistance from II
$4 " d.c. suppl%. The motor ta#es current of 5A on stalling and the stall&'(g tor)ue bei*g +.,-5
./m. 0he motor runs at 1000 r.p.m. ta#ing a current of 1 A. The value of I =4 X10-3 1g 2 ",1
and friction constant as 1.5 x10- 3.m/ 3rDd/sec
o (s) 2.9
(Ans.: V(s) =$(1+0.3s)'
--. for the closed loop s%stem shoo(ll below' draw the l(loc# diagram and determine
transfer fundion O~(s)/O,(s). 0he give* values are 4rror defector gain K.. = 8 "/rad' Amplifier
gain KA = 10 "/5 R, = 5 n, ! = +.$5 6' Kr =+.+5 .m/A' " # .,'o$ = +.+$ 1g / m2,
Determine the transfer /unction of the motor.
h, =31g / # 2 %O&o! o$ =+.+7 .m/3rad /sec.
% = 5.5 '# / 3rad/sec
&( Constant
+
0(1 ....
(A'.: Oc(s) = 32.12 )
9,,(5) (s& + 21.551 +30.35 +32.12)
12. 0he moment of inertia " # and the coefficient o f viscous friction %n. for IIfield controlled d.c. motor
are motor respectivel% 5 )1O-' *+ - m2 (n, 12.5)&O-''# 3tad/sec. 0he motor tor)ue constant
I{, being 2.5 '# /A. Determine the trdnsfer function rtlating th4 angular sp eed of the shaft and
the field current. (Ans! "" #$ 1!~~J
- '. , ':r( '
Control System Engineering Mathematical Modeling of Systems 463
13. Figure given below shows a block diagram oj speed regulator system. The accelertlting t01'queis the
difforence between ti,e torque developed by the controller and load TL. he controller gain is
0.00102 !m"rad lind the tachogenerator constant is 0.191 #"rad"sec. TIll! load speed is adjll$ted to
1000 r.p.m. he moment of inerti% is J and frictiim is negligi"&'e.
(alcull)te a* TIll! rtference voltage. 1,) ile speed if a constant load torque oj 0.001 Nm is
suddenly applied. (Ans.: a) 20V b}951 r.p.m.l
+
V,.(s)
14. Aninstrument servo con$isting of motor, spring loaded shaft, etc is SJ r(w1I below
R=1rad +
Where V = #oltage in volt+
R= ,otor resisumce 1 Q
L = ,otor inductance 0.1 H
K" = -l1'pr.fiergaitl =10 #"#
K = SprirrK COflsttlflt = 0.001 m!ra"
K" = /ack em#. rOflstar" = = 0.01 # (rtl"!sec$
KT = orque content = 0.01 !0m"-
J = ,oment of inertia = 0.001 Nms2
If the input is 1 Til"fullS% wl2lt l$ the steady state error3
1 1
(Ans. : !...= " K;1(:.# =1(1)
1+(Kx It)
$%1 , , : r I,
Control System Engineering 464 Mathematical Modeling of Systems
15. The positions serrxnneclurnism is shown in figure below. 6 r is refmnce position and 9 c : actual
tmgular displtzttmmt 0 / shaft inradUms. Obtmn its closed loop trtJns/tr function ~~:~
+
V-=-
I.~
I f constant !l
Give"
a
=
angular displacement 0 / the motor sha f t
K, = gain / error detector = 7.64V/ rad
Ra =
212, lu =mgligible
Kb
=
5.5 x1Q-2V/ rad/ sec
KT
=
6x10 -S m/!
J m
1 xl-! "# - r$2
f m
=
mgligible
J L =
4.4 X10 -3 "#- m2
f t.
=
4 X10 -2 %m/ &rad/ sec '
%t
=
1
%2
=
10
&($s.. ac &s' )= 42.* '
+ ,-s' 52 ./7.7 s . 42.*
"#. $oive the transfer function of a t%pical d.c. position control s%stem.
"&. $erive the trlm'/tr function of a t%pical d.c. speed control s%stem.
"(. Obtain the )*+lfhenurtical model oJ he,t transfor s0stem.
"-. Obtllin the rrurtlrmrllficlllmodel 0 / thermometer.
./. W 1u d is actu!tor 1 0xplain h%drtl+lit and pneumatic actuator s%stems.
.". 1ompttre h%drllulic a nd pnerlmlltic actuators.
(J(JO
r
"
Block Diagram Representation
5.1 Background
If a given system is complicated, it is very difficult to analyse it as a whole. With the
help of transfer function approach, we can find transfer function of each and every
element of the complicated system. And by showing connection between the elements,
complete system can be splitted into different blocks and can be analysed conveniently.
nus is the basic concept of block diagram representation.
Basically block diagram is a pictorial representation of the given system. It is very
Simple way of representing the given complicated practical system. In block diagram, the
interconnection of system components to form a system can be conveniently shown by the
blocks arranged in proper sequence. It explains the cause and effect relationship existing
between input and output of the system, through the blocks.
o draw the block diagram of a practical system, each clement of practical system is
represented by a block. he block is called functional block, It means, block explains
mathematical operation on the input by the element to produce the corresponding output.
he actual mathematical function is indicated by inserting corresponding transfer function
of the element inside the block.
!ey "oint# For a closed loop systems, the function of comparing the different signals is
indicated by the summing point while a point from which signal is taken Jor the feedback
purposeis indicated by takeoff point in block diagrams.
All these summing points, blocks and takeoff points then must be connected exactly as
per actual elements connected in the practical system. he connection between the blocks
is shown by lines called branches of the block diagram. An arrow is associated with each
and every branch which indicates the direction of flow of signal along the branch.
!ay "oint# The signal can travel along the direction of an arrow only.
It cannot pass against the direction of an arrow, $ence block diagram is an unilateral
property of the system.
(5 - 1)
r

Control System Engineering 52 Block Diagram Representation


In short any block diagram has following five basic elements associated with it :
1) Blocks 2) Transfer functions of elements shown inside the blocks.
3) Summing oints !) Takeoff oints ") #rrows.
5.1.1 Illustrating Concept of Block Diagram Representation
Pneumatic
valve
$%) Controller
ig. ".1 !i"ui# level control system
&or e'amle : (onsider the li)uid level system as shown in the &ig. ".1. So to
reresent this by block diagram* identify the elements which are*
i) (ontroller $ti% +neumatic valve $iii% Tank $iv) &loat.
In this system* the level of water is sensed by the float. ,ence the float osition acts as
the feedback. #ccording to the float osition* with resect to desired level of water* the
controller oerates the neumatic valve controlling the flow of water in the tank. -hen the
re)uired level is reached* controller oerates the neumatic valve in such a way that t&e
flow of water in the tank* stos. . the outut from the tank is taken i.e. the water from
the tank is drained then the float osition changes from the desired osition and
accordingly the controller oerates the neumatic valve to start the flow of water in the
tank.
,ence indicating all the elements by blocks* the block diagram of the system can 'e
develoed as shown in the &ig. ".2.
(l)ar# ath
g/ . * .... u .'0/ / :1
+ctual
level
Summing
point
2
ee#'ack pat&
0i0 0 I
ig. 5.2 Block #iagram of li"ui# level control
r
,
Control System Engineering 5-3 Block DiagramRepresentation
Consider another example of bottle filling mechanism. When bottle gets filled by the
contents upto the required level it should get replaced by an empty bottle. This system can
be made closed loop and hence can be as shown in the Fig. 5.3.
.---- .....----, Valvecontrolling
signal
Tank
sensor belt
Fig. 5.3 !tomatic bottle "illing mec#anism
n the system shown! conveyor belt is driven by the controller as well as valve
position is also controlled by the controller.
When empty bottle comes at the specific position! weight sensor senses the weight and
gives signal to controller. Controller stops conveyor movement and opens the valve so
bottle starts getting filled. When required level is achieved! again weight sensor sensing
the proper Weight sends a signal to controller which sends signals to start movement of
belt and also dosing t#e valve position with proper time delay till next empty bottle
comes at the proper position.
This system can be represented by a bloc" diagram as shown in the Fig. 5.#.
$a%
$e
sign
Comparator
%ositiono" valve
&
att#e'tank
erence(
Fille)bo
Controller r.-
*g#t
+, -
Belt )riving
mec#anism
F
$eig#t
sensor
ttle
ee)back
Fig. 5.- Block )iagram
r
.
Control System Engineering 5.4 Block Diagram Representation
5.1.2 Advantages of Block Diagram
The various advantages of block diagram representation are,
1) Verysimple to construct the block diagram for complicated systems.
2) The function of individual element can be visualised from block diagram.
! Individual as ell as overall performance of the system can be studied by using
transfer functions shon in the block diagram.
4) !verall closed loop T.". can be easily calculated by using block diagram reduction
rules.
5.1.# Disadvantages
The various disadvantages of block diagram representation are,
1! $lock diagram does not include any information about the physical construction of
the system.
2) %ource of energy is generally not shon in the block diagram. %o number of
different block diagrams can be dran depending upon the point of vie of
analysis. %o block diagram for given system is not uni&ue.
5.2 Simple or Canonical "orm of Closed #oop System
$ey %oint& A block diagramin whic/l,forward path contains only one block,feedbackpath
contains only one block, one summing point and one takeoff point represents simple or
canonical form of a closed loopsystem.
' can be acl(eved by umng
block diagram reduction rules R's!
ithout disturbing output of the
system. n(s form is very useful as its
closed loop transfer function can be
easily calculated by using standard
result. This result is derived in this
E's!
)'s!
11***.**+C,s!
)*+*)
-'s!
section.
The simple form can be as shon
in the "ig. 5.5.
"ig. 5.5
here , ,+s) -) .aplace of reference input r+t)
/+s) *+ .aplace of controlled output c+t)
0+s) -) .aplace of error signal e+t)
$+s) -) .aplace of feedback signal b+t)
1+s). *+0&uivalent forard path transfer function
2+s) -) 0&uivalent feedback path transfer function.
r
.
Control System Engineering 55 Block Diagram Repl'8Sentation
Key Point: G(s) and H(s) can be obtained by reducing complicated block diagram by using
block diagram reduction rules.
5.2.1 Derivation of T.F. of Simple Closed Loop System
Referring to te Fig. 5.5! "e #an "rite follo"ing e$%ations as!
&'s(
)
R's( * +'s( .. (1)
+'s(
)
C's(,'s( ... '2(
C's(
)
&'s(-'s( .(3)
+'s(
)
C's(,'s( and s%.stit%ting in e$%ation '1(
&'s(
)
R's( ) 1) C's(,'s(
&'s(
C's(
)
G(s)
C's(
R's( ) 1) C's(,'s(
G(s)
)
C's( =
R's(-'s( =t C's(-'s(,'s(
:. C's( '1 !-'s(,'s/( = R's( -'s(
C's( -'s(
R's( ) 10C's(,'s(
1se " sign for negative feed.a#2 and %se 3 sign for positive feed.a#2.
n%s #an #e represented as so"n in te Fig. 5.4.
R(s) $$$ ... % &::5 S(
Close' (oop ).*.
+$$$,'-C(s)
Fig. 5.4
Key .oint This can be used as a standard result to eliminllte such simple loops in a
complicated system reduction procedure.
/.3 R0les 1or Block Diagram Re'0ction
6ny #ompli#ated system i1 .ro%gt into its simple form as so"n in te Fig. 5.5! its
T.F. #an #e #al#%lated .y %sing te res%lt derived eariler. To .ring it into simple form it is
ne#essary to red%#e te .lo#2 diagram .%t %sing proper logi# s%# tat o%tp%t of tat
system and te val%e of any feed.a#2 signal so%ld not get dist%r.ed. Tis #an #e
a#ieved .y %sing follo"ing matemati#al r%les "ile .lo#2 diagram red%#tion.
r
,
Control System Engineering 5-6 Block DiagramRepresentation
Rule 1: Associative law : Consider two summing points as shown in the Fig. 5.7.
Fig. 5.7
Now change the position of two summing points. Output remains same.
So associative law holds good or +
summing points which are directl! R1 ~+ R1: R3 ~_
connected to each other "i.e. there is no
intennediate #loc$ #etween two summing
points or there is no ta$eo point in
R3 R
#etween the summing points%.
Fig. 5.&
Consider summing points with a #loc$ in #etween as shown in t!e Fig. 5.'.
"ey #oint: So associative law
is applicable to summing points
which are directly connected to
each other.
Rule ( : For #loc$s in series :
)he transer unctions o the #loc$s which are connected in series get multiplied with
each other*
Now interchange two
summing points* R1
--++..
Now the output docs not
remain same.
,
Fig. 5.'
+
Fig. 5.1-
r
$
---- --- -----
Control System Engineering 57 Block Diagram Representation
Consider system as shown in the Fig. 5.11.
R(s)
Fig. 5.11
C(s) = R(s) [Gl G G!"
#o instead o$ three di$$erent %locks& only one %lock with '.F. [G1 q 2 G !" can %e (sed
as shown in the Fig. 5.1.
R(#)
II
C{s}

Fig. 5.1
)(tp(t in %oth cases is same.
Key Point: If there is a taJ c eo f f or a summing po int in beiueen the blo c ks the blo c ksc anno t
be said to be in series.
Consider the com%ination o$ the %locks as shown inthe Fig. 5.1!.
Fig. 5.1!
*ey +oint, In this c o mbitUltio n G-G are in series Imd c an be c o mbined as G1G but G!
is no w no t in series with G.G as there is takeo f f po int in between.
'o call G! to %e inseries with G.G it is necessary to shi$t the takeo$$ point %e$ore
G1G or a$ter G!. 'he r(les $or s(ch shi$ting are disc(ssed later.
R(le ! , Fat %locks in parallel,
'he trans$er $(nctions o$ the %locks which are connected in parallel get added
alge%raically (considering the sign).
Consider system as shown in the Fig. 5.1/.
r
"
Control System Engineering 5-8 Block Diagram Representation
R(s)
R(s) G2
C(s)
R(s)G3
Fig. 5.14
C(s) = - R(s) G1 + R(s) G2 + R(s) G 3
= R(s) [G2+G3-Gl]
Now replace three blocks with onl one
block with !"#" G2 G! " G1 (#i$" 5"15)
C(s) #$$#$ R(s) %G2 G3-G%]
R& (& S)& .& "' ' ' G& G3'( ' ' ' """"( ) s)
Fig. 5.15
)*tp*t is sa+e" So blocks which are in parallel $et a,,e, al$ebraicall"
%,enti- the blocks in parallel correctl" !he con-*sin$ cases are ,isc*sse, here" .et
there e/ists a takeo-- point as shown in the #i$" 5"10 alon$ with blocks G1* G2 which
appear to +e in parallel"

Fig. 5.1,
-ey .oint$ But unless Imd until this takeoff point is Shifted before the block, blocks cannot be
said to be in parallel.
1hi-tin$ o- takeo-- point is ,isc*sse, ne/t" 1econ,l shi-tin$ a takeo-- point a-ter a
s*++in$ point nee,s so+e a,2*st+ents to keep o*tp*t sa+e" ' n abo3e case the takeo--
point cannot be shown a-ter s*++in$ point witho*t an alteration" !his tpe o- shi-tin$ is
,isc*sse, as critical r*les later as s*ch shi-tin$ +akes the block ,ia$ra+ co+plicate, an,
sho*l, be a3oi,e, as -ar as possible"
r
/
IIII Control System Engineering
" ,
+
(a) Avoid such shifting
as far as possible
59 Bloc !iagram "epresentation
#ot valid
$ig% 5.17
(b) &ithout any alteration
such shifting is invalid
Similarly consider a configuraion as shown in the Fig. S.lS.
This combination is not the parallel
combination of GI and I.
'ey (oint) For a parallel combination
the direction of signals through the blocks
in parallel must be same.
Inthis case direction of signal through
GI and I is opposite. Such a
combination is called minor feedbac!
loop and reduction rule for this is
discussed later.
$ig% 5.1"
"
II
*
#
$
+ C,s)- "*+y
%
$ig% 5.19
&ule ' ( Shifting a summing point behind the bloc!(
C(s) ) "* + .
*ow we ha+e to shift summing point behind the bloc!.
'ey (oint) The output must remain same, while shifting a summing point.
,& + -.G /# 0,s.
$ig% /%01
r
"
Control System Engineering 5-10 Block DiagramRepresentation
RG+xG = RG+y
xC = y
:. x = ~ so signa] y must be multiplied with ~ to keep output same.
R
C(s)=RG+y
Fig. 5.!
Thus while shifting a summing point behind the blok i.e. befo!e the blok" add a
blok ha#ing T.$. as !eip!oal of the T.$. of the blok befo!e whih summing point is to
be shifted" in se!ies with all the signals at that summing point.
%ule 5 : &hifting a summing point beyond the blok :
Conside! the ombination shown in the 'ig. 5.((.
C=(R +y)G
Fig. 5.
)ow to shift summing point afte! the blok keeping output same" onside! the shifted
summing point without any hange.
R
..
C=RG+yG
Fig. 5."
#. RG* x = RC * yG
x = y+
i.e. signal y must get multiplied with T.'. of the blokbeyond whih sumnting point is
to be shifted.
r
$
Control System Engineering 5-11 Block Diagram Representation
R
C=RG +yG
=(R +y)G
Fig. 5.24
Thus while shifting a summing point after a block, add a block having T.F. same as
that of block after which summing point is to be shifted, in series with all the signals at
that summing point.
ule ! " #hifting a takeoff point behind the block"
$onsider the combination shown in the Fig. 5.25.
R
I I I I
RG

G
L
.C
Fig. 5.25
$ = RG

%
RG
!ey "oint# To shift takeoff point behind the block villue of signlll takingoff must remain
same.
Though s$i%ting of takeoff point without an& change does not affect output directl&,
the value of feedback signal which is changed affects the output indirectl& which must be
kept same. 'ut without an& change it is (ust as shown in the Fig. 5.2!.
_ - - - - _
#,,- '.... ,
& R & & ' ( ' ' ) ) ) G I I * +
,
C=RG
y=R
Fig. 5.2!
'ut it must be e)ual to *. #o a block with T.F. * must be introduced in series with
signal takingoff after t$e block.
Thus while shifting a takeoff point behind the block, add a block having T.F. same as
that of the block behind which takeoff point is to be shifted, in series with a+ the signals
takingoff from that takeoff point.
r
*
Control System Engineering 5-12 Block Diagram Representation
R
t---..C::RG
Fig. 5.27
Rule 7 : Shifting a takeoff point beyon the blo!k:
Con"ier the !ombination "ho#n in the Fig. 5.2$.
R
to
y=RG ~ =R
Fig. 5.2$ Fig. 5.2%
&o "hift takeoff point beyon the blo!k' (alue of )y) mu"t remain "ame. &o keep (alue
of )y) !on"tant it mu"t be multiplie by )l*G).
+hile "hifting a takeoff point beyon the blo!k' a a blo!k in "erie" #ith all the
"ignal" #hi!h are takingoff from that point' ha(ing &.F. a" re!ipro!al of the &.F. of the
blo!k beyon #hi!h takeoff point i" to be "hifte.
Rule $ : Remo(ing minor feeba!k loop :
This in!lue" the remo(al of internal "imple form" of the loop" by u"ing "tanar
re"ult eri(e earlier in "e!tion 5.2.
Key Point: After eliminating such a minor loop if slimming point carries only one signal
input and one sigmzJ .output, it should be removed from the block diagram to avoid further
confusion.
C'(s)
G),S-
C'(s)
:!:G'(s) "'(s)
:!:
Fig. 5../
r
#
Control System Engineering 5 -13 Block DiagramRepresentation
Rule 9 : For multiple input system use superposition theorem :
R,
~91
I I
R2
System C
Rn
Fig. 5.31
Consider only one input at a time treating all other as zero.
Consider R1, R2 = R3 = R" =0 and ind output Ct.
R! = R" = Rn =0 and ind output C2. #hen $onsider R2!
%t the end &hen all inputs are $o'ered! ta(e alge)rai$ sum o all the outputs.
#otal output C * Cl +2 Cn
,ame logi$ $an )e e-tended to ind the outputs i system is multiple input multiple
output type. ,eparate ratio o ea$h output &ith ea$h input is to )e $al$ulated! assuming all
other inputs and outputs zero. #hen su$h $omponents o outputs $an )e added to get
resultant outputs o the system. .n 'ery e& $ases! it is not possi)le to redu$e the )lo$(
diagram to its simple orm )y use o a)o'e dis$ussed nine rules. I n su$h $ase there is a
re/uirement to shit a summing point )eore or ater a ta(eo point to sol'e the pro)lem.
#hese rules are dis$ussed )elo& )ut reader should a'oid to use these rules 0.!.C5, and
until it is the re/uirement o the pro)lem. 0se o these rules in simple pro)lems ma'
$ompli$ate the )lo$( diagram.
5.3.1 Critical R!les
Rule 10 : ,hiting ta(eo point ater a summing point. Consider a situation as sho'
in Fig. 5.32.

""1 2$
y z
Fig. 5.32 Fig. 5.33
3o& ater shiting the ta(eo point! let signal ta(ingo )e 4z4 as sho&n in the
Fig. 5.33.
3o& z = Rl "y
r
#
Control System Engineering 514 Block Diagram Repntlentatlon
But we want feedback signal as x = Rl only.
So signal 'y' must be inverted and added to CI to kee feedback signal value same.
!nd to add the signal" summing oint must be introduced in series wit# takeoff signal. $o
modified configuration becomes as s#own in t#e %ig. 5.&4.
:!:
+
:!:
y
%ig. 5.&4
Rule 11' $#ifting takeoff oint before a summing oint'
Consider a situation as s#own in t#e %ig. 5.&5.
.. c= R1.:!:y
x= R1.:!:y
%ig. 5.&5
(ow after s#ifting t#e takeoff oint" let signal takingoff be )* as s#own in t#e
%ig. 5.&+.
(ow , = Rl only because not#ing is c#anged.
R1
"

!=R1
%ig. 5.&+
But we want feedback signal x w#ic# is Rl - y. .ence to ," signal *y* must be added
wit# same sign as it is resent at summing oint" w#ic# can be ac#ieved by using
summing oint inseries wit# takeoff signal as s#own int#e %ig. 5.&/.
r
"
Control System Engineering 5-15 Block Diagram Representation
+
J---- C=R1y
:!:
v
Fig. 5.37
Thus it can be noticed that shifting of takeoff point before or after a summing point
adds an additional summing point in the block diagram and this complicates the block
diagram. No doubt, in some rare cases, it is not possible to reduce the block diagram
without such shifting of takeoff point before or after a summing point. part from such
cases, do not use such shiting! which "ill complicate the !imple block diagrams.
#ey $oint: Do not use these rules unless and until problem really needs them.
5.3.% Con&erting 'on(nlty Fee)*ack to +nity Fee)*ack
"onsider a nonunity feedback system,
sho"n in the #ig. 5.$%.
,t is minor feedback loop and hence,
C G
& ' l+() ... *1+
,t is to be con-erted to unity feedback
system. .ut the denominator of dosed loop
transfer function must remain same as
before.
R -
Fig. 5.3. 'on(nily ee)*ack system
"
1/0, with unity feedback
i.e, ) = 1 ..
R
=
.
1 / (1 = l+() i.e. (1 =()
Thus,
" ()
... *2+
R
=
l+()
1
.ut transfer function must remain same hence it is necessary to introduce block of )
inseries with & Thus con-erted unity feedback system is as shown in the #ig. 5.$3.
r
0
Control System Engin"ring 518 Block Diagram Representation
U n i t y f e e d b a c k
Fig. 5.39 Equivalent unity fee!ack system
C 1 GH G
R = H x 1+GH = 1+GH
... same as before
5.3.3 "roceure to Solve Block Diagram Reuction "ro!lems
Step 1: Reduce the blocks connected in series.
Step 2 : Reduce the blocks connected in arallel.
Step !: Reduce the minor internal feedback loos.
Step # $ "s far as ossible try to shift takeoff oints to#ards ri$ht and summin$
oints to the left. %nless and until it is the re&uirement of roblem do not
use rule 1' and 11.
Step 5: Reeat stes 1to ( till simle form is obtained.
Step ) : %sin$ standard *.+. of simle closed loo s,stem- obtain the closed loo *.+.
C%s&
R.s/ of the o0erall s,stem.
1. E'ample 5.( $ Determine the transferfunction C%s& ) R(s) of the system shown in the
Fig. 5.40.
c
~ - - - - - - - - - - - - - - ~ H 2 ~ - - - - _ ' - - - - ~
+i$. 5.('
r
"
Control System Engineering 5-17 Block Diagram Representation
Solution: The blocks G2 and G3 are inparallel so combining them as (G2+G3) we
get,
Blocks inseries
R-----1
+
Minor feedback loop
R-----1
Blocks inseries
R------i
G, G4 (G2 + G3)
1-G, G4H,
~---~--------c
Feedback loop
GI"G4 (G2+G3)
(s) I-GI ol HI
Rts) ! GI G4 (G2 +G3) H2
1+ I-GI Col HI
"
Control System Engineering 518 Block Diagram Representation
C(s)
R(s)
=
I-GI G4 HI +GI G4 (G2+G3) H:z
... Ans.
5.4 Analysis of Multiple Input Multiple utput Systems
In these problems t!e l!" o# s$perpos%t%on %s to "e $se& 'ons%&er%n( e!'h %np$t
sep!r!tel). *h%le !ss$m%n( the other %np$ts !s zero most o# the t%mes if onl) %np$t %s
!ppl%e& to the s$mm%n( po%nt s$mm%n( po%nt %s to "e remo+e& %# not ne'ess!r).
#ey $oint% While removing summing point if sign
of the signal present, at that summing point which is
to be removed is negative then it must be carried
forward in the further analysis.
&es)
R(s)
'
(ig. 5.41
n$s '!n be !'h%e+e& b) %ntro&$'%n( ! blo',
o# tr!ns#er #$n't%on )*%n ser%es "%th th!t
s%(n!l. -h%s is the %mport!nt step to be
remembere& "h%le sol+%n( problems on
m$lt%ple %np$t m$lt%ple o$tp$t s)stems.
.....- )(
e.(. 'ons%&er ! p!rt o# s)stem sho"%n( t"o
%np$ts R(s) !n& /es).
0ther &et!%ls !re not sho"n #or s%mpl%'%t).
+!en R(s) is 'ons%&ere& !lone &es) m$st be
!ss$me& zero !n& s$mm%n( po%nt !t /es)'!n be
remo+e& !s "%th /es) = 1 there rem!%ns onl) !
s%n(le s%(n!l present !t th!t po%nt so s)stem
(ets mo&%#%e& !s sho"n inthe 2%(. 5.42.
R(s,
3 3
(ig. 5.42
. 4o" s%(n o# s%(n!l #rom blo', Gl %s pos%t%+e !t the s$mm%n( po%nt "h%'h is
remo+e& hen'e there %s no nee& o# !&&%n( !n) other blo',.
4o" "hen R(s) = 1 "%th /es) !'t%+e the s$mm%n( po%nt !t R(s) !lso '!n be remo+e&.
5$t no" s%(n o# the s%(n!l 676 !t th!t s$mm%n( po%nt %s ne(!t%+e "h%'h m$st be
'ons%&ere& !n& '!rr%e& #or"!r& #or #$rther !n!l)s%8. -!is is poss%ble b) !&&%n( ! blo', o#
-1 %n ser%es "%th 7"%tho$t !lter%n( !n) other s%(n. -!is !+o%&s the 'on#$s%on !n& problem
'!n be sol+e& "%tho$t !n) error.
Blockof .)*.
&es)
2%(. 5.43
r
/
Control System Engineering 519 Block Diagram Representation
)... Example 5.2: Using block diagram reduction technique find the transfer junction from
each Input to the output Cfor the system shown in the Fig. 5.44.
X(s)-----t
Fig. 5.44
Solution: With X(s) =0, block diagram reduces as,
Minor loop Minor loop
c
- r:! t d..! ! ! . C(s)
R(s"
Senes blocks
R(s)
" # " ! ! ! 1! # ! # ! 2 ........ # ... ! ! , $ ! :! :! % ! r. ! # $ % &
' C(s)
(% ()*)(% ()s+sl
)*), o,
% (---=--
(% ( )2)(% ' )s +s)
....
--- Mlnor.oop Senes blocks
Fig. 5.44(&)
C/s)
0
Control System Engineering 5-20 Block DiagramRepresentation
C(s} Gl G2G3GS
R(s) = 1+G2+GsHs+G2GsHs+GzG3Gs
With R(s) = 0, G1 vanishes but minus sin at summin !"int must be #"nsi$e%e$ b&
int%"$u#in bl"#' "( - ), as sh"*n+
X(s)
Minor loop
Note this block
~
~
Minor loop
%(3l
2
)(3l
3

)(3l
5
C(s)
iii
,i+ 5+--(b)
Series blocks
r
t--~-C(s)
,i+ 5+--(#)
Rea%%anin the in!ut - "ut!ut *e et,
X(s)
-
Minor loop
,i+ 5+--($)
Control System Engineering 5-21 Block Diagram Representation
C(s) G4Gs(1+G2)
Xes) = 1+G5Hs+G2 +G2GsIi5+GzG)G5
5.5 Block Diagram from System Equations
The block diagram can be constrcted !rom the set o! e"ations re#resenting the
s$stem% The addition and sbtraction o! the &arios terms is re#resented b$ the smming
#oints 'hile mlti#lication !actors are re#resented b$ the blocks o! the res#ecti&e
trans!er !nctions%
Consider an e"ation(
)2 *+ 4)l - 5),
The corres#onding block diagram has blocks
o! trans!er !nctions 4 and 5 along 'ith a
smming #oint to re#resent the sbtraction% It is
as sho'n in the -ig% 5%45%
-ig% 5%45
The signals t an! " arc taken !rom their generating #oints and a&ailable # can be
sed !rther !or generating other &ariables and ot#t%
Combining the block diagrams o! all the s$stem e"ations( the com#lete block diagram
re#resentations o! the s$stem can be obtained%
$ey .oint+ The block diagrams of electrical systems can b e easily obtained b y this method.
As methods of writing equations for electrical system may vary, the block diagram is not
unique for the electrical systems.
%. E&ample 5."' Obtain the block diagram for the given electrical network.
R,
R3
r r
(l)t) R4 (o)t)
*
1 _
-ig% 5%4/
Solution' Con&ert the gi&en net'ork into 0a#lace domain and assme the crrents as
sho'n in the -ig% 5%4/(a)%
r
+
Control System Engineering 5-22 Block Diagram Representation
Fig. 5.46(8)
The KVLequations for the two loops are,
-laRa -, R!"l!R# "V$ = %
i.e
&&' Vi(Ra (R! )"h(Rl)R! )
.(1)
-&#R*-&#R4-l!R! "&&R! % i.e
+ R!, &
...(#)
=
h' && R!"R*"R4-
And v, = ! a
The .lo/0 1iagra2s for the three equations are,
E!"ation (1) E!"ation (2) E!"ation (#)
Fig. 5.46(.)
Thus the o3erall .lo/0 1iagra2 is as shown inthe Fig. 4.46(/).
V(4)
&
$
12
Vo(4)
R%R2
R2%R#%R&
$
R%$
Fig. 5.46(/)
' r
(
Control System Engineering 5-23 Block DiagramRepresentation
If the equations are written with different method, the different block diagramcan be
obtainedbut the transfer function of the systemremains same. For the abovenetworkwe
can write equations as,
12
v,= hR.
he overall blockdiagramis shown in the Fig.5.!"#d$.
Fig. 5.46(d
r
!
Control System Engineering 5-24 Block Diagram Representation
Examples with Solutions
J _ Example 5.4: Reduce the block diagram and obtam its closed loop T.F. C(s)!R(s).
C(s)
Solution: No blocks are connected in series or parallel. Blocks having transfer functions
G2 and H2 form minor feedback loop so eliminating that loop we get,
'<, 1G,H, !'
t----' .-.,-.------
C(s)
ey !oint: Always try to shift takeoff point towards right i.e. output side and summing
point towardsleft ie. input side.
"o shift takeoff point after GI to the right. #hile doing so, itis necessar$ to add a
block having %.&. e'ual to reciprocal of the %.(. of the block after which takeoff point is to
be shifted, in series with signal at that takeoff point. "o in series with H) we get a block of
*
G2 + 1G2H2
1! 1G2H2 i.e G2 after shifting takeoff point.
C(s)
r
"
Control System Engineering 5-25 Block Diagram Representation
Minor feedback loop
C(s)
Minor feedback
loop
R(s) C(s)
Simplifying,
C(s)
R(s)
'.... Example 5.5: Reduce tire block diagram to its simple form and lienee obtain C(s)/R(s).
C(s)
r
"
Control System Engineering 5-26 Block Diagram Representation
Solution: The blocks G4 and HI form minor feedbackloop,
Shifting takeoff point as shown I
G4 C(s)
1+G"H,
The signal from takeoff point A reaching to summing point Bis without any block Le
gain of that branch !oining A to Bis one So introduce block of T" '1' in between A and B
to a#oid further confusion as shown below
Blocks in parallel
Minor feedback loop
"
Control System Engineering 5-27 Block Diagram Representation
R(s) (1 + G, G2 H2) (1 + G4 H,)
G, G4 (G2 + G3)
1+ -:-:---:::'--:::'-:-:--:-:'~-:::--:-:-:"
(1 + G1 G2 H2) (1 + G4 H1)
After simplification
G2+G3
C(s) G2
t-...... -
C(s)
Minor
fee!ac"
leap
C(s)
-
C#s) G) G4 (G2 + G3)
R(s) = 1+ GI G2 H2 + G4 HI + GI C2 G4 HI H2 + GI G4 (G2 +G3)
".. Example 5.6: Reduce the block diagram using reduction rules and obtain C(s)!R(s).
R(s$ C(s)
Solution: Block %it& T.F. G3 and 'nit( gain block are in arallel so co!bining t&em%e
ge",
Minor fee!ac" loop
R(s) C(s)
"
Control System Engineering 5-28 Block Diagram Representation
R(s)
Shifting takeoff point,
Blocks in series
~----~~~------~
( )
R(s)
Blocks in parallel
R{s) C(s)
C(s)
qs)
___ ......H2 + H2G3 + 1
1+ G3
R(s)
H, (H2 + H2 G3+ 1)
(1+GJ)
R(s)
Minor
feedback
loop
C(s)
Control System Engineering 5-29 Block Diagram Representation
R(s) G2(1 + G3) C{s)
1 + G2 + H1 (H2+ H2 G3+ 1) (G2)
C(s)
R(s) =
I I " Example 5.7: Obtain C(s)!R(s) using block diagram reduction rules.
C(s)
Solution: Separatingout the feedbacksat different summingpoints we can rearrange
the abovebockdiagramas beow.
R(s) ""

Blocks in series
R(s)
C(s}
Minor
feedback
loop
C(s)
C(s)
Minor
fee Iback
loop
"
Control System Engineering ,..- 530 Block DiagramRepresentation

C(s) R(s)
C(s) R(s)
Minor
feedback
loop
R(
12 3
s)
! " 3H1 H2 " 23H,
C(S
1# 3H1 H2 H3
1"
1 " G3H, H2 " G2G3H1
C(s)
R(s) $
%. E&ample '.() Use of Rule No. 10, critical rule illustration. Reduce the blockdiagram
. C(s)
and obtaan R(s)'
.*
C(S)
*
Control System Engineering 5 -31 Block DiagramRepresentation
Solution: Shifting takeoff point before Gz .
. .
C(s)
Shifing takeoff point before summing point using critical rule No. 10,
Separating the paths in the feedback path shown dotted,
Minor
feedback loop
C(s)
C(s)
Key Point: Remember that though the paths through summing point are separated, signs at
the summing points to those paths must be carried as it is.
Hence after H2, carr the negati!e sign and then H" Gz to get - H" H2 #2.
$" %&
r

Control System Engineering 532 Block Diagram Representation ,I


Shifting summing point as shown and then interchanging the two summing points
using Associative law we get,
Blocks in parallel
Minor feedback loop
d C(s)
C{s)
G1 (G2 + G3) C(s)
(1+ G2H2)(1+ H1G1G2)
C(s)
R(s)
(1+ G2H2)(1 + HIG IG)
c Gt(G! +G3)("HtH2G2)
1+ (1+ G2H2)(1 + HIGIG 2)
C(s) Gl (G2 + G3)
#(s) 1+G2H2+HIGtG2"GIG2G3HIH2
r
!
Control System Engineering 5-33 Block Diagram Representation
'. Example 5.9: Obtain the expression for CJ and C2 for the given multiple input multiple
output system.
t-------c2
Solution: In this case there are two inputs and two outputs. Consider one input at a
time assuming other zero and one output at a time. Consider Rl acting, R2 = 0 and C2
not considered R2 = 0 and C2 is suppressed (not considered). C2 suppressed does not
mean that C2 = O. Only it is not the ocus o interestwhile C1 is considered. As R2= 0,
summing point at R2 can !e remo"ed !ut !loc# o '-1' must !e introduced in series with
the signal which isshown negati"e atthatsumming point.
I,..,..',. Forward path
$--%-... c$
(
Minor
loop
0=
&eed!ac#
path
'l 'l
l()'I* )-'2 '3 '4] = l-'t '2 '+ ',
C2
&or Rl I assume C$suppressed.
r
"
Control System Engineering 5-34 Block Diagram Representation
Forword
path
'",;",-.1' ;- "y
Minor loop
C2 -GlG3G2 -GlG2GJ
Rt = I+l-Gt G3 G2][G41= I-Gl G 2 G3 G4
Cl
For R2' Rl = 0 and C2 is suppressed
For! ord" t
path .~
Minor loop
# -GlG2G4 -GtG2G4
R2 = 1+" -GlG2G41[G3J = 1-GlG2G3G4
C2
For R2' Rl = 0 and Ct is suppressed
Feedback =
....... '" path
"'''C-'' '$'
; s ~ : : G2 ~.;)
#%#&-$- ## 'J'#'# '-'J
Minor loop
C2 G2 G2
R2 = 1+[G21 [-Gl G3 G4] = 1-Gl G2 G3 G4
r
Control System Engineering 535 Block Diagram Representation
,_ Example 5.10: Determine the transfer function C(s)!R(s) of the system shoum below by
block diagram reduction method. (M.U. : Nov.-94, Dec.-98)
Solution: Shifting summing point before G. n! t"eoff point fter G4 #e get,
R
$
%&chnging summing points n! t"eoff points using ssocitive '# n! combining
series b'oc"s #e get,
c R

()(
Control8yatem Engineering 5-36 Block Diagram R.p ..... ntation II
Eliminating minor feedback loops we get,
R
C(s) !"3#
R$s% & '('")'( 3#)"(t"3#)!)"("3)3
Series blocks
C(s)
R$s%
r !" u 3# '
ll(!")! *ll(3#)"*
&
'.r !" lr +# lr )3 ,
+''(!")l *ll(3#)"* l*#
1"+#
'... Example 5.11 : Obtain the closed loop transfer function.
R(s) C(s)
-ol.tion/ 0o blocks are inseries or parallel so s1ifting s.mming point towards left 2e,
before t1e block 1a3ing transfer f.nction l as s1own in 4ig.re.
R(s) C(s)
r
"
Control System Engineering '5 -37 Block Diagram Representation
C(s} R(s)
Using Associativelaw for two summing points we get,
Minorfeedbackloop
R(s} C(s)
R(s)
.,,-_.-. . . . . .
G1G2 Aftersimplification
Parallel blocks
The two blocks with transfer function 1 and with transfer function ~: are inparallel.
So they will add to each other so we have,
Rsl
.-
- !--.r"---A-ft-e-r#-i-m-p-lif-ica-t-io-n--!
$ C(s) ! "G"%."G.%%%2"&"G...%'"%.G...%2%%""
R(s) ( & G2 )2 & G(G2)(
C(s)
f""
Control System Engineering 5 38 Block Diagram Representation
'. Example 5.12: Obtain the closed loop transfer function C(s)!R(s)
Res) C(s)
(M.U. : Dec.-97, D.-2OO5, Dec.-2007)
Solution: Separating two feedbac fro! "econd taeoff point w#ic# i" after b$oc
#a%ing tran"fer f&nction '2 a" "#own, we get,
R(s) C(s)
R(s) C(s)
r
"
Control System Engineering 539 Block Diagram Representation
Shiftingsumming point behind the block having transfer function 'eI' as shown we
get,
R(s) C(s)
get,
Use Associativelaw for the two summing points and interchange their positions, we
R(s)
R(s}
C(s)
Minor loop
After simplification
r
"
Control System Engineering 5-40 Block Diagram Representation
Mmorloop
. .
L...- ~ -J
1 + G,Gz C i : 3 Hz
1+G,tI,-G,G2H; G,
+
C(s)
C (s)
=
R(s)
G4 +G4 G2 HI - G4 Gl G2 HI + G2 G 3 G4 H2 + GI G2 G 3
1+G2 HI -GI G2 HI + G2 C3 H2
'.... Example 5.13: Obtain the closed loop transfer function C(s)!R(s).
R(s)
"
Control System Engineering - 541 Block Diagram Representation
Solution: The blocks G1and Gs are in parallel, so add them.
R(s)
Shiftingsumming point behind the block 'G2', towards left as shown we get,
R(s)
Using Associativelaw for the two summing points in between and interchanging their
position we get,
R(s)
Minor loop
r
"
Control System Engineering 5-42 Block Diagram Representation
R(s}
R(s}
R(s)
After simplification
Minor loop
+
(G1 +G5) GG!G"
1 + GG!# + G!G"#!
+
The block G$ and equivalent block obtained are in parallel,
C(s)
=
R(s)
G(%+& & G(%H2 +G! G4 Gil H +(& +!") G# $ G& G'' Gil #( +(& +!") G# $ % G''
&+G2 GH2 +CC'' H +(& +!") G# $ C&G# # H'
)
Control System Engineering 5-43 Block Diagram Representation
I I " Example 5.14: Obtam the closed loop transfer function C(s)!R(s).
C(s)
(M.U. : May-97)
Soltion: The blocks G2 and H2 form minor feedback loop.
qs)
After SlmplJlicaliOO
G.G2G.H2-oG!G2
1+G~2
Minor loop
r
"
Control System Engineering Block Diagram Representation
C(s) G3 [C4 +G2 Col H2 + Cl C:d
=
R(s) 1+G2 H2 + G3 HI [G-I +G2 G4 H2 + Gl G2]
'. Example 5.15: Obtain tire resultant output C(s) inierms of the inputs R(s) and Y(s).
yes)
C(s)
Solution: As there are two inpts! "onsider ea"h inpt separatel#. Consider R(s)!
ass$in%#es) =o.
Minor loop
C(s)
t
C(s)
&
R(s)
'ow"onsider(es) a"tin%with R(s) =o.
'ow si%n o) si%nal o*tained )ro$ H1 is ne%ati+e whi"h $st *e "arried )orward!
tho%hs$$in%point at R(s)is re$o+ed!as R(s)& ,! so we %et!
Yes)
C(s)
r
Control System Engineering 5-45 Block Diagram Representation
Combining the blocks G1 HI and -1 as in series,
C(s)
Geq = G2
Hcq = -GI HI
Feedback sign positive at input summing
point.
Yes)
CCs) Geq
. !es" = 1-GeqHl#q
$o part o% C&s" due to 'es" alone is,
C&s" = '&$"(1)G**2H+
Hence the net output C&s" is given b! algebraicall! adding its t,o components,
C&s" = G1G2-&s")G2'&s"
1+ G1G2Hl
,_ Example 5.16: Tile system block diagram is given below.
Find i) C(s) if N(s) = . ii) C(s) if N(s) = . iii) NC(~) if R(s) = .
E(s) Ris) "
&/.0. /a!-l1+23"
Solution: i" 4ith 5&s" = . block diagram becomes
R(s)
rMinor,ooP
1.
C(s}
r
"
Control System Engineering 546 Block Diagram Representation
Minor feedback loop =
10
5(5+1)
- - ~ - - = =-::-~
10 52+ 8+58 52+ 65
1+ 5(5+ 1) 0.5s
10 10
R(s) C(s)
10
2
s +6s
Rs)
10
C(s)
2
s +6s
Ass!e o"p" of second s!!in# poin"s as $es).
%ence &(s)
=
'(s) - ((5)
((s)
=
$es) 10(s+4)
S2 + 65
$es) =
)
&(s) + - 4 Rs)
s+
... (i)
*.. (ii)
.*. (iii)
+bs"i""in# ,ale of $es)and '(s) fro! (i) and (ii) in (iii) -e #e".
52 + 6s ) )
10(5+ 4) ((s) = &(s) + s + 4 &(s)+ 5+ 4 ((s)
! "2 + 65 )/ C
= (1 + s0 4) &(5)
10(s+ 4) - (s + 4) (s)
-
# S$ + 6s - )0) (( )
=
(s+1) &(s)
10(5+4) s (s+4) .
%
((s) 10(s+1)
-2en 3(s) =0
%%
&(5)
=
s2 + 6s- )0
ii) 4o find ~ ~ 5~ . -.5 2a,e "o redce block dia#ra! sol,in# !inor feedback loop and
s2if"in# s!!in# poin" "o "2e lef" as s2o-n earlier in (i).
r
&
Control System Engineering 5-47 Block Diagram Representation
10(S+4)
2 t---...--
s +6s
So referring to block diagram after these two steps i.e,
R(s)
10
2
s +6s
Exchanging two .smming points sing associati!e law,
Res)
"
Parallel of 1 an! "#
s+4
C(s)
C(s)
"
10(s + 4#
$ % s2+6s
&inor loop "" '"("")$$$**+"
1+ %, 1+ 10(s $4)
s2+6s
"
'1+"#
s+4
.. %lock diagram becomes,
R(s)
"
10(S+4) C(s
1+"" r---
2
-+4
s +16-+40
C(s) &5 +7# &10 &5+4'# 10&5+7#
R(s) ( s +4 . -2 +1)5+40 ( -2 +1)5+40
iii# *ith +&s# '0 bldt,k diagram becomes,
Ces)
/
Control System Engineering 548 Block Diagram Representation
The ' block of '3' will not exist as R(s) = O. Similarly first summing point will also
vanish but stuent shoul note that negative sign of feeback must be consiere as it is!
though summing point gets elete.
-Parallel
T"o blocks are in parallel! aing them "ith signs.
C(s)
#$
s(s + 1)
C(s)
In general "hile
eleting summing point! it
is necessary to consier
the signs of the ifferent
Signals at that summing
points an shoul not be
isturbe. So introucing
block of '%#' to consier
negative sign.
C(s)
L...---i -(s +4) - 0.5 S & '' ...J
--____.' ......
---AfterSimplifiation(-1.55 - 4)
Removing summing point! as sign is positive no nee of aing a block.
C(s)
#$
S(S+1)
C(s)
L...-_-I_(1.5 s +4)
-10(1.55 + 4)
' ---! s(s + 1)
(
"inor loop #it$ % = 1
r
Control System Engineering 5-49 Block Diagram Representation
C(s)
1 1
= =
N(s)
1-[ -10(1.55+ 4)]
1+
15s+ 40
s(s+ 1) s(s +1)
C(s) s(s+ I)
N(s)
=
52+165+40
I ' " Example 5.1! Use block diagram reduction technique and obtain the transfer functions
.) C(S) " " C(S) .... " a
l R(s)' II xi: nn yes)
Also find total output of the system. (M.U. : Nov.-95)
C#s$
%#s$
Sol&tion' In such mult!l" n!ut s#st"ms$ it s n"c"ss%&# to us" su!"&!oston !&nc!l"
'n( t must )" not"( th%t *hl" &"movn+ summn+ !ont$ t s n"c"ss%&# to cons("&
s+ns o, v%&ous s+n%ls %t th%t summn+ !ont.
) Cons("& -(s) %ctn+ %lon"$ ./s)( #"s) ( 0
So )loc0s 1.. %n( 234 *ll v%nsh %lon+ *th th" summn+ !onts %n( s+ns o, %ll
s+n%ls %&" !ostv".
)inor loop
C#s$
C(s)
=
R#s*
... ()
COntrol System Engineering 5-50 Block Diagram Representation
ii) Consider Xes)acting alonge, R(s) = Yes)= 0
When R(s) = 0 , Summing point at R(s) will anish, !ut sign o" "eed!ac# signal at that
summing point is negatie$ So it is necessar% to carr% on that sign !% adding a !loc# o"
'-1' in series with that signal$
&ll ale
in series
Senes
~
,$$-------,
X(s)
Minor loop
C(s) ~.
1--...... - ~
-
-
C(s)
C(s) '( '$$ $$
)$ Xes) = 1+ * C+ ( ,* ,+ $$$ (ii)
C(s)
iii) Consider %es) acting alone, R(s) ! Xes)! 0
1-- --- ---- -- ---- - ----- ---- - - ..
"
#llare $
in series$
"
"
"
"
"
"
"
"
- ---------
Minolioop
"---....-C(%S) ~(~ 0
-
C(s)
&
iii
C(s)
& CCs) !
%es)
-'l '. '( ,. ,( (///)
-+'l Gz '( ,* ,. $$$$ W
0rom (i), (ii) and (iii) we can write the total output as
'1'. '( R(S)1'( '2 X(s)- 'l '+ '( ,+ ,(Y(s)
C(s) -. -+'1 '+ '( ,* ,+
'
Control System Engineering 5-51 Block Diagram Representation
'.... Example 5.18: Reduce the block diagram and obtain the transfer function C(sj!R(s).
(M.U. : May-96)
,.
C(s
3
'------t s ...8 t--------'
s-4
Solution: No series, parallel combination and no minor feedbac loop e!ists. So s"iftin#
taeoff point before t"e bloc of ($ S$ ) .
!" 1#
$inor loop
%it&'(1
an) *(+1+
s"1#
,arallel
-locks
C(s
3
'-------. s " %t--.-----'
S-4
+5 "+1+o ! :--5-,:(/ ..0 ,s:-:"+1 ..:.. & $ t-$ ..-'$ (S..)
5 " (( (5 " ()) (5 " 3(
23ter simpli3ication
!
!
r
Control System Engineering 5-52 Block Diagram Representation
C(s)
=
R(s)
S(25+ 7)
(5+11)(5- 3) S(25+7)(5 - 4)
=
(s+11)(s - 3)(s - 4)+ s(2s+7) (53 + 8) 5(25+7) (53 + 8)
1+ . .....,!.,...
(s+11)(s-3) (s-4)
C(s) s(2s" S 28)
=
R(s) "s5+754 + 53 + 2052 - #s+ 13"
,$ E%ample &.1#' Determine C!R ratio for the system shown below. (M.U. : May-95)
c
Sol(tion' S)i*ting takeoff po!t +)ic) is "efo#e $2 to% afte# ,l.
-arallel
.*ter simpli*ication
c
r
!
Control System Engineenng 5 -53 Block Diagram Representation
Shifting summing point before the block 'GI' and interchanging using associative law,
Minor loop
Series
~
MInor loop
.e e
G1(G2G3+G4)
1+G1G2H1
c
R =
C
R
G. (Gz G:l +Col)
"
Control System Engineering 5-54 Block Diagram Representation
I I " Example 5.20: For tire block diagram slrown, obtain C(s)!R(s) by using reduction rules.
(M.U. : Jan.-92)
Solution: Shifl the takeoff point, to the right of summing point. his is the !"riti#al
rule! $is#usse$ earlier as rule %0an$ %%. I n this pro&lemit is ne#essar' to use this rule,
whi#h is generall'not use$ to sol(e simple pro&lems.
New summing
point ue to -t-------)***.
critical rule
C!s"
+ew summing point gets a$$e$ $ue to use of #riti#al rule . #$is summing point #an
&e eliminate$ &' separating the two paths whi#h are linke$ &' that summing point. #$e
paths are shown $otte$. So &lo#k$iagramre$u#es as,
*,% , , : r -,
Control System Engineering 5-55 Block Diagram Representation
WhiJe removing summing point, as sign of one of the signal is negative, the block of
transfer function '-1' is connected series with that signal.
So now there exists a minor feedback loop
Shifting take off to the right of 2
Parallel
C(s)
!ombining blocks in parallel and shifting summing point to the left of 'et'.
C(s)
r
"
Control System Engineering 5-56 Block Diagram Representation
Interchanging the summing points using associative law.
Solving minor feedback loop. The block of 'I' and 'Ht' are in parallel so loop becomes,
+
}
The feedback
paths ! and "
G"
are in parallel
'-- ....1 b#t $ith
opposite signs
Combining the feedback blocks which are inparallel.
C%s&
+
_\ , , : r I,
Control System Engineering 5-57 Block Diagram Representation
C(s)
R(s)
C(s) ~ Gz G3 +~ G"
R(s) = I-G) Gz + G) Gz HI-G) c,G4 Hz +G) Gz G4H)Hz +G1 Gz G3+G) G4 -G2 G3Hz
,,... Eample 5.!" # $n the given block diagram, obtain the transferfunction of the system
Ct/R1
(%.&. # 'o(.-)*)
Sol+tion# Solving the minor ee!"#c$ loop o ,Col, #n! %1%&
c ,
'( C2 is not the oc)( o intere(t hence G* "ecome( me#ningle(( in the "loc$ !i#gr#m,
th)( it i( remove!&
r
-
Control System Engineering 558 Block Diagram Representation
Minor loop
Solving minor feedback loop and shifting takeoff point.
Series
G,G2
~
C,
G4GsH,G,G2G3
R, C,
.-oJ
(1 +G4)(1 +G,G2)
-
+
H2
G2G3
R,
G4GS
1+04
Gl G2 G3 G4 GS !
(1+Gl Gz) (1+G4)
1" Gl G2 G3 G4 G# $. 2
(l+G% G2) (l+G4) G2 G3
&'1 , , : r !,
Control System Engineering 559 Block Diagram Representation
t . . Example 5.22: Find C using block diagram reduction techniques
(M. U. - May-98, Dec. -D6)
c
+
Solution: Consider R) alone R21R3 IR4 are zero.
Not e : Wene!er Rl is zero, as t e si"n o# #eed$ac% at R) is ne"at i!e, &ile
re'o!in" s(''in" )oint at RIIdo not #or"et t o insert a $loc% o# *-+* t o consider e##ect o#
ne"at i!e si"n.
Minoro!p
c c
,),-
C
+.,-/- ,",-
..
0 0
1.,-/- .,+,-/l Rl
+. ,1,-/l
+.,-/-
C
,1,-Rl
. . . d(e t o Rl
..
0
1.,-/- .,1,-/l
Control System Engineering 5-60 Block Diagram Representation
Consider R2 alone, with R3 = RJ = R" = 0
Note
this
c
+
G2
C
1+G2H2
R2
=
1-(1 +~~H2}-GIHl)
C
=
G2R2
1+G2H2 +G1G2Hl
Consider R3 alone, Rl = R2 = R" = 0
Note
this
Co!inin" two s#in" $oints we "et,
Minor loop
R3
--
-
C
~
G2
+ -
C
II
I-G,H, I
+
c
%
nl
Control System Engineering 561 Block DiagramRep.... entation
=
-R3
C
-R3G2
C = 1+G2H2 +G1G2Hl
Consider Rl alone, with Rl =R2=R3=O.
C
=
1-(-G1H1l(1 +g~HJ
! - G"G2H"R#
1+G2H2 +G1G2Hl
!
R.t

$
Co%&ining all the 'al(es o) C, we get
C ! G"G2Rl +G2(R2 -R3*-G1G2H1R$
1+G2H2 +G1G2Hl
a
I'",+,a%-el 5.23 . Determine the transfer function R(s) U 6 i n f block diagram reduction
technique for the block diagram shown - (/.0.. /a12223*
C(s)
r-.--
r
"
Control System Engineering 5-62 Block Diagram Representation
Solution: Shifting takeoff point of G3 to the right of G2 we get,
Parallel
F.
'_....--C(s}
1+~
1-....--1 G2 '_......"j,__---C(s}
" " '.... ,,'
...._ - - - - - -
Shifting takeoff point towards C(s and inter!hanging takeoff points,
..'"i'#-".l$'" $",'$%'"' ... ,.,,,.." &inor loop
.:.. :. y~ G2 +G3 < 1 . i <
G2 !%%-.... -' "r$-$-- ....... ----- C(s
~.~: ,. i. After solving gives.
G2+G3
1+G2+ G3
r
"
Control System Engineering 5-63 Block Diagram Representation
=
'. Example 5.24: Determine the closed loop transfer junction for the system shown.
(M.U. : Df!c.-2003)
R(s)
Fig. 5.47
C{s)
t--+--..
Solution: Interchange the poition of t!o interna" #$$ing point% #ing aociati&e
"a!.
I---+- '' (())
Minor feedback loop
*()) I---+-''C(s)
Shift totheright
r
"
Control System Engineering 5 64
. . . . ,
Block DiagramRepresentation
1
- M:~~feedbaCk
t- - ?- - Cs!
"- Se" -> n e s
"##$#%%Cs!
qs)
C(s) _ Gl G2G3
R(s) - &'G2HZ 'G2G3H3 'GIG2Hl
.~~--------------~
(%%% E)ample 5%*5: Obtain the transfer function ~~:~.
(M. U. : May- 2004)
+ig%5%4,
Control System Engineering 5-65 Block DiagramRepresentation ~
Solution: Interchange the positions of two summing points using associative law.
~Parallelof
r--1
'1' and G,
G, f
Minor loop
R(s)
+
+~
s)
G!
res
r--l ",+"!
",
~
G!
1 + G!(",+"!)
Parallel
#(s) $ (1+ G%) G&
R!s - 1" G# !$I +$%
'' ()ample 5'!6* Find the overall transfer function.
!&.'. : &ay-%((), &ay-%(!1
#(s)
*ig,).+,
Solution :. Separating the fee-.ac/ paths,
+
" Control System Engineering 566 Block Diagram Representation
Separatingthe dotted paths from the summing point '5'.
C(s)
52
T2
C(s)

I~
e
I
,
_ _ . . .
I
I
I
I
I
: G >
~.__--_.,.
. . . _-- . . . . ----
Minor feedback
loop
r
"
Control System Engineering . 5-67 Block Diagram Representation
C(s) r------,
Shift the
summing point
C(s) --------.
Interchange
the
summing points
C(S)
Parallel Minor loop
G2G3
_1_--:::G:-=3,,::-H..;..' H....::2=--__ t-----1,........~C(S)
1
+ ~G3 x H H
1-G3H,H2 -, 2
, Control Syst.m Engineering 568 BlockDiagram Rep..... ntatJon'
C(s)
=
R(s)
[G1G2 - ! "#$G2%&G'l
1- G(!2 - G2G(#2
[G!G2 - H#(1-!- G2%&G(
#$))))--)))=-
1-G(!'* - G2G(!2
#--+...-+CCs%
C(s)
=
R(s)
'.. Example 5.2,- For the system represented by the block diagram shown, evaluate the
closed loop transfer functions when the input R is i) at station 1and in ii) at station II.
"../. - .ay-20"6%
!--1t--+C"S%
.... -........
,,' ...
" ,
, \
,2 'CCS% r-----.
3ig. 5.50
Sol4tion- i% !n54t 6 at station !
r
"
Control System Engineering 5-69 Block Diagram Representation
iii
iii
Minor loop
R(s)
=
ii) Input R at station II
Note that at station I, though input is zero while removing the summing point, keep
the negativesign of HJ as it is !hus a"" a #lo$kof %& l% as shown in the s'stem
r
Control System Engineering 5-70 Block Diagram Representation
,
I-_.....-_C(S)
..
,
,
. . . _ - - - - -
Shift the summing point to the left of G2 and interchange the two summing points.
R(s)
R(s)
CIS)
.
"
" BIoclIsin parallcl
+ C(S)
t--t--
C(S)
r
"
Control System Engineering 5-71 Block Diagram Representation
C(s)
C(s)
G1
..
R(s)
=
G;\ (- G1G;!H1- H2)
1-
1+ G2H3
C(s) G3 (1+G2H;\)
..
R(s)
=
1+ G2H3 + G3H2 + GtG2G;\H1
-G1G2H1- H2
1+ G2H3
Review Questions
1. What is block diagram representation ? Explain with suitable example.
2. State advantages and disadvantages of tile block diagram reduction technique.
3. Explain the block diagram reduction rules.
4. etermine C!R ratio for the s!stem shown below.
c
". etermine the tran.#er function of the s!stem using block diagram reduction of the s!stan shown.
C(s)
$. for the block diagram shown% obtain &'s()*'s( b! using reduction rules.
r
"
~.,ControlSystem Engineering 5-72 Block Diagram Representabon
C(s)
7. Use block dillgrllm reduction ndes fo obtain the tratlSferfunction of the block diagram shown below.
C{s)
8. Reduce the block diagram of the mllitilcop system slwwn in figure beloW.
C(s)
(
Ans . C(s) _ G 1 Gl G3 G4 )
R(s) -1+G3 c,IfJ +Gz G3 H2 +Gt c,Gl G4 III
9. Determine the uverall trmlSfa function relating C and Rfor the system whose block dillgram is
shown in figure.
c
Ans. ~ GIG1+GI G3 --
. R 1+G2 H1+GI G2111 -G I G2 G~ HI liz
r or
Control System Engineering Block Diagram Representation 5-73
c c
R'D
system zuhose output for t h e lind t h e total t h e 10. Determine mtio
block diJIgramis shown in follawing figure.
c
(
Ans .~ _ G1 G3 G4 +Gz G3 G4 _
R - 1 + G3 H2 + G 1 GJ G4 111+ Gz G3 G4 HI
C Goa(1+GJ H:!)
O=+GJ H2 +G1 G3 G4 H! +Gz G3 G4 H!
~ "a I t G1 G3 G4 + G 2 G3 G4 R
o o# $# %. 1+G3 Hz +G1 G3 G4 "& +G:z GJ G4 II!
G4 (l +G3 H2) ' )
+ 1 +GJ H2 +G1 G3 G4 HI +Gz G3 G4 111
11. Derive 1111 expression for the total output for t h e system represented by tire block diagram in
folIawing figure.
c
(
A C GIG( R1+G( R( )
ns.: =1+G1 G21111)+G212
72. se block diJIgramreduction methods "o obtain the e!uivalent tran.1erfunction from R to C.
_*1 * I" r I I
Control System Engineering 574 Block Diagram Representation
c
(
Ans ~ _ Gs G4 (G 2 + GJ) (GI) )
R - (1 +G" "2) (1+Gl HI) +GSG4 (G2 +G3) GI
13. Use block diagram reduction methods to obtain the equivalent transfer fundion from R to C
c
(
Ans .~ _ Gt GJ (Gl +G4) )
R - (1 +G, G2 liz) (1+G3 HJ ) +G1G3 (Gl +G4,
14. Finthe equivalent transfer function for the figure shown below.
c
r
"
Control System Engineering 5-75 Block Diagram Representation
15. Using block diagtam reduction, find the transfer function from ead: input to the output C.
c
An (A C G1 Gl G3 C G4 G3
s.: ns.:Ii = 1+G1 Gl G3 HI Hz' X = 1+ G3 GI Gz HI H2'
C -GI Gl G] Hz H3 )
Y = 1+G1 GIG] HI HI
DOD
r
"
(5 - 76)
r
If
Signal flow Graph Representation
6.1 Background
There is one more way of representating systems particularly when set of equations
describing the system is available. This representation which is obtained from the
equations, which shows how signal flows in the system is called signal flow graph
representation. As it uses the equations of the system which consist of various variables of
the system, the variables of the system plays a base role insignal flow graph. Thus we can
define signal flow graph as -
The graphical represenliltion of the variables o f a set o f linear algebraic equations reprn>enting
the system is called signal f l o w graph representation.
Let us see which are the important elements constituting the signal flow graph.
As variables are important elements of the set of equations for the system, these are
represented first in signal flow graph by small circles called nodes of signal flow graph.
Each node represents a separate variable of the system.
All the dependent and independent variables are represented by the nodes. The
relationships between various nodes are represented by joining the nodes as per the
equations. The lines joining the nodes are called branches. The branch is associated with
the transfer function and an arrow. The transfer function represents mathematical
operation on one variable to produce the other variable. The arrow indicates the flow of
signal and signal can travel only along an arrow.
e.g. Consider a simple equation,
V

IR
!here V

Voltage
V
I

Current
"ig. #.$
% %esistance which is parameter of the system.
This isnothing but simple &hm's law. (ow while representing this equation by signal
flow graph, first the variables voltage V and current I,are represented by nodes and they
are connected by the branch as shown in the "ig. #.$.
(6 - 1)
r

Control System Engineering 62 Signal Flow Graph Representation


1his represents that voltage V depends on value of
current I.And the relationship between the two is through
resistance R. So signal I gets multiplied by R to generate
variable V. So R becomes branch transfer function or
branch gain joining I and V. The direction of arrow is from
I to V. This is shown in the Fig. 6..
!o all the branches represent the cause and effect relationship e"isting between the
various variables. The branch transfer function is also called branch gain or branch
transmi#ance in signal flow graph terminology.
R
o-------~~----__ov
Fig. 6.
6.2 Properties of Signal Flow Graph
1) The signal flow graph is applicable only to linear time invariant systems.
$ The signal in the system. flows along the branches and along the arrowheads
associated with the branches.
3) The signal gets multiplied by the branch gain or branch transmittance when it
travels along it. .
e.g. %onsider signal flow graph shown in the "& ' "
Fig. 6.(.
The signal from )I gets multiplied by ' when it Fig. 6.(
travels along the branch joining )I to ). SO we can say value of ) is times the
value of )I. It shows dependence of ) on )l.
'$ The value of variable represented by any node is an algebraic sum of all the
signals entering at the node
e.g. %onsider the variable ). At that node&
( signals are entering from )I & )( and )'* "&
So value of ) depends on the variables )l& o+,,,,--,,.
)( and )'. The branch gains indicate the
e"act contribution of each variable in
generating ). So value of ) is algebraic
sum of all such signals entering. So we !an "3
write&
) / ')l , 2#3 $ 3"
0$ The value of the variable represented by any
node is available to all the branches leaving
that node. The number of branches leaving
a node does not affect the value of variable #I #s
~----~--~~----~----~.
represented by that node.
e.g. %onsider signal flow graph represented
in Fig. 6.0. The value of ) can be obtained
from signals entering at ).
Flg.6A
i.e. Fig. 6.0
r
%
Control System Engineering 6-3 Signal Flow Graph Representation
Now there are three branches leaving X2 , joining to X4, Xs and X6 that means x",
Xs and X6 variables depend on X2.
So we can write x" = X2 IX = 2X2 , X6 = 3X2
i.e. for all branches leaving !rom X2, the value of X2 available is same and number
of such branches do not affect the value of X2.
"ey #oint$ %he value of a variable represented by node depends only on the signnL-;
entering and this value is as it is available to all the branches leaving from that node.
6) For a given system signal flow graph is not unique. any other graphs can be
drawn by writing systemequations in different manner.
"ey #oint$ %he signal flow graph is not the unique property of tire system.
6.3 %erminology &se' in Signal Flow Graph
!onsider a signal flowgraph shown in the
Fig. 6.6
i)
" o #
$ource Node % %he node having only
outgoing branches is &nown as source
or input node. e.g. " o is source node.
ii) $in& Node % 'he node having
only incomingbranches is &nown
as sin& or output node. e.g. Xs is
sin& node.
Fig. 6.6
iii( !hain Node% ( node having incoming and outgoing branches is &nown as chain
node. e.g. " l ) " * ) " + and " , -
iv) Forward .ath % ) path from the input to output node is defined as forward path.
xe - " l / X3 - xs
First forward path
$econd forward path.
'hird forward path.
e.g. Xo - Xl - X2 - X3 - X.* - Xs
" o / " l / " + / " , / " s
Xo - Xl - "+2 - X3 - X,, Fourth forward path.
v) Feedbac& 0oop % ( path which originates from a particular node and terminating
at the same node) travelling through atleast one other node) without tracing any
node twi1e is called feedbac&loop. For example) " * /" + /" , /" *2
vi) $elf 0oop % ( feedbac&loop consistingof only one node is called self loop. i.e) t +.3
at " + is self loop. ( self loop can not appear while defining a forward path or
feedbac&loop as node containing it gets traced twice which is not allowed.
vii) .ath 4ain % 'he product of branch gains while going through a forward path is
&nown as path gain. i.e) path gain for path " o / " l / " */ " l / " , / " s is )
#" t#*" t*+" t +, " t ,5. %his can be also called forward path gain.
r
"
Control System Engineering 64 Signal Flow Graph Representation
Kay Point: While tracing a forward path or a feedback lo o p , no node is to be trtlCed twice.
viii) Dummy Node : H there exists incoming and outgoing branches both at first and
last node representing input and output variables, then as per definition these can
not be called source and sink nodes. In such a case a separate input and output
nodes can be created by adding branches with 'gain 1. Such nodes are called
dummy nodes.
e.g.
(a)
1
.... 0
Sorce
1
0
Sin!
(b)
Fig" !."
#n the signal flow graph $l and $% are input and output variables but as per definition
not input and output nodes. Such independent nodes can be generated by adding branches
of gain 1 as shown in the &ig. !." 'b).
Note: Such creation of dummy nodes is not necessary. (ithout this also, signal flow
graph can be analysed to get the overall transfer function.
Key Point: AdditiolI o f branches o f gllin # is possible only before starting node and after the
last node. #n between the chain nodes slIch branches o f gain 1 cannot be added,
ix) Non touching loops : I$ there is no node common in between the two or more
loops, such loops are said to be non touching loops.
)he &ig. !.* 'a)+'b) show a combination of non touching loops of two and three loops.
(a) %wo non toching loops (b) %hree non toching loops
Fig" !.*
Similarly if there is no node common in between a forward path and a feedback loop,
a loop is said to be non touching to that forward path. .
I
)he &ig. !., 'a) and 'b) shows such a loop which is non touching to a forward path.
.:;
&' ( s
)on toching loop
to $orwar* path shown
(a)
Fig" +",
'b)
r
-
Control System Engineering 65
Signal Flow Graph Representation
x) Loop Gain : The product of all the gains of the branches forming a loop is called
loop gain. For a self loop, gain indicated along it is its gain. Generally such loop
gains are denoted by 'L' e.g. L. I L2 etc.
-5
Gl G2 G3
3


X~X3

xl
X4
Xl x2 2 x5
-Hl -2
(a)
Fig. !."
#b)
$n the Fig. !."#a), there is one loop %ith gain L. & G2 x.:... 'l & ( G2'.
$n the Fig. !."#b), there are t%o loops %ith gains.
L) & * x ( 2 & ( + and other self loop %ith L2 & ( ,
6.4 Methos to !"tain Signal Flow Graph
6.4.# From the System E$%ations
Steps&
1) Represent each variable by a separate node.
2) Use the property that value of the variable represented by a node is an algtbraic
sum of all the signals entering at that node, to simulate the equations.
3) Coefficicnts of the variables in the eqJlations are to be represented as the branch
gains, oining the nodes in signal flo! graph.
") #ho! the input and output variables separately to complete signal flo! graph.
-xample: .onsider the system e/uations as say,
'. (20i ) 302 111(#)
02 & *0l 2303 2 202 111#2)
03 & 302 ) 0o
0o & !03
...(*)
...#*)
Let output be 0o and input be 0i %here 0l +02 , 03 are the system 4ariables.
-/uation #") sho%s that 0. depends on 0$
and 02. 3o there are t%o branches entering at
node 0t i.e. from 0i and 02. 5ut branch from 02
to 0$ has direction from output to input hence to
be sho%n as a feedbac6 path as in the Fig.
!.""#a). 3imilarly all e/uations are to be
simulated and 7oined tu get complete signal flo%
graph.
Fig. !.""#a)
r
, -,,.
Control System Engineering 66 Signal Flow Graph Representation
Signal from output side towards input i.e.
from, V2 to VI, V3 to V2 and so on are to be
indicated as feedback paths.
In equation (2) there is component of V2
itself, contributing to generate the variable V2.
his results in a self loop in a signal flow
graph. he complete signal flow graph is
shown in the !ig. ".##(b).
6.4.2 From the Given Block iagram
!!v
~ 0
2 3
"# "$ "%
Fig. 6.%%&'(
Steps)
1) Name all the slimming points and take off points in the block diagram.
2) Represent each summing and take off point by a separate node in signal floW graph
3) Connect them by the branches instead of blocks, indicating block transfer functions a.. s the
gains of the corresponding branches.
4) Sho the input and output nodes separatel! if re"uired, to complete signal flo graph.
E*ample)
!ig. ".#2
$aming summing and take off points
as shown in the !ig. ".#3.
+ey ,oint) #ake sure that if summing o%%&#'()('*''%"%(+%%)%%),*'(&(%%o
and take off points are near each other in i$ p
a gi%en block diagram, the! are. to be
represented by separate nodes in the
corresponding signal fl& W graph.
o/p
o'p
Fig. 6.%#
r
-
Control System EnSJineering 6-7 Signal Flow Graph Representation
6.5 Mason's Gain Formula
It is seen earlier that in block diagram representation, we have to apply reduction
rules, one after the other to obtain simple form of the system and hence overall transfer
function. We have to draw the reduced block diagram after every step. This is time
consuming. In signal flow graph approach, once S..! is obtained, direct use of one
formula leads to the overall system transfer function " " # " .$ This formula is stated by %ason
and hence referred as %ason&s gain formula. The formula can be stated as #
'verall T.. = n" A I
where ( )umber of forward paths
! ( !ain of *lh forward path
l ! . . ( System determinant to "e calculated as #
l ! . . ( +-,l- all individual feedback loop gains .including self
loops// # $ I%gain & gain product of all possible
combinations of two non touching loops/
' $I% gain & gain & gain product of
combinations of three non touching loops/ #....
l ! . . " ( 0alue of above l ! . . by eliminating all loop gains and
associated products which are touching to the lh
forward path.
12planation # H we have identified in signal flow graph following information that,
)umber of forward paths *( 3.
The gains i.e. product of branch gains involved in defining various forward paths are
denoted as TI(T4 and T3&
)ow number of loops including self loops are say 3 and their gains are say 5II L21
and )36
'ut of these three, 5l 54 and 5l 53 are the combinations of two non touching loops.
There is no combination of three non touching loops.
l ! . . ( +- $I%all individual and self loop gains7
# $I%gain 2 gain product of all combinations of
two non touching loops/
l ! . . = +- .5l # 54 8 53/8 .5l 54 # 5l 539
r
*
Control System Engineering 68 Signal Flow Graph Representation
Now for ilK i.e, ill, il2 and il3 consider each forward path separately.
For 11, say all loops !, 2 and 3 are to"ching to 11 hence all loop gains arc to #e
eliminated from $ to get $!.
For %2, say 2 is non to"ching and t, 3 are to"ching. So 1, 3 and associati&e
prod"cts i.e. ! 2 and l 3 are to #e eliminated to get $2, 2 will e'ist.
For %3, say l and 2 #oth are non to"ching and 3 is to"ching. So 3 and associated
prod"ct ! 3 m"st #e eliminated !, 2 and prod"ct l 2 as #oth are non to"ching to
%3 will e'ist in il3
(ence %.F. can #e o#tained #y s"#stit"ting these &al"es in
%F
) %!$!*%2+2*%3$3
. A
.... E'ample ,.1 - Find tire overall T.F. by using Mason's gain formula for the signal J luw
graph given in the Fig. 6.14
~
Fig. ,.1+
Sol"tion- %wo forward paths, K ) 2,
11 ) G1G3G.1GSG,
%2 = G1Gl G,
oops are, l = - G + (!
2 ) . G3 G+ Gs(2
3 ) . G2 (2
-H2
Fig. ,.1+/a0 Non to"ching loops
1"t of these, 1 and 3 is com#ination of 2 non to"ching loops
$ ) 1. 2l + 2* 33 * 2l 33
r
"
Control System Engineering 6-9 Signal Flow Graph Representation
Al = Eliminate LI , L2 , L3 as all are touching to Tl from A
Al = 1
A2 = Eliminate L 2 and L3 , as they are touching to T2, from
A !ut L is non touching hence "ee# it as it is in A.
G4
$
- 1 1 ,
Fig. 6.14(b) L1 Non touhing to T2
%u&stitute in 'ason(s gain formula,
T) =
T) =
6.6 !omparison o" #lo$ %iagram an& Signal Flow Graph 'etho&s
The com#arison of &loc" diagram re#resentation and signal flo* gra#h is gi+en in a
ta&ular form as ,
Sr. #lo$ %iagram Signal Flow -ra#h
No.
1 . #asi importane gi(en is to the #asi importanegi(en is to the (ariables o" the systems.
elements an& their trans"er
.
"untions )
2 Eah element III represente& &y a Eah (ariable is represente&by a separate no&e.
blo$.
3 *rans"er "untion of the element is *he trans"er "untion Is shown along the branhes
shown insi&e the orrespon&ing o'e!ting the no&es.
blo$.
4. Summing points an& ta$eo"" points Summing an& ta$eo"" points are a&sent Any no&e an
are se#arate I ha(e any number of inoming an& outgoing branhes.
. Fee&ba$ path +s present "rom +nstea& of "ee&ba$ path. (arious "ee&ba$ loops are
output to +nput. onsi&ere&"or the analysis.
r
,
Control System Engineering 6-10 Signal Flow Graph Representation
6 . For a minor feedback loop present, Gains of various forward paths and feedback loops are
the formula aGGH can be used.
ust the product of associative branch gains. !o such
formula 1: t G G A is necessary.
7. "lock diagram reduction rules can The #ason$s Gain Formula is available which can be
be used to obtain the resultant used directly to get resuHant transfer function without
transfer function. reductionof signal flow graph.
8. #ethod is slightly complicated and !o need to draw the signal flow graph again and again.
time consuming as block diagram %nce drawn, use of #ason$s Gain Formula gives the
is re&uired t o be drawn time t o resultant transfer functson.
time after each step of reduction.
9. Concept of self loop is not e'isting Self loops can e'ist in signal flow graph approach.
in block diagram approach.
10. (pplicable only to linear time (pplicableto linear til)*einvariant systems.
+nvariant systems.
,.- (pplication of the General Gain Formula between %utput
!odesand !on +nput !odes
It was derived earlier t hat Mason's gain for!la is !sed t o get a relat ion bet ween
o!t "!t node and in"!t node #alled t ransfer f!n#t ion.
$!t oft en% it is re&!ired t o #al#!lat e t he relat ion bet ween o!t "!t node variable and a
non in"!t node variable.
E'ample .
g6
'ow it is re&!ired t o #al#!lat e eo!l i.e. de"enden#e of t 'o!l on () and () is not in"!t
e2
variable.
*o e%/+can be e'pressed as
e2
0+
+fro ein t oe,
E120
+fro -.int o (/lit
r
2
Control System Engineering 611 Signal Flow Graph Representation
Since 6 is independent of inputs and outputs,
xr 6 I
K J < from '-'ill to CouL
zr A I
K K from ejn 10C2
,,.. Example 6.2 Calculate Yl of the system, w/w..<;e signal florv graph is given below.
Yl.
!"#
Sol$tion or!ard pat"s for # t to # $ arc t!o
% t & GtGZG3G4, ~ & G1G%'(
)ndi*idual feed+ac, loops are
G1 G# G1 G2 G#
&' &'
C
-
::J
.'/
!"1 !"2
Sel(
)
loop
!"#
*1+!G1 "1 *2+!G#"2 *#+!G1G2G#"# *,+!",
Com+inations of t!o non touc"in0 loops
Ll Lz & -'1 ' 12t l)z,
L2 L33 & - ' 12 Z"H4 ,
L1L4 + 5'132 I 24
La L33 & -'1 ' ' a 21 24
6ne com+ination of t"ree non touc"in0
Ll L2 L4 & - G t G12) H 24
A & 1 ! 7l3 -lz -L15 8,,957L: L2 5 Lt 3333 5 l2 La 5 L1Ld-7lt L2149
;t & 1 a< loops are touc"in0
;2 & 1 all loops are touc"in0
*. ... & % l 61-% 2 62 + '1 '2 '1'4 - 'l '2 's
/t A 6
r
0
Control System Engineering 6-12 Signal Flow Graph Representation
Now to find the ratio Y2
YI
Forward paths for YI to Y2 is one. T. = 1 Now A is same as above.
and
.. L2 and L4 nontouhin! to 1" # 1
= 1+ $ % &2 + &' + $% & 2 &'
$I $2 $% $4 + $. $2 $s (1)$% &2*
t + G% &2 + & 4 + G% &2 &4
Y 7
Y+
# #
6.8 Application of Masons Gain Form!la to Electrical "etwor#
To app+, -ason.s !ain formu+a' it is neessar, to draw the si!na+ f+ow !raph of the
networ/. This setion e0p+ains simp+e method of writin! networ/ e1uations from whih
si!na+ f+ow !raph an be easi+, obtained. App+,in! -ason.s !ain formu+a to the si!na+
f+ow !raph' transfer funtion of the networ/ is obtained.
$enera+ proedure to so+ve suh networ/ is as fo++ows'
i) Find out the lAplace transform oj tIre gioen fretwork and redraw the tzetwork in
s-domain.
ii) Write down tire equations for the different branch currents and "ode voltages.
iii) Simulate each equation b drawing corresponding signal flow graph.
io) !ombine all signal flow grapkt" to get total signal flow graph for the gtoen network.
v) #se $ason%s gain Formula to derioe tire transfer function oj the given network.
$. E%ample 6.&' Find tire &.F. for tire given network.
( t
2I L 2o
34 44'4444.443
r
)
Control System Engineering 6-13 Signal Flow Graph Representation '~
Solution: Laplace Transform of the given network is as shown infollowing figure.
r r
VI(s) sL Vo(s)
!0------1---1--01
Key Point: Assume the network variables alternately as the loop current and node voltage
and then write the equations by analysing the horizontal and vertical branches alternately.
11(s)
(Vi - VI)
... (I)
=
RI
VI
1
... (Il)
=
(11-1) -
s!
h=
(VI - " o)
###(111)
R$
Vo
=
1sL ...(IV)
%.&.'. for e(uation (I) :
+.1 .
v 1
VI
R1
1
-R,
%.&.'. for e(uation (II) )
1
1 1 sC
V,
-1
sC
%.&.'. for e(uation (III) )
-1
R
%.&.'. for e(uation (IV) :
r
"
Control System Engineering 614 Signal Flow Graph Representation
Total Signal Flow Graph for the network is as follows.
1 1 +1
R, 11 sC V1 R2 12 sL
VjOOO- .... ~CJCA:7~-~--+---9VO
-1 -1 -1
R, sC R2
Use Mason's gain formula to find ~~.
v, 1; TK aK N b f d ths
!i "# urn er 0rorwar pa =1
!o '$i "t
..
!i
=
"
T%
&
=
R. R2'
$ndi(idual feed)a*k loops are,
1 1
&. + , R '' &2 + , -R ''
S$S 2
&, and &- are non tou*hing.
a + 1 , f&. . &2 . &-/ . $&l &-0
1 1 sl, L
A = 1 +--+--- +- +=-=--=
sR$ ' sR2 ' R2 R. R2 C
As all loops are tou*hing to Tl , al =1,
L
+
1 1 sL L
1. , , . , , . , . -=-=---=
sR. ' SR2 ' R2 R. R2 '
(, s&
=
!i sR R2 ' . R2 . R. . s2 & R$ ' . s&
!o1s0 s&
!ieS0 + s2&R$'.s2&.R%R2'%.1R. .120
6.9 Obtaining Block Diagram from Signal Flow Graph
To o)tain the )lo*k diagram from gi(en Signal flow graph, it is ne*essar3 to write the
set of s3stem e4uations representing the gi(en signal flow graph. "ss~* suita)le node
(aria)les, write the e4uation for e(er3 node. 5hile writing the e4uation remem)er that the
!
"
Control System Engineering 6 -15 Signal Flow Graph Representation
value of the variable represented by a node is an algebraic sum of all the signals
entering at that node. The number of outgoing branches have no effect on the value of
the node variable. For example consider the part of signal flow graph shown in the
Fig.6.15.
Z : = iv:
Y x w
(a) (b)
Fig. 6.15
!n both the case
x = ay b"
#nd the number of outgoingbrancheshave no effecton the value of node variablex.
This set of e$uations can be represented by bloc% diagram& simulating each e$uation
separately.
!ey "oint# For any or - sign in equation, there exists a Slimming point uihile for each
branch gain of signal flow graph, there exists a block of slime transfer function as brandt gain,
in the block diagram.
For example&consider the e$uation.
VI = 2V2-4V:t
'ere ( and ) are the gains* correspondingly there exists the bloc%s of transfer
functions( and ) and a summing point for a minus sign. The bloc%diagramsimulationof
abovee$uationis shown in the Fig.6.16.
+& =(+& ,)+-
1,,,,.
Fig. 6.16
/imulatingall the e$uations in the same manner and 0oiningall of them& the re$uired
bloc%diagramcan be obtained.
r
$
Control System Engineering 6-16 Signal Flow Graph Representation
'. Example 6.4: Obtain tile block diagram for the signal J1uw grapll, shoum in the figure
bekno.
5
-5
Solution: Write the equations for various node variables y, z, wand u. The mput node
is x as there are only outgoing branches.
y = !s"x # $"z # %"u
z = (3)y - (3)w
... (1)
... (2)
w = !"z
u = l"w +(s)z
The bloc& diagram simulations of various equations are,
'quation ("
.(3)
... $"
Fromz
Fromu
'quation !",
y
Fromw
'quation )",
r
"
Control SystemEngineering 6-17 Signal Flow Graph Representation
Equation (4),
u

Comining all the simulations an! repositioning ran"hes #or "on$enien"e we get the
"omplete lo"% !iagram as,
u
Examples with Solutions
&&&&&&&& & C(s)
I,...,... Example6.5: Fmd R(s) f o r S.F.G. shown in following figure.
Solution ' (umer o# #orwar! paths = ) = *
2
+,)&6)
,&F&
-
K..I
using .ason/s gain Formula ..
&6
-I -
GIG2G3G4
T2
-
GI Gs
r
"
Control System Engineering ~~;
6-18 Signal Flow Graph Representation
Individual feedback loops,
G1 G2
~
-H3
L1=-G1G2H3
G2 G3
~
-H2
L2 =-G2G3H2
Gs
Loops Ll and La are non touching loops.
=!L" #L2#L3#L$% #!Ll L3%
= 1-[-GIG2H3 -G2G31-h -G$Hl #G&H'H21#!GIG2G$H(H31
)onsider *l , all loops are touching .. l =1
)onsider *2, all loops are touching .. 2 =1
: !F= 1+.6I,*2- . GIG-G3G .. /1"GIG0/1
t " G1Gz H3 " Gz G1.1H- " G-IHI - Gs HI H- " GI Gz G. HI H3
)2s( 3 Gl G2 G 3 G$ " Gl Gs
R#s$ - 1" Gl G2 H 3 " G2 G3 H2 " G$ HI - Gs HI H- "GI G2 G$ HI H3
%
E& I 66 45 d )2s( b y . 6 5 . fi I
'' amp e ( : In R(s) ustng ason5gam ormua.
R#s$
o
1 C#s$
0
Sol)tion: 7u8ber of for9ard paths * . 2
6ason5s gain for8ula,
*.4.
r
'
Control System Engineering 6-19 Signal Flow Graph Representation
li = Gl G2 G 3 G 4, T2 =GS G4
Individual feedback loops are,
Ll and L3 are two non toucin! loops"
: . A . = 1-#Ll$L2$L3%$#LlL3%
& 1-#-G2 'I -Gl G2 G3 G4 '2 -Gs G4 '2%$#G2 'I Gs G4 '2%
& 1$ G2 'I $ GI G2 G3 G4 '2 $ Gs G4 '2 $ G2 Gs G4 'I '2
(or T" all loops are toucin!
)" *+1 =1eli,inatin! all loop !ains and prod,)-cts fro, ll.
.onsider T2,
L1Is non touching to T!
C"s#
=
R"s#
./s0 GI .2 G3 G4$G4Gs /1$-G2 'I0
R"s# & 1$ G2 'I $ Gl G2 G3 G4 %2$ Gs G4 '2 $G2 .s G4 'I '2
r
&
_ Control System Engineering. 6-20 Signal Flow Graph Representation
I'" Example 6.7: Find 0:;)
R(:;)
R(s !
o ..
1 C"s
, 0
Sol#tion: Number of forward paths = K = 1
T.F. = 11 A]
A
.... Mason's gain formula
T] = 12!"#
$nd%&%dual feedba'( loops)
$%&
'( ) $G!G&G(%&
*omb%nat%ons of two non tou'h%n+ loops)
%, -. and -2 %%, -$ and 's %%%, -$ and -.
%&, -2 and -s &, -! and 's
*omb%nat%on of three non tou'h%n+ loops) %s -!* -2 and -).
A = l-/-$ 0 -20 -! 0-. 0-sl0/-]-2 0 -1-10 -$-.) 0-2-1 0 -!-sl-/-]-2-s2
11 ) !0 $ 3$ 0 ! 3! 0 . 2 ! 32 0 . 3. 0 s 3s
0] ! 3$ 3!0. . 3] 3.0. s 3$ 3#0! s 3! 3s
0 . 2 ! ') 32 3)0l ! ') 3$ 3! 3s
Now 'ons%der%n+ T. = t 2 ! . Gs
All loops are tou'h%n+ to th%s forward path hen'e)
111 = 1
r or
Control System Engineering '.. :1 621 , + Signal Flow Graph Representation
C(s)
=
R(s)
Tl.61 Gl G2 G 3 G4 Gs 1
- / } , .- = / } , .
C(s) C1C2G3G4Gs
R(s) = - = - 1- +"""G= - I- H= - 1- +- - = G:- 3- :H= - = - 3- +- G= - 1 - = C:- 2- G= - 3- = H- = - 2- +- - = G= - 4- :H= - 4- +- G= - S - = H- = - ;;
+GIG3HIH3+GIG4HIH4+GIGsHIHs
+ G3 Gs H3 Hs + Gl G2 G3 G 5 H2 Hs
+GI G3 Gs HI H3 Hs
, _ Example 6.! Construt the signal flow graph for the followi'lg set of system equation.
!2 = . !I +G3 !3
!3 = G4 !l +G2 !2 +Gs !3
... (1)
... (2)
... (3)
!4 = G6 !2 +G" !3
where Y4 is output. Find transfer function ~; .
Sol"tion! S#st$% no&$ '(r)(*l$s (r$ !1 I!2 I!3 I!4
Cons)&$r $+u(t)on 1 : nus )n&)(t$s !2 &$,$n&s on !I (n& !3
S.F.G. for equation (1)
Cons)&$r $+u(t)on 2 : T-)s )n&)(t$s !3 &$,$n&s on !I , ! 2 (n& !3
S.F.G. for equation (2)
Cons)&$r $+u(t)on 3: T-)s )n&)(t$s !4 &$,$n&s on, !3 (n& !2
S.F.G. for equation (3)
r
"
Control System Engineering 6-22 Signal Flow Graph Representation
Combining all three we get, complete S.F.G. as shown,
G4
No. of forward paths = K = 4
!K "K = !l fl.# $!2 "2 $!% fl.%$!4.tQ
!.F. = &
K-l fI. fI.
!l = G' C2 C(, !2= C" C(, !%=C' C", !4 = C)C ::IGt;
*ndi+id,al loops are,
SELF LOOP
No non to,ching loop combinations.
Consider !1, both loops are to,ching
!2, both loops are to,ching
!%, for this -Cs- self loop is non to,ching,
!I, both loops are to,ching
L2 non touching to !
... .ason-s gain form,la
.. "t = '
l!.2 = 1
/. "% = l-C 0
/. ''4 = 1
"
"
Control System Engineering 623 Signal Flow Graph Representation
Gl G2 G7 + G-a G7 + G1 G6(1- Gs)+ G4G3Gt;
l-G2G~-Gs
' E!ample 6" # Find c(s)
R(s)
R(s) 1 1 C(s)
Sol$tion# %$m&er o' 'orwar( paths ) * ) +
~ Tf( ,-*
.F ) /ason's gain 'orm$la
.l ) G) G2G3 G4 Gs
0n(i1i($al 'ee(&a23 loops #
Com&inations o' two non to$2hing loops
i) 4l an( 43 ii) 42 an( 43
L\ ) l-546 +42+43+478 +54l 43+42 438
Consi(er
9ll loops are to$2hing # ,-+ ) +
C(s)
)
R(s)
r If
Control System Engineering ~!.' 624 Signal Flow Graph Representation
C(s)
=
R(s) 1+ G 2 H. + G. G2H2+ G4H + G2 G 3 G4H4
+G2G4H!H+G!G2G4H~H
'. E"a#ple 6.1$ % Find C&~'
R(s)
R{s) 1
-H1
Sol(tion % )(#*er o+ +orwar, paths = - = 1
..F. = ... By /ason's gain +or#(la
!n,i0i,(al +ee,*a12 loops
G4
Q
-H3
3= 4G4 H
G2 G3 G4
34= 4G2 G G4H2
Co#*inations o+ two non to(1hing loops
i' 3l an, 32 ii' 3l an, 3 iii' 3. an, 3s
Co#*ination o+ three non to(1hing loops.
i' 3l 5 32an, 3s
t,= Gs
i0' 32an, 3s
6 = !473. + 32+ 3+ 4+ 48+73l 32+ 3.3+ 4~+ 323s8 473! 32~8
Ii= l+G!H. +GH ..+G ..H+9.2GG4 H24GS+G. G H' Hoi
+G' G..H! H4G. H! GS4G H..Gs4G. G Gs H! H4
r
"
Control Systems Engineering 625 Signal Flow Graph Repre&entation
~l = 1 ... All loops are touching to the forward path.
C(S)
=
R(s)
C(s) _ G G!" G# G$%
&(s) $ '( G) ) + G!* ).. ( G$%)# ( G!" G# G+ )!" + Gs ( G' G,* ) )..
(G G$%) ''#$G- ) GS$G# G5 )+$G- G# G5 ) )
n.. Example 6.'' ! Fill,t C(s)
R(s)
Solution : .u/0er of forward paths ! "
1.2. = ... #y 3ason4s gain for/ula
1 = G. G!" G# G+ and 1!"= G G5 G+
ndi6idual loops
SE$F $%%&
7 (G' G1 )-
.o co/0ination of non touching loops
.. ((
=
l$87 + 729 ) $G# G.- ) $ )2
Consider
11
~ 0oth loops are touching .. ll.
=
12
~ 0oth loops are touching .. ''2 = '
C(s) 1- ''' + 12~2 GG2G#G+'(GG5G .. '
..
R(s)
=
* ((
r
+
Control System Engineering :'~'\tI. 626 Signal Flow Graph Representation
C(s) G1 G2 G3 G4 + G. G1 G4
R(s) = 1- G 3 G4 HI - H2
'. Eample !.1"
F
'.,1a
" m R(s)
R(s) 1
10
1 C(s)
-1 -2
Sol#tion : Num ber of forwr! "t#s $ = "
C(s)
$.%. % R(s) %
... &so''s ()' form u*
.$+ = ,4210 = 400 '! $2 = 110210 = 200
I'!)-)!u* *oo"s r.
4 2
& &
'1 -2
/* = -4 /2 = -4
0 -'
SE(F ())*
(1%'1
1om b)'t)o's of two 'o'-tou.#)'( *oo"s re
)) /2 '! /3 '! ))) /2 '! /3
No .om b)'t)o' of t#ree 'o'-tou.#)'( *oo"s 3
+ = I-4/* + /2+ /35 + 4/+ /3 + /2 /31
= 1 - 4- 4 - 4 '11 + 44 + 41 = 1 + 6 + 7 = 17
1o's)!er $. 8 /3 *oo" )s 'o' tou.#)'(
9* = 1 - /3 = 1 - 4 -11 = 2
1
o 0
, $1 4 2 10
0 0 0 0
r
,
Control System Engineering 627 Signal Flow Graph Representation
Consider T2, Ll is nontouching
1 10
-1 L1(Nontouching)
t\2 = 1- Ll = 1- 1- 4] =5
C(s)
"
R(s)
TJ t\l +T2li2 4OO2+2005 !OO+l000
"l # 1$ # 1! # 100
))" E# " 61% F O d C&s' by ( ) $ J o l
-r ampe% tn R(s) aso n s gam 'mit Qo
R(s) G1 G* C(s)
Solution Num&er o' 'orwar( paths # ) # *
2
1+ +) il,
)=1
C(s)
#
R(s)
--- (*son)s g*in .onnul*
"ndi/idu*l loo0s *re,
- H e
L1,
Ge
Ll # - 16 !6
2oth Ll *nd L3 *re non touching to e*ch other-
r
"
~ : I;~:~ntrol SystemEngineering , .." r 6 ..28 'fit :,. Signal Flow Grap !e"l:ftsntation I it: #II
.,.U
Consider 1 1 IL2 is non touching
T G6
o ,~ : ; y ~.,~p 0
G3
l 2v : ;;
$%&
Consider T2 IL. is non touching
$%6
~
G6
~.~'
G3
.6.2 ! 1 " L. ! 1 # G6 $6
'(s)
*
%&s'
C&s' G. Gs G6 G( Gs &1 #G3 $3'#G1 G2 G,3G) Gs &1 #G6 $6'
%&s' ! 1 # G6 $6 + G.3$3 + G 3G6 $3 $6
,....... *+,- .e 6 14 /0 d C(s)
12, 3.: In R(s)
!,s) - - ',s)
4ol ution: 5u-6er o7 7or8,rd 3,ths ! 9! 3
r
"
Control System Engineering 6-29 Signal Flow Graph Representation
C(s)
=
R(s)
K '"' 1
1 1 0 6. 1+ Tz f l z + T3 o 6. ' J
fl
TI = CI G;? G3 G" GS G6
T::! = GIG z C 7 G"
T3 = CI GZ G G! Gil
Individu! "##d$%& !o o 's (# :
""" #y )so n' s *in "o (+u!
o
-H3
,7 = - GIG $ G G-I C %% -:.
Co +$in/io ns o " /0o no n /o u%1in* !o o 's (# :
i) ,! nd ,2 ii) ,-I nd ,s i!!) ,I nd ,6
3o %o +$in/io n o " /1(## no n /o u%1in* !o o 's
"" fl = 1& '(l + ($+ () + ,. 4 + ) + ((* + (* + ,"/5 + II. I J -! + 1 . 2 (%%+ ,4 ,61
6GIG2C3C2G7G61 !3 6CIG:!G3C2G8-7 6GIG2C7G6-7 +G%l+l
+ C-I -7 G2 C4;4 -2+ G9 G4 I1 Gi! -I + G7 :-1G4 C! G7Ci/ 1 1 3
r
"
: Control System Engineering 630 Signal Flow Graph Representation
Consider TIl all loops are touching :. \1 = 1
For T2 I only LI is non touching.
For T3,aUloops are touching :..6. 3 1
C(s) "Ii .6.1 + T2 .6.2 + T3 .6.3
:. R(s) = a
C!s" #l #2 #3 #olGs #o + Cl C2 G$#% (1+ #ol 1&Iol" + #1 #2 #3 Col #s
'!s( = 1 + #4 )* + #s #o )I + #2 #3 #% #s )2 + C2 #7 )2
+ #1 #2 #3 C* #s #o Ih + #1 C2 C3 Col Cs )3 + C. C2 C1 #o )3
+ #' )I +#2 #% #$ )2 )oi+ #2 #$ c+)I )2 + CI #2 C$ #* c+)3 )*
'. Example 6.15: Using Mason's gain formula,find the gain of the following system in
figure below.
I C!s"
-H2
Solution: The nu,-er o. .or/ard paths are 0 = 6
The .or/ard path gains are+
Tt = #I#2#"+ T2 = #*#1#6
T3 = #I#$#6+ T* = #*#g#3
Ts = #*#s2&)2"#$#61 T6 = #.#1!&)."#g#3
The .eed-ac3 loop gains are+
Ll 4 & #s )I + L2 = & 2 )2 + L3 = + G7 )I #s )2
r
"
Control System Engineering 6-31 Signal Flow Graph Representation
The two non touching loops are Lt ~
4 = I-fl, +l] +L31+[l, L2]
= 1+ GsH, +G2H2 -G7GgH,H2 +G2GsH,H2
or TI, LI is non touching!
4 1 = 1- L, = 1+ "s HI
or T2 # $ is non touching!
or T3 to T% all loops are touching to all forwar&
paths!
.. fl.3
=
fl.. =fl.s =fl.6= 1
"ain =
LTKfl.)i. = T'(11+ r242 +T343 +T-Ifl. 1+ T)ll!) + I#*ll!!,
..
Il fl.
..
G,G$G3 +I+GsH&+G,G,G% (I + G2H:d+
"ain
G,G,G% + G*GgG'-G4G-G.G%H2 - G,G3G,GslI,
!!! /ns =
I +G)H, +G]1-l2- G0G'lH,112+G$G0H,I$
) I _ Example %!1%1 Obtain the overall transfer function of the system shoum by block
diagram, using signal J lO' w graph technique.
R(s)--.c
+
I----t-- C(s)
Solution: 2a3e the 4arious su33ing an& ta5e off points to &raw the signal flow graph
as shown,
r
"
Control System Engineering 6-32 Signal Flow Graph Representation
R(s)-~~
Input
t----+-..C(s)
....._.... Output
+
The corresponding signal flow graph is,
Forward path gains are,
T) = GI G2 T2 = G3G2
The arious loops are,
!l =-GIG2 l !2 = -G2 G3 "2
#o co$%inations of non touching loops.
:.Ii = &-'!!+!2+!3)= &+GIG2"2+G(G3")+G2"l
For TI, *ll loops are touching, +. iiI= &
For T), *ll loops are touching, +. li) = &
*ccording to ,ason-s gain for$ula,
./s) T) *l " T2 &i2
R(s) = Ii
./s) G& G2 " G2 G3
R(s) = & +G) G2 ") +G2 G3 "2 +G2 "I
r
#
Control System Engineering 6-33 r'
IIII. t
Signal Flow Graph ~epresentation .
'.. Example 6.11: Determine the transfer junction for the block diagram shown below by
Mason's gain formula.
Solution: Name the various summing and takeoff points to draw the signal flow graph.
1 C(s) R(s) 1
-1
The fo~ard path gains arc,
The various loop gains are,
and
LI = - Gl ~ HI 14 = - ~ GJ H
La ! " GI~G# L. ! " G,
Ls = 1 x G$ x 1%&" H' %~ %&" HI' ! ~ G, HI H
r
"
Contiol8ystem Engineering 634 Signal Flow Graph Representation
No combinations of non touching loops,
A . = 1 - [Li + L2 + L. 1 + Lt + Ls] = 1 + t ! "i
+ ~ ~ 1-12+ # 3+ . - . "# "2
A ll th$ loops a%c touching 1&'l to '2 h$nc$,
(sing )ason*s gain fo%mula,
C(s) '#A . 1 +Tz A.2
R(s) = A .
C(s) #+3 +4
R(s) = 1+1,"l +,3", +1Z3 +-l-,4"#"2
!"E#ample 6. 1-. For the signal flow graph of figure below, determine the transfer junction
C(s]/ R( s) using Masons gain formula.
, t !
$
Sol%tion$ Numb$% of fo%0a%1 paths &= '
'l = l#+,2= t,
Tz = 13 . . -t= 3-l
'3 = #-#-6-4-1= 1 t3 . .
("" = t-3-s-,-l= , ) G*
#n1i4i1ual loops a%$
Ll = -, "#,
L"" = +G' "# G,-
L, = - 3 "2, L3= s 6
Ls = -l 6 "2
5ombinations of t0o non touching loops a%$
i6 an1 L,
ii6 No combination of th%$$ non touching loops
A . = 1+(ll+L, +L3 +L-# +Ls6+7L# L,6
= 1-[7- 2 "# -3 ", +8 6-4 G6 "# -# 6 ",).+/(+ , "# 67- 3 " z )#
9:1 , ,$ r #,
Control System Engineering 6-35 Signal Flow Graph Representation
.. ll. = 1+G2 HI +G3 H2 -Gs Gli +G4 G6 HI +GI G6 H2 +G2 G" HI H2
For all the forward paths an the loops are touchin g hen ce
ll.1=ll.2=ll.3=l!.4=1
Hen ce
C(s)
R(s)
C!s"
#
R!s"
... (2)
". E$ample 6.1% & Find ~~:~
(M.U. : M.a!"#)
R!s" 1
$
$ C!s"
Sol'tion & %u&'er of forward paths = ( = 2
C!s"
=
R!s"
... Mason )s gain for&ula
In di*idual feed'ac+ loops are,
G1 G2 G- !H.
(((
U
!H$ !H2 !H-
Gs
/l = ! 0I HI /2 = ! 02 l)h /- = ! 0- H- /., = ! 0s H.
Co&'in ation s of two n on !touchin g loops
i) /I an d /- ti" /I an d /. iii) /2 an d /. i*) /- an d /.
Co&'in ation s of three n on !touchin g loops 1 /2 ,/) an d /.3
r
"
Contr0I~Y"l"" Engineering 6-36 Signal Flow Graph Representation
Consider TI, For this '!!' is non-to"#hing
-H4
WL4
Gs
T ,
$
G, G2 G3
$
0
%
0

0 !! 0 !!
0
!! 0
For T&, & and 3 are non-to"#hing
C(s)
'
R(s)
C(s)
R(s)
G$ G& G ( )$ * Gs +!!, * G- Gs G6 )$ * G& (-h * G3 .-I3,
'
) $ +GI +I +G& Ih * G3 +3 * Gs I$-l * GI G 3 +I $$3*GI Gs +I +-I
*G& G/ $-0&+" *G3 G/ +3 +-I *GI G3 #, +I +3 +I1
II. Example 6.20: Find tile oatue of C(s). )2!l(! 3 2a4-05, 6e#!-06,
U(s)
R(s) 1 1 C(s)
W(s)
Sol"tion3 7s s4ste8 has three inp"ts, #onsidering ea#h inp"t separatel4,
7ss"8ing 9)s, ' :;s, ' 0, S!F!G! <e#o8es,
TI ' GI G&
and I = GI G& +I +&
R(s) $
G, G2
1 C(s)
0 !!

II 0
=>$ , , : r I,
6-37
~t .
Using Mason's gain formula,
Ii= 1- [LJ] = I-GIG2HIH2
IiI = 1 as loop is tou!ing to "J
"J iiI G# G2
= ~ = 1- G# G2 HI H2
. C(s)
R(s)
C(s)
$ssum% U(s) = R(s) = 0
& & '&(&G& )%om%s as s!o*n in follo*ing figur%
TI = HI GIG2
LI = I-! HI GJ G2
Ii= 1- LI = 1- GIG2 HJ H2
+,1 = 1
C(s)
=
W(s)
"J +,# HI GI G2
-ll- = I-G# G2 HI H2
C(s)
$ssum% R(s) = -.s/ = 0
LI = G2' HI H2 G#
+, = I-Ll = I-HI H2GI G2
+,1 = 1
C(s)
U(s)
TIiii
= -ll- =
Signal Flow Graph Rprsntation

... (1)
W(s)
&&& .2/
U(s)
1 C(s)
... (!)
1 "otal output is om)ination of all t!r%% in2i3i2ual output&
4.s/ = GI G2 5.s/6H# GI G2 -.'/6G2 U.s/
1- "# G2 IIIH2
r
$
Control System Engineering 6 -38 Signal Flow Graph RaprGS!"tattDn IJ
J ,;.
'. Example 6.21 : Construct the signal flow graph for the following set of simultaneous
equations.
X2 = AZI XI + A2.1 XJ
X3 = A.u XJ +A32 Xl +A1.1 X3
X4 = A42 Xz + A4_~ X.1
(M.U. : Dec.-96)
Solution: The value o !he va"#a$le #% !he al&e$"a#c%u' o all !he %#&(al% e(!e"#(& a! !he
(o)e, "e*"e%e(!#(& !ha! va"#a$le. The va"#a$le% a"e XI , X2I X;I I X+ ,h#le !he &a#(% a"e
A21, A 2;1, A 31, A 32, A 33, A ..2 a() A ..
3-
So %elec!#(&(o)e% "e*"e%e(!#(& va"#a$le% a() %#'ula!#(& )#e"e(!#al e.ua!#o(%.
,!
... /.ua!#o( (I)
... /.ua!#o((II)
... /.ua!#o( (III)
Ihe"eo"e "e%ul!a(! %#&(al lo,&"a*h #%
'.... Example 6.22: Find transfer functions for signal flow graphs gien !elow
i)
_ G1 " # ~2_
o__.~ __~~ __G~1~~~G~2~ __.~~o
-H2
(M.U. : 0ov.-91)
231 , , : r I,
ContrUl S~ Engineering 639 Signal Flow Graph Representation
~1W:ion: i) Forward paths K = 1
Forward path gain Tt = ! "
!ndi#id$al %eed&a'( loops are
n) H,
V Selfloop
*t = ) 1 " +"
There is no non to$'hing loop 'o,&inations-
.
! J . = 1) .*l/*"0 = 1) .) t"+") +t0 = 1/-"+"/+l
1oth the loops are to$'hing to the %orward path
2 3 -l = 1
Using 4ason5s ain For,$la
C6s)
R(s)
ii)
R
o
1

1 C
" 0
Solution: Forward paths K ::1
!"i = #, " G3 7
!ndi#id$al %eed&a'( loops
891 : , : r !:
6 40 Signal Flow Graph Representation ~t 1 \ II
~' . . ~
Control System Engineering
-H,
L4 =-Gl G2G)G4 III
All loops are touching each other, no combination of non touching loops
= 1 + G.j I + G! G4 " + G2 G#\ G.. 1 1 2+ Gl G2 G3 G4 I
All the loops are touching to the for$ar% path hence A& = 1
C(s)
=
R(s)
iii)
R
o
1

Solution: 'or$ar% path ( = 1


1 1 = G,G2G3G4
In%i)i%ual fee%bac* loops are
~
G, G: !
Ll = -G& G2 2
1 +
.. 0
r or
Control System Engineering 6-41 Signal Flow Graph Representation
The loops L2 and L4 forms combination of two non touching loops. Similarly Ll and
L.. also forms combination of two non touching loops.
L2 L4 = G2 G4 ! "
L. L4 = G. #2 #ol $ %%&
11 = 1-'Ll(L2(L)(L4*('L2L4(Ll L4*
= 1+ Gl G2 lh ( #2 !!. ( G2 G) G.. ( 4G4 ) ( G+ #ol ! !!%. + G. G2 #., + 1-1)
-.loops arc touching to forward path hence 111 = 1
C(s) Tll11
=
R(s) =r:
C(s) G. G2 G%. G.,
.. /0s$ 11 1( G1 G2 112+ c. li-2 -( G2l"., Col !! .. + G4 1 ( l.21G3i!.!-13 + G, G2 G. !b 11)1
,,.. Example .23 ! Use Mason's Cain Formula to obtain ~~~ of the system snoum below.
C(s)
04... % 5an-62$
Sol"tion ! 7ame all the summing and ta8e off points and assuming each as a separate
node draw the signal flow graph.
r
#
Control System Engineering 6-42 Signal Flow Graph Representation
1
C(s) R(s)
-1
Number of forward paths K = 2
Forward path gains
Individual feedback loops are
~-1.~

-1
-1
! .a = -G1 G..
! " = #1 #2 #$ I-%&
There is no combination of nontouching loops.
A = 1- '! l +! 2 +! ( +! .. +Ls +! ) + ! " *
+ll loops are touching to both the forward paths
,. -liminating all loop gains from +we get.
+I = +2 = 1
r
"
Control Systei.1 Engineering :..:: 643 Signal Flow Graph Representation
...Using Mason's Gain formula,
C(s)
R(s)
-
-
C(s) _ C. G2 G t (~t G..
r. !"(s) - 1- G# ('$2 - G2 G) %2 & G# C2 G3+ G# G4 + ~ G2 G4 !!! III& G1 G2 I-IJ - G. G2 Col #-#2
'. Exaple !."#$ Draw the signal flow graph and hence obtain the transfer f u n a i o n of the
systel1J shoum below. (M.U. : 'ul(-)#)
R(s) C(s)
*olution: +ame all t,e summing an- ta.e off /oints as s,o0n 1elo0 an- re/resenting
ea2, se/aratel( as a no-e -ra0 t,e signal flo0 gra/,.
...
R(s)
H,
R(s)
-H2
-%&
..
-1
+um1er of for0ar- /at,s are '("
3or0ar- /at, gains are
4l 5 G)G2G3
42 5 G4
1
C(s)
)I )I
_6# 6 "r ,
Control System Engineering 6-44

Signal Flow Graph Representation I~,
Individual feedback loops are
-H2
1 . . , , 2= -G2 G ! 2 " = +G2 G. I-II 1 #1 2
.
-1
"4 = -G1 G2 G)
$here are no co%bina&ion of non &ouching loops
-1
"s = -'. .

(ll &he loops are &ouching &o bo&h &he forward pa&hs

~1 = 1 1 2 = 1
. . . )sing *ason#s gain for%ula
'+s, Tt A) +$2 (2
-
-
-+s, 1 1

'+s,
Rs)
!" E#ample$"2%& The small signal equitalent circuit of Q common emitter transistor
alnplifier .s shoum below. The transistor amplifier includes a feedback resistor RI:Obtain a
signal flow grapll model of the feedback amplifier and determine the input-output ratio
(Va / Vinje +*. ). / 0ec. -16,
+
2ee
- -
-
I I
341 4 "r ,
Control System Engi ng 6-45 Signal Flow Graph Representation
Solution: Let input current be iin and load current be it .

"
I f
Rs hie
R,

A C F
I L
H
B

in
I ,
!ee
- -
-

E o J I
At node "#

lin + I f $ I b
... (1
Appl% &!L to loop A"C'EA #
... ()*
+ubstituting

lin $ ,b -I !
f ro- (1 in (" .e get
/in # (ib - if * Rs + I lie ib +hre !ee
!in $ ib (R# +hie* - if R# + hre !ee
... (0*
1he output e2uation is
!ee # iL Rl
... (4*
$t node 3#
... (%

it # - (ic + if * substitute in (4* .e get


!ee # -(ic + if * R&
... (6*
4o.
and
as 556e is ad-ittance

... (7*
+ubstituting (7* in(6* .e get
!ee # - 855ce!o9:ib9if ;Rl
Appl%ing &!L to the loop 3"C'I 3

... ('

I . e .
... (<*
I I
=>5 > "r ,
Control System Engineering 6-48 Signal Flow Graph Representation
Now the different variables are
Yin = input

Vee = output
and if lib are intermediate currents.
To setch the signal flow graph let us obtain the e!uations for ib " if and Vee. Vi# is
input hence no e!uation for Vin is e$pected.
%rom &'( we can write e!uation for ib as

)b =
Vin + if *f -Ilrc Vee
&*# + hie(
+
I . e .
+
)b =
1 VI." , .Rf h re V
) (
.- * ) ( .f - R ).( ce
R!+ -ic & s + .ie s + -)tt
.. 1"#
This can be simulated by signal flow graph as
1
%rom &8( we can write e!uation for if as"
. - hoe l/l Vee -0i *1 ib - Vee
)f = *l

I . e .
... 11#
This can be simulated as
+
-0(
+
V! )b )f
"

"-
$

&. + hoe*1(
-
*.2
+
E!uation &3( is directl4 the output e!uation

I . e .
Vee = if *f + ib hie + hre Vee
*f hie.
)f + )b
.- h re .- Ilre
. .. 1%#

I I
2 5. 5 "r ,
Control System Engineering 6-47 Signal Flow Graph Representation
This can be simulated as,

Combining all three signal flow graphs, the complete signal flow graph can be
obtained as,
1

I f Vee -J
-h!e
R + h.
s ~
I ndi"idual feedbac# loops are,

-~
-
I
$% = + Jl &f hre '
(&s ) I l ic* (1 - 11rc*
I . I
'+1 + "r ,
Control System Engineering 648 Signal Flow Graph Representation
L
-hie hre
4 =
(R, + 1 1 ic) (1- 1 1 re)
Ls = -Rf hie(l+h(lt>RI)
RL (R, +11i)(1- 11re)
!
!! A = 1 - [Lt + L2 +L3 + L.a + Ls
As there are no co!"inations o# t$o nonto%ching loo&s.
A') the loo&s are to%ching to "oth the #or$ar( &aths r. ~1 =2 = 1
... Accor(ing to )ason*s gain #or!%la
+ee ,1 A1 + ,l -2
-
-- -
"in #
.i!&li#/ing # ,
I!. = 1 0 1 Rf 0Rf(l0hoeRt2
(Rs 0 h ic.-) RL(1 - h r-)
3
4 3 Rf hre hie hre $%f Ili& (1+ hoe RL)
- 0 I ------&&&,&'---
(R, + 1 1ie) (1-1 1 re) (R, +11ic) (1- 11r-) RL (1%+ 1 1 ie) (1- 11re)
[RL (Rs + hie) (1 - hre) +1 Rf Rl (1- llre) 0 n, (1+ hoc Rt) (Rs + hie)
:. I!. = - Rr hre Rt 0 hie hre RL 0 Rf hie (1 0 hoe Rl5
(Rl (Rs +1 1ie) (1-11re)
:. I!. = Rf (1 + hoe RL2 (Rs + 2 hie) 0 Rl IRs (1 - h re) 0 h ie4 0 'i Rf Rl (1 - 2 h re)
RL (Rs+llie) (1- hre)
+c- (Rs +hie) (1 - hre)
. --------
!! "in - -
hie RL-1 3 R, RL
= - - - - - - - - - - - - - - - - - - - - - - - - - - - - - - - - - - - - - - - - - - - - - - - - - - - - -
R# (1 + hoe RL)(Rs + 2 hie) + Rl (Rs (1 -1 1 re) + hie0 -Rf R. (1-2 h,e)
In&ractice #or anal/sis o# s%ch net$or6 the 7al%es o# h oe an( 1 1 re are neglecte(.
8. .i!&li#ie( ratio -8 neglecting hoe an( h re is
! I& ! I
9:1 : "r ,
Control System Engineering 6-49 Signal Flow Graph Representation
n, (Rs + 2h ie)+RL (Rs + hie) +~Rf Rl
I I I . Example 6. 26 Find T.F. for the given network, using Mason's gain fonnula
(M.V. : May-97)
c
Sol!tion "apla#etransform of the gi$en network is,
% &arallel R'
( +sR'C
---r
Z
I ~
R2 ). *s+ R2 )o*s+
(
Rl x-
RI
z,
!
s"
!
# #+ Rl "s
R(+-
s"
I(s)
($i - $ol
C+sRI ")
! ! *)I - $o) Rl
,
)*I *s+ ! I *s+ R2
%or e&'ation (#) ( )%* is,
. . . *(+
. . . *2+
%or e&'ation (2) ( )%* is,
I *s+o
+,# , ,: r I,
Control System Engineering 8-SO Signal Flow Graph Representation
Combined S.F.G.
" " ' = ' !>:7v,

_ ( 1 +:~,c)
Use Mason's gain fom!"a. #!mbe of fo$ad %a&' ( ) ( 1
1i = *+ 1+S*, C)
! *"
_
*+(
1 +*S*,, C- )
,ndivid!a" "oo% ." (
"! /
1 +sR#C
R,
..
*+ 1+ S*,C)
1 1 = 1 - 0."1 = 1 + *& (
*" +*+ ( 1 +s*, C)
*&
R:r 1+ s*, C)
*" + *+ ( 1 +s*& C)
##$% E&ample '%(7: Draw the signal flow graph for the following network and find the
,I'. ft . Vo(s)
transfer tnction Vi ( S)'
Sol)tion: Conve& &'e given ne&$o2 in i&s "a%"ace fom and ass!me diffeen& "oo%
c!en&s and node vo"&ages as s'o$n.
1 1
sC# SC(
1
*1
+%, R1
!-
"!S, R( "os,
1
"
_ 31 , , : r ,,
.' Control System Engineering 6-51 Signal FlowGraph Representation
Writing down equations for 11, Vl ,h,Vo we get,
(Vi - VI)
I I = 1 = SCI (Vi - V 0)
sCt
VI = (lI-lz)Rl
h = (VI Vol = sCz (V1 - Vol
sCz
v, = 1! R!
Si"ulating a#o$e equations #% signal tlow gra&'(
V i 0
sC, 1,
$,
((
"'=
'7
For equation (I
" -sC,
"
R,
V ,
For equation (II
-R,
V , sC2
For equation (III
sC!
1! R! Vo For equation (I"
0 II 0
Co"#ining we get signal flow gra&' for gi$en networ)(
*o find *(+( a&&l% ,ason-s gain for"ula
*(+(
#E$% l
.u"#er of forward &at' = / = 1
=
&
*(+(
'Ii I
..
=
=x:
$l = sCt Rl sCz Rz = sZ Rt Rz Ct Cz
''' ((
...(ll
...(Ill
... (I"
011 , ,: r I,
Control Systems Engineering 6-52
>
Signal Flow Graph Representation ~,
Individual loops :
Lt = - Rt Ci 5, L2= - sRt C2
Out of three, Ll and L3 are nontouching
A = - !Lt " L2" L3#"!Lt L3#
= -! -sRt Ct -sRt C2-$R2 C2#"!$2 Rt Ct R2C2l
= " $ !Rl C " R% C2" R2C2# " &2 !Rl Cl R2Cd
V('I(s) At
'ie$( = )
All loops arc touching to "Ii, ! At = "
,,!!! E#ample 6*2&: Determine the transfer Junctions C%Rt and C!R2 from the block diagram
shoum by drawing its signal flow graph and using Mason's gain formula +,*-* : ,a.-//(
R,
c
Sol$tion Let us dra0 the signal flo0graph to use the ,ason1s gain for2ula*
3he signal flo0graph is,
~ G2
4or !Rt" let R2
=
5
R,
"
C
!! 3I
=
6I63
and 32
= 6263
L%
=
-63 7I
- H ,
A
=
l-!LI# = -!-63 7I#
=
"63 7I
Al
=
and A2:88:8
C "Ii At "32A2
= 6l 63"62 6%
63+6I"62(
!!
R
:88:8
=
Ii "63 7I "63 7I
4or CI R2, let Rl
=
5
9: , "# r I,
,.Control System Engineering 6-53 Signal FlowGraph Representation
R2
Hencesignal flowgraph reduces to,
TI = G3
Ll =
- G3 Ht
A
=
1-LI
=
1G3 HI
!t
=
t
=
R2
=
c
,_... Example 6.!" Pind the T.F. of tire given network. "#$%$ & #a'-2((3)
1 1
*I"+) R2 R, *o"s)
1(---1------(1
Sol#tion" $apla%eTransfor. o& the gi/en networ' is,
R, /, R3
01uations for different currents and /oltages are
1
1
+$2$G$ ()*
V i
R,
I, V ,
II = "*i - *I) 3 - ... ()*
RI
-1
I,
i&,
+$2$G$ "II) *I = "l4 -12) R2
$$$ "II)
1
-R2
V ,
R+
.
1
+$2$G$ "ill)
12 ="*I - *o) 5 R3
$$$ "III)
-1
R3
+$2$G$ "I*)
1
R"
V a
*o && 12 R, ,.. ()-*
. $$ /
r
0
Control System Engineering

6-55 Signal Flow Graph Representation
Solution: For the given current distributions,

For branch rl,


... (1)
For branch r3, ... (2)
_ Now current through f2 is (i2 - a it). Hence according to h!"s #aw,
. . $2 - $3
%2 - a %% =

...(3)
And . .. (&)
'he si!u#ations of a## the e(uations are,

a
%
-
%
-
1
Equation (1) Equation () Equation (!) Equation (")
'hus the overa## signa# f#ow gra)h is as shown,
a
% %
--
--
'here are two forward )aths,
,
'# =
# #
_*% * "r ,
Control System Engineering 6-56 Signal Flow Graph Representation

The various loops and loop gains are,


a
1
-
1 1
-- --
r ,
One combination of two non touching loops is Ll L3

ll loop are touching to all the forward paths,

!" = #$,% =1

-
-
r& f' + a r..
rl r% rl
- - - - - - - - - - - - - - - ~ ~ - - ~
- l-(L) *L%*L& *L ++l*(Ll L&,
-
-
rs r& )t a r+, r! f++"
1+ + + ..- ...+-..... _...
r- f% f% fl r" "-%

.& r&r'*ar'r%
= - - - - - - - - - - - - - - - - - - - - - - - - - - - - - -
.) ft r% +fl r% +r& rl +rl r' - a r% r- +r/, r'
$+ Example 6.31 : Obtain the tramp junction ~:~:; for the network shoum ill the figure
below. 01+2+ 3 1a4-%55&-
6
c c
6
~ - - - - - - - - - - ~ - - - - - - - - - - - - o
I. I
78" 8 "r ,
-
Control System Englne.ring Signal Flow Graph
Solution: The Laplace domain representation of the given network is shown below.
The various branch currents are shown,
R
V(s)
!"# 1
sC 1
!$2)
R
%
%%
!S# = = sC Vt !"# - sC Vr (s) ... (1)
.a
Then,
1 1
= RVI (s) - RV,,(s)
... !&#
... !'#
and also,
Vo(s)- (!S#
1 = sCVo(s)-sCV.(s)
sC
From this obtain the e)uation for Vo(s) as &!"# e)uation is alread* obtained.
+ote, -rite the separate e)uation for separate branch and each element must be
considered at least once.
%

... !.#
/ence the signal flow graph is,
+1
The forward path gains are,
1
0 sCR T& 0 sCR
" I I
12 2 "r ,
Control System Englne.rlng 6-58 Signal Flow Graph Representation
The various loop gains are,
1
Ll = - sCR L2 = -_
sCR
The loops Ll and L2 are non touching
1
LJ= --xRxl = - 1
R


Hence system determinant is,
t,. = l-Ll + L2 + L!"#L1L2$
= 1 %&' 1 1 1= !s%R#s2%2R( +
1
#5 '+sCR+ + s%R
) ) )
*1 = 1 all loops touching to .lj
112 = 1- Ll as L is non touching to T2
=!1# sCR
all loops touching to T+,
*ccording to -ason's gain .ormula,
"o!s Tll1$ #T2(2 #T!#!
-
-
/i !s l!.
!s%R#02%2R2 #1
sCR
02%$R2 #1#s%R#s%R
sCR
=---------
!s%R#s2%2R2 #1
sCR
1
sCR 1+sCR+1
-
-

-
-
02%2R2 #2s%R#l
s2%2R2 + !s%R# 1
%. Example #.&$' Obtain ooerall transfer function.
1
1
R!s
C!s
(M.V. : -ay - 21152
(. (
_ ,1 , "r ,
Control System Engineering 6-59 Signal Flow Graph Representation 'tr
... Only one forward path
-H3
L3 = -G,G2G3G4H3
No combination of non touching oop!"
""" # oop! touching to $
C(s)
=
R(s)
%i&% G'G2G3G4
----x -- = 1(G2H (G3H2 (G%G2G3G4H3
'" )*ampe 6"33+ Draw the signal flow graph and derive the transfer function using
M1lson's gain formula.
Solution Name the !umming and ta!e off point!"
C(s"
,-"." + -a/ - 20061
r
#
-
-
Control System Engineering
6-60
Signal Flow Graph Representation
The signal flow graph is,
R(s) 1
1 C(s)
... K = 2 forward paths
All loops are touching to each other, so no co!ination of nontouching loops.

... All loops touching to !oth forward paths


"
"
C(s)
R(s)
G1G2+G1G3
-
-
#$G%G2&l +G2H2 -GtG2G3&t&2
'(# ( I : r II
I
Control System Engineering 6-61 Signal Flow Graph Representation
Review Questions
1. Define signal fluw graph.
2 Defirrefollowitlg terms related to signal fluw graph
(i) Source node (ii) Sink node (iii) Chain node (iv) Forward path and its gain
(v) Feedback loop and its gain (vi) Self loop (vii) Nontouching loops.
3. !plain the various properties of signal flow graph representation.
". !plain how to construct signal fluw grap#$from
(i) Set of e%uations (ii) &lock diagram. 'ive silitavie e!ample.
(. State and e!plain )ason*s 'rin +ormula.
,. Compare block diagram representation witll signal fluw grapl! representation.
-. Construct the signal fluw graph for the folluwing .et of simultaneous e%uations.
X a =AI2 X 2 + An X 3
/. Find transfer 0unctions for signal fluw graphs given beluw
1
,
o
,
o
-H2
(
A C(s) Gt G1 )
ns. : R(s)= 1+GI Gz Hz+Ht
9.
1
,
1 C
, 0
R
o
10.
r

Control System Engineering .. 6-62 Signal Flow Graph Representation


11. Draw signalflow graphfor the system shown below.
Find overall transferfunction using Mason's Gain Formula.
C(s) R(s)
(
A C(s) 10 (s +7) )
ns, : R(s) :: 52+165+40
12. For system shown, obtain the closed loop transferfunction by Mason's Gain Formula
(
A C(s) GI G'1.G3 )
ns. : R(s) = I-G. G'1.HI +Gz G3l1z +G1 Gz G3
13. se Mason's Gain Formula tofind !".
1
(
Ans..' Xz.. G.GZG,G.G7G. +GIG CGG7G +G,GzG,GsG ... GIG.G,G. )
. XI 1- GZG3G,C7G,1f - G!G,G,C.H " - GzG,GsG.1f - G#GsG $l% - G,G6& 7Hz- C,GsHz
r
"
Control System Engineering 6-63 Signal Flow Graph Representation
14. Use Mason's gain formula to calculate C/R oj tile system shown below.
c
(
Ans . C _ G1 G2 G3 +GI G4 )
.. R -l+Gt G4 +Gt c,G3 +G1 Gz HI +Gz G3 Hz +G4 H2
15. Draw the signal f l ow graph and obtain tire transfer function of the system shawn below.
C(s)
16. UseMason's ain !ormula to obtain C"s#/R"s# oj the system shown below.
C(s)
r
"
Control System Engineering 6-64

Signal Flow Graph Representation
17. For the signal flow graph shown in following figure determine " I f ratio ~/Xl. Use Mason's gain
formula for signal flow graphs.
X1
A Xs G 1 G1G3 G4 +G1 G4 G, +G1 G7 (l-Gs)
ns. Xt ! t - G1 "1+G2 Gs "# - Gs
18. Using MIlSon's gain formula dete,nzine tile system output for input R and disturbances
Dt an$ ~ for the syste,,, described in the block d iQ gr D m

%l(s&
1
5+1
1
% _ _ - . . j1' --.R
s + ( Cs!
2"
#
Ans. Cs) $% Rs) + s + 1)"1s) + D&s)'(
(s)(s +1&(s +(&- (' *
1. !raw the signal flow graph lin$ obtain 1M transfer function.
A
R)s* 1
Cs)
. .
+
A+,+(A,
+ns. , t- +- '- 3+-
.I . .I
-.1 . "r ,
Control System Engineering 6-65 Signal Flow Graph Representation
20. Drau: tilt signal flow graph and obtain the transfer function.
ODD

I I
_\1 \ "r ,
(666)
_\1 , , : r I,
Time Response Analysis of
Control Systems
7.1 Background
Most of the control systems, use time as its independent variable, so it is important to
analyze the response given by the system for the applied excitation which is function of
time. Analysis of response means to see the variation of output with respect to time. The
evaluation of system is based on the analysis of such response. This output behaviour with
respect to time should be within specified limits to have satisfactory performance of the
system. The complete base of stability analysis lies in the time response analysis. The
system stability, system accuracy and complete evaluation is always based on the time
response analysis and corresponding results.
This chapter explains the concept of time response, steady state analysis, transient
response and derives the various transient response specifications.
7.2 Definition and Classification of Time Response
Time response of a control systemmeans, how output behaves with respect to time. So
it can be defined as below.
Definition : Time esponse : The response given by the system which is function of the
time, to the applied excitation is called time responseof a control system.
!n any practical system, output of the systemta"es some finite time to reach to its final
value. This time varies from system to system and io# dependent on different factors.
Similarly final value achieved by the output also depends on the different factors li"e
friction, mass or inertia of moving elements, some nonlinearities present etc.
$or example consider a simple ammeter as a system. !t is connected in a system so as
to measure current of magnitude %A. Ammeter pointer hence must deflect to show us %A
reading on it. So 5 A is its ideal value that it must show. &ow pointer will ta"e some
finite time to stabilise to indicate some reading and after stabilising also, it depends on
various factors li"e friction, pointer inertia etc. whether it will show us accurate 5 A or
not.
'asedon this example, we can classifythe total output response into two parts. $irst is
the part of output during the time, it ta"es to reach to its final value. And second is the
(7 1!
r
Control System Engineering 72 Time ResponseAnalysis of
Centrol Systems
final value attained by the output which will be near to its desired value if system is
stable and accurate.
This can be further explained by considering another practical example. Suppose we
want to travel from city A to city B. So our final desired position is city B. But it will take
some finite time to reach to city B. Now this time depends on whether we travel by a bus
or a train or a plane. Similarly whether we wilt reach to city Bor not depends on number
of factors like vehicle condition, road condition, weather condition etc. So in short we can
classifythe output as,
i) here to reach !
il) "ow to reach !
Successfulness and accuracy of system depends on the final value reached by the
system output which should be very close to what is desired from that system. hile
reaching to its final value, in the mean time, output should behave smoothly.
Thus final state achieved by the output is called steady state while output variations
within the time it takes to achieve the steady state is called transient response of the
system.
Definition : Transient Response
Tire output variation during the time, it takes to achiel'e its final value is called as transient
respoJlse. Tile time required to achieve the final value is called transient period.
This can also be defined as that part of the time response which decays to #ero after
some time as systemoutput reaches to its final value.
Key Point : The transient response may be exponential or oscillatory in nature. Symbolically
it is denoted as cr (t).
To get the desired output, system must pass satisfactorily through transient period.
Transient response must vanish after some time to get the final value closer to the desired
value. Such systems in which transient response dies out after some time are called Stable
Systems.
$athematicallyfor stable operating systems,
%im
ct&t) ='
t - - - + ''
(rom transient response we can get followinginformation about the system,
i) hen the system has started showing its response to the applied excitation!
ii) hat is the rate of rise of output! (rom this, parameters of system can be
designed which can withstand such rate of rise. )t also gives indication about
speed of the system.
iii) hether output is increasing exponentiallyor it is oscillating.
r
"
Control System Engineering 7-3 Time Response Analysis of
Control Systems
iv) If output is oscillating, whether it is over shootting its final value.
v) When it is settling down to its final value ?
All this inforation atters uch at the tie of designing the systes.
!efinition " #teady #tate Response
. It is that part o f the time response which remains after complete transient response vanishes
from the system output.
$his also can %e defined as response of the syste as tie approaches infinity fro
the tie at which transient response copletely dies out. $he steady state response is
generally the final value achieved %y the syste output. Its significance is that it tells us
how far away the actual output is fro its desired value.
Key &oint" $he steady state response indicates the accuracy of tire system. $he symbol for
steady state output is C / I 5 '
'ro steady state response we can get following inforation a%out the syste "
i) (ow uch away the syste output is fro its desired value which indicates
error?
ii) Whether this error is constant or varying with tie? #o the entire inforation
a%out syste perforance can be o%tained fro transient and steady state
response.
(ence total tie response c)t) we can write as,
The difference between the desired output and the actual oulput o f tire system is called steady
stau error which is denoted as e". This error indicates the accuracy and plays an iportant
role in designing the syste.
$he a%ove definitions can be shown in the wavefor as in the 'ig. 7.* )a), )%) where
input applied to the syste is step type of input.
c t) c)t)
+*)l)
css)t)-
#tep #tep
css)t)-
#t d lie
,
lie
ea y_
Transient
,
state of Transient
tie syste tie
(a) C1(t)' s exponential (b) t (t) +s osillatory
'ig. 7.*
+-
r
!
Control System Engineering 7-4 Time ResponseAnalysis of
Control Systems
7.3 Standard Test Inputs
In practice, many signals arc available which are the functions of time and can be used
as reference inputs for the various control systems. These signals are step, ramp, sawtooth
type, square wave, triangular etc. But while analysing the systems it is highly impossible
to consider each one of it as an input and study the response. Hence from the analysis
point of view, those signals which are most commonly used as reference inputs are
defined as tandard Test Inputs. The evaluation of the system can be done on the basis of
the response given by the system to the standard test inputs. !nce system behaves
satisfactorily to a test input, its time response to actual input is assumed to be upto the
mar".
These standard test signals are,
i) StepInput (Position function) :
It is the sudden application of the input at a specified time as shown in the #ig. 7.$.
%athematically it can be described as,
r&t' = A for t ~(
=0 fort)!
If * + ,, then it is called unit step function
and denoted by u&t'.
-aplace transform of such input is *.
s
Ii) Ramp Input (Velocity function) :
It is constant rate of change in input i.e. I'
gradual application of input as shown in the
#ig. 7...
%agnitude of /amp input is nothing but its
slope. %athematically it is defined as,
ret' + At for t z D
+ ( for t !
If * + ,0 it is called 1nit /amp input. It is
denoted as r!t". Its -aplace transform is 2.
s
iii) ParabolicInput (Accelerationfunction) :
T#is is the input which is one degree faster than
a ramp type of input as shown in the #ig. 7.4.
r t$
*,--------
o
%ig. 7.& Step
%ig. 7.3 Ramp
t$
'Slope=AI
o
%ig. 7.( )ara*oli
r
+
Control System Engineering 75 Time ResponseAnalysis of
Control Systems
Mathematically this function is described as,
A t2,
2
r{t) = for t ~o
= 0 I for t<O
where A is called manitude of the !arabolic in!ut
Key Point: Parabolicfunction is expressed as ~ t2 so that in Laplace trans/onns 0 / different
standard inputs, similarity will get maintained.
If A " #, i$e$ r{t) " ~ it is called unit !arabolic in!ut$ %ts &a!lace transform is ~$
Iv) Impulse Input:
It is the in!ut a!!lied instantaneously 'for short
duration of time) of (ery hih am!litude as shown
in the )i$ 7$5
It is the !ulse whose manitude is infinite while
its width lends to *ero i$e, t +, 0, a!!lied
momentarily $
Area of the im!ulse is nothin but its
manitude$ If its area is unity it is called -nit
%m!ulse %n!ut, denoted as B(t).
Mathematically it can be e.!ressed as,
r{t) " A, for t " 0
= 0, for t ~ 0
ret)
1
A
)i$ 7$5 Impulse
/he &a!lace transform of unit im!ulse in!ut is always #$ '0efer 1ha!ter+2)$ /he unit
im!ulse is denoted as l)(t).
r(t)
Symol
R(s!
"nit step
u(t# ##s
"nit ramp
ret) 1/52
"nit paraoli$ +
1/5
"nit impulse
%t) #
Tale &.1
r
'
Control System Engineering 7-6 Time ResponseAnalysISof
Control Systems
7.4 Steady State Analysis
As discussed earlier, steady state is that part of the output which remains after
transients completely vanish from the output.
Mainly the steady state response has following two specifications,
i) How much time system takes to reach its steady state which is called seHling time
which is discussed later in connection with transient response. It is related to
transient response also ecause same time will e re!uired y the transients to die
out completely from the system output.
ii) How far away actual output is reached from its desired value which is called
steady i""tateerror #ess)$
%ut of the two specifications, the steady state error is the most important
specification which is related only to the steady state. So let us see on which
factors it depends, how to calculate it and how to reduce it.
&efinition ' (teady (tate )rror ' It is the difference between the actual output and the
desired output.
*ow reference input tells us the level of desired output and actual output is fed ack
through feedack element to compare it with the reference input. Hence to e precise it
can e defined as the difference etween reference input and the feedack signal #actual
output).
and L(e#t) + =)#s) =,#s) - -#s), for unity feedack systems.
Mathematically it is defined in .aplace domain as,
L(e#t) + =)#s+ =,#s)--#s)H#s), for non unity feedack systems
7. !eri"ation of Steady State Error
-onsidera simple closed loop system using negative feedack as shown in the /ig. 7.6
C(s#
$%s#
*ow,
/ig. 7.6
)#s) 0 )rror signal, and 1#s) 0 /eedack signal
)#s) 0 ,#s) - 1#2)
1#2) = -#s)H#s)
where
1ut
r
&
Control System Engineering 7-7
"
E(s)
=
R(s) - C(s)H(s)
and C(s)
=
E(s)G(s)
.. E(s)
=
R(s) - E(s) G(s)H(s)
.., E(s) + E(s)G(s)H(s)
=
R(s)
Time Response Analysis of
Control Systems
R(s)
E(s) = 1+G(s)
for unity feedback
R(s)
E(s) = 1+ G(s)H(s) for nonunity feedback
Tis E(s) is te error in !apla"e domain and is expression in '5', #e ant to ca!cu!ate
t"e error #a!ue. $n time domain. correspondin% error i!! be e(t). &o steady state of t"e
system is t"at state "ic" remains as t -t ''(
)im
*teady state error, ess = e(t)
t-too
&o e can re!ate t"is in )ap!ace domain by usin% fina! #a!ue t"eorem "ic" states
t"a+
)im )im
,(t) = 's,(s)
t -'' s -
"ere $(s) = u $(t) t
."erefore,
!im )im
ess = e/t) = 'sEes) "ere E(s) is )$ e(t) $,
t -- *-
*ubstitutin% E(s) from t"e expression deri#ed, e can rite
!im s0(s)
e =
!i* * -to 1+ G(s)H(s)
,or ne%ati#e feedback systems use positi#e sign in denominator "i!e use ne%ati#e
sign in denominator if system uses positi#e feedback.
,rom te abo#e expression it can %e conc!uded t"at st1y state error depends on,
i) 0(s) i.e. reference input, its type an& ma%nitude.
i!) G(s)H(s) i.e. open !oop transfer function.
iii) 2ominant non!inearities present if any.
&o e 'ill study t"e effect of c"an%e ininput an& product G(s)H(s) on te #a!ue of
steady state error. As transfer function approac" is app!icab!e to on!y !inear systems, t"e
effe& of non!inearities is not discussed.
r
"
Control System Engineering
,.
78
Time RasponseAnalysis of
Control Systems
7.6 Effect of Input (Type and Magnitude)on Steady State Error
(StaticError Coefficient Metod)
Consider a system a!ing open loop T.F. G(s)H(s) and excited by,
a) Reference Input Is step of magnitude A :
A
t)
A
R(s)

"
ess !i"#!i$
s
p
A
)
lim s R(s)
Cst
..
s%#&
#$ G(s)H(s)
%esired
&im
output
SoA/s

s %# ' #$ G(s)H(s)
'
Time
&im
A

#$ G(s)H(s)
Fig. 7.7
s%#&
A
ess

..
(im
)$ 'G(s)H(s)
s%#
&im
For a system selected, ' G(s)H(s) is constant and called *ositional Error
s%#
Coe++icient o+ t,e system denoted as -.
(im
.p ( s %# ' G(s)H(s) (*ositional error coe++icient
/nd corresponding error is,
I e.- I
0o 1,ene2er step inp3t is selected as a re+erence inp3t, positional error coe++icient -
)ill control t,e error in te system along 1it, t,e magnit3de o+ t,e inp3t applied.
bJ Re+erem4e Input ,. tamp of magnitude "A' :
(im s./5s6

7%# ' )$G(s)H(s)


Actual
output
R(s) /576
(im sR(s)
s %# ' )$G8s)H(s)
Fig. 7.8
r
*
Control System Engineering 7-9 Time ResponseAnalysis of
Control Systems
Lim
A
=
s [1 +G(s)H(s)]
5--+0
Lim
A
=
s +s C(S)H(s)
5-)0
A
e~~=
.,
Lim
o sC(s)H(s)
s --)
Lim
For a selects.. ed system s G(s)H(s) i" constant an calle !elocity "rror Coe##icient
5-)0
as K ...
Lim
K,. = ) s G(s)H(s) = !elocityerror coe##icient
s --+\
An corres$onin% error is&
~
~
So '(ene)er ram$ in$*t is selecte as a re#erence in$*t& )elocity error coe##icient +&
'ill control t(e error in t(e system alon%'it( t(e ma%nit*e o# in$*t a$$lie.
! Reference input is parabolic of magnitude A :
Le, ,(t) = ~t-
,(s)
A
=
s.
c/l)
Lim s,(s)
ess
=
s~0 1 1G(s)H(s)
Lim
soA2s.
=
s -) 3 1 1 G(s)H(s)
Lim
A
=
4-)3
S- [1 1 G(s)H(s)]
Lim
A
=
s-10
S- +S- G(s)H(s)
Lim
A
ellS =
s-)0
S" G(s)H/s)
At#al
o#tp#t
e
- A
44 - $%
Fi%. 7.9
r
&
Control System Engineering 7 -10 Time Response Analysis of
Control Systems
Lim .,
So for a selected system 0 s- G(s)H(s) is constant and called Acceleration Error
s~
Coefficient as KI
~ - - - - - - - - - - - - - - - - - - - - - - - - - - - - - - - - - - - - - - - .
.
..
K =
a
Lim
O
S2 G(s)H(s) Acceleration error coefficient
s- t
And corres!ondin" error is,
So #$ene%er !ara&olic in!'t is selected as a reference in!'t, acceleration error
coefficient K" will control t$e error in t$e system alon" #it$ ma"nit'de of in!'t a!!lied.
So static error coefficients are "i%en in (a&le 7.2.
Static Error Coefficient Corresponding S.S. error
K! s lim G(s)H)s)
A
s....0
!" Kp
K" #rn s$%s&'%s&
A
*++0
K"
Ka ( Lim ,2 G(,)H(,)
A
..... 0
Ka
)ote * +A+ denotes t,e magnit-de of t,e inp-t applied
Ta.le /.0
/./ Effect of C,ange in $%s&'%s& on Steady State Error
%T12Eof a System&
T,is ~ .e st'died .y foc'sin" on to t$e dominant elements of G(s)H(s) from error
!oint of %ie#. S'c$ elements are constant of system +K+ and !oles of G(s)H(s) at ori"in if
G(s)H(s) is e-!ressed in a !artic'lar form called time constant form. ($is is as s$o#n
&elo#,
G(s)H(s)
K%!" Tls) %!" T2s)....
si (1" (asH1" T.s&
K .es'ltant system gain and 3 ((/0E of t,e system #$ere
"+T12E+ of t,e system means n'm&er of !oles at ori"in of o!en loo! (.1. G(s)H(s) of
t$e system.
r
"
Control System Engineering 7-11 Time Response Analysis of
Control Systems

So j = 0, 1'l'PEzero system
j = 1 I TYPE one system
j = 2 I TYPE two system

j = n , ITPE "n' system


Key Point: Th!IS 'TYPE' is the propertv ( if opel' loop " r . F . G( sj/-l( s) tt'/,i/e " Or der ' is the
. l;(s)
property of closed loop T.F. i C!s"#!s""
~ - - - - - - - - - - - - - - - - - - - - - - - - - - - - - - - - - - - - - - - - - - - - - - - - - - - - - - - - - - - - - - , - - - - ~
This is e!"#se, "s $e%ine$ e"rlier, or$er is the hi&hest 'o(er o% s 'resent in the
$enomin"tor 'olynomi"l o% !lose$ loo' T.). o% the system.
$%& Analysis of TYPE ', 1 an( 2 Systems
*ote: + 'o'#l"r metho$ to "ssess ste"$)' st"te 'er%orm"n!e o% ser,ome!h"nisms or
#nity %ee$"!- systems is to %in$ their error !o-e%%i!ients ./0 .1. "n$ .2
... , = Position error !onst"nt,
.1 = 3elo!ity error !onst"nt "n$
., = +!!eler"tion error !onst"nt.
4,io#sly in or$er to %in$ these error !onst"nts the system m#st e st"le, e!"#se %or
"n #nst"le system there is no ste"$y st"te "n$ .'l ., "n$ ., "re #n$e%ine$.
5en!e e%ore (e 'ro!ee$ to %in$ .', .1. "n$ .1 (e m#st ens#re (either )y 'ole
lo!"tion or )y 6o#th t"le o% the !lose$ loo' system" th"t it is st"le.
Th#s the !on!e't o% .', ., "n$ ., is "''li!"le only i%,
i) System is re'resente$ in its sim'le %orm.
ii) 4nly i% the system is st"le.
7onsi$er t*e in'#t sele!te$ "s ste' o% m"&nit#$e 'A'%
i) +et ,s ass,me t*at t*e system is of TYPE "''%
. .(l - . s" (1- T2s" %
/e 8(s)5(s) =
(1- T"s) !1- T)s" %%0%%%%
9im
1
2!s"#!s" = K
:;
)or ste' in'#t ... #sin& "o,e 8(s)5(s)
A A
0
00 e/ =
33=33
l<.' 1<1=
0 /% 0 /
>11 1 " r ,
Control System Engineering 7-12 Time Response Analysis of
Control Systems
i.e. TYPE "0' systems follow the step type of input with finite error 1~ K which can e
re!uce! by chan"e in 'A' or 'KI or oth as per re#uirement.
$ow 'K' can e increase! by intro!ucin" a %ariale "ain amplifier in the forwar! path
an! error can e re!uce!. &ut there is limitation on the increase in %alue of 'K' form
staility point of %iew which is !iscusse! later. &ut increase in 'K' is one way to re!uce
the error. 'orrespon!in" response will e as shown in the (i". 7.10 )a* or )*.
+
A + - - - - - - - - - - - ~ - - -

,
ess- 1+K
(finite) (finite)
o
Time
o
Time
(a)
(i". 7.10
(b)
ii* . f for the same input no T!"E is in#rease$to one' by a$$ing pole at origin
% In &(s)'(s)%
TYPE 1/ &(s)'(s)
K)l + Iis* (1+ T2s* .
(
-
s)1+T0 s* (1+ Tbs) %
,s input is step0 Kp )
1inl
*
&(s)'(s) - 00
2~
+
++
iii* 2imilarly . f no TYPE is further in#rease$ to 'to' i%e% &(s)'(s) with 2 poles
at ori"in0
T!"E+, &(s)'(s)
,s input is step0 Kp -
1im
3
4)s*5)s* )00
2~
e26 -
L\ ) - ) .
1+ K 00
p
I I
781 8 "r ,
Control System Engineering
'[J
7 -13 Time Response Analysis of
Control Systems
In general, for any TYPE of system mOTC than zero, Kp !ll "e !nf!n!te # $ $ % an& error
!ll "e zero' Tho(gh mathemat!)ally anser for error !s zero, pra)t!)ally small error !ll
"e present "(t it !ll "e negl!g!"ly small' *()h type of responses rna+ ta,e one of the
forms shon !n the -!g' 7'11 # a% an& # "% '
c(t) CCl%
Time
(a)
Fig. 7'11
(b)
Th(s TYPE 1 an& a"o.e systems follo a step type of referen)e !np(t of any
magn!t(&e, s())essf(lly, !th negl!g!"ly small error'
/et (s no )hange the sele)te& !np(t from step to ramp of magn!t(&e ' t\' so K,' !ll
)ontrol the error'
iv) Let the system be of T!E ".
K# 1# $%s% # l # T&s) ....
0# s% 1# s% 2
($ t T.. s3($# T''(s) .
Lim
K,' 2 s 4 $ s 0# s% 1# s% 2 $
!'e' TYPE $ systems !ll not follo ramp !np(t of any magn!t(&e an& !ll g!.e large
error !n the o(tp(t h!)h may &amage the parameters of system or may )a(se the
sat(rat!on in parameters' 1en)e ramp !np(t sho(l& not "e appl!e& to TYPE '$ ' systems'
The o(tp(t may ta,e the form as shon in the -!g' 7'15 # a% an& # "% '
c(t) c(t)
Time Time
(a)
-!g' 7'15
(b)
'61 , , : r ' , ale')i*$
11/".
Control System Engineering 7-14 Time Response Analysis of
Control Systems
v) If TYPE 1 System is subjected to Ramp input ten!
G( )
K(I+1js)(I+T2s) .
s)H(s =
s "1#Ta 5) "1#TbS) .....
$im
o
s G(s)lI(s) = l"
S%
l" &
v
:. e~~
A A fini
&' = - te
Ky K
i.e. TYP 1 syste!s "ollo# t$e %a!& ty&e o" i'&(t o" !a)'it(*e (A( #it$ "i'ite e%%o%
+,K #$i-$ -a' be %e*(-e* as *is-(sse* ea%lie%. T$e o(t&(t !ay ta.e t$e "o%! as s$o#'
i' t$e /i). 7.10 (a) a'* (b).
1 1 t) - t)
Time Ti!e
"a)
/i). 7.10
(b)
vi) If TYPE) system Is e*cited by Ramp Input ten!
i.e.
K =
v
$im
o
s G(s)H(s) = 22
s%
A A
e3 S = ~ = -4 4 = 2
Tis is true "o% a'y syste! o" TYP!o%e t$a' o'e. He'-e all syste!s o" TYP 2 a'*
!o%e t$a' t#o "ollo# %a!& ty&e o" i'&(t #it$ 'e)li)ible s!all e%%o% a'* !ay ta.e t$e
"o%! as s$o#' int$e /i). 7.14 (a) and (b).
- t) c"t)
Time
"a)
/i). 7.14
(b)
r
+
Control System Engineering 7 -15 Time Response Analysis of
Control Systems
Let us now change the selected input from ramp to parabolic input of magnitude A
hence coefficient K.l will control the error.
vii) Consider TYPE 0 system :
G )
K(l+ rls) (1+Ts) .
(s !(s) "
(1 T.. s) (1 Tb#) .
Lim .$
!" # s% $(s%&(s% " " '
s&o
A A
.. c #(#(#''
s$ Ka 0
'iii) Consider TYPE 1system :
! (1 T)S) (1 T*s) .
$(s%&(s) "
s(l Ta s) (1T+s) ,
!- =
Lim .$
o %( $.s)&(s) " '
S~
A
:, Css " !" ''
/ 0
)or both TYPE 1'1 and 111 s*stems$ error will be 'er* large and uncontrollable +f
parabolic input is used. !ence parabolic input should not be used as a reference to e,cite
T-./ 000 and T-./ 010 s*stems. The output ma* ta1e the form as shown in the
)ig. 2.13 (a) and (b) if e,cited b* such input.
2(t) 2(t)
Time Time
(a)
3ig, 7,15
(b)
i4) )f TYPE * system is 5sed i,e,
G(s)!(s) " K (1 Tls) (1T*s) .
s*(1 T%s) (1T+s) ,
Lim
K$ " s* G(s)!(s) # K
s~O
Then
A A fini
.. c&$" " ! # ! te
a
!ence T-./ * s*stems will follow .arabolic input with finite error A6! which can be
controlled b* change in A or K or both and output ma* ta1e form as shown in the
)ig.2.14 (a) and (b).
r
7
Control System Engineering 7 -16 Time Response Analysis of
Control Systems
c(t) c(t)
e
- A
SS- K
(finite)
e
- A
SS- K
(finite)
Time Time
(a)
Fig. 7.16
(b)
And for any system of TYPE3 or more if parabolic input is used, error will be
negligiblysmall. Theresults can besummarisedas shown in theTable7.3.
Typeof Error Coefficients Erroresa for
System
Kp K Ka
Step inp!t Ramp inp!t "arabolicinp!t
K #
A !" !"
1 $ K
#
%&
K
A
-
K
$
##
K #
A
K
Table 7.3
7.' %isa()antages of Static Error Coefficient *et+o(
T+e (isa()antages of Static error Coefficient *et+o( are ,
#% &ethod cannot gi'e the error if inputs are other than the three standard test
inputs.
$% &ost of the times, method gi'es mathematical answer of the error as (( or
(infinite( and hencedoes not pro'ideprecise'alueof theerror.
3% &ethod does not pro'ide 'ariation of error with respect to time, which -ill be
otherwise'ery useful fromdesignpoint of 'iew.
R(s)
---$.
$ C.s)
Fig. 7.17
r
/
Control System Engineering 7 -17 Time Response Analysis of
Control Systems
4) Error constants are defined for the loop transfer function G(s)H(s), strictly, hence the
method is applicable to only the system configuration shown in the Fig. 7.17.
) !s final "alue theorem is used to calculate steady state error #$ before applying it
is necessary to chec% if sEes) has any poles on the &ro a'is or in the right half of the
s(plane. )his means before applying this method, the system must be chec%ed for
its stability. )he method can not be applied to unstable systems.
6) *hen the system configuration is different than as shown abo"e then it is
necessary to establish the e'pression for the error signal and apply the final "alue
theorem directly, without the use of error coefficients.
7) )he method isapplicable only to stable systems.
+ecause of these disad"antages ,ynamic error coefficient method (Error series method)
is de"eloped which eliminates abo"e said disad"antages.
7.10 Generalised Error Coefficient Method (or ynamic Error
Coefficients!
!s e'plained earlier,
R(s)
E(s) - 1+ G(s)H(s)
.et us assume that this is the product of two polynomials of/s/.
E(s) = F,(s)0F2(s)
1
12((3(- (1(1((.(, F4 5s6.- 7(s)
1+ 8(s)H(s)
*here
F9(s) -
:ow if, F(s) = F9(s)0 F4 (s) then using con"olution integral,
t
.(99F(sH - F(t) =IF9()) l3(t( )) dr
(I
#imilarly
I I
e(t) - IFI(T} F2(t(t;dT= <f&(t) R(t-T) dr
(< =
7(t ( or)can be e'panded by using )aylor series form as,
4 >
7(t ( or) - 7(t) ( or7/(t) + 3? 7@(t) ( 2?RIII(t) + .....
#ubstituting
9 5 4 > 6
e8t) - ?11 (T) R(t)-TR'(t)+ 3? R"(t)- 2? R"'(t)+ ....... <t
r
"
Control System Engineering 7-18 Time Response Analysis of
Control Systems
I I
= fR(t)F1(T)d'C - J TR'{t) F({T)d'C+ .
o 0
Nmv
ess =
Lim
e(t)
t-+co
.. 00 GO .,
= R(t) JFt(T) dT-R'(t) fTl)(T) dT+R"(t)J ~~ F('C) dr .
000
Were
-
!l = - J T F" (T) dt#
o
...
!$ = J 'C% F.(T) de .....
o
&'(stit'ti)* tese v+l'es ,e +ve-
es~ = !o R(t)+ !l R'(l)+ ~~ R"(t) + .
,ere K o -!1 ' !% +re c+)ed d.)+mic error coefficie)ts.
To c+lc'l+te tese coefficie)ts 'se te follo,i)* metod/
0ccordi)* to defi)itio) of L+1l+ce tr+)sform-
..
F(2) = f F" (T) e-"''t dt
o
No,
3'lti1l.i)* (. c - &'t to (ot sides-
eo
!o c-) = JF1(T) dr e-~"'f= F(s)
o
T+4i)* limit +s s -+ 5 of (ot sides-
Lim !o e - &'t = Lim F((2)
s -to 6 -to
!o = Lim F" (s)
5_0
,ere
1
Fl (2) = """"1 +--= C-/-(s//-/"")7= (s"'""'")
T+4i)* deriv+tive of !o e - 5t ,.r.t. '&f ,e *et-
r
"
C;:ontrolSystem Engineering 7-19 Time Response Analysis of
Control Systems
Substituting
-
- 't J F i (t) dr e- s'(
o
i.e,
Taking limit as s -+0 of both sides,
~ = lim dF 1_(s)
-to ds
!n general,
This method eliminates the disad"antages of stati# error #oeffi#ient method.
$d"antages of this method is,
i) !t gi"es "ariation of error as a fun#tion of time and
ii) F or an% in&ut, other than standard in&ut error #an be determined.
. ~~+V
' _ Example 7! : A unrty feedback system has "(s# ( (_ !# (- $#'
s)*+ )*+
Determine (i)Type of the system, (ii) All error coefficients and
(iii) Error for ramp input with magnitude 4.
Sol%tion: To determine type of s%stem arrange ,(s)-(s) in time #onstant form.
$&(s +'# $&('# (!+0...)
,/s)-(s) ( s(s +!# (s +0) ( s(! +s# ($# (! +&'(s#
10(1 +2..s)
(
s(1 + s) (1 +0.1. .)
3om&aring t)is 4ith standard form,
,(
5(l +lis) (1+T6s) .
s)-(s) (
si (!+Ts# (!+T*s# ++++
4here = T%&e of s%stem
( 1so gi"en s%stem is t%&e 1s%stem.
r
,
Control System Engineering Time ResponseAnalysis of
Control Systems
7-23
f
es.(t) = KoR(t)+ K. R'(t) + ~~ R"(t)
R(t) = 6 + 5t + 6; 2 I R'(t) = 5 + 6t I R"(t) = 6
7.346x 10-3
es,(t) = 0.1428[6+5t+312)+0.0857{5+6t1+ '" x6
.....
= 0.8568+ 0.714t + 0.4284t2 + 0.4285....0.5142t + 0.02203
e"(t) = 0.4284 t2 +1.2282I + 1.3073
!."" Transient Response Analysis
Transent res!"nse s t#e !art "$ t#e t"ta% "&t!&t '#(# (an n"t )e se!arate* "&t an*
#en(e t" ana%+se t, t s ne(essar+t" (a%(&%ate t"ta% "&t!&t ((t) $"r t#e a!!%e* n!&t. T#s
("ntans stea*+ state ,a%&e a%"n- 't# transent res!"nse. .r"/ t#e ex!ress"n "$ (0%) "n%+,
t#e transent res!"nse 0t(%) (an )e ana%+se*.
!.""." $et#o% to &etennine Total '(tp(t )(t)
. . C(s)
1. 1ete/&ne t#e (%"se* %""! trans$er $&n(t"n "$ t#e s+ste/ R(s)'
0(s)
R(s)
C(s)
=
"2 C(s)*(s)
2. .n* t#e ex!ress"n $"r R(s) $r"/ t#e n$"r/at"n "$ re$eren(e n!&t ret) t" )e
a!!%e* t" t#e s+ste/.
3. 3&)stt&te R(s) n t#e (%"se* %""! T... t" ")tan ex!ress"n $"r 0(s).
4. Ta4e 5a!%a(e n,erse "$ 0(s) +y &sn- !arta% $ra(t"n /et#"* t" ")tan t"ta%
((t) "$ t#e s+ste/ t" t#e a!!%e* n!&t ret).
As
T#e transent "&t!&t (an )e st&*e* $r"/ t#e a)",e ex!ress"n. Transent res!"nse
/a+ )e ex!"nenta% "r "s(%%at"r+ innat&re.
5/
Key ,oint- TRking o f c(t), the final steady state value Css o f the output also can b e
t -+00
obtained.
r
"
Control System Engineering ,"
J !...
7-24 Time Response Analysis of
Control Systems
7.12 Analysis of First Order System
Order: Order of system is the highest power of 's' in the denominator of a closed loop
transfer Junction.
Consider a simple system shown in the
Fig. 7.18 (a).
!"t# = 1 t 2! "
= " t $ "
Find #oCt) i.e. r$spon$%e if it is e&$ited %y
'nit step inp't.
&! "s# ' 1()
Fig. 7.18 "a#
*ow first $al$'late system +.F. +he ,apla$e
networ- is shown in the Fig. 7.18 (.).
R
/ies)
1
=
1()) 0 ( sCres)
..."1#
&o"s#
1
= s)*"s#
... (2)
&o"s# 1 1
&!"S#
=
1( sRC ' 1( +s
..
Fig. 7.18 (.)
where + ' 0C
7.12.1+nit Step Response of First Order System
,et inp't applied /i "t# is 'nit step #oltage.
1'.stit'ting /2(1) = 1() in the transfer f'n$tion
1 A' 5'
= = - ( .....---= -= .
s"1(sRC# 1 1( sRC,
3' ' 1 and 4' = - 0C
1 1
=
%- )5(1 ( RC#
r
"
'., . . '''-
Control System Engineering 7-25 Time ResponseAnalysis of~;(
Control Systems
Taking Laplace inverse,
v oCt) = 1 - e -lIRe ~ Css +ci (t) form
c, = 1an c1(t) = e-t!RC I "o
#o$ transient term is totall% epenent on t&e val'es of R an C an its rate of
e(ponential eca% $ill get controlle )% '1IRC' $&ic& is pole of t&e s%stem*
+e% ,oint- The values of R and C will not affect the steady state part.
T&e response $ill )e as s&o$n in t&e .ig* 7*1/ 0c)*
t !"t)
1 1
RC 1*232
2RC 1*/21
3RC 1*451
5RC 1*4/2
-
1
.ig* 7*1/ (c) 6nit step response of first
or#er system
T&e response is p'rel% e(ponential*
#o$ s'ppose inp't is c&ange to step of 'A' 'nits*
T&en 7ies)
A
=
5
7o0s)
A A' /'
..
=
= +
5(1+sRC) s 1+ sRC
.. A $ A '01 +sRC) +s/'
.. A'RC+ %' $ 1 an A'$ A
%
.
ARC ..
=
&.,(s)
A ARC
=
8-
..
1+sRC s
A A
=
s9:l IRC) s
** vo0t)
=
A ;l8e-t!RC<
r
'
Control System Engineering
:~~'Io. f.,.? 41'';~ '.=
726 Time ResponseAnalysis of
Control Systems
So rate of decay is not changed but the
steady state value has changed. he
corres!onding res!onse can be sho"n as in the
#ig. 7.$%.
&ut if values of 'and C are changed i.e.
location of !ole s ( ) l/RC is changed, the
transient out!ut "ill behave in a diffurent "ay as
rate of decay will get affected "ithout change in
its steady state.
he transient tenn is vanishing as it contains e*!onential ter+ of negative inde*
"hich is only because !ole of the syste+ is having real negative !art.
#ig. 7.$%
Key ,oint: In general transient response depends on,
i) Poles of the closed loop T.F.
ii) Location o f the poles of the closed loop T.F. and it is independent of the agnit!de of t"re
inp!t applied. #n$ change in the agnit!de o J the selected inp!t %ill not ha&e an$ effect
on the transient response o f the s$ste.
7. 12.2Closed Loop Poles of First Order System
he closed loo! transfer function of a syste+ is given by,
C-s. -/-s.
Rs! ( 1"#s!$s!
he e0uation "hich gives !oles of syste+ is called characteristic e0uation "hich is,
% $& #s!$s! = ' %
#or first order syste+ this e0uation is also first order in general of the for+,
% $ & s ( 1 %
2s closed loo! !oles are the roots of characteristic e0uation, so for first order syste+
there isonly one dosed loo! !ole i.e.
% s ( ) i I+ag.33y
he !ole) 4ero !lot is as sho"n inthe #ig. 7.21.
-1f T
Real
#ig. 7.21
r
(
Control System Engineering 7 -27 Time Response Analysis of
Control Systems
Key Point: The value of closed loop pole appears as an exponential index in the transient
output.
e.g, in the system considered above the closed loop pole is s = - it and in the output
c(t) = 1 - e-I/I{{', the exponential index is - d e which is nothing but the dosed loop pole.
onsider two systems, one with closed loop pole at s, = - !while other having closed
loop pole at "! = - #. $n the output o% two systems we will get the terms (,- !& in %irst
system and e- #1 in second system. 'ow as t -+ ( ( ,) *,- $$ wiu vanish earlier than e- !1. +ence
second system transients will vanish ,uic-ly and will stabilise to its %inal value earlier than
the %irst system.
.rom this we can conclude that locations o% closed loop poles a%%ect the transient
behaviour.
Key Point: /s closed loop pole moves in tire left half anoayfrom the imaginary axis m tile
s-plane, transients die out more quickly making system more stable.
). Eample 7.!: FaT a first order system, find out the output of the system wi0en the input
applied to lire system is unit ramp input. Sketclr ti,e rtt) and dt) and show the steady state
error.
Sol"tion: 1et the %irst order system has trans%er %unction
(s) #
=
2(s) (s34)
r(t) = $nput
R$s) =
52
unit ramp input = t
%
1\ & C
- :- - - = 53- 3- - 4
s!(s31) s- s s3
. . A (6 + 4) + &s $s + T) + C 6! = %
'' $& + ) s2 + $A + &T) s + AT = %
AT = %
(s) =
%
A=
4
A+&T = ( &=
&+C = (
1
C=)
4!
C$s)
(T! 74! 74!
= -----+--
6! " "+ r
r
*
Control System Engineering 7-28 Time Response Analysis of
Control Systems
Taking Laplace inverse,
.. c(t) = (~ } - T~ + T~ e-rt
The sketch of c(t) and ret) is shown elow. Take T =!for plotting c(t).
c(t), r(t)
" " l) forT=!
7.13 Analysis of Second rder System
#ver$ practical s$ste% takes finite time to reach to its stead$ state and d&ring this
period, it oscillates or increases e'ponentiall$. The ehavio&r of s$ste% gets decided $
t$pe of closed loop poles and locations of closed loop poles in s-plane, The closed loop
poles arc dependent on selection of the para%eters of the s$ste%. #ver$ s$ste% has a
tendenc$ to oppose the oscillator$ ehavio&r of the s$ste% which is called da%ping. (ow
this tendenc$ controls the t$pe of dosed loop poles and hence the nat&re of the response.
This da%ping is %eas&red $ a factor or a ratio called da%ping ratio of the s$ste%.
This factor e'plains &s, how %&ch do%inant the opposition fro% the s$ste% is to the
oscillations in the o&tp&t. !n so%e s$ste%s it will e low in which case s$ste% will
oscillate &t slowl$ i.e. with da%ped fre)&enc$. !n so%e s$ste%s it %a$ e so high that
s$ste% o&tp&t will not oscillate at all and not onl$ that it will e e'ponential, so slow that
it will take ver$ long ti%e to reach the stead$ state. T"is da%ping ratio, e'plaining s&ch
ehavio&r is denoted $ a greek s$%ol (*eta) l#. (ow as this %eas&res the opposition $
the s$ste% to the oscillator$ ehavio&r, if it is %ade +ero, (t# = $) s$ste% will oscillate with
%a'i%&% fre)&enc$. ,s there is no opposition fro% s$ste%, s$ste% nat&rall$ and freel$
oscillates &nder s&ch condition. -ence this fre)&enc$ of oscillations &nder ~ =$ condition
is called nat&ral fre)&enc$ of oscillations of the s$ste% and denoted $ the s$%ol
(l)n rad . sec .
/or a second order s$ste% the deno%inator of closed loop T./. is )&adratic and the
coefficients of this e)&ation are directl$ related to ~ and (l)n as e'plained elow.
r
%
Control System Engineering 7-29 Time Response Analysip.
Control Sy9l..
The CL.T.F. (closed loop transfer function) for a standard second order system ~.Ikcs
the form as,
C{s)
=
R(s)
(fJ2
n
... standard 2nd order system
Where characteristic euation is, s2 + 2! (")n s + (")# $
The standard second order system is that %here in C.L.T.F. numerator is (")#.
!ey "oint# In practice it is 1I0t necessary tha! numerator must be alllJay.t;(J)#. It may be
other constant or polynomial of &'& but denominator middle term coefficient and last ten"
coefficient always reflect &2!(")n& and &(J)#& of the system respectively.
(ence al%ays denominator of a ).*. must +e compared %ith the standard form
,2 +2$% (")n s + (fJ# & to decide the -alues of! and (l)n of the system. The numerator
should not 'e used for comparison to o+tain the -alues of t% and (")n&
e... /
(& C(s)
()+()()+2)
*i.. 7.2"
(&
C{'0 1(s0 (s+ l)(s + 2) "$
..
R(s)
= = =
"...C(s)((s)
"+ "$
'2 2 *s 2 (2
(s2 () (s +, 2)
This CL.T.F. is not standard as numerator term is not ())# +ut denominator al%ays
reflects ! and (J)n& The -alues can +e decided 'y comparin. the denominator %ith the
standard characteristic euation ,2 + 2! (l)n s +())# o.
..
(")2
"2 i.e. ())n = -.2 rad 3 ,(.&C
n
While
2#(")n 4 ..
~

J u = $.544
2 "2
/.(+ E..ect o. ,on Secon0 1r0er System "er.ormance
Consider input applied to the standard second order system is 2nit step.
6(s) = lis
While
C(s)
=
R(s)
r
3
Control System Engineering 7 -30 Time Response Analysis of
Control Systems
(Il
C(s) = n
S(S2 +2~(On s+ro~)
Finding the TOOts of the equation S2 + 2; ron s + (O~= 0
- 2~ ron .J4l;2 ro! - 4ro~
ie! 2
ie!
"e #an $rite! C(s) =
(02
n
%o$ nature of these roots is de&endent on da'&ing ratio l; Consider the fo((o$ing
#ases!
! #ase)*)+(i+-)
The roots are!
ie rea(! unequa( and negati,e! sa- - .( and - .2
2
C(s) = (On = / + _B_+ "s "
S(0 + #)) (s + #2) S of #$ S + #~
Ta1ing 2a&(a#e in,erse! #C()$i(( ta1e the fo((o$ing for'!
#C() = Cs!+3e-.4t+Ce-.25!
$here C67 = Stead- state out&ut = /
The out&ut is &ure(- e8&onentia( This 'eans da'&ing is SO high that there are no
os#i((ations in the out&ut and is &ure(- e8&onentia( 9en#e su#h s-ste's are #a((ed
:O,erda'&ed6
9en#e nature of res&onse $i(( ;e as sho$n in the Fig 722
C(5)
O%er&ampe&
Criti'ally &ampe&
o
(ig 722 l;~)
r
*
Control Sysmm Engineering 7-35 Time Response AnalYSIs of
Control Systems
The partial fraction can be calculated for the Laplace inverse as below,
C(s) =
a1 a2s+a3
-+--=---=--_..:;:__-:-
S S
2
+ 2; (J)ns + (J)~
a(s2 + 2;(J)n !"+(J)~) + s(a2 s+ a #)
=
$($2 + 2; (J)nS+ ro~)
ro2
n
%% S(S2 + 2; ron s + (J)~)
e&uatin' nu(erators on both sides,
ro~ = s2(a +#2)+s)a 2;ron +a#)+alro~
e&uatin' constant
e&uatin' coefficients of 52
e&uatin' coefficients of s
*% a = , a 2 = + , a # = + 2l;ron
As assu(ed earlier for ease of co(putations*
at = , a 2 = - , a 3 = - 2a
1 + s+ 2a
C(s) = + + -::-----::-
S S2 + 2a . . c ; + (1)~
C(s) = ~+ )s2 +s~~rod
,ow consider s
2
+ 2a s+ (1)2
n'
**** (Ta-in' ne'ative si'n outside*)
.or co(pletin' s&uare,
((iddle term):! /a 2 ~
Last ter( = = 01 1 =a-
/ x first term "t
So ad2ustin' deno(inator as s
2
+ 2a*s + a2 + ro~+ a2
i*e* deno(inator = [s + a)2 + (J)~+ ex:!
but
Substitutin' in above we 'et, (s +a)2 + ro~3;2 ro;
i*c* deno(inator = (s + a) 2 + (J)~ (3;2)
,ow (J)d = (J)nJt 3;2
Substitutin' this in the e1pression of C(s) we 'et,
C(s) = ~+ )(s+4~;4rod
r
"
~~!'trol~.tem Engineering
7-37 Time ResponseAnalysis of
Control Systems
{
H)
e = tan-I ~ radians
Key Point : Substituting l; = 0 in this expression, we can find out expressionfor the output
for undamped systems.
Important Remarks
1) The result derived is applicable for standard second order systems which is
underdamped and ecited by unit step input.
!) Ifin the problem" the input is #iven as step of $ units rather than unit step then
each term of the final epression of c%t) #ets multiplied by $ as"
o c%t) = $ & 1 ~"'n%''dt(e))
... *or step of $ units
3) If the system is not in the standard fonn i.e. numerator of closed loop transfer
function is not %+ )~but some other constant as"
<:(s !(
R(s = ,! ( !~%+ -ns(%+ )!
K = constant
Then the above result can be applied by epressin# .s)/0%s) as"
$nd in such case" the standard epression of c%t) #ets multiplied by the constant
1/ %2)!.
c%t) = ~& 1- e~,in3%+ -dt(4))
%+ -n ""1-~-
The values of ~ and (ln must be obtained by comparin# denominator of 5%s)/0%s)
witll the standard form.
-6) Ifnumerator of 5%s) / 0%s) has some polynomial in s as"
5%s) 7%s). 5%s) !s(6
-- " 2.e." -- = -8 8 ----
0%s) s!(!~%2)s(%2)! R(s s!(19:(;6
#:t n n
Then the above result cannot be applied to #et c%t). Insuch case" thou#h l; and (ln
values are to be obtained by comparin# denominator with the standard fonn" the
epression for c%t) must be obtained by actually calculatin# partial fraction
coefficients"after substitutin# the proper value of 0%s).
r
$
Control System Engineering 7-38 Time Response Analysis of
Control Systems
5) For any other input, other than step the derivation is not applicable but the steps
of the derivation can be used as a guide line to obtain expression for c(t) in any
other condition.
7.16 Transient Response Specifications
The actual output behaviour according to the expression derived can be shon as in
the Fig. 7.!".
c t)
Peakovershoot Mp
Toleranceband.t !#
$ %nsteady stale, output
&''#-li----f-i(-)-*-*-*)*+*,-*-*-*-*--. -* .... re.ainsithin(.!#error
!!i.!!!t band
"ig. 7.#6 Transient response specifications
/et us define the various ti.e response specificationsreferring to the Fig. 7.!".
1) Delay Time Ttl : %t is the time requiredfor the response to reach 50% of the /tnal
value in the first attempt. $t is given by,
$T% & 1' $
2) ise Time T, 0 $t is the time requiredfor the response to rise from &''1 to !0% of
the final "#due for overdamped systems and 0 to 100% of the final valm for
underdamped systems. The rise time is reci. $$Dcalof the slope of the response at
the instlmt% the response is equal to 5'# of the final value. #t is given by,
r
(
Control System Engineering 7 -39 ~ Time ResponseAnalysis of
Control Systems
1t -0
sec where 0must be in radians.
3) Peak Time Tp : It is the time requiredlor the response to reach its peak value. It
is also defined as the time at which response undergoes the first overshoot which
is always peak overshoot.
T =
p
1t 1t
- ro- d = ron ~1_~2 sec
4) Peak Oversl,oot M" : t is t,e largest error !etween reference input and output
during t,e transient period.
It also can be defined as the amount b which output o!ershoots its reference
stead state !alue durin" the first o!ershoot.
Mp = # c$t% & t c 1p' - 1 1 is for unit step input
") "ettling Time (s : This is defined as the time requir. ..d lor the response to decrease
and stay within specified percentage of its final value #within tolerance !and).
(ime constant of sstem
(s = ) * (ime constant
+racticall the settin" time is assumed to be ) times, the time constant of the sstem.
..... for a tolerance band of - ./ of stead state
Key Point: 1 Time constant 7' is the time required!y tire sstem output to reach!"2 #
of its final value during the first attempt.
7"17 $eri%ations of Time $omain Spe&ifi&ations
7.17.1 0eri!ation of +ea1 (ime (p
(ransient response of second order underdamped sstem is "i!en b,
-~w I
e n
&'t( = 1- r.2--2:r sin 'rod t )0%
,,1_2.
r
*
: Control System Engineering 740 Time Response ~nalysisof
Control Systems
Where
~
e = tan"! "l/l-'l.
~
As at t =Tp' c(t) will achieve its maxima. According to maxima theorem,
So differentiating c(t) w.r.t. "t' we can write,
- ~(I) t
C n (-~ron)sin(rodt+e)
t.e, - r:--:
,,! " ~#
.. sin (rodt + e) - ~! -#cos(rodt + e) = $
tan (rodt + 8)
=
~! " ~#
..
;
%ow e
=
t -1~1-#
an ~
~! " #
=
tane ..
;
,
.. tan(rodt + e)
=
lane
&rom trignometric form'la,
tan(n(t +e) = tan e
where n = I, #, )
*'t T+ , time re,'ired for first +ea- overshoot. .. n = !
rod Tp = (t
r
"
Control SystemEngineering 7-41 Time ResponseAnalysis of
Control Systems
7.17.2 Derivation of Mp
Fromthe Fig. 7.27, Mp = qTp) - 1
tl
Fig. 7.27
{ -to.T,
sin (ol. Tp + O l } -1 Mp
=
l_c
~1_~2
-CilnTp
Mp
e
sin (ro Tp +!)
=
..
~1_~2
"#t
Tp =
~ , s#$stit#ting %$o&e 'e get,
(()
-l!!n Tp
Mp
-e
sin (n +!) ..
=
~1_~2
)o', sin (rt *+)
=
- sin (+)
J 1 - ,,2
-CllnTp
Mp
e
sinO ..
""""
~1_~2
,,
(l -~2. h . the # 72-
,ig. 7.2-
.e /no', t%n +
=
~ %ss o'nm 1$%%%
sinO
oppositesie ~ g 2 2
..
=
h $#t h0poten#se =( 1-~ ) *~ = 1
0poten#se
.. sinO
=
(l -~2
-tllln Tp
~1_~2
Mp
e
=
..
~1_~2
1
=
-(l&n Tp
e
1 2
r
'
,. :j Control Systemengineering
Time ResponseAnalysis of
Control Systems
Substitute T = ~
P rod
-n~
% M = 100 e J l- ~ 2
p
7.17.3 Derivation of T
Ti!e re"uire# b$ output to a%&ieve 100% of its final value starting fro! 'ero #uring
t&e first atte!pt is the rise ti!e.
c(t)
(ig. 7.2 )
i.e. *%+t~l ..T, = 1for unit step input
-I,Clln r
1 = 1- e sin+-d Tr.)/
J 1- ~ 2
0"uation !ill get satisfie#1onl$ if
sin +-d Tr " #) = $
Trigono!etri%all$ t&is is true onl$ if,
!d Tr.2 = n %t -&ere n = , 2 ..3.
r
&
Control System Engineering 7-45 Time Response Analysis of
Control Systems
defining the term sensitivity for the system. Let us study the effect of parameter variations
in an open loop control system.
7.18.1 Effect of Parameter Variations in an Open Loop Control System
Consider an open loop control system shown in the Fig. 7.30. The overall transfer
function of the system is given y!
C"s#
R(s)
.
C(s)
$"%#
!
&
'"s#
"ig. 7.#$
.. C"s#
&
$"s#('"s#
Let )) $"s# e the change in $"s# due to the parameter variations. The corresponding
change in the output e )) C"s#.
C"s# * A. C"s+ % ,$"s# * A. $"s#- '"s#
C"s# * A. C"s# & $"s#('"s# * A. $"s#('"s#
C(s) * A. C"s# & C"s# * .$"s#( '"s#
& A C"s# & A $"s#('"s# ' .! (1)
The e/uation "0#! gives the effect of change in transfer functi01r12due to the parameter
variations! on the system output! in an open loop control system. Let us discuss now! the
effect of such parameter variations in a closed loop control system.
7.18.( Effect of Parameter Variations in a Close) Loop System
Consider a closed loop system as shown in
Fig.7.33. T*e signal 4"s# is the Laplace transform of
the error signal e"t+. The overall transfer function of
the system is given y!
& C"s# 5 $"s#
'"s# - 1* $"s# lies#
C(s)
T(s)
Fig. 7.33
Let .$"s# e the change in $"s# which is due to the parameter variations in the
system. The corresponding change in the output +e .C"s#.
,$"s# ,)) $"s#-
C"s# * A. C"s# & . '"s#
3*, $"s# , .$"s#- lies#
,$"s# *.$"s#-
% 3 * $"s#i-i"s#*)) $"s#6"s+. '"s#
C"s#*. C"s#
r
-
Control System Engineering 7-46 Time Response Analysis of
Control Systems
The term AG(s)H(s) is negligibly small as compared to G(s)H(s), as the change AG(s) is
very small compared to G(s). Neglecting the term A G(s)H(s) rom the denominator, !e get,
G(s"+A G(s) . R(s)
#(s) $ A #(s) % 1$ G(s)H(s)
#(s) $ A #(s) % G(s) &( ) $ A G(s) &( )
'$ G(s)H(s)( s '$ G(s)H(s)( s
AG(s)
#(s)$ A )s) % #(s) $ 1 $ G(s)H(s) . &(s)
AG(s)
A #(s) % 1$ G(s)H(s) . &(s) ... (2)
The e*+ation (,), gives the change in the o+tp+t d+e to the parameter variations in
G(s), in a closed loop system.
Inpractice, the magnit+de o '+G(s)H(s) is very m+ch greater than +nity.
-G(s)H(s)' 1
Hence it can be observed rom the e*+ations (') and (,), that in a closed loop system,
d+e to the eedbac., the change in the o+tp+t, d+e to the parameter variations in G(s), is
red+ced by the actor /'$G(s)H(0)1. In an open loop system, s+ch a red+ction does not
e2ist, d+e to the absence o the eedbac..
7.18.3Sensitiity of a Control System
The parameters o any control system cannot be constant thro+gho+t its entire lie.
There are al!ays changes in the parameters d+e to environmetal changes and other
dist+rbances. These changes are called parameter variations. 0+ch parameter variations
aect the system perormance adversely. 0+ch an eect, in the system perormance d+e to
parameter variations can be st+died mathematically deining the term sensitivity o a
control system. The change in a partic+3ar variable d+e to the parameter variations can be
e2pressed in terms o sensitivity as belo! 4
5et the variable in a system !hich is varying be 6T, d+e to the variations in the
parameter 676 o the system. The sensitivity o the system parameter T to the parameter 7
is e2pressed as,
0 % 8 '9 change in T
! " ! change in 7
... (3)
:athematically, it can be e2pressed as
0T ; dln(T)
3( - din (#)
r
$
Control System Engineering 7-47 Time Response Analysis of
Control Systems
(a T/1)
(aK/K)
... (4)
The symbolic representation s~represents the sensitivity of a variable T with respect
to the variations in the parameter K. In practice, the variable T may be an otpt variable
while the parameter K may be the !ain, the fee"bac# factor etc. The representation si is
also calle" the sensitivity fnction of a system. $or a !oo" control system, the vale of
sch a sensitivity fnction shol" be as minimm as possible,
%et T(s) be the overall transfer fnction of a control system. The forwar" path transfer
fnction &(s) is varyin!. Then the sensitivity of overall transfer fnction with respect to the
variations in &(s) is "efine" as,
a T(s) / T(s)
a &(s) / &(s)
ST ' &(s) a T(s)
& T(s) . a &(s)
$or the open loop system,
T(s) ' &(s)
ST ' &(s) a &(s)
G &(s) . a &(s)
' 1
.(. ())
Ths the sensitivity of T(s) with respect to &(s) for an open loop system is nity.
%et s fin" ot the sensitivity fnction for a "ose" loop system. $or a close" loop
system,
T(s)
*(s) G(s)
=--=
+(s) 1, &(s)-(s)
aT(s) .1, &(s)-(s/.10-. &(s)0.-(s)0 '
1
a&es)
=
.1, &(s)-(s)t .1, &(s)-(s)r
ST
&(s) aT(s)
&
=
T(s) . a &(s)
&(s)
1
'
. &(s) 0 . .1, &(s)-(s)1t
1, &(s)-(s) .
2 3
r
!
Control System Engineering 7-48 Time ResponseAnalysis of
Control Systems
ST _ 1
G - 1+ G(s)H(s)
. (6)
Comparing the two equations (5) and (6), it can be obsered that due to the !eedbac"
the sensitiit# !unction gets reduced b# the !actor 1$ %1+ G(s)H(s)& compared to an open
'oop s#stem( )nd 'ess the a'ue o! sensitiit# !unction, J ess sensitie is the s#stem to the
ariations in the !orward path trans!er !unction G(s)(
Sensitiity of T(s) !it" respe#t to $(s)
*et us ca'cu'ate the sensitiit# !unction which indicates the sensitiit# o! the oera''
trans!er !unction +(s) with respect to the !eedbac" path trans!er !unction H(s)( ,uch a
!unction #an be e-pressed as,
ST _ H(s) a +(s)
%% - +(s). a H(s)
/or a c'osed 'oop s#stem,
+(s) 0
G(s)
1+G(s)H(s)
aT(s)
a H(s1
%1+ G(s)H(s)&%21-%G(s)&%G(s)&
0
%1+ G(s)H(s)t
H(s) -%G(,)t
s3 0 ---(--3--33
+(s) %1+ G(s)H(,)&4
%1+G(s)''(s) t
H(s) -% G(s)t -G(s)H(s)
0 % G(s) &( %1+ G(s1H(s)r 0 1+ G(s)H(s)
15G(s)H(s)
((( (7)
6t can be obsered !rom the equations (6) and (7) that the c'osed 'oop s#stem is more
sensitie to ariations in the !eedbac" path parameters than the ariations in the !orward
path parameters( +hus, the speci!ications o! the !eedbac" e'ements must be obsered
strict'# as compared to the speci!ications o! the !orward path e'ements(
&.1'.( Effe#t of )ee*+a#, on Time Constant of a Control System
Consider an open 'oop s#stem with oera'' trans!er !unction as,
-
G(s) 0 1+s+
r
.
Control System Engineering 7-49 Time Response Analysis of
Control Systems
When this system is subjected to unit step input, its response can be obtained as,
C(s)
K
..... as open loop system
R(s)
=
1+sT
R(s)
1
unit step input
=
s
C(s)
K
..
s(l+ s1)
c(t)
=
L!"C(s)#= c,"$ Kl#= K"le ' I T ]
..
(s+T)
%o T is the time constant o& the open loop system.
'o( the &eedbac) is introduced in the
system (ith &eedbac) trans&er &unction as
*(s)= h. This is sho(n in the +i,. -../.
Let us obtain the response o& this closed
loop system &or unit step input. The o0erall
trans&er &unction o& the system is,
K
C(s) &+%T
R(s) = K
1+--h
l+sT
C(s}
+i,. -../
C(s) _ K = K = K/ T
R(s) 1+sT+Kh t+Kh+sT "s+e+$ )#
1
R(s) =
s
(K!1+Kh>j
(
1+Kh)
s+ -T-
K K t (I+ Kh) K [ t#
= .cT = 1 1 1e (T /1+.Kh)
1+Kh l+Kh 1+Kh .
r
"
Control System Engineering 7-50 Time Response Analysis of
Control Systems
Thus it can be observed that the new time constant due to the feedbackis (T/ 1+ Kh).
Thus for positive value of hand K1! the time constant (T/ 1+ Kh) is less than T. Thus it
can be concluded that the time constant of a closed loop s"stemis less than the open loop
s"stem.
Key Point : Less the time constant faster is the response. I fence the feedback improves the
time response oj a system.
7.18.5 Effect of Fee!ac" on #$erall %ain
#onsider an open loop s"stem with overall transfer function as Cfs&' The overall $ain
of such s"stem is nothin$ but %(s).
(f the feedback with transfer function &(s) is introduced in such a s"stem! then its
overall $ain becomes '%(s)/ 1(%(s)&(s)) . The positive or ne$ative si$n in the
denominator $ets decided b" the si$n of the feedback.
*or a ne$ative feedback! the $ain #(s) reduces b" a factor'1/ (1+ %(s)&(s+) .
,ue to the ne$ative feedbackoverall $ain of the s"stemreduces.
7.18.) Effect of Fee!ac" on Sta!ility
(t is discussed earlier that the feedback reduces the time constant and makes the
s"stemresponse more fast. &ence the transient response deca"s more -uickl".
#onsider an open loop s"stemwith overall transfer function as!
K
#(s) * s.T
The open loop pole is locatedat s/ -T.
0ow let a +nity ne$ative feedback is introduced in the s"stem. The overall transfer
functionof a closed loop s"stembecomes!
K
#(s)
*
R,s&
-S+T . K
1+/ 0 s+,K+ T&
s.T
Thus the closed loop pole is now located at s*-(K. 1). This is shown in the
*i$. 7.11(a) and (b).
(m1 (m1
000/'..0+000Real
0,K + T&
-T
0/(00000+000000 Real
,a& #pen loop system ,!& Close loop system
Fig. 7.22
r
3
Control System Engineering 7-51 Time Response Analysis of
Control Systems
Thus the feedbackcontrols the time respono;ei.c, dynamics of the systemby adjusting
locationof its poles. The stabilityof a systemdepends on the locationof poles in s-plane.
Thus it can be concluded that the feedbackaffectsthe stabilityof the system. The feedback
may improve the stability and also may be harmful to the system fromstabilitypoint of
view. The closedloop systemmay be unstable though the open loop systemis stable.
Key Point: Thus the stability of the system can be controlled by proper design and
application of thefeedback.
7.18.7Effect of Feedbac on !ist"rbance
verycontrol systemhas some nonlinearitiespresent in it. The dominant nonlinearitics
like friction, dead !one, saturation etc. affect the output of the system adversely. "ome
e#ternal disturbance signals also make the systemoutput inaccurate. $he e#amplesof such
e#ternal disturbances are high fre%uencynoise in electronicapplications, thermal noise in
amplifier tubes, wind gusts on antenna of radar systems etc. The disturbance may be in
the forward path, feedbackpath or output of a system.
#a$ !ist"rbance in t%e for&ard pat%
&et us assume that there is a disturbance in the forward path of a control system
produced due to varying properties of forward path elements or due to effect of
surrounding conditions. 'ig. 7.() shows the disturbance signal Tel *s+ produced in the
forwardpath.
C#s$
Fig. 7.()
,ssuming -*s+ to be !ero, let us obtain the ratio .*s+/ Td*s+ to study the effect of
disturbanceon output. 0ith R#s$''''' 1, systembecomes.
C#s$
Fig. 7.(5
r
(
Control System Engineering 7-52 Time Response Analysis of
Control Systems
The resultant elements are,
Gv;) = G2(S)
H'(s) = - G1 (s)H(s)
Positive feeda!"
#e$ative in%ut
C(s) G2(s)
..
- = Td(S)
=
1- [ G2(s)( - G1(s)H(s
C(s) - G2(s)
&&
T!(s)
=
1"G1 G2H(s)
C(s)
=
- T!(s)G2
&&
1"G1 G2H
#n the denominator assume that 1$$G1 G2 H hen!e 'e $et,
- T!(s)
C(s) = G& H(s)
Thus to ma"e the effe!t of disturan!e on the out%ut as small as %ossile, the G#(s)
must %e sele!ted as lar$e as %ossile&
(%) &ist'r%an(e #n t)e fee!%a(* pat)
These are %rodu!ed due to the nonlinear ehaviour of the feeda!" %ath elements& The
(if)' 7&*+sho's the disturan!e si$nal T!(s) %rodu!ed in the feeda!" %ath&
C(s)
T!(s)
(i$& 7&*+
,ith -(s)= ., the effe!t of T!(s) on out%ut !an e otained&
'The s/stem e!omes,
(i$& 7&*+(0)
r
+
Control System Engineering 753 nme ResponseAnalpls of
Control Systems
C(s)
Td(s)
For large values of GI' G2' HI' H2, in!e denom"naor # $an be negle$ed%
C(s) 1
Td(s& = - H#(s)
T!us des"gn"ng proper feed'a$(
elemen HI(s), !e effe$ of d"sur'an$e
in feed'a$( pa! on oupu $an 'e
redu$ed%
C(s)
(c) Disturbance at the output
Cons"der !a !ere is d"sur'an$e
Td(s) affe$"ng !e oupu d"re$l) as
s!o*n in !e F"g% 7%37%
*"! R(s&+ ,, *e ge
F"g% 7%37
C(s}
F"g% 7%3-
C(s)
+
Td(s)
# #
-:------=;;.+ /o/+ .
#/0 / G(s)H(s)1 #+G(s)H(s)
For large values of G (s) H (s), # in denom"naor $an be negle$ed%
Td(s)
22 C(s) + G(s)H(s)
T!us if d"sur'an$e is affe$"ng !e oupu d"re$l) !en ') $!ang"ng !e values of
G(s), H(s) or 'o! !e effe$ of d"sur'an$e $an 'e m"n"m"sed%
he feed'a$( m"n"m"3es !e effect of d"sur'an$e s"gnals o$$urr"ng in !e $onrol
s)sem%
r
!
-
Control System E;ngineering
..... "1
7- 54 Time ResponseAnalysis of
Control Systems
'-.. Example 7.5: The negative feedback control system has the forward path trallsjer
function as,
10
G(s) = s (s+1)
While the feedback path transfer function H(s) is 5. Determine the sensitivity of the
closed loop transfer function with respect to G and H at (1) = 1 radtsec. (A.M.LE.- 97)
Solution: For the gie! closed"oo# s$ste%&
10
G(s) = s(s+ 1)' H(s(=5
10
= G(s) ) s (s+1) = 10
1+ G(s)H*s)- 1+ 10 .5 +,+ s+ 50
s(s+1)
.Te se!sitiit$ o- T(s) .ith res#e/t to G(s) is gie! 0$&
+T ) 1 = 1 :1 s (s+ 1)
G - 1+ G(s)H(s) 10 s, + s+ 50
1+ s(s+ 1(25
T(s)
To /3"/4"3te +5 3t (1)=6& s40stit4te s = ! (6)to /o!ert the ti%e 7o%3i! -4!/tio! to the
-re84e!/$ 7o%3i!.
s5 =
! (1) "#+ 9 ro)
=
) (1),+ 9 (6)
..
"!ro),+ ! (1)+50
(50) (1),) + ! (6)
For the 3"4e o- "#$=6&
+T
=
- 1+ !#
%
49+ ! 1
& S' &
.(f)T
..
=
549, +1
=
0.0,::5
The se!sitiit$ o- T(s)& .ith res#e/t to H(s) is gie! 0$&
10 .5
+5 = - G(s)H(s) = s(s+1)
1) G(s)H(s) 1) 10 .5
+(5+1)
- s o
=
;e#"3/i!g s 0$ ! (1)& to /o!ert the -4!/tio! to the -re84e!/$ 7o%3i! .e get&
- 50 - 50
+<6 = = -:----=-:--
=(9 ro),+9 (")+50 (5>- (6),)+9 (")
r
*
Control System Engineering 7-55 Time ResponseAnalysis of
Control Systems
At 00=1,
S~I
- s o
=
49+jl
I Si ll
I-SO l
..
=
~492 +(1)2
...
= 1.0202
I t can be obsere! t"at S#I i s $ore t"an S~ i .e. t"e s%ste$ i s $ore sensi ti e to t"e
ari ati ons in &(s) rat"er t"an '(s).
'. Example 1.6: A position control system is shown in tile figure.
~KI
s s + a!
Cs"
(
I
Evaluate the sensitivities s~IS) .
K = 20 and a = 4
Sol#tion : *or a gi$en s%ste$,
'(s) = an! &+s)= 1
S (5+ a"
K
,o-
K
.(s)
Cs" 5(5+a)
= /+s)=I+ K
%(s+a)
0
=
52 + as+ K
a .(c1)
aK
(52 + as+ K" l"&K 1"' 52 + as
= 2 - (
(52 + as+ K" (52 + as+ 02
5(5+a)
51+as+0
%(s+ 4)
(52 + 45+ 20)
r
)
Control System Engineering
u :
762 Time ResponseAnalysis of
Control Systems
0.005
0.1
=K
For any value of K greater than 20, e'5 will be less than 0.005. Hence the range of
value of K for cs s0.005 is,
20 s K!oo
'. Example 7.11: For a unity feedback system G(s) = /008) and ret) = 2t determine
s s +
steady state error. If it is desired to reduce this existing error by 5"find ne !a"ue of gain
o f the system.
Solution : #he in$ut is 2t i.e, ra%$ of %agnitu&e 2, so K, will ontrol the error.
'i% 'i% 200
Ky = 0 s()s*H)s* = 0 s+ ) ,*1 =25
s+- s. ss-
/ 2
012 = != 25 = 0.0,
!
#his error is to be re&uce& by 5" of e3isting value, with new gain of ()s* as K2
instea& of 200.
)
5* 534.0,
e" o# $ % & l = s + 100 3e,,1 =0.0, + 100 = 0.076
5ew error is 0.076.
!
5ew ()s* =
2 an&
H)s* = 1 with sa%e in$ut
s)s-,*
'i% sK2 K..
.. Ky
=
s()s*H)s* =$ $ = +1
S40 5)s-,* '
/ 2 16
..
e... (,1
=$ =$ $ =$
Ky
)11*
K2
16
0.076 =
K2 = 210.52
So new gain is 210.52.
r
(
Control System Engineering
7 - 63
~.'I .t.ILli~1
Time Response Analysis of
t Control Systems
). Eample 7.1!" The control SlJstem is shown belota. If ti,e input to the system is
i) Ilni! step and ii) Unit ramp, find e.,.". (M.U. : Nov. - 96)
C(5)
1
0.1 200
t-----t # - I-------t
$ %$#1)%$#!)
.___--,
Sol&tion" G(s) = (1 0.1) ( 200 ) =
+ S(5+ 1) (s+ !)
%s #0.1)200
s%s# 1) %s# !)
H(s) 0.2
G(s)H(s)
200(s #0.1) 2
= --- 00 '
S(5+ 1) (5+!) .
0.2(1#105)
s(l #s) (1#O.5s)
200x 0.02x 0.1 %1#lCs)
-~----"~- - -
1! s(1+s) (1+0.5s)
i) o! "#it st$% i#%"t
&i'
(% = s ---#0 (%s))%s) =
_1_= 0
1+ (%
&i' 0.2(1 # lOs)
'00
s ---#* s%l #s) %1#*.$$)
$ =
S)*
ii) o! "#it !+'% i#%"t
&i'
(, = 0s(%s))%s) =
s---#
&i' 0.2( I#lOs) ' 0.2
s -, 0 s(I#s)-1#0.55)
$ =
~
1 1
-'- = $
x, 0.2
'. Eample 7.13" Find the steady state error E, if T is unit step input and = +.
.
r
,
Control System Engineering. ,. 7-64 Time Response Analysis of
Control Systems
Solution : When system is not in the simple closed loop form then we can not apply the
error coefficients.
Insuch case, we have to use final value theorem
Lim Lim
i.e, e = e(t) = sEes}
"'~ t~oo s~O
I-
In the system iven E(s) = ! "(s) when #(s) = $
i....-.
%ow let us find out i~: ; , so that for unit step distur&ance we can calculate "(s) and
.
hence E(s). When # = $, summin point at #(s) can &e removed and &loc' of '! (' is to &e
added to consider sin of the sinal at that summin point.
)hiftin summin point to riht.
(+ $.(s
($
C(s)
C(s)
$.(s*($
'----t (+ $.(+ (! ! ! ! ! (
"om&inin the two summin points and redrawin the diaram.
! ($$
(1 +O.1s)
($$
(! ! ! ! ! ((( + ,.l)) t! ! ! ! .-
CIs)
r
"
Control System Engineering 765 nme Response Analysis of
Control Systems
Negative sign of (1~l~s) can be taken out to change sign of the signal at the
summing point from positive to negative.
Now the two blocks are in parallel, in the feedback path.
C(s)
100+ s
(1+0.1s)
100
C(s)
7
..
T(s)

100 (100! s)
1!"#
$% (1!0.1s)
.
C(s) 100 (1 ! &.ls)
T(s)
=
(0.ls' ! $%! 100s! 10000)
1
C(s)
1
100 (1 ! &.ls)
(or T(s) ",

")
s $
(0.1*3 ! s% ! l00s! 10000)
Now +(s) " C(s) =-
100 (1 ! &.ls)
=
s(0.ls' ! $%! l00s! 10000)
,im
.
c""

&$ +(s)
s-+
,im - ,"100 (1! &.ls) . 100
e';'1 = s"!&s) s(0.ls'!s%!100s!10ooo) =-10000
$tead/ state error " 0.01
' .. Example 7.14: Determine Kp, KI1 alld KaJor a system with
Solution:
G
f ) 100 , H(s) =1
ts = *(*! 0.5) (4-*) (*! 1000)
100
(0.1. 2 Nov. " 3*)
4(s)5(s)
s) &.$6 (7 !&~$)68 6 (7 "i)6 l&&6 (l!l~)
4(s)5(s)
0.*
"s( 1"!".22%,.....,s),."(~1""9 "0.%*s) (1+&.:1s)
r
"
Control System Engineering:' I~t , ' 7 - 66 Time Response Analysis of
Control Systems
Lim Lim
0.5
00 K = o G(s)H(s)
=
=00
p
s~ s ~ 0 s(l + 2s) (1- 0.25s) (1+ O.Ols)
Lim Lim
sO.5 = 0.5
K"
=
sC(s)H(s) =
s~o s-+O
s(l + 25) (1- O.25s) (1+ O.01s)
Lim .. LIm ~2 x 0.5
K = s~G(s)H(s) = = 0
oJ S-40 S~ 0 s(1 + 2..) (1- O.25s) (1+ O.oIs)
'. Example 7.15: Find the steady state error for various types of standard test inputs for a
lmity feedback system witll
G(s) = K
s(s+ 5) (s+ 10)
(a) K = 10 (b) K = 200 (M.U. ~May-1(95)
Solution:
K
G(s)H(s) = s(s + 5) (s+ 10) =
K
=
(5~)
s{1+ 0.2s) (1+ O.ls)
Lim
O
G(s)H(s) =
S-4
(5~)
,.---=-:-_ = 00
s -+ 0 s(1!" 0.2s) (1+ O.ls)
Lim
Lim
O
sC(s)H{s)
s-+
Lim Lim s2(5~)
K" = 5-40 s2C(s)H(s) = s -+ 0 s(l + 0.25) (1+ O.ls) = o.
. #o$ s%&p i'p(% o) m*+'i%(,& I-
I 1
& = --=-- = 0
S. l+K 1+-
p
///)o$ *'0 1*l(& o) K.
#o$ $*mp i'p(% o) m*+'i%(,& I-
1 50
&o!s = K" = K
a) #o$ K = 10- ~~= 5
2) #o$ K = 200- 3445-= 0.25
!
"
Control System Engineering 7 -67 Time Response Analysis of
Control Systems
For parabolic input of magnitude 1,
1 1
e.... = K= ' 0 = 0 0
. .
... for any value of K.
I I ' " Example 7. 16 1 1 1 the system gioen, the command input is R = 1 0 and disturbance signal
is li. = 4, what is tire steady state error? (M.U. : ov. ! "4#
$
1!". 1s
1""
C#s$
t!%! ...
#s ! 1$#1 . . "%s$
Sol&tion Using superposition principle, consider inputs separately.
a# R acting,
$
1. . ". 1s
1""
C#s$
#s ! 1$#1 . . ". %s$
&(s#'(s#
$0 0
. .
=
#1!0 .1$# ($) 1# (* ) 0 .+s#
For step input
,im
K. '
=
&(s#'(s# =S( (
s-+O
- . 10 10
. .
e" " ' l
1) .p
/0ere A =magrutude of step = !! =--
1)$0 0 $0 1
b# T) acting, R
=
0
$ 1""
C*s$
!!
1!". 1s #s ! 1$*1 !". %s$
T)
1(s#
=
! 2(s#
r
"
Control System Engineering 7 -68 Time Response Analysis of
Control Systems
As systemis not in standard fonn, error coefficientmethod cannot be used.
5 100
C(s)
......._ - -
1 + 0.1s (5 + 1)(1 + 0.2s)
100
C(s)
-5
1 +0.15
+
(s + 1) (1 + 0.2s)
100
(1+ 5) (1+ 0.25)
1- 100 x ( -5 )
(1+ s) (1+ 0.2s) 1+ .!s
100(1 +.!s) C(s)
"
T(s)
#o$
(1+ s) (1+ .2s)x (1+ .!s)+ 500
-4
s-%e si&n as '(. a))!ied $ith -%e si&n. T(s)
C(s)
-*00(1 +.!s)
"
5+(1+ 5) (1+ 0.25), (1+ 0.15)+ 5001
" - C(s) " +*00(1 + 0.15)
5.! + s) (1+ 0.2s) x (1 + .!s)+ 5001
but E(s)
/im

sE(s) "
s!
/im "00(1 +0.15)
s 0 0(1 + s) (1+ 0.25) # (1+ .!s)+ 5001
1. 'ota! error
" +02 $ +*00
1+500 501
10 "00 *10
ess " cu( + css2 " 501 +50r " 501 "
0.8183
%........ En &717 A d rd t . -, by 4(s) 250 F' d i . .
...'. mpe. ( secon 0 er sys em IS glvm R( ) " 2 05 In ,ts rise time,
s s +6s+25
p tJ lk time, p eak overshoot and settling time if subjected to unit step inp ut. Also calculate
exp ression for its outp ut resp onse.
Sol)tion ( 4om)arin& t*e denominator of T.+. $6th t*e stan,ar, form 72 + 2- (l)n 5 + (()0
.,
25 an, 28 (l)n 6
( 1 ) -
"
"
n
. % 1 .
i
.&/
61
(l)n
"
5 .. l- $
0.6
69
r
'
Control System EngJneeri{'l9 t 1 7-69 Time Response Analysis of
Control Systems
e = tan "
[
J N 2 ] ~ = 0.92 72 radians
Tr
7t-9 7t-0.92 72
=
--= = D.5535 sec
rod
4
Tp
7t 7t
=- = - = 0.785 sec
rod 4
%M =
e -"f!Jl-f"# $ 100 = 9.48 %
p
T&'
4
= -- = 1.33 sec
l("ro n
and c)t*
e-f""'nt e-+t
= 1 - sin )rod t!9" = 1 - sin#4t! 0.92 72 "
""1-l("# ~1-),-*#
= 1 - 1.562 5 e-31 sin #4t .0.92 72 "
$... %&a'()e 7.18* For the system shown in the figure obtain the closed loop T.F., damping
ratio, natural frequency and expressionfor the output response if subjected to unit step
in~t. .
#, "R)s*
#5!1"#5!4"
2 0
Sol/tion&
1 . 2 0 s
2
.5s .2 4
#s! 1"#s!4"
0ey 1oint & Now tlwugh T.F. is not in standardform , denominator always reflect #~ ron
and ro2 from middle term and the last term respectively.
C)s*
=
+#s"
#s! 1"#s! 4" 2 0
=
ro# =
2 4 )'* = 4.8989 )$ad),$ec
n n
r '
#~ron = 5 l(" =
0.51031
rod
= ron
J l ~! = 4.2 12 9 rad)sec
N o - .o r c#t" -e can /se standard e&(ressin .o r * in standard .o r'. So -ritin0
r
"
Control System Engineering 7-70 Time Response Analysis of
Control Systems
CCs) 20 { 24 }
-R(s) = 24' s2+5s+24
For the bracket term use standard expresson!
mu"tp"#n$ ths expresson by constant %&'
20 ( e -~fIIn l
'' c(t) = -2 1- ~
4 !!)*&2
and then c{t) can be obtaned b#
e = tan-) & radans = )'0+radans
c(t) = ~~ ()- )'),2-e - 2'5) sn (4'2)2.t + )'0+)/
''' 0xamp"e 7').1 For a control system shown in figure, find the values of KI andK2 so
that Mp = 2! and Tp "" " sec. Assume unit step input.
C#s)
Sol$tion% &(s) = & ! '(s) = 1+ 22 s
32
4'F' =
1' 5omparn$ 6th standard 7onn!
8o6 9p is 7uncton o7 & a"one!
(( ! 9p = e-:!;<=-7'2 x"00
25 = e-:';<"-7!!2 x )00
0'25
r
)
Control System Engineering 774 Time Response Analysis of
Control Systems
Now the time for 1 cycle is known and if it is known to us that what is the time
required by the system to achieve steady state, we can find how many cycles output will
perform before reaching steady stale.
4
!""!#sec
II ~ron
$o 1.%7&#sec for one cycle, how many cycles output will perform in#sec.
#
'. otal no. of cycles !1.%7&#= 1.1&1
(utput will perform 1.1&1 cycles before reaching the steady state.
This can be shown as in)ig. 7.*&.
c+t,
time
)ig. 7.*&
he frequency in -. if required can be obtained as,
1
fd !/.0&00 -.
fd ! )requency of damped oscillations in -..
'. Example .!! " A second order system is represented by the transfer junction,
1+s, 1
2+s, ! 3$#4 fs4 5
A step input of #$ Nm is applied to the system and the test results are,
a) Maximum overshoot !% 6
b) Time at peak overshoot !#sec
c) The stetldy state value of the output is 0.5 radians
etermine the values of #% f and K.
7 8 2 , ,! r" ,
'l.. Control System Engineering 7-75 Time Response Analysis of
Control Systems
Solution : Arrange the given transfer function as,
Q(s)
=
l(s)
ro2
K
i.e.
ron =I f =
T
n
and 2~(I)n
f
i.e.
f
=
j ~ = 2JKf
Now
Mp =
6% i.e., .6
.6 =
-1tVJl-~2
.. e
.
In (.6)
-1t~
=
Jl-~2
...(1)
.!. (")
#olving for ~ , ~ = .66$
...(3)
%t
ron = = &."%6' rad(sec
~ %)(.66$)"
... (&)
*he +aplace transfonn of output is Q(s).
Now input isstep of % N, hence corresponding +aplace transfor, is,
I (s)
%
=
-
s
Q(s)
%
e ~ )
=
-s$.fs./
Q(s) =
%
#(-#" . fs. /)
*he stead0 state of output can be o1tained 10 final value theore,.
#tead0 state output = +i, sQ(s)
.... 0
.' = +i, s2% = %
.... os(-s$.fs./) /
34% , , : r I ,
Control System Engineering 776 Time Respons4f Analysis of
Control Systems
.. K = 20
Equating (1) and (4),4.2165 = ~
.(5)
.. 4.2165
=
F 1
.
J =
1.124
F ro! equation (2), 0.667 = "
2 KJ
0.667
f
.. =
2.J20#1.124
.. f
=
6.$274
,.. Example 7.23 : The open loop T.F. of unity feedback system is G(s) = %(! Ts) . For this
l:>ystem overshoot reduces from 0.6 to 0.2 due to change in 'K' only.
TK -1
Show that = 4$.$$ where Kl and K, are values of ! for ".# and ".$
TK2" !
overshoot respectively.
#(s)
&olution' Clo%ed loo( ).F . = 1 $ *(%+ , ,(%) = 1
%
%
s(l $Ts)
%
T
=
%
=
=
2 %
$
s2T$s$%
s(l $Ts)
& $Ts$ T
fii
Co!(aring -it. %tandard /onn 2 n 2
& $ 20 (J)n &$ (l+n
002
%
(J) =! =
)
n
n )
210(l)n
1
10=
1 1
=
=
)
2~ 1)
2.JKT
2o- /or
" (
=
0.6 3et l0 = l0J
.. 0.6
=
e 4/t~ t2l4~
i.e, 40.51
=
7t l0t
~1-l;~
...
561 , % :r 7,
Control System Engineering 7.78 Time ResponseAnalpls of
Control Systems
.,
As l; > 1, system is overdamped, hence the output will not contain any oscillations.
Hence standard expression for c(t) cannot be used.
Now input is unit step, so es) ! l"s #ubstitutin$ in %.&.
1'
yes) !
s(s') 7s) 1')
*se partial fraction method
!
1' A+,
--:----::=-:--=! - + -- + --
s(s) .) (s ) /) s s ) . s ) /
where, A =1, +=-/, , =.
143
yes) ! s- s ) 3 ) s )
%a0in$ 1aplace inverse,
yet) ! 1- /e- .1 +.e- ct
... 2xample7.'34 For a control system shown in figure, find the values o f K and K, 50 that
the damping ratio o f system is 0.6 and settling time is 0.1 sec. Use T. - ..' Assume
- 56(!",,#
unit step input.
C(s"
Solution :*sin$ bloc0 dia$ram reduction rule, reduction of inner loop is,
177
1) 7.'s
177
100
1) 1) 7.'s . 8,
=
1) 7,'s) 177x,
9verall :(s)
! $. %&& .'%'! ($
1)7.'s) 1778t '7s s;1)1778t )9.'s<
H(s=
!
1
38
%.&.
)(s" s (1) 1778, ) 7.'s)
! !
1) :(s)H(s)
1) 38
s(l ) 1778> ) 7.'8)
r
*
Control System Engineering 7 -79 TIme Response Analysis of
Control Systems
5K
=
0.252+5(1+100K, ) +5K
But coefficient of s2 in the denominator must be 'unity' to compare it with the
standard form. Sodividing it b 0.2.
!ow
"sing
i.e.
and
C(s)
25K
R(s)
52 +5s (1+100 Kt) + 25K
o l 25 K,
ron = 5.#K
n
2 $ron 5 (1 + 100 %&t)
l!
=
5(1 + 100Kt)
l"#$
% =
0.' and (s = 0.1 sec
r!
3 . 2
$ron
0.1
).2
=
0.' ron
ron =
5).)) rad*sec
ron =
s&#$
K
=
11).+,
%

5(1 +100 1(1) = 0.'
10.#i(
Kt = 0.11,
'& E'ample +.2'- For the system shown in figure show that system is always overdamped,
independent of the selection of Rand C&
Solution ( (o prove that sstemis overdamped means to prove $ ) * and not dependent
on the va.ues of /and 0.
So first to find its 0.1.(.2. $$$-$use signa. f.ow graph method.
r
+
Control System Engineering
-,
7-80 Time Response Analysis of
Control Systems
I} (s) =
Vj -VI
..
R
Vt(s)
1
= (It -12). sC
12(s)
=
VI -Vo
R
Vo(s)
1
= 12.-
sC
r, Signal flow graph is shown inthe following figure.
V I
o
1 1 R

-l/R -1 -11R
sC
1
L1 and L.) is comination of 2 nontouching loops.
!sing "ason#s gain formula,
$t = 1as all loops are touching to %,
1
C(s)
=
R(s)
1
=
&omparing with standard form,
1
&2 '2
1
wn= CR rad(sec
r
"
Control System Engineering 7-81 Time Response Analysis of
Control Systems
2J: "'n = 3
':t'" CR"
3 1 3
l; = CR.2 : . CR= 2 " = 1.S
As l; > 1system is overdampcd and l; is independent of R and C values
!or t"is# t$o resistances must be e%ual# and t$o capacitor values must &e e%ual
,,.. Example 7.2 7: A position control system driVes a load through a 5 : ! gear ratio. The
inertia of motor shaft is 2' ( 1'-) k.~ nl and friction is *'( 1'-) N-m sec. Tne torque
constant is 0.0 N-m per degreeof error! if output speed is 2' rpm. "alculate
iJ l; ii) ro" "
+olution: ,et -= -or%ue produced proportional to '8.-8''
$"ere# /i = Reference input position
8' = 0utput position
1sed to drive load of 23 T and friction '4'
T = J #2 eo $ %#eo
dt2 dt
5 6' 7-90] = J d2*' $ 4 d0o
dt2 dt
...
-a8in9 ,aplace $e "ave#
5 e& :s; 50o:s<$ J s2 'o:s< $ 4s0 ':);
'
-
~ - - - - - - = --==-=
<+2 $ %s$ ' 2 % '
s $ TS$ T
'
5 = ''> N-m/deg = ''> x )73 ?-m@radian
Aear ratio = 1#$0
: . ' at motor s"aft = ''> ( $%.&#$0
r
"
Control System Engineering Tim. Response Analysis of
Control Systems
0.04x 57.3
I-----=-5 = 15.13 radlsec
5Ox20x10-
B
~ = -- = 0.099
2J f K
I ) " ExMI ple 7.28: A system has the following transfer function
C(s) 20
=
R(s) s +10
Determine its unit impulse, step and ramp response with zero initial conditions. Sketch the
responses. (M.U. : May-%)
Solution : a) Unit implse inpt
1
20
s! 10
1
-,.
s
" 20
s#s!10)
$ ==2"
R(s)
=
C(s)
as
R(s)
=
C(s) =
.. c(t)
=
.. c(t)
=
%) Unit step inpt
.. R(s)
=
C(s) =
&'ere
2
C{s) ==
20
s! 10
L-1 {~}
. s! 10
e-tDt
A (
+ -
)!10
--
s
2
c*l) = +-1 {~- , - lO.
c(t) = 2 - 2e-lOt
c) Unit ramp inpt
1
R(s) =
2
C(s) =
20 $(*
-=---==- +- +--
52#5!10) 52 , 5!10
!
"
Control S,.eem engineering 7-83 Time R_pon .. Analysis of
Control Systems
..... -,
A(5+ 10) + Bs(5+ 10) + Cs2
=
20
.. B+C =
0, A + lOB = 0, lOA = 20
.. A= 2, B = - 0.2, e= 0.2
.
C(s)
2 0.2 02
=
---+--
52 5 5+ 10
.. c(t)
=
L-1 { .3 .. _ 0.2 + ~}
52 5 5+10
.. dt)
=
2t - 0.2+O.2e -lot
The responses are,
c(t)
20
Unit impulse
o
c(t)
2t---------~~~==~
Unitslep
c(t)
_ --_ .
.. -
......
....
....
....
....
....
,,
Unit ramp
r
"
Control System Engineering 7-84 Time Response Analysis of
Control Systems
'... Example 7.29: Derive an expressionfo r the time responseof a second order system
subjectedto a unit impulse input for ~ < 1, ~ >1, where~ is a damping ratio.
Solution : The second order system transfer function is
Cs!
"
R(s)
o l
n
The characteristic e#uation is,
s$%$~&!ns%ro~ " '
(here ~ " )am*ing ratio
The roots of this e#uation are,
- $~&!n+ ~ 4~
$
&!~- 41!,
-1$ " $ " -l.&!n+&!n~l.$-1
" - ~l!n+ l!n/ l -I;
$
l!d l!n~1-l.$ 01 (here
!ase i!: ~"1, under dam*ed0
The roots are com*le2 con3ugates0 The in*ut is unit im*ulse,
4s!
"
1
1!$
Cs!
"
n
-$ % $1. 1!n-% 1!~
( J l
Cs!
"
n
S$ % $ ~ro -% ~$1!$% 1!$5 ~$1!$
n n n n
1!$
.#
1!-
rod
Cs!
"
n
" 00$,00
6S%~ron!$ %&!,15~$! rod 6s% ~ l!n!$ % &!~
$-16 % o &
"
e - at sin l!t
..
s%a!$%1!$
7 $ 8 $-17ees9
"
$-l'. C&ld
..
/ !d s9 l/ 0C&:t!$ ('
$
ct!

m.. 'e -tco n lsin / ! t
..
C&l; J l - ~$ d
< ;
)
*
C~~ISysam engineering
~ .T_' ,. _'I
7-85 Tim. ResponseAnalysis of
Control Systems
ct!
Case ii! ~ " #, critically $ampe$
R(s)
"
#as %nit imp%lse inp%t
Cs!
ro&
~
;;;;;
n
"
5&'&ro s'ro& s' ron!
&
n n
(o)
*-#+e-altl
#
"
s'a!&
*-#,Cs!-
" L - ' [ & 1 ]
ron S'ron!&
ct!
"
ro&. t e-lIInt
n
~ >
#, o.er$ampe$
R(s)
= # as unit imp%lse inp%t
Case iii!
T/e roots are real an$ ne0ati.e
1in$in0 partial fractions )e 0et,
..
*-#,Cs!2
"
ct!
ct!
"
~ [ J- e .. - .. N-' -3 4~ . . , , _ J ~ 2 _ , ] . ]
..
&C4!n~~&-#
cCt!
"
ro , . [ -e-..5.=i..~- R=i) ]
..
&~~& -#
r
!
7-86 Time Response Analysis of
Control Systems
The responses are shown inthe following figure.
r(t)
Unit impulse
c(t)
'... Example 7.30: The open loop transfer function of a unity FBCS (Feed Back Control
System) is given by G(s)= '(: 1)'
s s +
i) By what factor the amplifier gain K should e multiplied so that damping ratio is
increa:sed from 0.2 to O.S.
ii) By what factor the time constant ! should e multiplied so that the damping ratio is
reducedfrom "#$ to "#%# (M.U. : Jan.-92,. May-97)
Solution: Given G(s) = s (!"#) an$ %(s) = #
&
'(s) G(s) s(sT" #)
..
((s)
= =
#"G(s)%(s)
#" &
s(sT" 1)
& &)T
= =
Ts2 "s"&
2 # &
"-s"-
T T
'o*paring $eno*ina+or wi+h 2 "2,())n s"ro,,
2 K
()) = - an$
n T
and
r
"
Control System Engineering
- . : - - ' ! ' - , -
7- 87 Time Response Analysis of
Control Systems
Case i) : 2 ; 1 = 0. 2 , K = K} !ile ; 2 = 0. 8, K = K2 , " = #onstant
1
0. 2 = 2 $1 %. , = 1 -
1
an&
0. 8 = 2 . J K2 "
1
0. 2
0. 8 -
1 K2
. . . s'(aring )ot! si&es ..
1 6
::;
~
K2 =
1
..
1 *K1
So K m(st )e m(lti+lie& )y ,l* to in#rease; -rom 0. 2 to 0. 8.
Case ii) : ; 1 = 0. *, " = "} !ile ; 2 = 0. ., " = "2 , K = #onstant
. . 0. * = 1 an& 0. . = 1
2 . J K"/ 2 . J K T z
!f;
0. . = . . rr;
0. *
. 2 rr;
/. C. = V " T;
i. e.
So " m(st )e m(lti+lie& by 0 to re&(#e; -rom 0. * to 0. ..
'. Eample !."1: For the system shown in the given figure, calculate the sensitivity o f the
closed loop system with respect to the function G(s). Does the sensitivity depend on U(s) or
M(s) 1
#$s%
&1
Sol(tion' : 2rom gi3en signal -lo gra+!,
"l = '$s%($s)*$s)+ "2 =#$s%*$s)
4l = - ($s%*$s%+
T$s%
= ,$s% - '$s% ($s% *$s% .#$s% *$s%
R$s% & 1.($s% *$s%
*$s)/'$s%($s% .#$s)0
::;
l.($s%*$s%
"5s)
r
"
tK. Control System Engineering 7-91 l!Time Response Analysis of
Control Systems
u.. Example 7.34: A second order system lias unity feedback and open loop Irmrsfi'" [unction.
G(sJ= 500
s(s+ 15
a) Draw the block diagram for closed loop system.
b) What is characteristic equation
c) What is damping ratio and natural frequency !alues
d) "alculate Tp #peak time)$ M" #peak o!ersl%oot) and &' #settling time) for the system output
response tohen e(cited by unit step input.
e) )ketch the transient response for unit step input.
/) If input is ramp of *.+ rad,sec$ calculate steady state error. (M.U. : Nov. - 95)
Solution : a Block !iagram is,
C"s
#
$
b) Characteristic e!atio" is
1 % C(s)#(s) $ % i.e. 1 % s& :' ( 5)$ % i.e, s:) % 15s%511%$ %
c) Co*+ari", above -ith sta".ar. e!atio"
/' % & 1' "$ns% "$( $ 0
.)
"$( $ 5%% i.e, (l)" * ' ' .01%12ra.3sec
i.e.
/4ste* is !".er.a*+e..
.)
(l). (l)" ( 1-l5' $ & & .3+ 6 ,-"0.335-1 7 '
$
' 1.%182 ra.3sec.
Tp
1t
%.1891 sec.
$
- *
(l).
9Mp
$
e-r. ./ -l'.6 1%% $ 0' .1779
r5
4 4
::
0 ' "$n1 :: :.005;6i' .01 :: %.5000sec.
r
2
Control Sysl!.m Engineering 7-92 Time Response Analysis of r..
Control Systems
e}
c(t)
._. __. ./. 2%
1t=t=~t==t==~~~====
--r------- ---

-fI---...;..._----__._,-----_ time
0.533
sec
0.149
sec
f) ret) ! 0.5t
R(s)
05
=
7
0.5
Lim sR(s) Lim
s _
?
eS.~ = =
s-
s-!
1 "(s)#(s)
s-!
1
500
s(s 15)
Lim
0.5
0.5 $IS
=
s$S!!
=
500
s-!
s
s(s 15)
Css
=
0.015
"_ E#ample 7.35 % For a unity feedback system G(s) = s(s :'~7i)'Determine characteristic
equation and hence calculate damping ratio, peak time, settling time, peak overshoot and
number o f cycles completed beforeoutput settles Jor a unit step input. (&.'. % &() - 93)
Sol$tion % *+(r(cteristic e,-(ti./ is & ' ((s))(s) = *
.
1
36
0
s(s 0.72)
!
i.e. s2 0.125 30
! 0
*.m1(ri/2 +it,
? ?
s - ' 2l!ns !" ! 0
?
30,
c./ ! 0
r(34sec ..
!# %%%
..
n
(/3 25;c./ ! 0.72, .. 5; ! 0.00
CO d
! 67)/ ~5- ;2
..
ro-
! 5.9891 re34sec.
9 :
r
.
: . e q ; w i r o l S y s t e m Engi ne e r i ng
,. ~ I '.
7-93
. ,
t_ Ti m e Re s po ns e Anal y s i s o
!o nt r o l S y s t e m s
Peak time Tp
"t "t
. .
= rod = 5.9891 = 0.5245 sec.
Settling time Ts
4 4
#--#
0.06 x 6 = 11.11 sec.
l;ron
Peak oversoot
!"
#
#rt$%&#T._i
=
0.82'9 e
i.e( )!" #
82.'9)
rod =
5.9891 rad*sec(
# $t %
ro
E% =
-$~= 0.95+1 c,cles*sec.
Td
1
1.0491 sedc,cle ... Time -or one c,cle.
=
0.95+1 #
.o/
i.e.
Settling time is 11.11 sec tere-ore o0t"0t /ill "er-orm(
11.11
1.0491 # 10.59
... 1,cles 2e-ore it settles.
'. E&am pl e 7. 3': Followitl8 expression denotes the lime response of a servomechanism.
c(t) #1 (D.2e-60t - 1.2 e#l3t
i J Obtain tire expression for tire closed loop transfer junction of tire system.
i i ) Determine tile undamped natural freuency and dampin! ratio. "ssume unit step input
4!.5. 6 .ov. # 967
S o l )t i o n:
i8
c4t7 = 1 (0.21#6l9 # 1.2c#"*l
Taking :a"lace trans-orm
14 7
1 0.2 1.2
S = S ( s (60 # s (10
=
+s (107+s (407 (0.2S48 (107# 1.254S (407
s; s< 607 +s ( 107
=
S2 ('0s (600 (0.2s2 (25# 1.252 # '25
s4s< 607 4s< 107
r
,
Control System Engineering ,;. 1-94 Time ResponseAnalysis of
Control Systems
1
R(s) =
5
600
Closed loop transfer function
C(s) s(s +60}(s +10)
= = =----= -----
R(s) 1 52 +70s+600
s
ii) Characteristicequation is
52 +70s +- 600 = 0
Coparin!"ith
#2 +2~ rons +(!~ = 0
.,
600$ "#$n =2%.%&%' rad%sec&
(!~
=
n
and 2~(!n = 70$ ~ =1&42''
600
(s l; ) 1$ s*ste is o+erdaped hence output is purel* e,ponential "ithout
oscillations&
(.... -,aple 1&.1/ A position servomechanism has as error detector gerlerating )V/rad error.
It is followed by an amplifier o f gain 'K'. The amplifier feeds the armature of a motor which
has armature resistance o j )nand negligible inductance. TILemotor generates .IV/!rod/sec"
bac# e.m.f and lias 1= $.% #g 01$2 and viscous damping is negligible. The motor is coupled
to the load through a reduction gear 'luitl& ratio '. Tire load has
1= )#g(m) and viscous damping constant * =$.' +m,!rad/sec". -etermine the closed loop
transfer function of the system. Also find the damping factor. Ta#e K = %$$. .hat is the
effect of increase in K on damping factor. (*.+. " 2o+& - 95)
Sol,tion " T-e circuit dia!ra is
Error .ete/tor
Re.,/tion gear
2$ f
0=)
1 =0.2
J e q
J olor + h3 3 d (4 / r
c r
0&145!-2
=
= 0&1+ 1, 51 =
6t&1)
6l)73 r + 6l8ad (4 / r
c r
0&12%(rad%sec) =
=
09 0&5, 5
=
: 1
r
3
Control System Engineering 7-95 Time Response Analysis of
Control Systems
The block diagram is
Ke Error
detector
Kb = 0.1V!(rad!sec), K 100
K\! = ] V! rad I
Assume torque constant KT of the motor as 1.
1
(!"1#$!"1)
1+ %(!"1#+!"1) x !"1s
1
(!"1#+!"1)
1
&!x --:-::--::---,--:,- 1'''""""'""""
% (!"1#$ !"&)
&!
eC < 5 ) (!"1'1$!"&)
ares)
=
1+ &!
(!"1#$ !"&)
(c(s) &! 1#&")*
= =
(r(s)
!"1#s& $ !"&$ &! %& $ 1"#&)$ 1#&")*
The characteristic equation is
& $ 1"#&)$ 1#&")* = !
+om,aring -ith
& +2e ron +ro. = 0
r
"
Control. System Engineering
I ~. l" 1 li
7-96 Time Response Analysis of t:.').
Control Systems
.,
(J)~ == 142.857, r o n ==11.952r ad/sec
an d 2~m" == 1.4285, :. ~ ==0.0597
Dampn ! "ac#o r is 0.0597.
If !an $ s n cr eased, #%en (O~ &a'(e )'' n cr ease *(# '21; (l)n' )'' r eman co n s#an #.
+%er e"o r e n cr ease n $ )'' ca(se decr ease n dampn ! "ac#o r .
n E!ample 7.,8 : A system has 30% ooersnoo: and srttlirtg time of " seconds,Jor a unit
step input. Determine the transfer function. Calculate peak time and output response.
Assume <; as 2-. (../. : 0o &. - 95)
Sol#tion$
,0 == e ,uJ. t:! ." 1 100
== e-r r 2,3J# - 1.2
0.,
.In(O.3) ==
~1 - 4 5 :
l;
%
== 0.128, .e, l; == 0.,578
.&
+. == --==5
35 (J)n
(J)n == 6~ = 2.2,58 r ad/sec
: l . "
C(s)
+.7. ==8 (9 )
.,
(1)- ".9988 = n~ = .66 6
"% : 2):(1) . ..:: I, ;! . "% : 1.599s: #.$$%%
., .,
;%er e
a ' #an -1 (~) r adan s '1.2048 r ad
c(#)
<-=>81
' 1 - sn (2.=9 77# * 1.2048)
~' - (0.,578)2
c(#) ' 1 - 1.07 e-=.8# sn (2.0877# * 1.2048)
r
"
- l-
.iControl SystemEngineering
~:t:
7 -97 Time Respon.. Analysis O f
Control System.
'... Example 7.39: A closed loop system has two complex conjugate poles at 51' ~2=-2j.
Determine the form of transfer function and values of 0>" ' 'fp' T, ' !" and #$ assuming
standard second order system. (M.U. : Nov. - 95)
Sol%tion:
C&s'
0>2
n
R&s'
=
s( ) (!O *ns) &+.'!
.
while -2jl
'.
Sl ' 52 =
Deo!i"to# o$ %.& ,(5' 2 ) jl) (( ' 2 - jl) =(5' 2)2 - (j))2
= s( ) -s).
":&s' 5
=
R&s' S2 ) -s) .
*o!+"#i, /it0 (t"-"#- $o#!
..
&.'(
=
5 , :. &l.'n=(.(31
n
"-
(!&+.'n
4 ,
.
=
..
~= -- = 2.39--
(x O *n
&l.'4 , &l5'n~)- !( = 2.9999 #e-/(e0 "" ra46se7
G
[ R ' ]
#"-i"(
,
t" - ! #"- ,2.-131
%+
7t
,
- ,3.-. se7
ro4
%(
.
=
-,(se7
!ron
r8
7 t - G
, -- = (.1779 se7
ro4
1 M+
=
c -f r 1 j J I -F . 2 2 100 = 0.13451
.-
'. Example 7.-2: find the impulse responseof electricalcircuit given below. &#.9.: M"6-95)
:
9
;
+

el
2U 2&
e o
1
'$ ''$+ .
r
$
Control System Engineering 7 -98 Time Response Analysis of . 'F
Control Systems
Solution: Taking Laplace of the network
1/5 V1 eo
r 1I-------r---'\MI\r--r------o 1
fo- s _ > _ _ I _ J _ _ 2 _ _ . _ _ _ ' ) _ _ - - - , _ ; _ 5 _ - 1 ! ' 5 )
I I "
#j - ! 1
" s $c, - %I >
1
s
&I "
2(1]-12 )
'2
"
()I - eo
1
1 2
1
eo
"
* -
2s
.. (1)
. . . . '2 )
. . . . . '+)
. . . . . ',)
-s -2
[I
"
1
L- - 2 s
L2 =- 2 L+ "
1
=
- 2 5
.. /ignal flow graph is
L) L+ " 0on to1ching = ~
" 1 22 /22 22 1 21
s
1
" ,22 s 22 s
L \ 1 "
1 as all loops are to1ching to T]
eo's )
"
T) L \ 1
3s ing 4as on' s gain for51la
ej 's ) =r:
1 2 5
" "
1
4s2 + 8<:+ 1
4+ 2 5 2' 2
s
eo's ) . 5 5
"
#, 's )
/2 + 2 5 2 .25
r
!
Control System Engineering 7-99 , Time RespOnse Analysis of
Control Systems
For impulse input ells) = 1
e.,(s)
O.5s 0.5s
=
S2 + 2s+ 0.25
=
(s+ 0.1!) (s+ 1."##)
A $
=
% +
(s&t& 0.1!) (s + 1."##)
A
=
- 0.0386, B = 0.5386
(',,(s)
-0.0386 0.5386
= +
..
(s+ 0.1339) (s + 1."##)
.. cClt!
((()
*%1"c,,s!!
eo(t)
=
0.5386 e-1.6661 - 0.0386 e#.13391
eo(t)
,+ E,ample -..1 ( A system has unit step response of c(t) = 1 % e-l$l. Determine its
impulse response and ramp response. Assume zero initial conditions. %.&. ' (une - 9))
Sol/tion ( For unit step input
R(s) = 0
s
*+,in- .+pl+ce o/ ct),
Cs) '''0 # - s+10.1 1. SS##.l)
0.1
T1 Cs) '''0
. R(s)
s(s + 0.1) 0.1
1 &. 5+0.1
s
For impulse response, 2s) = 1
Cs! =
0.1
s3 0.1
ct! = .-1 "Cs)! = 0.1 e-)4l$
r
&
Control System Engineering 7 -100 Time Response Analysis of ~J;""
Control Systems
For ramp,
1
R(s) =
ABC
=-+-+--
52 S s+0.
A(s +0.) +!s(s +0.) +Cs2 =0.
B+C
=
0, A +O.IB = 0, O.lA = 0.1
.. A
=
1, B= -10, C = 10
C(s)
1 to 10
..
=
- - - +_ .._--
s" # S-4-0.1
.. $(t)
=
%-J (C(s)J
=
t - 10 +10e-l).u
n& E'ample (.)"* T+eopen loop transfer function of lmity feedback system is
ao = - - : = , . . . . . K - - - = :
*(f,-+)
By what factor the gairz K should be multiplied so that damping ratio is increasedfrom 0.3
to 0.8. By U!ratfactor time constant slzould be multiplied so that damping ratio is reduced
from ". # to ". $. (.!. " J a#.-$2, a%-$7)
Solmon * T+e &'ara&ter()t(& e*uat(o# () ++()),()) =0
+ . =0
-- 5(.5+ 1)
.S2 +) +. =0
/(0(1(#2 /y ., to &ompare it 3(t' 52 +24(1)#5+(1)4= 0
2 .
5 +-s+- = 0
T T
(J )2
.
". (J )# = 4
..
=
r
#
a#1 24(1)#

=
15
..
4 = 2m
a) 4 () (#&rea)e1 6rom 0.7 to 0.8
l(t = 0.7, let9= 9t a#1 42 =0.8, 9 = 92
l ; J
1
=
42 = 2..J 9,..
..
2.J 9:.
r
"
C9ntrol System Engineering 7-101 T"me Response Analysis of
Control Systems
~J
=
~
0.3=~
..
l i 2
..
O . B KJ
..
K2 =
0.1406 KJ
So gai n shoul d be mul ti pl i ed by 0.1406.
b) ~ i s reduced from 0.6 to 0.4
~l = 0.6 l et!= !t and ~2 = 0.4 ! = !2
l i 1
1 1
..
=
2 .JK!J
~l = 2JKT2
lil
=
~
O . b=~
..
li2
..
0.4 !l
..
!2 =
2 .2 " !J
So ti me constant shoul d be mul ti pl i ed by 2 .2 ".
II. Example 7.43: For tile centro system shown
Show
i) Trajectory on which parabolic error constant K# is 40 ~ec-2
ii) Trajectory on wlziclz rol l = 40 rad/sec.
on the parameter plane K, - Kn' Tae K!J on " a"is and Kp on ya"is. (M.U. : Nov.-96)
Solution:
$%S)&%")
;;::
10%Kp +Kns)
52
Lim
"2 $%s)&%s' i )
Ka
;;::
"(()0
*i m
s2 .
l +%Kl +K,)s)
=
S()0
~
s~
40
--..
10 Kp
Kp
=
4 i s l i ne on /hi ch Ka =40 sec(2
r
"
Control System Engineering 7-102 Time Response Analysis of
Control Systems !.
ii} Characteristic equation is 1 + G(s}H(s) =0
lO(Kp+ Kosj
1 + 2 = 0
S
52 + 10KDs+ 10Kp = 0
Compare with
s2 + 2~rons+ ro! = 0
ro~ = 10Kp
40 = 3 . 1 6 2 . J K- ; :
ron= 31!2 " K#
.J K; = 12!4$
i
Kp = 1!0 is line on which ron= 40 ra%&sec
Kp
Kp= 160 for (110= 40 rad/sec
Kp= 4 for Ka= 4
~ - - - - - - - - - - - - - - - - - - - - - - - - - - - - - + K o
,,... Example 7.44: A seroomechanism is used to control the angular posuion 6 0 of a mass,
using 0 i as tire referettce input signal. The moment of inertia of the moving parts referred to
load is 200 kg '()2 and the motor torque at the load is 2 x 10 Nnr/rad of error. *he
damping torque coefficient referred to load is ! + 103 Nm/(radlsec). Find:
i) The step response of tire system to a step input of one radian.
ii) The natural freq e! l" # of ascillaiions, damped frequency of ascillations, peak time and
overshoot.
iii) Tire steady state error $hich e%ists $hen a step torque of ! &&& Nm is applied to t oad
shaft'
iv) The steady state error if the reference input i( a constant angular velocity of 1r.p.m.
Solution : *he open loop trans,er ,unction o, the s-stem with)
torque constant K* = 2 . 104 /m&ra% o, error
moment o, inertia " 0 = 20012 3 m
2
%ampin2 torque constant 4l ((3 + 103 /m&(ra%&sec) is
K
"(s# = *
s(s *l + 4l)
r

Control System Engineering 7-103 Time Response Analysis of


Control Systems
Assume lies) = 1
The system can be shown as below :
KT 0o(s)
s (s JL + BL)
K,
0..(5)
0; (5)
(
K T)
It
=
S2 + s(~ )+ (K 1 )
J I. J J
Substituting the alues o! K T, " I an# $"
%n(S) 1 00
= ..
&;(5)
S' + 1 55+ 1 00
.,
1 00 i.e( 1 0 )a#*sec .,
( 1 ) -
=
(l)n
=
n
an# 2l; (l)n 1 5 i.e. l;
1 5
= = '+l, = 0.-5
..
(l)# = (l)n ~ 1 -l;
'
=1 0~ 1 . (0:-5)' /./1 01 )a#*sec
i) The ste2 )es2onse !o) in2ut o! one )a#ian is
-~Wnt
&(((t) = 1 gsin(3l)" t+ ,)
l-l;2
whe)e 0
4( 5~ 6l74( 5~ l.3,-5)7 1
use )a#ian mo#e
=
tan l; - an 0-5
= 0.-'' )a#ians
/0(t) =
1 . 1 .51 1 e. -.5 t sin (/./1 01t+ 0.-''))a#ians
Tp
1 t
= .= 0.0-sec
Wd
r
"
- s . Control' System Engineering 7-104 Time Response Analysis of
Control Systems
and
= 2.83%
iii) Load torque is 1000 Nm i.e, T1. = 1000Nm
But
K... =
fad of error
. di TL 1000 0! d" . 2.8#0
error m ra tans = KT = $ 2 % 10$ = . ra &ans i.e,
i') T(e in)ut no* is ram) in)ut of ma+nitude 1 r.).m.
i.e,
7t
,-) = 1r.).m. = 30 red.se/
0s 1 re'o&ution i.e. 27t radians in #0 se/.
1. 2a+nitude of ram) in)ut = 0 = 3 0
K, = &im s 4,s)5,s) = &im s- KT
s... o .....0 s,s6. +BI.)
No*
= KT 7 2%&$ 8 =
19-3%&19
#.#7
e =
ss
0 ( : 0 )
K..-= #.#7 = 0.01:7 rad = = 0.;$
'. Example 70!1 A s ervomechanis m is des igned to keep a radar antenna pointed at a flying
aeroplane. If the aeroplane is flying with a velocity o f 600 Km/hr, at a range of 2 Km and
ttu maximum tracking error is to b e within 0.1<, determine tile required velocity error
coefficient. (ate)
Sol!tion " T#e &inear 'e&o/it= of aero)&ane is,
$ = %00 &m'#r
T(e ran+e of t(e aero)&ane is,
r = 2 &m
5en/e t(e an+u&ar 'e&o/it= of t(e aero)&ane is,.
$ %00
(l) = (=( = 300rad.(r =
r 2
300 1 dl
)%00 = = Iira se/
T#is is t(e ram) t=)e of in)ut to t(e s=stem.
r
*
Control System.,Engineering 7 -105 Time ResponseAnalysis of t,Jr
Control Systems
ret}
I
= wt =12 t
R(s) =
12s:!
The velocity error coefficient for system is to be calclate!, for t"e ram# in#t. $ence
for finite e%istence of velocity error coefficient, system mst be of TYPE 1.
K
G(s)H(s) = s(l+sf)
&it" $'s( ) 1, t"e system can be s"o&n as,
(s)
!
I
Now K"" ) lim s G(s)H(s# ) lim s. ** K$$$ ) K
s ..... C( + ..... (I s (%+s T)
,
&'' K&"ere , ) magnit!e of ram# in#t
(
e
'..
'1-12.
/
an! +is == )"%0 allo&e!
%
)"% ) %*K
%
K ) 1.2 - !egree
,s %, = /, t"e re0ire velocity error coefficient is 112I !egree.
2... E%am#le7.34: The open loop transfer function of a unity feedback system is given by,
G(s) ) I+,2,. Sketch the output of the feedback system for a unit step input. Assume that the
system is initially relaxed. (Gate)
Soltion : ,s system "as -nity fee!bac5,
$ts. ) 1
(s)
)
R(s)
G(.) G(s)
1+6's.(:-7's.) 1+6'5.
r
'
"J Control System Engineering 7-106 Time Response Analysis of
Control Systems
=
Dividing both numerator and denominator by e-2s,
C(s) . 1
R(s) = 1+ e2s
The system is excited by unit ste inut
ret) = 1 !or t"#
1
R(s) = -
s
$ubstituting in C(s)
C(s)
1 1
..
=
%&1+ e2s
.
C($)'(+ e2s)
1
=
-
s
C(s)+ C(s) e"ls
1
..
=
s
*o+ , -c (t+ T).
=
C(s) eTs
.. L-t - C(s) e2s.
=
c(t+ 2)
/ence ta0ing ,a(ace inverse o! e1uation (1),
c(t)+ c(t+ 2)
=
u(t)
+here u(t)
=
unit ste !unction
.. c(t+ 2)
=
u(t) - c(t)
... (1)
... (2)
To obtain the outut +ave!orm (et us obtain the va(ues o! c{t) !or various va(ues o! t.
*o+ at t = 2, c(t) has its initia( va(ue. 3sing initia( va(ue theorem,
. 1
c(#)= ,im cCt) = (im s C(s) = (im s- = 2
t-.o s-.- "-... $ ( 1+ e2")
c(#) = 2
$ubstituting t = 2 in e1uation (2),
c(2. = u(t)-c(#)= 1-2
= 1 as u(t)= 1 at t = 2
r
"
Control System Engineering 7 -107 Time Response Analysis of
Control Systems
Substitutingt =2 inequation (2),
c(4) = u(t)-c(2)=1-1=O
Substitutingt =4 in equation (2),
c(6) = u(t)-c(4)=1-0=1
Substitutingt = 6 in equation (2),
c(8) = u(t)- c(6)= 1-1 = 0 and so on.
Hence the sketch of the outut !esonse of the s"ste#fo! unit ste inut is as sho$n
in the figu!e.
c(t)
Input
---1.. ___ ;~I
o
2 4 6 8 10
u_.. Example 7.47: For the control system shown ill figure.
R(s) %
1) Determine
i) &'i for a unit step input
ii) e!OS for unit ramp input.
11) Calculate tire steQdy state value of the output when the input slUlft is held fied and a
sudden tor!ue "#. = 1 (# is applied. ().*. + ,ec. - -8)
r
"
Control System Engineering 7108 Time Response Analysis of
Control Systems
Solution : Assume TL as zero for [I]
i) To calculate p!
G(s) "
_--::-6 __ H(s) " 1
0#1$s2 + 0#%s
6
C(s) G(s) $&0#1$ s'0#%)
..
R&s)
=
"
1'G(s)H(s)
1'
6
s &0#1$$'0#%)
( 40
" "
0#1$$)'0#%$'( $)'(s'*0
Comparing t+e c+aracteristic e,uation -it+!.
S2 ')l/ron5+(,l)n
2
" 0
&0)1 " *0 Le! (l)n " (#2)*$
~ =
6
)&1)n
i#e# 1 "
0#*7*2
3o-!
-nxa44!
"#t-$44!12 4l55
18#*610
ii) 7or %n&t ramp input!
', = Lim $ 8&s)9&$) " Lim s# &0 1( 0%)#1 " (#(7
s.#o !.#o $ #$$'#
t
ess " : " 0#1$ ra;
<=
( 7or TL is t 3: m! assume R&s) as zero# >it+ R&s) zero! t+e system gets mo;ifie; as
1 C(s)
1: : : ##..:
7rom t+e figure -e can -rite#
1
8&s) "
0#1$$)+ 0#%$
r
)
Control System Engineering 7 -110 Time ResponseAnalysis of
. Control Systems
K K -at
c(t) = _ -_ e
a a
From graph, ~~ c(t) = 2
2 = K_ Ke- = K
a a a
From graph,
c(t) = 2 - 2 e- at
c(t) = 0.8 at t = 2
0.8 ::;;;; 2 - 2 e- 2a
0.2554 and K = 0.5108 o!"#ng $e get, a =
C(s}
=
R(s)
0.510%8
5&0.2554
'. Example 7.!" For what mlues of K is tire time constant of the closed loop system less
titan 0.2 sec. (#.$. " #ay%2&&')
~_ ,,' ) r"%(_) ( *
) ~2&1: +
C(s)
*
Sol,tion " +he c!o,ed !oop tran,-er -.nct#on #,,
C(s)
=
R(s)
'K - 'K .
2,&1 (K 1&(K
(K = 2,/1/(K= :::/ 0 -= --2--= ~1
1& 25& 1 1/1/(K5
2
2ompar#ng denom#nator $#th !&+,, the t#me con,tant #, 1/ (1( $h#ch m.,t 3e
ma4#m.m 0.2.
0.2
K 2: (
* * 0 Example 7.1&" A pair of complex conjllg'ltepoles in the s-plane is required to meet the
various specifications.For each specificatitt s!etch tire region ill tire s-plane ill which poles
should "e located
i); 2:: 0.#0#$ ro/ 2: 2 md%sec &positive damping'
ii) 0 ,; ,0.#0#$ ro/ ,2 rad%sec &positive damping'
iii' ( 0.)$ 1 roll 5 rad%sec &positive damping'
io' 0.) S2 S 0.#0#$ roll s 1 *ad%sec&positive and negative damping' (#.$." #ay%2&&')
_ 51 , $+ r 6,
Control System Engineering 7 -111 Time Response ~alysis of
Control Systems
Solution:
----.. --- .--. j 1.4144
-1.4144
-------. - j 1.4144
j2
-1.4144
-j2
-2.5
jO.866
------------- j 4.3301
-0.5
- - - - - - - - - - - - - - - j 4.3301
i) ~ "" 0.707 con = 2
O l i : : ; (l l , ~1-l ; 2
"" 1.4144 rad/sec
: : ; - 1.4144 j 1.4144
As ~ increases and eco!es "ni#$
o#% #%e &ol es con'er(e a# - 2 and
#%erea)#er eco!e real .
ii) ; = 0.707 and < q , = 2
.*. +ol es = - 1.4144 , j 1.4144
-"# )or 0 s l; s 0.707 #%e &ol es
re!ain co!&l e. and a# ; = 0 eco!es
&"re i!a(inar$. /or ron 0; 2, #%e
i!a(inar$ &ar# o) #%e &ol es !"s# e
l ess #%an 2. 0o re(ion ins-&l ane is as
s%o1n inadjacen# )i("re.
iii) ~ : s 0.5 and 1 0 ron 0 5
2e# l; = 0.5 and ron : : ; 1
#%en rod = 0.344
r.+ol es are - 0.5, j 0.344
/or ~ = 0.5and (5)n : : ; 5
then (5)d = 4.3301.
: . +ol es are - 2.5, j 4.3301
0o re(ion is as s%o1n in #%e
adjacen# )i("re.
678 , ,:r', oIj
7 -112 Time Response Analysis of
Control Systems
iv) 0.5 S ~ S 0.707 and O h S 5
For ~ = 0.5 and ron = 5 th e poles are - 2.5 j 4.3301.
For ~ = 0.707 and ron = 5 th e poles are - 3.535 j 3.536.
For positive and negative daping th e region is as sh o!n inth e "ig#re.
1 1 . + Example 7.51: Consider the ullity feedback control system whose open loop transfer
function is $%s) = s %1&~.1s) . Determine tile steady state error and its variation with time
when the input is r(t) =1+ t + t2
Solution: For stead' state error e5-~($%s))%s) = s %1&~.1s)*
%+.,.& -e..-2003)
/p = 0i $%s))%s) = -
5-.0
0i s$%s))%s)(= 0i
50
/v
= s---10
s---1O
s2s%l1O .ls) = 50
~a
=
0iO
s2$%s))%s) = 0
s---1
3n th e inp#t ret) = rl%t)1r2%t) 1 r3(t)
r1%t)
=
1 i.e. 43 = 1
r2%t)
=
t i.e. 42 = 1
r3%t)
5 1 5
i.e. 43 = 2 =
t- = -62t-
2
... step
... rap
... para7oli.
r
"
Control System Engineering 7-113 Time Respons~A~alysisof
Control Systems
A.. 1
e= - - < 2 = ~ = 50'
For obtaining variation of e"i!'with time use general error series method.
1 1
F. s! = 1+ "s!#s! = 1+ 50
51$ 0.1s)
es~ t! = %o rt! + %. r't!$ ~ ~ r"t!$ ....
0.1s& + s
= ----= ----
0.152 + s+ 50
0.1 '&+ s
% - (im F. s! = (im
o - 5--$0 s--$) 0.15& $ s$ 50 = 0
(
. d F* 5! (' +0.ls& $ s$ 50! 0.&s$ 1!- 0.15& $ 5! 0.&s$ * !,
lin - - = 1m
%l = s--$) ds 5--$0 0.ls&$5$50!&
(im + 50 0.&5$ 1) = !2 "# = - !- = 0.0&
= 5--$0 0.15& $ s$ 50!& 50!& 50
. +.0.15& $ 5$ 50!& 10!- 10 s$ ')!&! 0.15& $ s$ ')!0.&s$ * !,
- (im $ /
- 5--$0 0.1'-$5$')!
%& = 50!& %1&)- 5)!&!')! %1) = 0.003&
50!/
ret! = 1+ t + t&0 r't! = 1 + &t 0 r"t! = &
0.003&
.1 e23st! = 0.0& 1$ &t!$-&- &! = ).0/t$ 0.0&3&
r
'
Control Syst~";'Engineering
.!
7 -114 Time Response Analysis of
Control Systems
'.. Example 7.52: In the block dillgram given below, determine fire output rate factor that
yields a response fa a step input command having a maximum overshoot of 10%.
(M.U. : Dec.-20(3)
Solution : Solve the internal minor feedback loop
2
1+--xsK
s(s !2)
2
"# """"2___;;;"'_-
s +2s+2sK
2
s(s!2)
C(s)
C(s)
$t%& a unity feedback &'&tem (ith )(&) = , 10
&- +2&! 2&*
]0
C(s)
+
R(s)
S2 !2&!2&*
1! ,0
S2 +2&! 2&*
10
=-::-------
S2 ! S (2! 2*)! 10
-omparin. denominator (ith 52 +2l/(n&!(0
w; + ,0 i.e (n + .J 1 + 3.11222 rad3&ec
4::2+2K=1+K
2l/(n =2 +2 * i.e. 0 2#..456 .J !
.". (1)
... (2)
)iven
Mp +
10030
.. 10
+
100 e-*03b-f.2
Solvin. for #;, ~=0."711
U&in. (2) 0."711
#+K
+
J !
.. K
+
0.81722 ... 69tp9t rate factor
$
"
Contt:ol System Engineering 7-115 Time Response Analysis of
Control Systems
'. Example 7.53 : A feedback control system is represented by the differential equation,
d2c de
-2+ 6.4-d = 160 e where e = r - 0.4 c.
dt t
The ariable c denotes output. !ind the alue of tire dampin" ratio and what information
does this COlrvey about the trans#t behaiour o f the system.
(M.U. : Dec-2003, Dec.-2006)
Solution : Substituting value of e,
d2e + 6.4de = 60 (! -0.4 e)
dt2 dt
dc de
:. -2+ 6.4dt +64c = 60 !
dt .
"a#ing $a%lace t!ansfo!& of bot' t'e sides and neglecting initial conditions,
5 ((s))6.4s ((s)) 64((s) = 60 *(s)
((+) .60
*(s) ! 5 ) 6.4 s) 64
... "!ansfe! function
(o&%a!ing deno&inato! ,it' S2) 2-!on s)!o-,
!o2
=
64 and 2-!on= 6.4
n
i.e.
.ln =
/!ad0sec. and
- = 26-4/= 0.4.
"#t$
1s t'e da&%ing !atio l% is less t'an unit2,
t'e s2ste& is unde!da&%ed and ,ill %!oduce
t'e ti&e !es%onse ,it' da&%ed oscillato!2
t!ansients fo! t'e ste% in%ut as s'o,n in t'e
figu!e.
'.. Example 7.5&: 'or a second order undamped system, show that c$t% = 1 - cos !o,l.
(M.U. : Ma2-20(4)
Solution : 3o! an unde!da&%ed second o!de! s2ste&, t'e out%ut is given b2,
-(mnt
e(t) ! '-sin (!odt)4) ... ()
1-l;2
r
)
c(t) = 1- cos (J)n t ... Proved
Control SystemEngineering 7116 Time ResponseAnalysis of~
Control System~
For an undamped system, ~ = O.
(J)d = (J)n and 0 = tan - 1 .,.; = ; rad = 90.
!s"n# "n (1),
'... Example 7.55 For the block diagram shown below first obtain steady state output and
then obtain steady state error. ($.!. % $ay-&0$)
&0.000
C(s)
(1' ()(1' 0.1()(1' 0.&s)
S l!ti " t)e e" t #( ) &0000 *(s)= $
o on rom e #+ven sys em% s = (,'s) (1' 0.1s) (1' 0.& s)-
&0000
C(s)
=
R(s)
.(s) ($'s) ($' 0.1/) (1' 0.& s)
=
1' .(s) 1' &0000
(1' /)(1' 0.1s) (1' 0.& s)
C(s)
=
R(s)
&0000 & R() 1000
(1' ()(1' 0.1()(1' 0.& s) ' &0000 an ( = -s-
(C(s) ) * &0000 0 x 1000 = &0000000
1. - (1' s)(1' O.+s) (1' 0.&s) ' &0000 s s 2(1' () (1' 0.1s) (1' 0.&/) ' &0000J
+
, C() +' -. &0000000 3
"m s 4 = "m s5 --= -~--%-.,......-= --%%%--%--= ---%--= -%--= =
6s... = s~ 0 S~ 0 s 2(1' s) (1' 0.1s)(1 ' 0.& s)' &0000J
.
C..
= &0000000 =/// /5
1' &0000 1
.... 4teady state output
e/4 = 7e8eren9e "nput - 6 = 1000 -; 999.9/ = 0.0/
r
.
-
Control System Engineering
"
7 -117 Time Response Analysis of
Control Systems
- -
).. Example 7.56: The open loop transfer junction of a 1lnity feedback control system is
G(s) = s (:~010)and input applied ;s r(t) = a + bt + c~2. Obtain generalised error series.
(M.U. : May-2004)
Soltion : For generalised error series,
!" (5) =
."
1 1 s"+ 10s
-#-- $ ---- $-----
1+ G(s)(s) 1+ 100 52 + 10 5+ 100
!(5+ %&)
-
-
."
"i# F$(s) = "i# 5- + 105 = 0
s-+% 5-+0 s2 + 105+ 100
-
-
'
( dFl (5) '(
r# ds $ rm
s&o s-e %
(!2 + 10s+ 100) )*s+ 10)- (!2 + 105)(25+ 10)
(!2 + 10s+ 100)2
-
-
"i# 100 (25+ 10) ' $ "i#
5-+ 0 (52 + 105+ 100)2 s-+ 0
200 s+ 1000
-
-
. +*()(s) . +
"*# ds2 $ "*# ds
s#, s#,
200++ 1000
(52 + 105+ 100)2
-
-
"i#
5-)0
(52 + 105+ 100)2(200)- (200s+ 1000) (2) (52 + 10 s+ 100) (2s+ 10)
$ 0
(52 + 105+ 100)4

,,
-
-
-
-o r($)+ -l r.($)+ 2&r"($) $ 0+ 0.1 /0+ c $1+ 0
es, $ 0.1 )-+ e$1

... error s2nes

.. Example 7.57: control design has to meet follo!ing specifications: damping ratio of
0."# natural fre$uency of / T, rad%sec and steady state error of 10 3. &or Q unity feedback
- )s+a)
control syste' 1( 0)s) $ * . find )alues oj *# a and $4 .
(5+a) (M.U. : May-20M)
Soltion : T1e 2losed loo5 $rans(er (6n2$ion is,
2)s+a)
C)s) 0)s) (!+&)2 2)s+a)
- - ----------
- - -
7(s) 1+ G(s)(s) 1+- (s+a) 5* +s (28+ -)+(9a +52)
(!+5)2
, , &. , ." , (":l. .....
3 3 3 3 43 3 3 3 .43 3 2 ,
" .
3
-
Control System Engineering 7118 Time Response Analysis of. t , ! ,
Control Systems '
Comparing denominator with S2 + 2~(l)ns+(I)!,
fJ)2
Ka+p2

(l)n = ~ Ka +13 2
.. (!) - I . e .
-
n
"
"
2~ (1)n
# 2!3 +"

2#+"
(2) I . e .
$
-
2 x J Ka +13 2
"
"
$or %nit step inp%t, & # 1
%im &(s)'(s) = %im K(s+a) # K~
"p = ('+13 )2 ~ -
'- +( s- +o

A ! p2
Cs)* # ~ # Ka # Ko
~2
.. (()
+%t el%~is 1((, ( hen-e,
~2
(1 # Ka
ie Ka #1(~ 2 (.)
$rom (1),
JW
~ 1(/02 +13 2
) !(2 1(

~ = (1'3 . $
Ie # 11
I . e .
$
$rom (2), ('
22 ((1'3 .)+ "
ie K = 12''2
$
$
2 x J 342 ((1'3 .)2 + ((1'3 .)2
$rom (.), 12''2 2 a = 342 (*.+,(-)2 ie e2#72.15
. . d2 yll) 8dl/(t)
E.ample /.,0 1 A control system I6 descnbed by - ,+ d + 2,ylt) = ,* x tt).
dt- t
Eval~te the output reSpOJ1Se y(t) and its peak. value for Q input x lt) #2' JI(t).
(78 ) 7a9- 2((., :e-- 2((5)
Sol2tion 1 ;a<ing =ap3a-e trans/orm of >oth sides,
~
'?3(s)+ 0 s 3(s)+ 2, 3(s) #,* 4(s)
9es) '(
$
-
4es)

,* 2,
$$$$#
,2+8'+2' 26 ,2+8'+2'
Comparing denominator with S2 +2~ (0)n s+ro~ ,
=e, (1)n = ' rad, se-
ie, l; = (8

,
~ (?)3 ?,
" " " " 5" " " " .5" " -

"
Control System Engineering
J
7 -119
. .
Time Response Analysis of
Control Systems
Hence the output response iSI

= 2{1-1. 6b7e-41 sin(3t+ 3b.86!"#


$(t# = %& 3.3334 e- 4 t sin (3t + 36. 860)
'
. . . Output response
I the input is 2. ! u"t) #e$ step o 2. ! inste%& o (nity then the response ch%n'es %s$
y(t# = 2. !( "2-3. 33. Ie-4t sin "3t)36. *69+,) = !-*. 332!e-. J - sin"3t )36. *69+)
.he pe%/ 0%1ue is %tt%ine& %t t = Tp = 1t ) ..* * .3 sec
ro+

n
2"t)It-c%3 = !- *. 332! e- . . )"-3 sin " 3(i + 0. 6434 r%&)
= !. 07!* . . . 4sc r%&i%n 5o&e to c%1cu1%te sin
,. E-ample ../0 For the signal flow graplt shoum beloto obtain the impulse response.
"6. 4. 7 3{%2-20048
R(s# 1
1
-
s
1
3 1 C(s#
1
-
s
-12
Sol(tion 0 .he #%p1%ce tr%nsor5 o i5pu1se respon. c9eis the tr%nser unction o the 'i0en
s2ste5.
4sin' 6%son-s '%in or5u1% or the 'i0en si'n%1 1o: 'r%ph$
113
.1 ) 1(-(-(3(1= 2
S S S
1 1
.. $ ) 1( - ( 1( 1 = -
s s
1
-
s
1
-
s
1
&
!
On12 t:o in&i0i&u%1
1oops :ithout %n2
nontouchin' cornbin%tion.
-7
1
& 7
1&&&
S
3. ";71. ;$.
I I I I 2I I I I .2I I c

-
Control System Engineering 7 -121 Time Response Analysis of
Control Systems
and
As ~< 1, the system is underdamped in nature,
I'" Example 7.61 : Find value of Kl and K2 so as to obtain peak time = 2 sec, and settling
time = sic.
50
s!s " 2)
C!s#
R!s# "
(M.V. : May-200S, May-2007)
Solution: Solving inner minor eed!a"# loop,
50
s!s " $#
0$ ~%&&%
1
%#! s&
" s!s " $# 2
C!s#
50
R!s#

''
C!s#
R!s#
'
'
5$5(2)( % &$s % )l
- - - - - - - - - - - - - = - - - - - - - - ~ - - - - - - -
1 50)1 $ ( s $2 ( 50 )$# ( 50 )1
( 5$*( 2)( 50 &25
+omparing denominator ,ith $ " 2~$1)ns($-).,
ro. = 50)% i%e% ron =.J 5 0 K .
2l&ron = 2 "50)2 /e% ~ 0 2 (S1)2
2%-S1)-
...!1#
%%%$2)
2iven:
Tp
' 2s e c and 3,0 5 se"
'
4o,,
Tp
7t
0 2
and 3s = (
5
=5 i.e% ron
0%*
'
'
- -
$t)d
( ron


'
'
i%e%
' ' 1 :11.1 1 67 1 $8:l% 8,6
I I I I )I I I I .)I I " '
' 1
Control System Engineering 7 -122
Time Response Analysis of _.
Control Systems

3.8553 i.e, 4.8553 ~


2
= 1
~2
0.20595

~ = 0.4538 and ron =1.76277 radl sec

- I . e . II
-
.J 5OKI
Using (1), 1.7627

x, = 0.06214 - i.e,
-
sing (2, 0.4538 -
2!50"2
i.e. K2 = - 0.008
-
2xJSOx 0.0621
#$.. E%ample &.'2( A feedback system wllich uses a rate feedback controller is shown in the
figure.
)otor
* C+s#
Amplifier
K! *- ...
s +s !2
R+s#
- -
a) For K! = 10, in absence of derivative feedback (Ko = 0 deterine the da!ing ratio lind
natural fre"uenc# of oscillations. Also find the s.s error for unit ra! in!ut.
b) $eterine the constant Ko if the da!ing factor re"uired is 0.6% witll &'( = 1.0. )ill. this
oalue of "o$ deterine ss error for unit ra! in!ut. (M.V. : "a#-2006
Sol,tion ( i $i%& Ko =0- %&e s#s%e' is,
KA
C+s# s+s! 2#
-
-
((s 1) K!
s(s)2
R+s#
*
K! .,* * ,.
s+s ! 2#
+o',aring deno'ina%or -i%&
s2 ! 2~(lns)(./,
i.e. ron = ..0K1 = 23O = 3.16 radl sec
i.e. ~ = ~ = 0.3162
2x 10
4(s5(s =
KA
= ~---
s(s) 2 6(5) 2
10

7i' 10
K, = 5-)0 s4(s5(s = 2 = 5

A *
-=-=0.2
K8 5
...9or :ni% ra', ! =1
1 ;;l..1 1 .< 1 (=;l. =,.
I I I I _ I I I I ._ I I c
1

I
I
Control System Engineering 7-123
. .
Time Response Analysis of
Control Systems
ii) With K o , the system beco mes,

Minor loop
R(s)
C(s) R(s)
- - - -
1
-
s(s + 2)
10

C(s)
R(s)
S2 + s(2+K o >
1+ 10
s2 + S (2+ K o >
=- - - - - -
52 + S(2+ K o )+ 10
10
-
-
. .

i. e. r o n =Jf r !"# sec


!n "
$%t &i'en
J( - 0) h 0. ) * 2+ I o
+ -. en ce, 2M

* 1. 7,-7

!(s)"(s) *
10 10
- - - - - - =- - - -
52 + 5(2+ 1. 7,-,) 5(S+ #$%&'&)

.im 10
=s+ 0 s/(s)0(s) =3. 7,-, =2. )351

A 1
ess * + =2. )351 * 0. 37,5

1
1
5(5 + 2)
S K q
1+ s(s +2)
I
C(s)

Control System Engineering 7-124 Time Response Analysis of


Control Systems
Review Questions
1. What is tile d~'Jerence between steady state response mId transient response of a control system?
2. Define steady stafe response and steady state error.
3. How steadystate error of a control system is def ennined? Ho it can be reduced ?
4. State how type of a control system is determined? How it affects the steady state error of a
system?
! Deri"e the e#pressions for static error coefficients? How these coefficients are lise$ul in determinin%
steady state error? State the limitations of static error coefficient method .
.
&. How dampin% ratio affects the time response of a second order system?
'. Show the locus 0/ closed loop poles of a second order system as ~ is "aried from 0 to DO.
(. Define t)*e fo11ou*in% systems s+etchin% their output wa"eform for , unit step input -
i) )lnderdamped system ii) .nd/mped system
iii) Ooerdamped system io) 0ritically d)l'*rped system.
1. With a neat s+etch e#plain all the time domain specifications.
10. Derive the relaf ionslzip between maximum oversloof and dampin! ratio of a second order systeln.
Draw the %raph or D"ershoot "ersus dampin% ratio
11. Deri"e the e#pressions for pea+ time and rise time interms of 2 a11d (t)n for a second order control
system!
12. Discuss the ad"a*rtll%es of feedbac+ in a control system.
13. /n !nity feedbac+ system has the loop trans$tr function
K
a4 = ( )
5 s+ a
If its time response is to ha"e "i) an o"ershoot of less than 5% and "ii) seulin% time Ilot e#ceedin%
4 seconds I determine silitable "alues for K and Q. lAnse : K = 2;a ~ 2]
14. / position control system is stabilised by means of acceleration feedbac+. If the system has a
moment of inertia of 10-5 Kg 2; "iscous frictional !!!!!1"~of !0! 5m6rad 7 se#-$ and the motor
tor8ue is %ioen by
% "4 # 10-$ K &% 00 ' - m
n! e+ &t2
where
K &% '0 " /cceleration fi7eedbac+
&t2
i) Draw its bloc+ dia%ram.
(() Write the system D.9.
iii) Determine : f;< critical dampin% and
i"= Determine the steady state error e when the input is a constant "elocity of 20 rpm.
(Ans. : K = 2.)4* 10-) se+I 3#$
, 1 :11.1
) )
! )*. ( -:lo ..
) ) +) ) ) ) .+) ) # ,

Control System Engineering 7-125 Time Response Analysis of


Control Systems
15) Determine the step, ramp, and paral ' o l ic erro r c o nst' l nts o f the ! o I I O l v i,rg Ilnily - feedl 1ac k c o ntrd
systems. The o pen l o o p transfer func t io ns are g il l en
a) G ( s )
1 0 0 0
-
-
(1+ 0.15) (1+ lOs)
b) G ( s )
100
-
-
S(52 + 105+ 100)
c) G(s)
100
-
-
5(1+ 0.15) (1+ 0.55)
d) G ( s )
100
-
-
52(52 + 105+ 100)
e) G ( s )
1000
-
-
5(5+ 10) (5+ 100)
f J
G ( s )
K(l +25) (1 + 4s)
-
-
S2(S2 + s+ 1)
IAns. : a) Kp =1000 K,..=0 K., =0
b) K., = 00 K, =1 Ka= = 0
!) K., = 00 K, = K Ka = 0
d) K., = 00 "= 00 K.1 = 1
e) K., = 00 K, =ao Kd =0
f) K., = 00 , , = 00 K.1=1 KJ
16. A to phase ac . serv o meter hao ing (l to r!ue c o nstant o f "."#5 $,n! % c o ntro l s It po sitio n l o ad
thro ug l l a g ear ratio o f 1"&1. The effec tittt mo ment o f inertia t' ( ,1 c o )effic ient o f o isc o us fric tio n
referred to l o ad side are ".*5 +g ) m2 and 1." Ntn(rad/sec). TIlt syneJ ITo (n,tgslipJ t"9T detec to r
pro duc es an erro r sig nal o f ".1 % per deg ree l..'TrOT in misal ig nment. Dev el o p tI,e ,l o c k diag ram
representatio n o f the c o ntro l systenl and there fro m o ,tain the transfer func tio n.
o - .al c ul ate the natura! fre!/l nrc y o f o sc il l atio ns and damping ratio .
,) If damping is to ,e made c ritic al using a deril 1ativ e erro r c o ntro l , determine the time c o nstant o f
the phase adv anc e c irc uit. (Ans.: (l)n # .21 !adls"c , #= 0.62 I T$ = 0.24 s"c$
!. "or the c o ntro l system sho um %n fig ure.
R(s) +
-
-
f -
1
C(s)
..
6
-
..
... 2
0.15s + 0.&5
J
...
-
."-
I l ) Determine (i) "'#, fo r a uni& step input (ii) es.0fo T unit ramp inpu!
,) .al c ul ate the steady state v al ue o f the o utput i$t%en the input shaft 's hel d fi1ed and sudden
to r!ue Tl = 1$m is appl ied (Ans.: )* -1+.&0,0, "ss = 0.15 !ad, -ss = - 0.166 !ad$
. . ".
% %
.... (.:l/ , .
% % &% % % % .&% % c
.
I
I
Control System Engineering 7-126 Time Response Analysis of
Control Systems
J
I
"
~
r
I
18. The open loop transfer function of II unity feedback control system is given by
K
G(s) =s{l + sT)
The input to the system is 1R.P.M. lind the steady slate error being 0.25.. Calculate the natural
frequency of oscilllltions if the system is critically damped. (Ans. : 48 od/s
19. The block diagram of a position control system witl, velocity feedback is shown in figure. Determine
the value of a so that the step response has maximum overshoot of 10 percent. WIUlt is the steady
state error?
R(s C!s
10
s (s + "
1+#$s
[Ans. : a = 0.179!ess =0"
!. "he closed loop transfer function of a second order system u#itll proportional plu$ error%rate
feedback is given by ~~s~= ~(s +% where parameters & and % are ad'ustable
s s # 4$# 8
a( If rit) = t) determine the values of & and & sud* thai the steady state error is +ero.
b( For these txdue$ of & and %! determine the steady state error to a unit parabolic input; i.e.
rt) = - 1,l.t-
lAnse : (a K =4! % =2 (&) 0.2$1
1. .or the system shown in 'igrlre determine tlte steady state error for
R(s + E(s
'!s
0.4
C(s ((."s + 1
((.)s + 1
2
' + *s + +
,otor
-
Amplifier Amplifier
.
i) / unit step input
ii) / unit step #0elocity input
iii) / unit step acceleration input (Ans. : (i) 0.047$ (ii) 00 (iii) oa "
. "he block diagram of a fire control system t()ith unity feedback is described i1 figJ 1re. 2sing
generalised error series determin$ the steady slate error of the system lo!ren the systenr input is
/reamplifier
and 0emod1lator 2et3or4
/reamplifier
,otor and load
+
s
C(
..
10
s+1
$
"(( 0.01$
.. ... .. .. ..
-
.. ...
- - -
...
s+) () + 2) (s + 1
-~
*
Amplidyne
+(
s
. . ::lI..
I I
..,. (,:lo !.
I I 5I I I I .5I I - *
" .
6
Control System Engineering 7 -127 Time~~spons.~"alysisof
Control Systems

3t2
r(t) = 5 + -It + ?
-
lAnse : 0.5 +0.373t +0.09315t2)
23. The block diagram of a simple servo system is shoum in !ollou1ing figure. Determine tI re
cnaracteristic equation of tI le system. Hence calculate the undamped frequency of oscillations,
,illnlpillg ratio, damping factor, maimum overshoot first undershoot, time intervals after !hich
maimum a"#d minimum $%%#lT&, settling time and the number of cycles completed before the output
is settled toiihin 2% of the final talue. The input to the system is a unit step.
R(
E( )
)
+
s
s)
1.2
C(s
20
-
-
.. .-
- ..
-
...
5 (s + 1) (0.25 + 1)
-
- .
5
-
6
[Ans. : 51 + 65 + 25 =0; (1)n = 5 ra!se";
~= 0.6; ro = #; $% =9.5&; 0.9#"0;
0.7'5 se"; 1.57 se" i 1.33 see 0.'5 "y"les)
2'. The open loop transferfunction of a unity feedback control system I & given by
(
G(s) = s(s) * 1)
i) !y !hat factor theamplifier gain ( should be multiplIed so that the danrping ratio is increased
from 0.2 to O.B.
;i) By wlUlt factor fl,e time constant ) sllould be multiplied so that the thI )"ping ratio is reduced from
*.+ to *.3.
iii) ,or the system overshoot of tire unit step response 10 reduce from 6()+, to 2*-. &l!! that
T"-1
)(: - 1 = #3.22 !here (l and (. are the values of ( for +*- lind 2**.'
1
.Ans. : 16# #/
2/. %alculate static error coefficients for a unity feedback system !ith 01s2 = 0(0~ 6)$ If input given is
r(t) .. + 3t determine steady stale error. ,or the above system of ess is to be reduced to 3*- of
eisting value, !hat u"ould be the percentage chilnge in gain4
.Ans. : I%p = 001 (1. =21 ess = 1.519000!0 "2an3e is nee)
26. 0iven5
6hat7 is the steady st!!teerror"
(Ans. : 0.1721
R=O +
4
&
-
T' = (.1)2
. . ~.
I I
5 555 (":l. 1.
I I *I I I I .*I I " 5

Control System Engineering 7-128 Time Response Analysis of
Control Systems
.i ,\.
27. Given:
E
2
R
100
c
(0.05 s+ 1) (0.05 s+ 1)

s
1
+0.25
-
The input is a ramp, R 1.St . !",af ;s the steady-state error?
28. Gioen :
[Ans. : 0.0151
T(s)
R c
If R = 0, uJJuztis the sttady - state error?
29. Find error oe!ii"#""ts for ti"e $ioen ""ni"y feed%a& system haoin$
a 4(52 + l? s + 100)
S(5 + #) (s$ + 2s + 10)
lAnse :-11
(Ans.co, , 0)
'(. !etermine error oeffiients for the system haoin$
(s !2)
"(s)#(s) 5(1 + 0.55) (1 + 0.2s)
(Ans. 00, 2, 0)
'1. Find error oeffiients for a system I"a)in$ "(s) =
10
and steady state
52 (1+ s)
error i*input to the system;s $o+ $. t! $% t2% (Ans.:oo,oo, 10, $%&l')
'2. + syste"n has 40( overs",oot and re-uires a sett"in$ time of & sees .hen $iven (I step input. The
steady sta"e error 2(. !etermine fire transfer funtion of the seond order system. +"so find rise
time and pea& time. ().*.: +o,.--4)
(Ans.: 0.540- sec, 0.-1./ se')
''. + standard seond order system has /(0 overshoot and sett"in$ time of / seonds- !etermine
01, l2, 1( ( $n3 t; ().*.: )an.-*&) +
(Ans., ..1-, 0.215&,0.5/.-7, 1.2-.
1 ::l4.1
+ +
1 151 (1:l. ,4
+ + -+ + + + .-+ + c
1
Control System Engineering
7 -129 Time Response Analysis of
Control Systems
34. The open loop transfer function of a Ullity feedback control system is given by
K
G(s) = ~(5 T+ 1)
iJ By fuha' factor the amplifier gain K S I 'O llld be multiplied so that the damping ratio is increased
from 0.2 to 0..
ii) By !hat factor the time constant T should be multiplied so tllnt damping ratio is reduced from 0."
to 0.3.
(M.U. : May-97)
(Ans. : V16, 4)
3#. TI $e block diagram of a speed control system is sho!n in the figure. The disturbance is present in
its %onoard path.
-
-
V1(s) : ~!""-# 1
. ~ G1(s)= s+ 2
$(s)
...:
-
-
~ - - - - - - - - - - - - - - ~ + K b ~ - - - - - - - - - - - - - ~
&etermine the sensitivity of the system
i) ~% ii) ~%
!here Md (5) is the ratio of (1)(5) and Td(s).
/ &
\ A. '( + 35+ 2 K Kb
ns. 2 ' 2 ( )
! 5 + 35+ (K Kb + 2) 5 + 35+ K Kb ' J
3". The block diagram of a position control system is sho!n in the figure. &etermine the sensitivity o f
closed loop transfer function T(s) !ith respect to G(s) and 1-1(5) fO T (l) " 1#ad / s$%.
(Ans. : &.&29! 1.&2)
R(s)
-
-
C(s)
...
1&
)(s) " s (s + 1)
'''''--t *(s) = '
3'. ( fly!heel is driven by an electric motor. I t is controlled I lutomatically by a mcroement of
handtoheel and follo!s its m&l) ement. The effective moment of inertial of the fly!heel is
) #0 +g- (
2)
The tor*ue developed by the motor is 2400 +m,rad o f misalignment bet!een fly!heel
and hand!heel. -iscous friction of the system is "00 *( #ad-1 s$%. If the I u2nd!heel is suddenly
moved through ,rad from rest$ determine f-e e.pression + o# the subse*uent angular position of the
fly!heel. / (Ans. : 1.&5,1- $- 2 t %os3.5 t + &.-sin 3.5 t) )
&
) 1 :11.1
& &
1 . . .
!.:/ o .:: # # "# # % )
.#
Control System Engin_rlng 7-130 Time Response Analysis of
Control Systems
38. A servo mecMnism shown in the figure has moment o f inertia of the moving paris referred to load
slulft as 150 kg - m
2
The motor load torque is 4 x 104 Nm/rad of e"!. The dnmping torque
coefficient referred to load slulft is 4 x tel Nm/rlld/sec.
e~s)

-
-
2
Js +Bs
Find :
a" !he step response if input is step t#pe 0/ one radian.. Also determine rise time$ pea% time and p eak
uoershoot.
&" The '.'. error if input is constant angular velocit# of 1r.p.m .

c) The '.'. error which e(ists when II t)rque of 1200Nm is applied to the load shaft.
(An!"1- 1#7$e-13#4t in (%#3&t' 0#(1)!0#(0! 1#7r)
000

1 ""l)#1 1 )* 1 (+"l# +!)


I I I I _ I I I I ._ I I c

1
Stability Analysis
8.1 Background
As we have seen earlier that every system, for small amount of time has to pass
through a transient period. Now whether system will reach to its intended steady state
after passing through transients or not? The 311..c;!rto this "uestion means to define
whether system is stable or unstable. This is stability analysis.
#or e$ample, a meter is connected in a system to measure a particular parameter.
%efore showing the final reading, the pointer of meter will pass through the transients. The
final reading is the steady state of the pointer. %ut during transients, it is possible that the
pointer may become stationary due to certain problems in the moving system of that
meter .. So to achieve steady state, the system must pass through the transient period
successfully.
Key &oint' Tile analysis of tohether the gioen systell1 call reach steady state; IJQssing
through tile transients sZlccessftllly is called Stability A"alysis of the systet".
8.2 Concept of Stability
!onsider a system i.e. a deep container with an ob(ect placed inside it as shown in the
#ig. ).1.
Force
F
(a)
#ig. ).1
(8 1)
..... l! " # # .
r > ; ~ ...
: . ti' : .. J
....,.. ....,..
$
% $

..& . ...*......
$
~ I ~.
' !
( (
$ $
./.......r".....
$ $ $
# ( . .
..... + .... , ,.e- *
+ .
- .. * ..
.... .
.
*
(b)
. . ).
! !
/ /// 0- 'lo -
! ! *! ! ! ! .*! ! c /

Control System Engineering 82 I Stability Analysis
- - _ .
Now if we apply a force to take out the object, as the depth of the ,c~ntainer _I e; more, it
will oscillate and will settle down again at its original position.
I f we assume that the force reuired
to take out the object tends to infinity
i. e. always object will oscillate when
force is applied and will settle down but
will not come out, such a system is
called absolutely stable system. No
change in parameters, disturbances,
changes the output. !s against this,
consider a container which is pointed
one, on which we tty to keep a circular
objects shown in the "ig. #. $.
I n this case object will fall down without any e%ternal application of force. &o if we try
to keep the circular object, we will always fail to do so. &uch system is called unstable
system.

'f

(a) (b)
"ig. #. $
'hile in certain cases the container is shallow then there e%ists a critical (alue of force
for which object will come out of container.
I
o
F F F
F) Fcritical
"ig. #. *
!s long as " ) "criticat, object regains I ts original position but if " > "+,lti+l. - object will
corne out. &tability depends on certain conditions of the system hence system is called
conditionally stable system.
I
,here are few systems e. g. . /endulum where system keeps on oscillating when
certain force is applied. &uch systems arc neither stable nor unstable and hence called
critically stable or ",arginal'y stable systems.
Now let us see on which factors e%actly the stability depends in a control system.
. . ~.
I I
01.lo ,.
I I _I I I I . _I I c


. .

. ~
. . . . .
Control System Engineering 83 Stability Analysis
. . . -
8. 3 Stability of Control Systems
The stability of a linear closed loop system can be deter mined from the locations of
close~~loop poles in the s-plane.
For example: If system has closed loop T. F.
C(s) 10
-
-
R(s) (s+2) (!"#
$et %s find o%t& o%tp%t response for %nit step inp%t .
+. '(s# = l(s

)(s# =
10 !C
- - - - - = + +- -
(!*#(!"# s!* !"

C(s) 10
1(+ 1(" 1(+
-
- +
-
S !* s!"
C(s)
1. * 25 1. *

-
- + -
s s! * s! "
. . . Findin, the partial fractions
c(t# = 1. * - *. e-*t +1. *e- "t =)-S +)t (t)
Steadystate Transient
As closed loop poles are located in left half of s-plane& in o%tp%t response there are
exponential terms .ith ne,ati/e indices i&e& e-*t and e- "t
0o. as t - +00 both exponential terms .ill approach to 1ero and o%tp%t .ill be steady
state o%tp%t .

i. e. as t ~ co& c& (t) = 0


Transient o%tp%t = 0
S%ch systems are called absol"tely stable systems.
0o. transient terms are exponential terms .ith ne,ati/e index beca%se closed loop
poles are located in left half of s-plane. For the abo/e system %nder consideration& the
closed loop poles are s =- * and s 2 - " and the ne,ati/e indices of exponential terms are
also - * and - ".
#ey 3oint: TI'!lS if closed loop poles are located in left half, exponential indices itl output
are negative. And if indices are negative, exponential transient terms will vanish when t ~00.
0o. let %s ha/e a system .ith one closed loop pole located inri,ht half of s-plane.
)(s# 10
-
-
R(s) (s- 2) (s+")
Find o%t %nit step response of abo/e system.
10 !C
C(s) 2 s(-*#(~"#&~ s$ !-*! !"
.
1 % 11. 1 1 I4 1 (5:l. 5&I
& & & & '& & & & . '& & c
1
Control System Engineering 8-4 Stability Analysis
.
C(s)
1.25 0.833 0.416
= - + +--
s s-2 s+4

c{t) = - 1.25 + 0-833 e+2t + 0.416 e-4t


Now due to pole located in rigt al!" tere is one e#ponential ter$ wit positi%e
inde# in transient output.
wile &'c()t* = - 1.. 25
,.
t c(t)
0 0
1 + 4.+1
2 + 44.23
4 + 2481.88
00 00
,s it is clear !ro$ te table tat instead o! approacing to steady state %alue as t ' co"
due to e#ponential ter$ wit positi%e inde#" transients go on increasing in a$plitude. So
suc syste$ is said to be unstable"
Key -oint. So it is clear that if any of the closed loop poles lie in righ! half of s-plane, then
it gives the exponential term of positive index and due to that, transient response of increasing
amplitude, making systenz unstable.
n suc syste$s output is uncontrollable and unbounded one. /utput response o!
suc syste$s is as sown in te 0ig. 8.4.
c(t)
00
....................................................
!"
Stea#y state o$t%$t
c(t)
00
Stea#y state o$t%$t
1"..--------' t
(&) ncreasing e'%onentially
~ - - - - - - - - - - - - - - - - - - - - ~ t
(() ncreasing am%lit$#e oscillations
)ig. &.4 *ncontrolla(le res%onse
0or suc unstable syste$s" i! input is re$o%ed output $ay not return to 2ero. ,nd as
soon as input power is turned on" output tends to 00. 3! no saturation ta4es place in syste$
and no $ecanical stop is pro%ided ten syste$ $ay get da$aged and !ailed.
5e$e$ber tat te stability depends on locations o! closed loop poles. ,nd te closed
loop poles are te roots o! te caracteristic e6uation o! te systenl.
So" &losed loop poles = 5oots o! te caracteristic e6uation
7 7 '. 7 77 7 )8.l. 8".
+ .+ c 7

Control System Engineering 1176
Bode Plots
1 1 1
v) --s-' simple pole, T2 = 20' c.oC2= T2 = 20.
1+20
The straight line of slope - 20 !"ec for c.o#20.
vi) $1+ : 0 > , simple %ero, T& = i o ' c.oC&= i 3 = &0.
The straight line of slope + 20 !"ec for ')# &0.
vii) 1s ,simple pole , T( = )0 ' c.oC(= ) = *0
(1+ 90) (
The st+raight line of slope - 20 !"ec for c.o#*0.
,es-ltant slope ta.le +
,ange of (I) ,es-ltant slope
o < (I) < 1$roc,) - 20 !"ec
1< C'/ 20 $ron) - 20 -20 = - (0 !"ec
20 < $0) / &0 $1e&) - (0 - 20 =- 60 !"ec
&0 / (I) / *0 $1c() - 60 + 20 = - (0 !"ec
*'/ro/00 - (0 - lO =- 60 !"ec
2tep & + 3hase angle ta.le
4 4 o+t =
5 e-'.16ro (1+j~)
30
6ro (1+j~ (1+j ;) (1+j ~)
ro
1
-'.1rora -tan-1 ro
- tan-1#
-1 (I)
-tan-1 ~ I!
7 C6) 20
+ tan 30
90
0.8 - g o o - 0.08 ra - 26.8ao - 1.(&0 + 0.99: - 0.&1: - 120.29:
= - 2.96:
10 - 900 -1 ra - 9(.29: - 26.sao + 19.(&: - 6.&(: - 2(6.08:
= - 87.&:
8 - 90" - 0.8 ra - 79.6go - 1(.0&: +*.(6: - &.1ao - 211.&1:
= - 29.6:
2 - 900 - 0.2ra = - 6&.(&: - 8.71: + &.91: - 1.2;0 - 1690
- 11.(0
(
-*$)0
- 0.( ra -78.*6: - 11.&: + 7.8*0 - 2.8(: -1*(.78:
= - 22<2'
"I ~# r$ $ $ ll% & & '
Control System Engineering 11 81 Bode Plots
At (I) = 1, the magnitude is 36.123dB as obtained by equation (1).
:. 36.123= - 60 Log (I) + 2 i.e. 2 = 36.123.
:. !agnitude in dB = - 60 "og (I)+ 36.123. ... (2)
At (I)= 5, the s#o$e %hanges by + 20 dB&de% hen%e the'e is sim$#e (e'o at (l)C2 = 5.
)hus )2 = *1* = 0.2. )he +a%to' is (1+ 0.2,).
(l)C2
)he magnitude at (I)= , can be ontained by equation (2).
!agnitude in dB (at (I) = ,) = - 60 "og , + 36.123 = - ,.81, dB
)he equation o+ the thi'd #ine is - .0 "og (I) + 3/
r. - ,.81, = - .0 "og , + 3 i.e. 3 = + 22.1.3 dB
r, !agnitude in dB = - .0 "og (#) + 22.1.3. ... (3)
At (I)= 10, the s#o$e %hanges by - .0 dB&de%. As %o''e%tion is (e'o (0 = 0), it is a
quad'ati% +a%to' 1ithout midd#e te'm. )he %ome' +'equen%y is roo = 10. 2en%e
ro. = roo = 10.!"s the +a%to' is [ 1 1 (13 4 2 )5 i.e. 6 u(13 0:o)7
2en%e the o8e'a## t'ans+e' +un%tion is,
ii) 9ith %o''e%tion (4 = 0.316)
"et the quad'ati% +a%to' is s
2
+ 2# (l)n :+ (l)n
2
$
As %ome' +'equen%y is 10, 00n = 10 'ad& se%.
;; 20(I)n = 2< 0.316< 10 = 6.32
=a%to' * 6 1 5
- s2 3 6.32 s3 100
%ntime %onstant +o'm = 1 2
s
(13 0.0632,3 1& & )
't!er ana#ysis 'emains same. 2en%e the o8e'a## t'ans+e' +un%tion is,
>(s)2(s) = 6. (1+ 0.2 s)
s2 ( 13 ,) (13 0.0632 s3 0.01 ,2)
$$ % ()
) * ) $$ r+ + + lio ,,,
Control System Engineering 1182 BodePlots
'.. Example11.22: Derive the transfer function from the given Bode plot.
(M.U. : Dec.-200!
-12
-21
"ig. 11.#
Sol$tion:
%&e inital slope is - 20 '()'ec &ence t&ere is one pole at t&e origin (*s!. %&e e+$ation
o, t&e initial line is y - - 20 .og (I) + C1/&ere y - mag in '(.
y - -20.og(l!0C1 .. (1)
Consi'er secon' line /&ose e+$ation is y - - #0 .og (I) + C2 . 1n' on t&is line
y - 0 '( at (I)- #.
o - - #0 .og # + C2 .e2 C2 - 2#.082
3 = - #0 .og (I)+ 2#.082 ... (2!
1t It)=cot2 3- 4 '( /&ic& is common to 5ot& the lines.
4 - - #0 .og 6 Dt + 2#.082 i.e. (1!1 = 0.707.
Using (1!1 in (1!2 4 = - 20 .og 0.707+ C1 i.e. C1 = 0.0#
y - -20 .og (I)+ 0.0# ... (la!
1t C I.) =. 12 t&e e+$ation (1a! gi8es y = 0.0# '( /&ic& is t&e contri5$tion 5y 9 .e2
20 Log 9. 26
.
.. 20 Log 9 = 0.0# i.e. 9 - 1.:48;
The slope c&anges by - 20 '(I'ec at (I) = (1!1 = 0.707. %&$s t&ere is simple pole /it&
(l!Cl = 0.707. %&$s %l - C iJ~ - 1.<84.The ,actor is =(1 +1><84s! ?
I ~"
" _ " r : : : lio !
Control System Engineering 11-83 Bode Plots
Now on the second line, at (I)= ~ , the magnitude y = - 12 dB.
Using in (2),
- 12 = - 40 Log roz +24.0823 Le, roz = .!8...8
The sloe changes !y +20 dB"dec at ()2"##" 8. Thus the#e is simle $e#o with
%%% $ 8. Thus T2 $ &1& $ %.12'%&e (acto# is (+0.2'').
(l)C2
The e)uation o( thi#d line is y = -2% Log %% +*3. +t (1)2 $ 8, ,= - 12 dB hence,
- 12 $ ( 2% Log 8 +*3 i.e, *3 $ -.%-18
y = - 2% Log (I)+ -.%-18 ... (3)
)n this line, at %% = (1)3the magnitude is - 21dB.
- 21 .//. - 2% Log ()3 +-.%-18 i.e, ()3 = 22.'4
The sloe changes !y - 20 dB"dec at %%3. Thus the#e is simle ole with roo $ 22.'01.
Thus T3 $ &1& $ %.%003. The (acto# is 2 -3
roo 1+%.%003' .
Thus the o4e#all t#ans(e# (unction is,
5( )6( ) 31.1-8 (17%.12'') $
s s $ s(1+1.88-')(1+%.%003s)
0'.13' ('78)
s(s +%.'%3')(s +22.'01)
II.... E*+m,le 11.23/ Obtain the Bode plot for,
G(s)H(s) = 1% (1- s)
'('72) ('2 72s72')
Hence find G.M. and P.M.
Sol-tion # 9te # Time constant (o#m o( .(s)/(s).
1% (l-s)
( :.;./ :ay - 2*%-)
5(s)6(s) $
%.2(1-s)
' 2 '2
s 0 2 0 (1+2)0 2' (1+2' s +2')
9te 2 / +nalysis o( (acto#s
i) < $ %.2, 20 Log < // - 13.88dB, st#aight line a#allel to the Log roa=is.
u) ~ , one ole at o#igin, st#aight line o( sloe (20 dB"dec ass>ng th#ough the
=
'(17%.'') (17 %.%8'7 %.%0'2)
inte#section o( (I)$$ 1and % dB.
1
?iii) (1- s), simle $e#o, Tl = >, roCl $ Tl = 1
The st#aight line o( sloe +2% dB"des (o# ro11.
2I 13
3 4 3 22r###ll. ,,5
Control System Engineering Bode Plots 1188
Corresponding to (J)pc1=20,AB = +64 dB
.. 20LogK = 64
.. K = 1585
Corresponding to (J)pc2= 400, C = 100 dB
.. 20 Log K = 100
.. K = 100000
!o r"nge o# $"l%es o# K is,
I 1&8&' K ' 100000
Review Questions
1. WIult are Bode plots?
2. State the advantages of Bode plots.
3. Explain tIre nature of Bode plots for
i) Poles at origin ii) Simple pole iii} Simple (ero
. Explain the !on!ept o f gain margin tmd phase margin. Explain ho" these l#tliues help in stud$ing
relative sta%ilit$.
&. Write a note 011fre'uen!$ domain spe!ifi!ations.
(. )ra" lhe Bode diagram for
G(s) = l))().02s +1)
(s+1)(0.1&* 1)(0.01&* 1)2
*ar+ the follo"ing on the Bode diilgram, re!ording the numerialll#tllues.
1) -ain !ross.ooer fre'uen!$
2) Phase margin
3) Phse !/ross.overfre'uer0!$
4) CI Zin margin (Ans.: l+ , c = -2. (l)pe = 60. /.0.1. 10. P.0. 2 1&3 st"4le)
7. Given
1) )ra" the Bode diagram.
%) Is the s$stem sta%le? (Ans. 5 2lse = 4&. Qlp ! = 00. /.0. =00. P.0. =181 st"4le)
200 (0.1& +1) C
!(0.2&* 1)(0.0&& +1)
"ig. ##.4$
%I &' r:::lio ( ( )
Control System Engineering 1442 Compensation of Control Systems
IKI = 1
1-0.4+jO.693IIO.6+jO.693113.6+jO.6931
K =1
0.8x 0.9167 x 3.667
K = 2.688
The transfer functin f unc!"ensate# s$ste! is%
2.688
G(s) =
s&s+'(&s+4(
= )un s * s( + ,! s.2.688
Ky s- &- ....0 /&.+1(&s+4(
= 0.672
It is #esire# t ha0e Ky f . sec-(. The factr by 1hich static errr cnstant is t 2e
increase# is%
3actr =
K.. #esire# 5
= 0.672
Kv f unc!"ensate# s$ste!
= 7.44
)et us chse this factr be 10.
- = 10
4'ace the 5er an# "'e f the 'a6 c!"ensatr 0er$ c'se t the ri6in.
Let 5er f c!"ensatr at s =- 0.1
7. 4'e f c!"ensatr at .= - 0.01
/ transfer functin f the 'a6 c!"ensatr is%
*c8s( = i s+0.1
c s+O.OI
9ence the transfer functin f the c!"ensate# s$ste! 2ec!es%
G ( )G( ) K 8+0.1. 2.688
c s s = c s+O.01 s&s+1(&s+4(
!
:0
K&s+0.1(
/&.+1(&s+ 4(&s+0.01(
;here% K = 2..688 Kc
"I #$
$ % $ "" r& & & lio ' ' (
.c
Control System Engineering 1444 Compensation of Control Systems
The new dominant pole is - 0.43 +j 0.67.
Apply magnitude condition at this new location of dominant closed loop pole.
IGc(s!(s1f;=-O.43+jO.67 ;.: 1
. I ! II-O .33+j ".
671
# 1
1-0.43$% 0.671&0.'7$( 0.67113.'7$( 0.67I1-O 4)$( 0.671
*+".746, = 1
..
0.7-61 + 0.,7,6 + 3.63)3 + 0.7-07
.
*
#
).6-
.ut !
#
).6,,
.
i/ c =
1.001-
!ence the compensated system "as the t0ansfe0 function1
G G ).6-('$0.1
c(s (s # s('$0.01 (s$l (s$4
This gi2es static e00o0 constant of 6.7)' which is g0eate0 than '.
14.1#$esigning %ag-%ea& Compensator 'sing (oot %o)'s
It is 3nown that the lead compensato0 inc0eases the speed of the 0esponse and
imp0o2es sta4ility. The lag compensato0 imp0o2es the steady state 0esponse. &f
imp0o2ement in 4oth t0ansient as well as the steady state 0esponse is desi0ed then the
lag-lead compensato0 is used. 5ag-lead compensato0 com4ines the ad2antages of 4oth lead
and lag compensato0s.
Assume the t0ansfe0 function of the lag-lead compensato0 as1
f* + 1, - + 1
5et us conside0 two cases to design such compensato0.
6asel7y8p
Inthis design1 dete0mine the locations of dominant closed loop pol8 f0om the gi2en
specifications.
90om uncompensated G(s1 calculate the angle deficiency 4: which must 4e cont0i4uted
4y lead po0tion of the compensato0.
6hoose T) sufficiently la0ge so that magnitude of lag po0tion is unity.
.I /0
0 1 0 .. r:::lio ,,2
Control System Engineering 14-45 Compensation of Control Systems
= 1
Where s = sl is one of the dominant closed loop pole. From the angle deficiency ,
determine values of Tl and y.
I
5. +T,"
L =

5. +T,"
!
The determine value of I< c from the magnitude condition.
I
s. +T.
" ## I$%s& = t
c Y 1
5. +T.
I
'f the static velocity error constant ", is given then.
", = (im s $e%s& $%s&
s#o
= (im
..... 0
(
I) ( I) )+- 5+-
s'*c T.. T+! $%s&
%5+ i.& %5+, - .
= (im s"c . - $%s&
" ... / y
0s "c and y are 1no2n, f3 can 4e o4tained.
5sing this value of 13 ,choose T+ such that the magnitude of the lag portion is unity.
6 ase+7 y=13
8etermine the locations of dominant closed loop poles according to given
specifications.
0s y =13 .the transfer function of compensator 4ecomes,
P>l
From the re9uirement of "y, c can 4e determined.
I !"
" # " r$ $ $ lio % % &
Control System Engineering 14-46 Compensation of Control Systems
To have the dominant closed loop poles at desired location, calculate the angle
deficiency~
Choose T2 very large at the end so that
( 5 1 + * )
( S + P ~ J
= 1
!here, s " st is one of the dominant closed loop pole#
$o% " 1
and
" & 1 '
These t%o e(uations give the values of Tl and ( ) #
*nce + is ,no%n, choose large T2 so that the magnitude of the lag portion is
appro-imately unity and
1
5 1 + y;
1
sJ + + T2
+ T2should .e high .ut should .e physicallyreali/a.le#
0so < L
1. What is compensation 1 What is compmslltor 1 Which are the various compensation schemes rLwd
in practice 1
2. Which are the "important electrical networks used practiC4lly for the compensation o f the control
systems 1
3. Derive the transfor junctions o f,
II. Ltad ~twork . La! network c# La!"lead network
4. #raw and e$pltlin the polar plot o f,
tl. Lead network . Ln! network c# Ln!"lead network
Review Questions
I ~ " r : : : l io ,,'
Control System Engineering 14-47 Compensation of Control Systems
5. Draw Imd upltlin the Bode plot of,
a. Lead network b. lAg network c. lAg-lead network
6. Derive fhe reltltion between .m and afor the lead compensator.
7. Compare the characteristics of three types of compensators.
. !"plain the design in fre#uenc$ domain of,
a. Lead compensator b. %ag compensator c. lAg-&ead compensator
'. !"plain the design of three compensators using (oot locus metlwd.
10. !"plain the effects and limitations of the three types of electrical compensators.
)). Design a suitable lead compensator for s$stem with,
G(s) =-( 4 2) to meet the specifications as,
ss*
-1-1 b
Q. K =20sec P.M.=+ 5(J' c. C.M.';! + 10d
)+. Design Q suitable lag compensator for a s$stem with,
,-s. =lO(s+1) (~+0.55) to meet the following specifications/
Q. K ';!5 sec-1 b. P.M. : < ! : + 0-12 c. C.M. : < ! : + 10d
)3. Design a lead compensator using root locus for the s$stem with,
,-s4 = (42) to meet the following specifications/
ss*
a. Damping ratio =5.5 b. 6etting time =2 sec
)0. Design a suitable lag compensator root locus for the s$stem with,
,-s4 =s-s* ;f(s+ 2) to meet tire fol7wing specifications/
-1
a. Damping ratio = 5.5 b. 8 C !: 5 sec c. 9ndamped natural fre#uenc$ = 0.7 ,ad&sec
000
~
Control System Engineering P-47 Polar and Nyquist Plots
Q.iS Obtain Nyquist plot -
(i)G(s)H(s) = S(;~6) (ii)G(s)H(s) = : g = ~ ~
(May-2ooS, 12 Marks)
. ./
Hence comment on stability and number of poles on R.11.5. of jro axis.
Ans.: Reer e!a"#$e 12.%&.
Q.i' Explain: !"
(i)#rinciple of ar$uments
(ii) %election of Nyquist pat& is s-plane
(iii) 'appin$ of Nyquist pat& in (-plane
(iv) Nyquist stability criteria.
Ans.: Reer se()ions 12.*, 12.1&an+ 12.11.
Q.i7 Ho) is $ain Imar$in and p&ase mar$in found from m.a$nitude-p&ase plot*
(May-2ooS, 1& Marks)
(May-2&&', S Marks)
Ans.: Reer se()ion 12.1,.
a.iS
,
Draw t&e approximate polar plots of t&e follo)in$ transfer ftmctions :
(
' J 1 (.. ) 1
t 1 . T. II . (1 . T.)
+-ro . -ro +-ro 1
(iv) 1
(jro)2 (1++ro1+)(1 ++ro,-)
/.-
(iii) (1++ro,1)(1 ++.,-)(1 !/-+ro,3)
1
(v) .
(joo) % (1 +jro01)
(May-2&&', 1& Marks)
Ans. : Reer e!a"#$e 12.2, 12.%, 12.4an+ 12.,or #ro(e+1re.
, Q.i* 0iscuss t&e stability of t&e system usin$ Nyquist plot +or G(s)H(s) :1::1 2(s - .
(,21)
(May-2&&', 1& Marks)
Ans. : Reer e!a"#$e 12.%%.
Q.2 0ra) Nyquist plot for G(s) 3(s) = ( ~ )~ ~ )and &ence comment on stabili&4.
, ,+ ,- (3e(.-2&&', 3e(.-2&&1, 1&Marks)
Reer e!a"#$e 12.%'.
If G(s)3(s) = 2(s +1) usin$ polar plot determine t&e ran$e of 2 for stability.
!2 (s22)(, 24) .
5erify yOUT results by 3out&6s criterion. (3e(.-2&&',1& Marks)
, Ana. : Reer e!a"#$e 12%4.
"ns. #
Q.21
$
4 . 5.
% & % '' r###ll. .& & % #,
Control System Engineering P-48 Polar and Nyquist Plots
Q.22 If G(s)H(s) = s (1+2S)~1+0.1s) using polar plot determine the range of K fo r stability.
Verify your results by Routh criterion. (!ay-200" 10 !ar#s)
$ns.% &e'ere(am)le 12.*+.
I
I
II
II
"
II
II
II
II
,
I
Q.2* Draui lire Nyquist plot for ;
G(s)H(s) = (4s+1
52(+,1)(2+,1)
(!ay-200" 10 !ar#s)
!ence" comment on stability.
$ns.% &e'ere(am)le 12.8 'or t-e )ro.ed/re and 0eri'y 1 = + 2 /nsta2le.
Q.24 Dra# the polar plot for a system gi$en by"
%(s!(s = o S (++2)(~0~4)(+ +S)3 &ind #hether tlte system is stable and if so find G.!. and
'.(. 14 100 is replaced by K find critical $alue of K by Routh criteria and also $erify %.(.
and '.(. (5e..-200" 12 !ar#s)
$ns. % &e'er e(am)le 12.*2'or t-e )ro.ed/re.
QQQ
.
6
6tt.l
.
I ~"
" _" II r : : : l io , , '
Technical Publications Pune~
Contents
Control System Analysis
Examples of control systems, Open loop conrrol system... Closed loop control systems Transfer
function Types of feedback and feedback control ~stem characteristics Noise ,!ection. "ain.
Sensiti#ity. Stabtlity.
$athematical $odelin% of Systems
&mportance of a mathematical model. 'lock dia%rams, Signal flow (raphs, $a~an!s %mn formula
and its applicallon 10block diagrdm reducoon State space method. Solving time-invanam
system.Transferrnamx
Transient and Steady State )esponse Analysis
&mpulse response function. Firsl order *+,.lorem. Second order ,stem. Time domain SpeClll-dllons
of .o,l!tems.Analysis of transient/response ~,% second order model
Classlflcllion of control systems accordin% 01 'Type of S+. !2.mcSlloddy tar errors! Static rror
constants Steady . *0 te analy is of different &ypb of ~~&ems us"ngstep. ramp and parabolic
"nput si%nal!
Stability Analysis
#oncept of ~tabillty! ~wblhly anai~l$ uSin% )outh!s stability entenon %h.'"&iulec'tablhty.) 3000..,
stability. )oot4-56cus plo~ ~ummary of %enua! rull * f5 c!.lnrnac..in )#o, )oo /-ocu
analysis of control +7ystemcCompensauon t chru(ue /-o%, lead. Io q -I d
8re(uency/)esponse Analysis
l ro(uenr/ domam specif).chlons. ), "dll#' ,..&t and pe*l) n! onCiun% l" ..u C! , )elauonsl "P
b..t9 en time md fnlfuenc# domain speC&fication of i terns 'ode plo., :aldr plote,. -o%
ma%nltud .. +s phaseplo~. ,y(ul~ stab"lity cit "on. !&abihr.ydnc-li~ . otll / bihtu
flam mar%lO. :hase margm. Stab..lty 3mal++ u. /n ;7ln% 'ode 0!01 Ci d/loop
fre(ut'n/+rcsponse/Constent %ain and phe 10. Nichol!+7ch. rl and then 0.. &n . .."ty &ud.
OfS<,lst-&n
Control Components and Controller
=.C and A C s -rvomosors. Ser#oamplifler. :lt .. nnorneter! C nchro tran..mlttl.:.< Synchro
recei#ers. Synchro control transformer! otcpper motors 1" .onnnuous controller mode>
Cuntinuous colltroll !&! mode Composue 0..1n,,100
First 2dition- 3004
)s.*?1@/
&S'~ ABC/C0/CDE0/D?E/F
G 1! Amit )esidency. 513 Shani9ar :eth, :une4 511060. $.S., &ndia.
Telefa7- 891 :030; 3559<59=>9?! 2mail- technical@vtuboo)s.com
+isit usat- www.vtuboo)s.com

Potrebbero piacerti anche